Download as pdf or txt
Download as pdf or txt
You are on page 1of 156

Practice Set 1

BITSAT
Instructions
1. There are 150 questions in all. The number of questions in each part is as follows Subject No. of Questions
Part I Physics 1-40
Part II Chemistry 41-80
Part III (a) English Proficiency 81-95
(b) Logical Reasoning 96-105
Part IV Mathematics 106-150
2. All questions are multiple choice questions with four options, only one is correct.
3. Each correct answer fetches 3 marks while incorrect answer has a penalty of 1 mark.

PART I
Physics
æ az ö
a ç- ÷ 4. A body A starts from rest with an acceleration a1. After
1. In the relation, p = e è kq ø , where p is pressure, z is two seconds, another body B starts from rest with an
b
acceleration a 2. If they travel equal distances in 5th
distance, k is Boltzmann constant and q is second after the start of A, then the ratio of a1 : a 2 is
temperature. The dimensional formula of b will be equal to
a. [M0L2T 0] a. 5 : 9 b. 1 : 3
b. [ML2T] c. 3 : 1 d. 9 : 1
c. [M0L2T -1]
5. A smooth block is released at rest on a 45° incline and
d. [ML0T -1] then slides distance d. The time taken to slide is n
-2
2. A cube has a side of length 1.2 ´ 10 m. Calculate its times as much to slide on rough incline than on a
smooth incline. The coefficient of friction is
volume.
1 1 1 1
a. 1.7 ´ 10-6 m3 a. 1 - b. 1 - c. -1 d. +1
n2 n2 n2 n2
b. 1.73 ´ 10-6 m3
c. 1.70 ´ 10-6 m3 6. A long horizontal rod has a bead which can slide
d. 1.732 ´ 10-6 m3 along its length and initially placed at a distance L from
one end of the rod. The rod is set in angular motion
3. Maximum height reached by projectile is 4 m. The about A with constant angular acceleration a. If the
horizontal range is 12 m. Velocity of projection is coefficient of friction between the rod and the bead is
g g m and the gravity is neglected, then time after which
a. 4 b. 5 the bead starts slipping is
2 2
1 g 1 g 1 1
c. d. a. m / a b. m / a c. d.
4 2 5 2 ma m a
1196 SELF STUDY GUIDE BITSAT

7. A particle tied to string describes a vertical circular 14. A temperature of equal masses of three different
motion of radius r continually. If it has a velocity liquids A, B and C are 12° C, 19 ° C and 28° C,
( 3gr ) at the highest point, then the ratio of the respectively. The temperature when A and B are
respective tensions in the string holding it at the mixed is 16° C, when B and C are mixed is 23° C.
highest and lowest points is What is the temperature when A and C are mixed ?
a. 4 : 3 b. 5 : 4 a. 26.02°C b. 26.60°C
c. 1 : 4 d. 4 : 1 c. 20.26°C d. 21.62°C

8. A shell of mass 20 kg at rest explodes into two 15. 1 mm 3 of a gas is compressed at 1 atmospheric
fragments whose masses are in the ratio 2 : 3. The pressure and temperature 27° C to 627° C. What is the
smaller fragment moves with a velocity of 6 ms -1. The pressure under adiabatic condition? (g for gas = 1.5)
kinetic energy of the larger fragment is a. 27 ´ 105 Nm-2
a. 96 J b. 216 J b. 36 ´ 105 Nm-2
c. 144 J d. 360 J c. 56 ´ 105 Nm-2
9. A wire of length l and mass m Y d. 80 ´ 105 Nm-2
is bent in the form of a

æ
semicircle. The gravitational 16. If one mole of a monoatomic gas ç g = ÷ is mixed with
field intensity at the centre of è 3ø
semicircle is X æ 7ö
O one mole of a diatomic gas ç g = ÷, the value of g for
Gm Gm è 5 ø
a. along X-axis b. along Y-axis the mixture is
pl pl
2pGm 2p Gm a. 3.7 b. 1.63
c. along Y-axis d. along X-axis
l2 l2 c. 1.50 d. 1.40

10. A spherical liquid drop of radius R is divided into eight 17. In the given diagram, S1 and S 2 are identical springs.
equal droplets. If surface tension is T, then work done The frequency of oscillation of mass m is f . If one
in the process will be spring is removed, the frequency will be
a. 2pR 2T b. 3pR 2T S1 S2
c. 4pR 2T d. 2pRT 2 m
11. A body of density D1 and mass M is moving downward
in glycerine of density D 2. What is the viscous force a.
f
b.
f
acting on it ? 2 2
a. Mg D1 b. Mg D2 c. 2f d. 2f
æ D ö æ D ö 18. A capacitor of capacitance 1mF is filled with two
c. Mg çç1 - 2 ÷÷ d. Mg çç1 + 2 ÷÷
è D1 ø è D1 ø dielectrics of dielectric constants 4 and 6. The new
capacitance is
12. A 2 kg copper block is heated to 500°C and then it is a. 10 mF b. 5 mF
placed on a large block of ice at 0°C. If the specific
c. 4 mF d. 7 mF
heat capacity of copper is 400 J/kg°C and latent heat
of fusion of water is 3.5 ´ 10 5 J/kg, the amount of ice 19. A small oil drop of mass 10-6 kg is hanging at rest
that can melt is between two plates separated by 1 mm having a
7 7 potential difference of 500 V. The charge on the drop is
a. kg b. kg
3 5 a. 2 ´ 10-9 C b. 2 ´ 10-11C
7 8 c. 2 ´ 10-6 C d. 2 ´ 10-8 C
c. kg d. kg
8 7
20. Three plates of area A each are connected as shown
13. A same force is acting on two wires made of same in figure. The effective capacitance becomes :
material. One wire has length l and diameter d. The
æl ö d
other wire has length ç ÷ and diameter 2d. If the d
è 2ø
extensions in the two wires are l1 and l 2 such that e0A 3e0A
l1 + l 2 = 1 cm, then the values of l1 and l 2 are a. b.
d d
a. 0.80 cm, 0.20 cm b. 0.89 cm, 0.11 cm 1.5 e0A 2 e0A
c. 0.90 cm, 0.10 cm d. 0.95 cm, 0.05 cm c. d.
d d
PRACTICE SET 1 1197
21. Two capacitors C1 = 2m F and C 2 = 6 mF in series are 26. In the diagram I1, I 2 are the strength of the currents in
connected in parallel to a third capacitor C 3 = 4 mF. the loop and straight conductor respectively.
This arrangement is then connected to a battery of OA = OB = R . The net magnetic field at the centre O is
emf 2V as shown in figure. How much energy is lost zero. Then, the ratio of the currents in the loop and the
by the battery in charging the capacitors? straight conductor is
C1 C2
I1 R
A
C3 R
B
I2
2V
a. p b. 2p c. 1/p d. 1/ 2p
-16
a. 22 ´ 10 J b. 11 ´ 10-6 J
27. The magnetic field intensity due to a thin wire carrying
32 16
c. ´ 10-6 J d. ´ 10-6 J current I in the figure shown, is
3 3
22. In a circuit, galvanometer G shows zero deflection. If
the batteries A and B have negligible resistance,the O
value of R will be
R
G 2a
500 W
12 V R 2V m 0I m 0I
a. ( p - a + tan a ) b. (p - a )
2pR 2pR
a. 1000W b. 500W m I m I
c. 0 ( p + a ) d. 0 ( p + a - tan a )
c. 200W d. 100W 2pR 2pR
23. In the given figure, the internal resistance of the 28. Electromagnets are made of soft iron because soft
battery is 1.5W and VP and VQ are potential at P and Q iron has
respectively. What is the potential difference between a. low susceptibility and high retentivity
the points P and Q ? b. high permeability and low retentivity
20 V 1.5 W c. low permeability and high coercivity
d. low permeability and high retentivity
3W 2W 29. A magnetic dipole of moment 0.72 Am 2 is placed
A B
P horizontally with the north pole pointing south. The
2W 3W magnetic field of earth is 18 mT. The neutral points is at
Q a. 0.1 m on axial line b. 0.2 m on equatorial line
c. 0.2 m on axial line d. 0.16 m on equatorial line
a. Zero b. 4 V (VP > VQ )
c. 4 V (VQ > VP ) d. 2.5 V (VQ > VP ) 30. Two coils have a mutual inductance 0.005 H. The
current changes in the first coil according to equation
24. In the given figure, the potentiometer wire AB has a I = I 0 sin wt , where I 0 = 10A and w = 100p rad / s. The
resistance of 5W and length 10 m. The balancing maximum value of emf in the second coil is
length AJ for the emf of 0.4 V is a. 2p b. 5p c. 6p d. 12p
5V 4.5 W K 31. In a series LCR circuit, voltage across R is 100V and
R = 1kW with C = 2mF. The resonant frequency w is
200 rad/s. At resonance the voltage across L is
a. 4 ´ 10-3 V b. 2.5 ´ 10-2 V
J
A B c. 40 V d. 250 V
G 32. A thin lens of glass (n = 1.5) of focal length 10 cm is
0.4V immersed in water (n¢ = 1.33). The new focal length is
a. 0.4 m b. 0.8 m
a. 12 cm b. 20 cm
c. 1.52 m d. 4 m
c. 40 cm d. 48 cm
25. A milliammeter of range 10 mA has a coil of resistance
33. Time taken by sunlight to pass through a window of
1W. To use it as a voltmeter of range 10V, the
thickness 4mm and refractive index 3 / 2 is
resistance that must be connected in series with it is
a. 2 ´ 10-4 s b. 2 ´ 108 s
a. 9W b. 99W
c. 999W d. 1000W c. 2 ´ 10-11s d. 2 ´ 1011s
1198 SELF STUDY GUIDE BITSAT

34. The size of image of an object, which is at infinity, as 37. The transition from the state n = 4 to n = 3 in a
formed by a thin convex lens of focal length 30 cm is hydrogen atom results in ultraviolet radiation. Infrared
2 cm. If a concave lens of focal length 20 cm is placed radiation will be obtained in the transition from
between the convex lens and the image at a distance a. 3 ® 2 b. 4 ® 2
of 26 cm from the convex lens, calculate the new size c. 5 ® 4 d. 2 ® 1
of the image.
38. In a transistor the base is
a. 1.25 cm b. 2.5 cm c. 1.05 cm d. 2 cm a. an insulator
35. A slit width a is illuminated by red light of wavelength b. a conductor of low resistance
6500Å. If the first minimum falls at 30°, the value of c. a conductor of high resistance
a is d. an extrinsic semi-conductor
a. 6.5 ´ 10-4 mm b. 1.3 ´ 10-6 m 39. In a common emitter transistor, the base current
c. 2.6 ´ 10-4 m d. 1.3 ´ 10-4 m Ib = 2mA, a = 0.9, then Ic is equal to
a. 18 mA b. 20 mA
36. The energy (E ) of a photon is equal to the kinetic c. 22 mA d. 24 mA
energy of proton. Let l1 be the de-Broglie wavelength
of the proton and l2 be the wavelength of the photon. 40. The antenna of an AM broadcast transmitter
modulated by 50% is 11A. Find the carrier current.
The ratio of l1 and l2 is proportional to a. 9.25 A b. 10A
a. E -2 b. E -1 c. E1/ 2 d. E 0 c. 10.35 A d. 5.5 A

PART II
Chemistry
41. SOCl2 B
45. O
COOH A CONH2
(i) NaOMe
CH3 P
(ii) H+, D
H3O+

D
R OH
C The product P is
The product D has functional group a. acetone b. a , b-unsaturated ketone
a. amide b. alcoholic c. ether d. ester c. a, b-unsaturated acid d. None of these
42. Choose the correct statement from the following. 46. The decreasing order of boiling points of the following is
a. BF3 and NF3 are covalent compounds and both are I. butanal II. butan-1-ol
polar in nature III. pentane IV. diethyl ether
b. BF3 and NF3 are covalent compounds but BF3 is a a. IV > III > II > I b. IV > II > III > I
non-polar, while NF3 is polar compound c. II > I > IV > III d. I > IV > II > III
c. BF3 and NF3 are covalent compounds but BF3 is polar 47. Consider the following reaction,
while NF3 is a non-polar compound OH
Reagent
d. BF3 and NF3 are covalent compounds and both are O2N COOH O2N
non-polar in nature
43. The freezing point depression of a 1.00 ´ 10-3 m The reagent is
–3
. ´ 10 K. Calculate the
solution of K x [ Fe(CN)6] is 710 a. LAH b. B2H6 /THF
value of x.(K f = 1.86 K kg mol–1 for H2O) c. Both a and b d. None of these

a. 2 b. 3 c. 4 d. 1
48. NH N H N
44. Given below are several electronic configuration that
may be correct for oxygen atom. Which of the following
represents the ground state configuration for oxygen? O
I II III
a. ­¯ ­¯ ­¯ ­ — ¯
b. ­¯ ­¯ ¯ ¯ ¯¯ — The correct decreasing order of basic strengths of
c. ­¯ ­¯ ­¯ ­¯ — — I, II and III is
a. I > III > II b. II > III > I
d. ­¯ ­¯ ­¯ ­ ­ —
1s 2s 2p 3s
c. I > II > III d. II > I > III
PRACTICE SET 1 1199
49. Which of the following is/are correct? 54. A mixture of solid SrSO4 and solid BaSO4 is shaken
I. XeF6 has lone pair of electron. up with water until saturated equilibrium is
II. XeF4 has two lone pairs of electrons with established. Given that,
sp 3d 2 hybridisation. K sp( SrSO4 ) = 7.5 ´ 10-7, K sp ( BaSO4 ) = 1.5 ´ 10-9
III. XeOF2 has dsp 2-hybridisation. Then, the concentration of SO24 - is
IV. XeO3 has square planar geometry. a. 8.52 ´ 10-3 M
a. Only I b. I and II b. 8.67 ´ 10-4 M
c. III and IV d. Only IV
c. 1.73 ´ 10-6 M
50. Which one undergoes fastest nucleophilic substitution d. 1.82 ´ 10-4 M
reaction?
a. Benzyl chloride 55. The value of E ° of some elements are as
b. Allyl chloride I2 + 2e - ¾® 2I- ; E ° = 0.54 V
c. Formaldehyde
MnO24 - + 8 H+ + 5e - ¾® Mn2 + + 4 H2O;E ° = 1.52 V
d. Acetaldehyde
Fe3 + + e - ¾® Fe2 + ; E ° = 0.77V
51. Match the following columns and choose the correct
code given below. Sn4 + + 2e - ¾® Sn2 + ; E ° = 0.1 V
Column I Column II On the basis of values of E °, which of the following
P. H2(g ) + I 2(g ) s 2HI(g) (i) K p = KC (RT ) reactions will be spontaneous?
Q. N2(g ) + 3H2(g ) s 2NH3 (g ) (ii) K p = KC (RT ) 2
a. Sn4+ + 2Fe2+ ¾® Sn2+ + 2 Fe3+
b. 2Fe2+ + I2 ¾® 2Fe3+ + 2 I-
R. PCl5(g ) s PCl3 (g ) + Cl2(g ) (iii) K p = KC (RT ) - 2
c. Sn4+ + 2 I- ¾® Sn2+ + I2
S. NH4 Cl(s) s NH3 (g ) + HCl( g ) (iv) K p = KC d. Sn2+ + I2 ¾® Sn4+ + 2I-

Codes 56. NBS Mg


A B
P Q R S P Q R S O
a. (i) (iii) (iv) (iii) b. (iv) (iii) (i) (ii)
c. (iii) (iv) (ii) (i) d. (i) (iv) (ii) (iii) H+ CH3 C OCH3
(i) Trace of C
D
52. OH Grignard
reagent
CO2H (ii) H+
K2Cr2O7 What would be the product D in the above reaction
D Product
sequence?
What is the product of the following reaction? a. 1° alcohol
O b. 2° alcohol
c. 3° alcohol
d. cyclohexene
a. b.
57. The monomer of the polymer nylon-6, 10 is
O
a. hexamethylene and sebacic acid
c. COOH d. COOH b. hexamethylene and adipic acid
c. caprolactum
d. None of the above
53. O–
58. A dye absorbs light ray of wavelength ( l) 4530 Å and
(i) CO2 re-emits radiation of wavelength 5080 Å. If we assume
A
(ii) H– that 47% of energy absorbed is emitted, then the ratio
of quanta emitted and absorbed is
Which one of the following statements is true about a. 0.231 b. 0.527
the reaction? c. 0.412 d. 0.612
a. Para isomer is major, if PhOK is used
59. On the basis of electronic configuration, if the ion X 3 -
b. Ortho isomer is major, if PhONa is used has 14 electrons, then the number of electrons
c. Product formed is further used for preparation of present in X 2+ is
drug aspirin a. 16 b. 14
d. All of the above c. 12 d. 18
1200 SELF STUDY GUIDE BITSAT

60. Cl Cl Cl 67. The number of electrons exchanged in the reaction


Cl Cl Cl Cl Cl given below
Al + Fe3O4 ¾® Al2O3 + Fe
a. 36 b. 20 c. 24 d. 32
Cl Cl Cl
1 mol C2H5 Br NaNH2 CH3I
I II III IV 68. HC ºº CH ¾¾® A ¾¾® B ¾¾® C ¾® D
Ethyne NaNH2
The correct increasing order of dipole moment of the
following compounds is The product D of the following reaction is
a. I < II < IV < III b. II = IV < III < I a. pent-2-yne b. pent-2-ene
c. II < IV = III < I d. III = IV < I < II c. but-2-yne d. but-1-yne
61. 69. The separation of a racemic mixture into the pure
CH2CH CHCH3 enantiomers is known as
X a. resolution b. racemisation
c. isomerisation d. equilibration
The correct IUPAC name of compound X is
NaBH4 LAH
a. 1-(3-butenyl) cyclohexene 70. B ¬¾¾ Ethanoic anhydride ¾® A
b. 1-(2-butenyl) cyclohexene
c. 2-(4-butenyl) cyclohexene ½H / Pt
d. 2-(3-butenyl) cyclohexene ¯ 2
C
62. Standard heat of formation ( DH °f ) of PCl 5 and PCl3
The product A, B and C respectively are
are x1 and x 2 respectively. The DH ° for the reaction a. 2CH3CH2OH , no reaction, 2CH3CH2OH
PCl3(g ) + Cl2(g ) ¾® PCl5(g ) is
b. 2CH3CH2OH , no reaction, no reaction
a. x1 + x 2 b. x1 - x 2
c. 2CH3CH2OH , 2CH3CH2OH, 2CH3CH2OH
c. x1x 2 d. x1 ¸ x 2 d. no reaction, no reaction, 2CH3CH2OH
63. A unit cell of sodium chloride has four formula units. 71. Consider the following reaction,
Calculate the density of sodium chloride, if the edge H 3C
CN

length of the unit cell is 0.564 nm. H 3C Cl H 3C CN +
a. 2.165 g/cm3 b. 2.165 g/m3 I
CN
c. 2.165 mg/cm3 d. 4.81 g/cm3 II

64. Which of the following reactions occurs at the anode Which of the following statements via S N 2 is correct?
during the recharging of lead storage battery? a. Formation of (I) and (II) proceeds via SN1
a. PbSO4 + 2e - ¾® Pb + SO24 - b. Formation of (I) proceeds via SN1 while (II)
b. Pb + SO24- ¾® PbSO4 + 2e - c. Formation of (I) and (II) proceed via SN 2
c. Pb + PbO2 + H2SO4 ¾® 2PbSO4 + 2 H2O d. Formation of (I) proceed via, SN 2 while (II) via SN1
d. PbSO4 + 2 H2O ¾® Pb + PbO2 + 2 H2SO4 + 2e - 72. The equivalent mass of which of the following
65. Which of the following acids on heating loses a (underlined) is 8 g equiv - 1?
molecule of H2O to form an a, b-unsaturated acid? a. H3PO4 + Ca(OH)2 ¾® CaHPO4 + 2H2O
a. HOCH2COOH b. CH3CHOHCH2COOH b. H3PO4 + 3NaOH ¾® Na 3PO4 + 3H2O
c. CH3CHOHCOOH d. HOCH2CH2CH2COOH c. H3PO4 + NaOH ¾® NaH2PO4 + H2O
d. 2H2 + O2 ¾® 2H2O
66. For a reaction,
2NO(g ) + 2H2(g ) ¾® N2(g ) + 2H2O (g ) 73. Analysis of chlorophyll shows that it contains
2.68 per cent magnesium. How many atoms of
[NO]
[H2] (mol L- 1 ) Rate (mol L- 1s - 1 ) magnesium does 1.00 g of chlorophyll contain?
(mol L- 1 )
a. 8.12 ´ 1021 b. 5.92 ´ 1021
1. 5 ´ 10-3 2.5 ´ 10-3 3 ´ 10- 5
c. 6.72 ´ 1020 d. 5.21 ´ 1020
2. 15 ´ 10-3 2.5 ´ 10-3 9 ´ 10- 5
74. The artificial sweetener used in soft drinks is
3. 15 ´ 10-3
10 ´ 10 -3
3.6 ´ 10 - 4 a. fructose
b. cellulose
Order of reaction w.r.t NO and H2 respectively, are c. glucose
a. 1, 2 b. 1, 1 c. 1, 3 d. 2, 2 d. aspartame
PRACTICE SET 1 1201
75. Silver chloride dissolves in a solution of ammonia but 78. Out of TiF62 - , CoF63 - , Cu2Cl2 and NiCl24 - , the
not in water because colourless species are (Given atomic number of
a. ammonia is a stronger base than water
Ti = 22, Co = 27, Cu = 29 and Ni = 28)
b. ammonia is a better solvent than water
c. silver ion forms a complex ion with ammonia a. CoF63 - and NiCl24 - b. Cu2Cl2 and NiCl24 -
d. the dipole moment of water molecule is higher than c. TiF62 - and CoF63 - d. TiF62 - and Cu2Cl2
that of ammonia molecule
79. NH3 has a net dipole moment but boron trifluoride
76. When NaCl or KCl is heated with conc. H2SO4 and
( BrF3 ) has zero dipole moment, because
solid K 2Cr2O7, the product is
a. F is more electronegative than H
a. chromous chloride b. chromic chloride b. B is less electronegative than N
c. chromic sulphate d. chromyl chloride c. NH3 is pyramidal, while BF3 is trigonal planar
77. The IUPAC name of the complex d. BF3 is pyramidal, while NH3 is planar
fac – [Co(CH3 NH2 )3 Cl3 ] is
80. The antibiotic streptomycine is specific against which
a. fac -trichlorot ris (methylamine) cobalt (V)
of the following disease?
b. fac -trichlorot ris (methylamine) cobalt (III)
c. tris (methylamine) trichlorocobalt (III) a. Typhoid b. Malaria
d. tris (methylamine) trichlorocobalt (V) c. AIDS d. Tuberculosis

PART III
a. English Proficiency
Directions (Q. Nos. 81 to 84) Read the following passage c. there is an unnecessary repetition in the reference to
carefully and choose the most suitable option from the given the old man
ones. d. the figure of the old man is brought under a
An old man steel-rimmed spectacles and very dusty clothes sharp focus
sat by the side of the road. There was a pontoon bridge Directions (Q. Nos. 85 to 87) In the following sentences,
across the river and carts, truck and men, women and there is an error. Spot the error and indicate your answer.
children were crossing it. The mule-drawn art staggered up 85. Order has been issued (a)/for his transfer to another
the steep bank from the bridge with soldiers helping to push district (b)/but he has not received them so far. (c)/No
against the spokes of the wheels. The truck wound up the error (d)
away heading out of it all. The peasants plodded along in the
ankle-deep dust. But the old man sat there without moving. 86. Although she has studied (a)/English for almost a year
(b)/she is yet to learn the alphabets. (c)/No error (d)
81. The term ‘pontoon bridge’ means
a. a temporary bridge constructed with the help of ropes
87. There are two scores of books (a)/which are lying
b. a bridge made by soldiers during a war (b)/unused in the library. (c)/No error (d)
c. a bridge supported by floating flat-bottomed boats Directions (Q. Nos. 88 to 90) Find the word from the
d. a bridge made with wooden planks options that is synonym to the word written in capital letters.
82. The soldiers were ‘helping to push against the spokes 88. We should not allow ADVERSITY to discourage us.
of the wheels’ because
a. Poverty b. Darkness
a. they wanted to stop the carts c. Time of trouble d. Unfriendly criticism
b. the spokes of the wheels were broken
c. the mules refused to draw the carts 89. He is very OBLIGING by nature.
d. there was a steep elevation a. Helpful b. Nice c. Thankful d. Compelling
83. The mule-drawn carts staggered up because 90. She thought that RUNNING INTO her old friend was a
a. there were too many mule-carts strokes of fate.
b. the mules were undisciplined a. Meeting by chance b. Colliding
c. it was a steep uphill journey c. Travelling with d. Quarrelling with
d. the carts were blocked by the peasants
84. The reference to the old man in the beginning and the
Directions (Q. Nos. 91 to 93) Find the word from the
options that is antonym to the word written in capital letters.
end of the passage indicates that
a. the writer want to compare between the bridge and 91. The COMPLAINANT was not supportive of providing
motionless old man all facts in the court.
b. the description that takes place between the first a. Defendant b. Advocate
sentence and the last sentence is irrelevant c. Indulgence d. Servant
1202 SELF STUDY GUIDE BITSAT

92. Living in a SOLITARY place brings in some kind of 100. Select a figure from the given four alternatives which
satisfaction. when placed in the missing portion (?) of problem
a. Limited b. Exotic figure (X ) would complete the pattern.
c. Healthy d. Populous Problem Figure
93. They spent a DISTURBED night after hearing the
tragic news.
a. Restless b. Sleepless
c. Mournful d. Peaceful
Directions (Q. Nos. 94 and 95) Find one word for the set ?
of words given below.
(X)
94. One who knows everything.
a. Pedantic b. Omniscient Answer Figures
c. Knowledgious d. Educated
95. Aversion from water
a. Hydrophobiac b. Aquatic
c. Hydrophilic d. Exaquatic
a. b. c. d.

b. Logical Reasoning 101. Mohit is 14th from the right end in a row of 40 boys.
What is his position from the left end ?
96. In a certain code, SOBER is written as RNADQ. How a. 24th
LOTUS can be written in that code? b. 25th
a. KNSTR b. MPUWT c. 27th
c. KMSTR d. LMSRT d. None of these
97. Find the missing number in the following figure. 102. The number of triangles in the figure is

93 27 79 38 67 16

63 37 42

3 4 ?
a. 24 b. 21
c. 14 d. 11
a. 5 b. 19 c. 27 d. 9
98. Find the missing number in the following figure. 103. Which of the following is the water image of the given
figure?
5 7 18
6 93 15 9 ? 5 4 50 1

3 6 8

a. 5 b. 19 c. 27 d. 89
99. Find the figure will that replace the question mark (?).
Problem Figures a. b. c. d.

Directions (Q.Nos. 104 and 105) Choose the odd one out.
?
104. a. Home
(i) (ii) (iii) (iv)
b. City
c. Town
Answer Figures d. Village
105. a. Parrot
b. Owl
c. Eagle
d. Hawk
a. b. c. d.
PART IV
Mathematics
106. The sum of order and degree of the differential 5 8 4 1
a. b. c. d.
equation y 32/3 + 2 + 3 y 2 + y 1 = 0 is 36 36 9 3
a. 2 b. 3 c. 4 d. 5 116. If [x ] and {x } represent integral and fractional parts of x,
2000
| x + 2| {x + r }
æ 1 ö2 - | x |
> 9 is
then the expression [x ] + å is equal to
107. The solution of the inequation ç ÷ r =1 2000
è3ø 2001 2001
a. (2, 4) b. (4, 6) c. (2, 6) d. None a. x b. x + 2001 c. x d. [x ] +
2 2
108. If z 2 + z | z | + | z |2 = 0, then the locus of z is 117. The value of lim x m (log x )n , m , n Î N , is
a. a circle b. an ellipse x ® 0+
c. a pair of straight lines d. None of these a. 0 b. m /n c. mn d. n /m
109. If a, b, g are the roots of x 3 - x 2 - 1 = 0, then the value ì 1 - |x |
(1 + a ) (1 + b) (1 + g ) ï , x ¹1
of + + is 118. If f (x ) = í 1 + x , where [ ] represents the
(1 - a ) (1 - b) (1 - g ) ïî 1, x = -1
a. -2 b. -3 c. -4 d. -5 greatest integer function, then f ([ 2x ]) is
110. If AM, GM and HM of the first and last terms of the a. continuous at x = - 1
series 100, 101, 102, …, n - 1, n are the terms of the b. continuous at x = 0
series itself, then the value of n (100 < n £ 500) is 1
c. discontinuous at x = ,1
a. 200 b. 300 c. 400 d. 500 2
111. A train time table must be compiled for various days of d. All of the above
the week, so that two trains twice a day depart for
119. If f (x ) = (1 - x )n , then the value of
three days, one train daily for two days, and three
trains once a day for two days. How many different f ¢¢( 0) f n ( 0)
f ( 0) + f ¢( 0) + + ... + is
time table can be compiled ? 2! n!
a. 120 b. 210 c. 133 d. 72 a. 2n b. 0 c. 2n - 1 d. None

112. If C 0, C1, C 2, ..., Cn are the binomial coefficients, then 120. Let F (x ) = f (x ) g (x ) h(x ) for all real x, where f (x ), g (x )
2 ´ C1 + 23 ´ C 3 + 25 ´ C5 + ... equals and h(x ) are differentiable functions. At some point x 0,
3n + ( -1)n 3n - ( -1)n 3n + 1 3n - 1 if F ¢(x 0 ) = 21f (x 0 ), f ¢(x 0 ) = 4f (x 0 ), g ¢(x 0 ) = - 7g (x 0 ) and
a. b. c. d.
2 2 2 2 h¢(x 0 ) = lh(x 0 ), then l is equal to
113. In DABC, if a. 12 b. -12 c. 24 d. -24
1 1 1 121. If y = 4x - 5 is a tangent to the curve y 2 = px 3 + q at
A B C (2, 3), then
cot cot cot = 0, a. p = 2, q = - 7 b. p = - 2, q = 7
2 2 2
c. p = - 2, q = - 7 d. P = 2, q = 7
B C C A A B
tan + tan tan + tan tan + tan
2 2 2 2 2 2 122. If f (x ) = x a log x and f ( 0) = 0, then the value of a for
then the triangle must be which Rolle’s theorem can be applied in [0, 1] is
a. equilateral b. isosceles a.-2 b. -1 c. 0 d. 1/2
c. obtuse d. None of these 123. The minimum value of 27 cos 2x
× 81 sin 2x
is
114. If A and B are square matrices of the same order and a. 1/243 b. -5
A is non-singular, then for a positive integer n, c. 1/5 d. 1/3
( A -1 BA )n is equal to 124. If f (x ) = x 3 + bx 2 + cx + d and 0 < b 2 < c, then in
-n n n n n -n
a. A B A b. A B A ( -¥, ¥ )
c. A -1BnA d. n ( A -1BA ) a. f (x ) is strictly increasing function
3 2 f (x ) has a local maxima
115. Consider f (x ) = x + ax + bx + c . Parameters b.
a , b and c are chosen, respectively, by throwing a die c. f (x ) is a strictly decreasing function
three times. Then, the probability that f (x ) is an d. f (x ) is unbounded
increasing function, is
1204 SELF STUDY GUIDE BITSAT

125. (1 + x )n £ 1 + x n , where 134. If the point (a 2, a + 1) lies in the angle between the
a. n > 1 b. 0 £ n £ 1 and x > 0 lines 3x - y + 1 = 0 and x + 2y - 5 = 0 containing the
origin, then the value of a is
c. n > 1and x > 0 d. x < 0
æ 1ö
a. a Î( 3, 0) Ç ç1, ÷ b. a Î(5, 0) Ç ( 0,5)
126. If y = f (x ) makes positive intercepts of 2 units and è 3ø
1 unit on x and y -coordinate axes and encloses an æ1 ö
2 c. a Î (–3, 0) È ç , 1÷ d. None of these
area of 3/4 sq unit with the axes, then ò x f ¢(x )dx is è3 ø
0
3 5 -3
a. b. 1 c. d. 135. If a = – $i + $j + k$ , b = 2$i + 0$j + k, then a vector x
2 4 4
satisfying the conditions that it is coplanar with a and
127. ò x - 3 {sin-1(ln x ) + cos -1(ln x )} dx is equal to b; perpendicular to b and a × x = 7, is
p 1
a. (x - 3)3/ 2 + C b. 0 a. -3 $i + 5$j + 6k$ b. ( -3$i + 5$j + 6k$ )
3 2
1 $
c. Does not exist d. None of these c. ( 3 i + 5$j - 6k$ ) d. 3 $i - 5$j + 6k$
2
128. Let f be a positive function. If 136. Let a = $i - 2$j + 3k$ , b = 3 i$ + 3 $j - k$ and
k k
I1 = ò x f {x (1 - x )} dx , I 2 = ò f {x (1 - x )} dx , where $ $ $
c = d i + j + ( 2d - 1) k. If c is parallel to the plane of the
1- k 1- k
I1
2k - 1 = 0, then is vectors a and b, then 11d is equal to
I2
a. 2 b. 1 c. -1 d. 0
a. 2 b. k c. 1/ 2 d. 1
137. If the vectors a = ( 2, log3 x , a ) and
129. The equation of the line which is parallel to the line b = ( -3, a log3 x , log3 x ) are inclined at an acute angle,
common to the pair of lines given by 6x 2 - xy - 12y 2 = 0 then
and 15x 2 + 14xy - 8 y 2 = 0 and at a distance of 7 units a. a = 0 b. a < 0 c. a > 0 d. None
from it, is 138. The coordinates of the foot of the perpendicular drawn
a. 3x - 4y = 35 b. 5x - 2y = 7 from the point A (1, 0, 3 ) to the join of the points
c. 3x + 4y = 35 d. 2x - 3y = 7 B ( 4, 7, 1) and C ( 3, 5, 3 ) are
a. (5 / 3, 7 / 3, 17 / 3) b. (5, 7, 17)
x2 y2
130. If ax + by = 1 is tangent to the hyperbola - = 1, c. (5/7, -7 / 3, 17 / 3) d. ( -5 / 3, 7 / 3, - 17 / 3)
a2 b2
2 2 139. The solution set of the inequality
then a - b is equal to
a. 1 / a e 2 2
b. a e 2 2 log3 (x + 2) (x + 4 ) + log1/ 3 (x + 2) < (1/ 2) log 3
7 is
c. b 2e 2 d. None a. ( -2, - 1) b. ( -2, 3) c. ( -1, 3) d. ( 3, ¥ )
131. The normal at a variable point P on the ellipse 140. Let A = sin8 q + cos14 q, then for every real q,
2 2
x y a. A ³ 1 b. 0 < A £ 1
2
+ = 1 of eccentricity e meets the axes of the
a b2 1 3
ellipse at Q and R. Then, the locus of the mid-point of c. < A £ d. None of these
2 2
QR is conic with eccentricity e¢ such that p
a. e ¢ is independent of e b. e ¢ = 1
141. Ify = (1 + tan A ) (1 - tan B ), where A -B = , then
4
c. e ¢ = e d. e ¢ =
1 ( y + 1)y + 1, is equal to
e a. 9 b. 4 c. 27 d. None
132. Let y = f (x ) be a parabola having its axis parallel to the 1
Y-axis, which is touched by the line y = x at x = 1, then 142. If sin q, cos q, tan q are in GP, then q is equal to
6
a. 2f ( 0) = 1 - f ¢( 0) b. f (0 ) + f ¢(0 ) + f ¢¢(0 ) = 1 (n Î Z )
c. f ¢(1) = 1 d. f ¢(0 ) = f ¢(1) p p
a. 2n p ± b. 2np ±
133. In DABC, right-angled at A, on the leg AC as diameter, 3 6
a semi-circle is described. If a chord joins A with the p p
c. np + ( -1)n d. np +
point of intersection D of the hypotenuse and 3 3
semi-circle, then the length of AC is equal to 143. sin-1(sin 5 ) > x 2 - 4x holds, if
AB × AD AB × AD a. x = 2 - 9 - 2p
a. b.
AB + AD 2 2 AB 2 + AD 2 b. x = 2 + 9 - 2 p
AB × AD c. x > 2 + 9 - 2p
c. AB × AD d.
AB 2 - AD 2 d. x Î( 2 - 9 - 2p, 2 + 9 - 2p )
PRACTICE SET 1 1205
144. In DABC, if a 2 + b 2 + c 2 = ac + ab 3 , then the 147. If X is a binomial variate with parameters n and p,
triangle is p (X = r )
where 0 < p < 1 such that is independent
a. equilateral b. isosceles p (X = n - r )
c. right angled d. None of these of n and r , then p equals
145. Let f (x ) = x 2 and g (x ) = 2x . Then, the solution set of a. 1/2 b. 1/3
the equation fog (x ) = gof (x ) is c. 1/4 d. None of these
a. R b. { 0}
148. For n Î N , x n + 1 + (x + 1)2n - 1 is divisible by
c. { 0, 2} d. None
a. x b. x + 1
146. The negation of the proposition “If a number is c. x 2 + x + 1 d. x 2 - x + 1
divisible by 15, then it is divisible by 5 or 3”, is
a. If a number is divisible by 15, then it is not divisible 149. If mode of a data exceeds its mean by 12, then mode
by 5 and 3 exceeds the median by
b. A number is divisible by 15 and it is not divisible by 5 a. 4 b. 8 c. 6 d. 10
and 3 150. In a series of 2n observations, half of them equal a
c. A number is divisible by 15 and it is not divisible by 5 and remaining half equal -a. If the standard deviation
or 3 of the observations is 2, then | a | equals
d. A number is not divisible by 15 or it is not divisible by 2 1
5 and 3 a. b. 2 c. 2 d.
n n

Answers
Physics
1. (a) 2. (a) 3. (b) 4. (a) 5. (a) 6. (a) 7. (c) 8. (a) 9. (c) 10. (c)
11. (c) 12. (d) 13. (b) 14. (c) 15. (a) 16. (c) 17. (a) 18. (b) 19. (b) 20. (d)
21. (b) 22. (d) 23. (d) 24. (c) 25. (c) 26. (d) 27. (a) 28. (b) 29. (c) 30. (b)
31. (d) 32. (c) 33. (c) 34. (b) 35. (b) 36. (c) 37. (c) 38. (b) 39. (a) 40. (c)

Chemistry
41. (d) 42. (b) 43. (b) 44. (d) 45. (b) 46. (b) 47. (b) 48. (a) 49. (b) 50. (a)
51. (b) 52. (a) 53. (d) 54. (b) 55. (d) 56. (c) 57. (a) 58. (b) 59. (c) 60. (b)
61. (b) 62. (b) 63. (a) 64. (a) 65. (b) 66. (b) 67. (c) 68. (a) 69. (a) 70. (a)
71. (c) 72. (d) 73. (c) 74. (d) 75. (c) 76. (d) 77. (b) 78. (d) 79. (c) 80. (d)

English Proficiency
81. (c) 82. (d) 83. (c) 84. (d) 85. (a) 86. (a) 87. (a) 88. (c) 89. (b) 90. (a)
91. (a) 92. (d) 93. (d) 94. (b) 95. (a)

Logical Reasoning
96. (a) 97. (d) 98. (d) 99. (c) 100. (c) 101. (c) 102. (a) 103. (c) 104. (a) 105. (a)

Mathematics
106. (d) 107. (c) 108. (c) 109. (d) 110. (c) 111. (b) 112. (b) 113. (b) 114. (c) 115. (c)
116. (c) 117. (a) 118. (d) 119. (b) 120. (c) 121. (a) 122. (c) 123. (a) 124. (a) 125. (b)
126. (d) 127. (c) 128. (c) 129. (c) 130. (a) 131. (c) 132. (a) 133. (d) 134. (c) 135. (b)
136. (c) 137. (d) 138. (a) 139. (b) 140. (b) 141. (c) 142. (a) 143. (d) 144. (c) 145. (c)
146. (b) 147. (a) 148. (c) 149. (b) 150. (c)
Practice Set 2
BITSAT
Instructions
1. There are 150 questions in all. The number of questions in each part is as follows Subject No. of Questions
Part I Physics 1-40
Part II Chemistry 41-80
Part III (a) English Proficiency 81-95
(b) Logical Reasoning 96-105
Part IV Mathematics 106-150
2. All questions are multiple choice questions with four options, only one is correct.
3. Each correct answer fetches 3 marks while incorrect answer has a penalty of 1 mark.

PART I
Physics
1. Frequency f of oscillations of a mass m suspended 4. A body of mass m is accelerated uniformly from rest to
from a spring of force constant k is given by f = cm x k y a speed v in a time T. The instantaneous power
where, c is a dimensionless constant. The values of x delivered to the body as function of time is given by
and y are 1 mv 2 2 1 mv 2
a. t b. t
1 1 1 1 2 T2 2 T2
a. x = - ,y = b. x = ,y = -
2 2 2 2 mv 2 mv 2
1 1 1 1 c. 2 t 2 d. 2 t
c. x = - , y = - d. x = , y = T T
2 2 2 2
5. A ball of mass (m) 0.5 kg is attached to the end of a
2. Given X = (Gh / c 3 )1/ 2, where G, h and c are string having length (L ) 0.5 m. The ball is rotated on a
gravitational constant, Planck’s constant and the horizontal circular path about vertical axis. The
velocity of light, respectively. Dimensions of X are the maximum tension, the string can bear is 324 N. The
same as those of maximum possible value of angular velocity of ball
a. mass b. time (in rad/s) is
c. length d. acceleration a. 9 b. 18 c. 27 d. 36
3. A projectile can have same range R for two angles of 6. A sphere of mass 1 kg is connected to a spring of
projection. If t1 and t 2 be the time of flights in the two spring constant 5.0 Nm - 1 as shown in figure. A force
cases, then the product of time of flight is proportional of 0.5 N is applied on the sphere along X-axis. What is
to the velocity of the sphere when it is displaced through
1 1 a distance of 10 cm along X-axis ?
a. b.
R R2 a. 0.11 ms- 1 b. 0.22 ms- 1
c. R d. R 2 c. 0.11 cms- 1 d. 0.22 cms- 1
PRACTICE SET 2 1207
7. Two identical balls A and B, collide head on elastically. 14. A vessel contains 1 mole of O2 gas (molar mass = 32)
If velocities of A and B, before collision are + 0.5 ms - 1 at a temperature T. The pressure of the gas is p. An
and - 0.3 ms - 1 respectively, then their velocities after identical vessel containing one mole of helium as
the collision, are respectively (molar mass = 4) at a temperature 2T has a pressure of
p
a. - 0.5 ms- 1 and + 0.3 ms- 1 a. b. p c. 2p d. 8p
8
b. + 0.5 ms- 1 and - 0.3 ms- 1
c. + 0.3 ms- 1 and - 0.5 ms- 1 15. A refrigerator works between 2° C and 27° C. To keep
the temperature of the refrigerated space constant,
d. - 0.3 ms- 1 and + 0.5 ms- 1
660 calories of heat are to be removed every second.
8. A rod PQ of mass M and length L is hinged at end P . The power required is
The rod is kept horizontal by a massless string tied to a. 60 W b. 55 W
point Q as shown in figure. When string is cut, the c. 252 W d. 231 W
initial angular acceleration of the rod is 16. In an isothermal process, the pressure of a gas is
decreased by 20%. The percentage change in the
volume
a. increase by 20% b. decrease by 20%
c. increase by 25% d. decrease by 25%

P M Q 17. An object suspended from a spring exhibits


L oscillations of period T. Now, the spring is cut in two
halves and the same object is suspended with two
3g g 2g 2g halves as shown in figure. The new period of
a. b. c. d.
2L L L 3L oscillation will become
9. A satellite is moving with a constant speed v in a
circular orbit about the earth. An object of mass m is
k¢¢
ejected from the satellite such that it just escapes from
the gravitational pull of the earth. At the time of its k¢
ejection, the kinetic energy of the object is m
1 3
a. mv 2 b. mv 2
c. mv 2 d. 2 mv 2
2 2 m
10. A solid sphere of volume (V ) and density (r ) floats at T T T
a. b. c. d. T
the interface of two immiscible liquids of densities r1 2 2 2 2
and r 2, respectively. If r1 < r < r 2, then the ratio of the
volume of the parts of the sphere in upper and lower 18. Four particles each having charge q are placed on the
liquids is four vertices of a regular pentagon as shown in figure.
r + r1 r - r1 r -r r + r2 The distance of each corner from the centre is a. The
a. b. c. 2 d. electric field at the centre of the pentagon is
r + r2 r2 - r r - r1 r + r1

11. Two rods of same material having same length and


?
area. If heat DQ flows through them for 12 min when
they are jointed side by side and now both rods are
(i) (ii) (iii) (iv)
joined in, then the same amount of heat DQ will flow in
a. 24 min b. 18 min c. 6 min d. 3 min 1 q 1 q
a. along OE b. along EO
2
12. A hole of area 1mm opens in the pipe near the lower 4pe0 a2 4pe0 a 2
1 q
end of a large water storage tank and a stream of c. d. None of these
water shoots from it. If the top of the water in the tank 4pe0 a2
is 20 m above the point of the leak, then the amount of 19. Two equal charges q are kept fixed at a and + a along
water escapes in 1s is
æq ö
a. 87.5 cm3 b. 43.1 cm3 c. 27.5 cm3 d. 19.8 cm3 X-axis. A particles of mass m and charge ç ÷ is
è 2ø
13. The pressure of a medium is changed from brought to the origin and given a small displacement
1.01´ 105 Pa to 1.165 Pa and change in volume is along the X-axis, then
10% keeping temperature constant. The bulk modulus a. the particle executes oscillatory motion
of the medium is b. the particle remains stationary
a. 204.8 ´ 105 Pa b. 102.4 ´ 105 Pa c. the particle executes, SHM along X-axis
d. the particle executes SHM along Y-axis
c. 51.2 ´ 105 Pa d. 1.55 ´ 105 Pa
1208 SELF STUDY GUIDE BITSAT

20. Consider a non-spherical conductor shown in the 26. A conducting loop carrying a current I is placed in
figure which is given a certain amount of positive xy -plane. A uniform magnetic field pointing into
charge. The charge distributes itself on the surface Z -direction is switched on. The loop will have a
such that the charge densities are s1, s2 and s3 at tendency to
regions 1, 2 and 3 respectively. Y
×××××××××
3 B××××××
×××
×××××××××
×××××××××
1 2 ×××××××××
××××××××× X
3 ×××××××××
×××××××××
Then, ×××××××××
×××××××××
a. s1 > s2 > s3 b. s2 > s3 > s1
c. s3 > s1 > s2 d. s2 > s1 > s3 a. contract
21. Two batteries A and B each of emf 2v are connected b. expand
in series to an external resistance R = 1W. If the c. move towards + ve X-axis
internal resistance of battery A is 1.9 W and that of B is d. move towards -ve X-axis
0.9 W. What is the potential difference between the 27. The magnetic susceptibility of a paramagnetic
terminals of battery A ? material at - 73 ° C is 0.0075, its value at - 173 ° C will
A B be
a. 0.0045 b. 0.0030
c. 0.015 d. 0.0075
28. Two similar poles of 60 Am and 240 Am are 12 cm
R apart. The magnetic field is zero at a position on the
a. 2 V b. 3.8 V line joining them at
c. Zero d. None of these a. 4 cm from 60 Am b. 6 cm from 240 Am
c. 3 cm from 60 Am d. 8 cm from 240 Am
22. 32 cells each of emf 3 V are connected in series and
kept in a box. Externally, the combination shows 84 V. 29. A conducting circular loop is placed in a uniform
The number of cells reversed in the connection is magnetic field 0.04 T with its plane perpendicular to
the magnetic field. The radius of the loop starts
a. 0 b. 2 c. 8 d. 16
shrinking at 2 mm/ s. The induced emf in the loop
23. In the circuit, the voltmeter resistance is 10000 W and when the radius is 2 cm is
the ammeter resistance is 2W. The voltmeter reads a.4.8 p mV b. 0.8 p mV
12v and ammeter reads 0.1 A. The value of R is c. 1.6 p mV d. 3.2 p mV
P R Q
A 30. An AC voltage, e = 200 sin 100 t is applied to a series
combination of R = 30 W and an inductor of 400 mH.
V The power factor of the circuit is
a. 118 W b. 122 W c. 10022 W d. 10018 W a. 0.01 b. 0.2 c. 0.5 d. 0.6

24. In the circuit shown in figure, the heat produced in 5W 31. The network shown in figure is a part of a circuit. What
resistor due to a current flowing in it is 10 cal s . The -1 is the potential difference VB - VA , when current I is 5A
heat produced in 4 W resistor is and is decreasing at a rate of 103 As - 1?
4W 6W 5 mH
A B
1W
15V
5W
a. 10 V b. 5 V
c. 15 V d. 20 V
a. 1 cal/s b. 2 cal/s c. 3 cal/s d. 4 cal/s
25. Force per unit length between parallel wires in the 32. In a two slit experiment, with monochromatic light,
-3 fringes are obtained on a screen placed at some
circuit is 3.6 ´ 10 N/ m. Resistance of the circuit is
distance from the slits. If the screen is moved by
1.2 m 5 ´ 10- 2m towards the slits, the change in fringe width
18 V is 10–2m. Then, the wavelength of light used when
8 mm
r=0 distance between slits is 0.3 mm will be
a. 4000 Å b. 6000 Å
a. 3 W b. 1.5 W c. 4.5 W d. 6 W c. 5000 Å d. 7000 Å
PRACTICE SET 2 1209
33. The intensity of the central fringe due to the 36. If the wavelength of the first line of the Balmer series
interference of light from two similar, slits is I. When of hydrogen is 6561Å, the wavelength of the second
one of the slit is closed, the intensity is I 0. What is the line of the series should be
relation between I and I 0 ? a. 1312 Å b. 3280 Å c. 4860 Å d. 2187 Å
a. I = 4I 0 b. I = 2I 0
c. I = I 0 d. I = I 0 / 2 37. Half-life of radioactive substance is 140 days. Initially,
the mass of radiactive substance is 16 g. Calculate the
34. Which one of the following showing correct time for this substance when it reduces to 1g.
relationship between power of a lens and wavelength a. 140 days b. 280 days
of incident light? c. 420 days d. 560 days
38. When a hydrogen atom emits a photon in going from
a. P b. P n = 5 to n = 1, its recoil speed is almost
a. 10- 4 ms- 1 b. 8 ´ 102 ms- 1
l l
c. 2 ´ 10- 2 ms- 1 d. 4 ms- 1

39. In the ratio of the concentration of electrons and of


c. P d. P holes in a semiconductor is 7/5 and the ratio of current
is 7/4, then what is the ratio of their drift velocities ?
l l 4 5 4 5
a. b. c. d.
35. The surface of a metal is illuminated with the light of 7 8 5 4
400 nm. The kinetic energy of the ejected photo 40. An 80 MHz carrier is modulated by 400 Hz sine wave.
electrons was found to be 1.68 eV. The carrier voltage is 5V and the frequency deviation
The work function of the metal is is 20 kHz. Find modulation index.
a. 1.41 eV b. 1.51 eV c. 1.68 eV d. 3.09 eV a. 25 b. 50 c. 400 d. 5

PART II
Chemistry
41. On heating 1.763 g hydrated BaCl2, 1.505 g 44. Determine the enthalpy change that occurs in the
anhydrous salt is obtained. The number of water following isomerisation reaction,
molecules in BaCl2 is Propene ( g ) ¾® Cyclopropane ( g )
a. 3 b. 2 c. 5 d. 1
If BE ( C ¾ C) = 348 kJ/mol and BE ( C == C)
42. The electron represented by the following sets of
quantum numbers: = 6 15 kJ/mol
I. n = 4, l = 0, me = 0, ms = + 1/ 2 a. 81kJ/mol b. - 81kJ/mol
II. n = 3, l = 1, me = 1, ms = - 1/ 2 c. 75 kJ/mol d. - 75 kJ/mol
III. n = 3, l = 2, me = 0, ms = + 1/ 2 45. In the sequence of reaction,
IV. n = 3, l = 0, me = 0, ms = - 1/ 2 (i) CH 3 MgI
Mild
The correct increasing order of energy is ( CH3 )2 CHOH ¾¾® X ¾¾® Y , Y is
Oxidation [O] (ii) Hydrolysis
a. IV > I > II > III b. III > I > II > IV
c. II > I > IV > III d. I > IV > III > II a. tertiary butyl alcohol b. n-butyl alcohol
c. isobutylene d. isobutyl alcohol
43. The phenol is not obtained in which of the following?
MgBr
46. The (—COOH) group is not present in which of the
following?
Cl
NaOH H3 O+ a. Benzoic acid b. Picric acid
a. b. c. Aspirin d. All have (—COOH) group
633K, 300 atm
47. Ethanamine is treated with nitrous acid at ordinary
N2Cl temperature, then the product will be
OH a. only ethanol
H 2O NaOH/CaO
c. d. b. acetic acid, ethane and H2O
D
c. ethanol, ethene, N2 and H2O
COOH d. ethanol, acetic acid, N2 and H2O
1210 SELF STUDY GUIDE BITSAT

48. Choose the correct statement from the following. 55. The emf of the following cell is 0.81 V.
a. H2S and SO2 both act as oxidising and reducing M | M n + | | H+ | H2( Pt); E °M / M n + = 0.76 V
agent 0.002 M 1M 1 atm
b. H2S acts only as reducing agent while SO2 acts both Then, the value of n for metal ion is
as oxidising and reducing agent a. 1 b. 4 c. 2 d. 3
c. H2S acts only as oxidising agent while SO2 acts as
reducing agent
56. Consider the reaction given below,
d. SO2 acts only as oxidising agent while H2S acts both D H2O
A + N2 ¾® B ¾® C + NH3
as oxidising and reducing agent Alkali metal ¯ HCl ½ HCl
¯
49. The weight of CaO required to remove hardness of D
NH4 Cl
106 L of water containing 1.62 g of Ca(HCO3 )2 in 1L, is White fumes
The alkali metal A is
(MW of Ca(HCO3 )2 = 162 and MW of CaO = 56)
a. K b. Li c. Na d. Cs
a. 5.6 ´ 102mg b. 5.6 ´ 105g
57. In the following reaction,
c. 4.2 ´ 104mg d. 4.2 ´ 104g
xZn+ y HNO3 (dil. ) ® aZn(NO3 )2 + bH2O + c NH4 NO3
50. For concentration of sulphide ores, froth floatation the sum of the coefficients (a + b + c ) is
process is used which is based on a. 6 b. 8 c. 4 d. 5
a. specific gravity of ore particles
b. electrical properties of ore particles 58. Arrange the following three chlorides in decreasing
c. wetting properties of ore particles order of reactivity towards S N1 reaction.
d. magnetic properties of ore particles
I. II. III.
51. Which of the following will show optical activity? Cl Cl
a. trans -[ Co(NH3 )4 Cl2] + Cl
b. trans -[ Co(NH3 )2( en)2] 3+ a. II > III > I b. III > II > I c. I > II > III d. II > I > III
c. [ Cr(H2O)6] 3+ 59. The % CaSO4 in the sample containing 0.455 g
d. cis - [ Co(NH3 )2( en)2] 3+ æ 1 ö
sample of plaster of Paris ç CaSO4 × H2O ÷, when
è 2 ø
52. O
treated with excess of Ba(NO3 )2 gave 0.617 g of
anhy, BaSO4 is
A
a. 87% b. 92% c. 80% d. 75%
Br
60. Choose the incorrect statement from the following.
a. Chlorine is less selective and more reactive
The reagent A is
b. Vinyl free radical is more stable than allyl free radical
a. NaBH4 + PtCl4 c. Bromine is more selective and less reactive
b. (i) N2H4 / H+ (ii) C2H 5O–Na + d. Benzyl free radical is more stable than allyl free
c. Zn – Hg / HCl radical
d. LAH + AlCl3 O
½½
53. The geometry of [Ni(CN)4 ] 2 - , [NiCl4 ] 2 - and [Ni(CO)4 ] 61. CH3 ¾ C ¾ O ¾ CH3
a b
are
2-
a. [Ni(Cl)4] is square planar and The relation between bond lengths a and b is
[Ni(CN)4] 2 - , [ Ni(CO)4] are tetrahedral a. a = b b. b < a
b. [Ni(CN)4] 2 - is square planar, c. b > a d. impossible to predict
[NiCl4] 2 - and[ Ni(CO)4] are tetrahedral 62. The maximum number of emission lines when the
c. [ Ni(CO)4] is square planar, [Ni(CN)4] 2 - and [NiCl)4] 2 - excited electron of a H-atom in n = 6 drops to the
are tetrahedral ground state is
d. None of the above a. 14 b. 36 c. 15 d. 10
54. The volume of 3.7 g of a gas at 25°C and volume of 63. Choose the correct option from the following in case of
0.184 g of H2 gas at 17°C are same at same pressure. van der Waals’ adsorption.
The molecular mass of gas is a. High temperature, low pressure
a. 52 b. 41 b. High temperature, high pressure
c. 45 d. 55 c. Low temperature, low pressure
d. Low temperature, high pressure
PRACTICE SET 2 1211
64. If the solubility of calcium carbonate in water is 73. Which one of the following order is not in accordance
0.0305 g/L, then its solubility product is with the property stated against it?
a. 8.40 ´ 10- 6 b. 9.30 ´ 10- 8 a. F2 > Cl2 > Br2 > I2 : Oxidising power
c. 7.21 ´ 10- 8 d. 9.21 ´ 10- 7 b. F2 > Cl2 > Br2 > I2 : Electronegativity
O c. F2 > Cl2 > Br2 > I2 : Bond dissociation energy
(i) CH3 MgBr
½½ (excess) d. HI > HBr > HCl > HF : Acidic property in water
65. C2H5 ¾ O ¾ C ¾ O ¾ C2H5 ¾¾¾® P 74. In corundum, oxide ions are arranged in hcp array and
(ii) H3 O+
the aluminium ions occupy two-thirds of octahedral
The product P is voids. The formula of corundum is
O a. Al2O3 b. Al3O2
½½ c. Al2O6 d. Al2O4
a. CH3 ¾ C ¾ CH3 b. CH3 ¾ CH2 ¾ CH3
O OH 75. The freezing point depression of 0.1 molar solution of
½½ ½ acetic acid in benzene is 0.256 K. K f for benzene is
c. CH3 ¾ C ¾ O ¾ C2H 5 d. CH3 ¾ C ¾ CH3 5.12 K kg mol–1. What conclusion can you find about
½ the molecular state of acetic acid in benzene?
CH3
a. Benzene is doubly associated
66. On applying pressure to the equilibrium, b. Acetic acid is doubly associated
Ice s Water c. Both are equally associated
d. None of the above
Which phenomenon will happen?
a. More ice will be formed 76. Which of the following is not an addition
b. More water will be formed homopolymer?
c. Water will evaporate a. PVC b. Teflon
d. Equilibrium will not be disturbed c. SBR d. Natural rubber
67. From the reaction, 77. Choose the correct structure of the drug paracetamol.
P(white) ¾® P(red) ; DH = - 18.4 kJ Cl OH
which of the following statements is correct?
a. Red P is readily formed from white P a. b.
b. White P is readily formed from red P
c. White P cannot be converted to red P CONH2 CONH2
d. White P can be converted into red P and red P
is more stable Cl OH
68. From which of the following mixtures, buffer solution
can be prepared? c. d.
a. NaH2PO4 + Na 2HPO4 b. NH4OH + NH3
c. HCl + NaCl d. CH3COOH + NaCl COCH3 NHCOCH3
69. Which of the following has regular tetrahedral structure? 78. The same osazone is formed by which of the following
a. SF4 b. XeF4 pairs?
c. [ Ni(CN)4] 2 - d. BF4- a. Glucose and fructose b. Glucose and galactose
70. If the reaction rate at a given temperature becomes c. Glucose and arabinose d. Lactose and maltose
slower, then 79. The rate of change of rms speed of a gas is twice the
a. the entropy changes rate of change of absolute temperature. Then, the rms
b. the initial concentration of the reactants remains speed of Ne is (Atomic mass of Ne = 20)
constant a. 344 ms- 1 b. 302 ms- 1 c. 381ms- 1 d. 312 ms- 1
c. the free energy of activation is lower
d. the free energy of activation is higher 80. The absolute configurations of the centres (1 and 2) in
the molecule given below are
71. The number of sigma and pi-bonds in
peroxydisulphuric acid respectively are 1 COOH
a. 10 and 4 b. 11 and 4 c. 13 and 2 d. 11 and 2 2
H Br
72. Calculate the resonance energy of benzene, if the
enthalpy of hydrogenation of benzene is DH1 and for 3
H CN
cyclohexene, it is DH 2.
a. DH1 - 3DH2 b. 3DH2 - DH1 OH
c. 3DH1 - DH2 d. 3DH1 + DH2 a. 2 R, 3S b. 2 S , 3S c. 2 S , 3R d. 2 R, 3R
1212 SELF STUDY GUIDE BITSAT

PART III
a. English Proficiency
Directions (Q. Nos. 81 to 84) Read the passage and 86. A series of shocks
choose the most suitable option from the given options. P : is known as earthquake
When vegetation sprouts in the desert, it is a good sign but Q : which can be recognised through seismic waves.
when the ice in the Arctic and Antarctic begins to turn green, R : that result from sudden Earth movements or
there is something terribly wrong. Reports say that an tremors
iceberg, approximately the size of New York city, has broken S : causing widespread destruction of life and property
off from the icy continent. An Argentine term discovered The proper sequence should be
huge cracks in the polar ice caps. These developments can a. PQRS b. RPQS c. RSPQ d. RQSP
have serious implications. If polar ice fields melt, our coastal 87. P : in this world
cities might be submerged and sea levels across the world Q : a man has
could rise between 3.65 m and 6.09 m in different parts of R : it is possible that the best friend
the Earth. This is probably due to global warming. S : may turn against him
81. What can be considered as a ‘Good sign’ in the desert ? The proper sequence should be
a. When shrubs and trees grow there a. QPRS b. PQRS c. RQPS d. QRSP
b. When it shows in the desert 88. P : in estimating the size of the earth but they were
c. When ice caps melt and there is water Q : hampered by the lack of instruments of precision
d. When there is vegetation found in the desert R : ancient astronomers
82. The melting of ice fields in the Arctic and Antarctic S : used method which were theoretically valid
regions is dangerous because The proper sequence should be
a. the ice will turn green and poisonous a. RPQS b. PRQS
b. it will cause huge floods which will destroy coastal c. RSQP d. RPSQ
regions
c. it will create global warming and will badly affect our Directions (Q. Nos. 89 to 91) Spot the error in the given
climate sentence and indicate your answer.
d. huge cracks will develop all over the world 89. Children are prone (a)/ to making mischiefs (b)/ if they
83. Polar ice caps develop huge cracks because have nothing to do. (c)/ No error (d)
a. of the movement of the Earth 90. Sheeps are economically useful(a)/ and so they are
b. of the breaking off of Earth reared (b)/ in the hills. (c)/ No error (d)
c. of the crowding of cities like New York
d. of rising temperatures 91. I have not gone through (a)/ the letter and so I am not
aware (b)/ of its content. (c)/ No error (d)
84. What do you think about the intention of the author?
a. To describe strange phenomena in nature Directions (Q. Nos. 92 and 93) Find the synonym of the
b. To report findings of research teams working in the word written in capital letters.
polar regions 92. The convict’s INGENUOUS explanation brought tears
c. To make us aware of the dangers of global warming in every eye.
d. To compare development in deserts and Arctic a. Candid b. Secret c. Insincere d. Consistent
regions caused by global warming
93. The ENORMITY of the population problem is irksome.
Directions (Q. Nos. 85 to 88) Rearrange the jumbled up a. Intensity b. Vastness c. Cruelty d. Fragility
parts of the sentence to make it meaningful and then choose
the most suitable option. Directions (Q. Nos. 94 and 95) Find the antonym of the
85. It seemed to him word written in capital letters.
P : like seeing one’s reflection 94. We have CREATED a beautiful new house from out of
Q : an endless quest the old ruin.
R : two mirrors a. Destroyed b. Built
S : while standing between c. Constructed d. Planned
The proper sequence should be 95. A large number designer clothes are REVOLTING.
a. PRSQ b. SPQR a. Inviting b. Beautiful
c. RSPQ d. QPSR c. Fashionable d. Delightful
PRACTICE SET 2 1213

b. Logical Reasoning
96. Choose the odd one out. 102. Which figure from the given alternatives will replace
a. BDFH b. MOQS the question mark (?)
c. SUWY d. TUZE Problem Figures
97. Find the missing number.
?
56 22 46 10 34 14
41 39 ?
(i) (ii) (iii) (iv)
15 8 9 6 11 6
Answer Figures
a. 12 b. 25 c. 48 d. 52
98. Find the figure that would replace the question mark (?)
Problem Figures
a. b. c. d.
103. How many different triangles are there in the figures
? shown below ?

(i) (ii) (iii) (iv)

Answer Figures
a. 28 b. 24 c. 20 d. 16
104. Find the wrong number in the series.
1, 9, 36, 81, 99, 121
a. b. c. d. a.1 b. 121
c. 99 d. 36
99. If KASHMIR written as 8142753, how RIMSHAK can 105. In the following question, select a figure from amongst
be written in that code? the four alternatives, which when placed in the blank
a. 3574218 b. 3571842 space of figure (X ) would complete the pattern.
c. 3521478 d. 3574812
Problem Figure
100. If ‘PAPER’ is written as ‘OZODQ’ how ‘PENCIL’ can
be written in that code?
a. QFODJM b. OFOBHM c. ODMDJM d. ODMBHK
101. Which one of the following diagram best depicts the ?
relationship among elephants, wolves, and animals?

Answer Figures
a. b.

c. d.
a. b. c. d.
PART IV
Mathematics
106. The integrating factor of the differential equation 113. A bag contains n balls out of which some balls are
dy 1+ y white. If probability that a bag contains exactly i white
+y = is
dx x ball is proportional to i 2. A ball is drawn at random
x ex from the bag and found to be white, then find the
a. b. probability that bag contains exactly 2 white balls.
ex x
c. xe x d. e x é 6 ù æ 23 ö 2
ê ú çç ÷÷
2
107. Let a and b be the roots of x - 6x - 2 = 0 with a > b. If ë n(n + 1) û è n ø 3(n + 1)
a. b.
a10 - 2a 8 2 é 6 ù æ 23 ö
an = an - bn for n ³ 1, then the value of is 3(n + 1) ê ú çç ÷÷
2a 9 ë n(n + 1) û è n ø
a. 1 b. 2 æ 6ö é 23 ù
ç ÷ ê ú
c. 3 d. 4
c.
èn ø ë n(n + 1) û d. None of these
108. Let a1, a 2, a 3 , ¼, a100 be an arithmetic progression with 2
p 3n
a1 = 3 and S p = å ai , 1 £ p £ 100. For any integer n
i =1
114. The domain of definition of f (x ) = cos -1(x + [x ]) is
Sm a. [ 0, 1) b. R - Z
with 1 £ n £ 20, let m =5n. If does not depend on n,
Sn c. ( 0, ¥ ) d. None
then a 2 equals 115. Let f :R ® R be given by
a. 6 b. 5 f (x + y ) = f (x ) - f ( y ) + 2xy + 1 for all x , y Î R . If f (x ) is
c. 7 d. 4 everywhere differentiable and f ¢( 0) = 1, then f ¢(x ) is
109. If r , s , t are prime numbers and p , q are the positive equal to
integers such that the LCM of p , q is r 2 t 4s 2, then the a. 2x + 1 b. 2x - 1
number of ordered pair ( p , q ) is c. x + 1 d. x - 1
a. 252 b. 254 d 2y
c. 225 d. 224 116. is equal to
dx 2
110. The expression [x + (x 3 - 1)1/ 2] 5 = [x - (x 3 - 1)1/ 2] 5 is a æ d 2y ö
-1
æ d 2y ö -3
æ dy ö
polynomial of degree a. çç 2 ÷÷ b. - çç 2 ÷÷ ç ÷
è dx ø è dx ø è dx ø
a. 5 b. 6
c. 7 d. 8 æ d 2y ö æ dy ö -2 æ d 2y ö æ dy ö
-3
c. çç 2 ÷÷ ç ÷ d. - çç 2 ÷÷ ç ÷
111. Let a , b and c be three real numbers satisfying è dx ø è dx ø è dx ø è dx ø

é 1 9 7ù 117. The equation of a curve is y = f (x ). The tangents at


[a b c ] ê8 2 7ú = [ 0 0 0]. p p
(1, f (1)), ( 2, f ( 2)) and ( 3, f ( 3 )) makes angles , and
p
ê ú
êë7 3 7úû 6 3 4
respectively with the positive direction of the X-axis.
If the point (a , b , c ) with reference to (E ) lies on the 3 3
plane 2x + y + z = 1, then the value of 7a + b + c is Then, the value of ò f ¢(x )f ¢¢ (x ) dx + ò f ¢¢ (x ) dx is equal
2 1
a. 0 b. 12 to
c. 7 d. 6 1 1
a. - b.
xp + y x y 3 3
112. The determinant yp + z y z = 0, if c. 0 d. 1
0 xp + y yp + z x2 1
118. If f (x ) = ò dt ; x ¹ 0, x ¹ 1, then f (x ) is
a. x , y , z are in AP
x (log t )2
b. x , y , z are in GP monotonically
c. x , y , z are in HP a. increasing on ( 2, ¥ ) b. increasing on (1, 2)
d. xy , yz , zx are in AP c. decreasing on ( 2, ¥ ) d. decreasing on (0, 3)
PRACTICE SET 2 1215
119. If ò log ( 1 - x + 1 + x ) dx = x f (x + B sin-1 x + C , then 128. If S = {x Î N : 2 + log2 x + 1 > 1 - log1/ 2 4 - x 2 }, then
a. f (x ) = log ( 1 - x + 1 + x ) a. S = {1} b. S = Z
b. A = 1/ 3 c. S = N d. None
c. B = 2 / 3
129. The area of the parallelogram contained by the lines
d. B = - 1/ 2 4 y - 3x - a = 0, 3 y - 4x + a = 0, 4 y - 3x - 3a = 0
1 1 and 3 y - 4x + 2a = 0 is
120. If I = ò dx , then
0 1 + xp / 2 2 2 3 2
a. a sq units b. a sq units
p 7 8
a. ln 2 < I < b. I < ln2 5
4 c. 2a 2 sq units d. a 2 sq units
p 4
c. I > d. None of these
4 130. The two circles which pass through ( 0, a ) and ( 0, - a )
121. The area between the curve y = 2x - x , the X-axis 4 2 touch the line y = mx + c , will intersect each other at
right angle, if
and the ordinates of two minima of the curve is
a. a 2 = c 2( 2m + 1) b. a 2 = c 2( 2 + m 2 )
a. 7/120 sq unit b. 9/120 sq nuit
c. 11/120 sq unit d. 13/120 sq unit c. c 2 = a 2( 2 + m 2 ) d. c 2 = a 2( 2m + 1)

122. Let a = $i + $j + k$ and r be a variable such that r × $i , r × $j 131. Two mutually perpendicular tangents of the parabola
y 2 = 4ax meet the axis at P1 and P2. If S is the focus of
and r × k$ are positive integers. If r × a £ 12, then the total
1 1
number of such vectors is the parabola, then + is equal to
12 12 12 12 SP1 SP2
a. C3 b. C4 c. C9 - 1 d. C8
4 2 1 1
a. b. c. d.
123. Given, | a | = | b | =1 and | a + b | = 3. If c is a vector a a a 4a
such that c - a - 2b = 3( a ´ b), then c × b is equal to
x2 y2
1 1 3 5 132. Tangents are drawn to the ellipse + = 1, (a > b )
a. - b. c. d. a 2
b2
2 2 2 2
and the circle x 2 + y 2 = a 2 at the points, where a
x y z
124. If the plane + + = 1 cuts the coordinate axes at common ordinate cuts them (on the same side of
2 3 4
A, B , C , then the area of DABC is X -axis). Then, the greatest acute angle between these
a. 29 sq units b. 41 sq units tangents is given by
61 sq units 71 sq units æa -b ö æa + b ö
c. d. a. tan-1ç ÷ b. tan-1ç ÷
è 2 ab ø è 2 ab ø
125. If d1, d 2, d 3 denote the distance of the plane
æ 2ab ö æ 2ab ö
2x - 3 y + 4z + 2 = 0 from the planes c. tan-1ç ÷ d. tan-1ç ÷
ç a -b ÷ ç a +b÷
2x - 3 y + 4z + 6 = 0, 4x - 6 y + 8z + 3 = 0 and è ø è ø
2x - 3 y + 4z - 6 = 0 respectively, then 133. The chords of contact of a point p with respect to a
a. d1 + 8d 2 + d 3 = 0 b. d1 + 16d 2 = 0 hyperbola and its auxiliary circle are at right angle.
c. 8d 2 = d1 d. d1 - 2d 2 + 3d 3 = 29 Then, the point P lies on
126. The median of the series 8, 12, 15, 7, x, 19, 22 lies in a. conjugate hyperbola
the interval b. one of the directrix
c. one of the asymptotes
a. [12, 15 ] b. [ 7, 15] d. None of these
c. [15, 17] d. [9, 12 ]
134. Through a point A on the X-axis, a straight line is
127. The variable X takes two values x1 and x 2 with drawn parallel to the Y-axis, so as to meet the pair of
frequencies f1 and f2 respectively. If s denotes the straight lines ax 2 + 2hxy + by 2 = 0 at B and C. If
standard deviation of X , then s2 is equal to AB = BC , then
f12x12 + f22x 22 æ f1 x1 + f2 x 2 ö
2
a. h 2 = 4ab b. 8h 2 = 9ab
a. - çç ÷÷
f1 + f2 c. 9h 2 = 8ab d. 4h 2 = ab
è f1 + f2 ø
f1f2 135. If a and b are positive quantities such that a > b , then
b. (x1 - x 2 )2
(f1 + f2 )2 the minimum value of a sec q - b tan q is
f1f2
c. (x1 - x 2 )2 a. 2ab b. a2 - b2
(f1 - f2 )2
d. None of the above c. a - b d. a2 + b2
1216 SELF STUDY GUIDE BITSAT

136. Let x = sin 1°, then the value of expression 2x - 1


143. If S is the set of all real x such that is
1 1 1 2x 3 + 3x 2 + x
+ +
cos 0°× cos 1° cos 1°× cos 2° cos 2°× cos 3 ° positive, then S contains
1 a. ( - ¥ , + 3 / 2) b. ( - 3 / 2, - 1/ 4)
+ ¼+ is equal to
cos 44 °× cos 45 ° c. ( - 1/ 4, 1/ 2) d. (1/ 2, 3)
a. x b. 1/x 144. For any three sets A, B and C, the set
c. 2 /x d. x / 2 ( A È B È C ) Ç ( A Ç B ¢ Ç C ¢ ) Ç C ¢ is equal to
137. If tan x = n tan y , n Î R + , then the maximum value of a. B Ç C ¢ b. B ¢ Ç C ¢
2
sec (x - y ) is equal to c. B Ç C d. A Ç B Ç C
(n + 1)2 (n + 1)2 145. Let w denotes words in the English dictionary. Define
a. b.
2n n the relation R by R = {(x , y ) Îw ´ w : the words x and y
(n + 1)2 (n + 1)2 have atleast one letter in common} then, R is
c. d.
2 4n a. not reflexive, symmetric and transitive
3 3 b. not transitive, reflexive and symmetric
sin q - cos q cos q not symmetric, reflexive and transitive
138. - - 2 tan q cot q = - 1, if c.
sin q - cos q 1 + cot 2 q d. reflexive, transitive and symmetric
æ pö æp ö 146. Let f : [ 2, ¥ ) ® x be defined by f (x ) = 4x - x 2, then f is
a. q Î ç 0, ÷ b. q Î ç , p ÷
è 2ø è2 ø invertible, if x is equal to
æ 3p ö æ 3p ö a. [ 2, ¥ ) b. ( - ¥ , 2]
c. q Î ç p, ÷ d. q Î ç , 2p÷
è 2ø è 2 ø c. ( - ¥ , 4] d. [ 4, ¥ )
139. If sin-1 x + sin-1 y + sin-1 z = p, then 147. Which of the following is always true?
4 4 4 2 2 2
x + y + z + 4x y z = k (x y + y z + z 2x 2 ),
2 2 2 2
a. ( p ® q ) @ (~ q ® ~ p ) b. ~ ( p Ú q ) @ (~ p Ú ~ q )
where k is equal to c. ~ ( p ® q ) @ ( p Ú ~ q ) d. ~ ( p Ù q ) @ (~ p Ù ~ q )
a. 1 b. 2 c. 3 d. 4
148. If 49n + 16n + l is divisible by 64 for all n Î N , then the
140. Two medians drawn from the acute angles of a right
angled triangle intersect at an angle p/6. If the length least negative integral value of l is
of the hypotenuse of the triangle is 3 units, then the a. - 2 b. - 1
area of triangle (in sq units) is c. - 3 d. - 4
a. 3 b. 3 c. 2 d. 9
149. Between any two real roots of the equation
141. The minimum value of the sum of real numbers e x sin x - 1 = 0, the equation e x cos x + 1 = 0 has
-5 -4 -3 8 10
a ,a , 3a , 1, a and a with a > 0, is
a. atleast one root
a. 7 b. 8 c. 6 d. 10 b. no root
142. A man walks a distance of 3 units from the origin c. atmost one root
d. exactly one root
towards the North-East (N 45° E) direction. From
there, he walks a distance of 4 units towards the 150. If the mean and variance of a binomial variable x are 2
North-West (N 45° W) direction to reach a point P . and 1 respectively, then the probability that x takes a
Then, the position of P in the argand plane is value greater than or equal to 1 is
a. 3ei p/ 4 + 4i b. ( 3 - 4i )ei p/ 4 2 4
i p/ 4
a. b.
c. ( 4 + 3i )e d. ( 3 + 4i )ei p/ 4 3 5
7 15
c. d.
8 16
Answers
Physics
1. (a) 2. (c) 3. (c) 4. (d) 5. (d) 6. (b) 7. (d) 8. (a) 9. (b) 10. (c)
11. (d) 12. (d) 13. (d) 14. (c) 15. (c) 16. (c) 17. (b) 18. (a) 19. (c) 20. (d)
21. (c) 22. (b) 23. (a) 24. (b) 25. (b) 26. (b) 27. (c) 28. (a) 29. (d) 30. (d)
31. (c) 32. (b) 33. (a) 34. (c) 35. (a) 36. (c) 37. (d) 38. (d) 39. (d) 40. (b)

Chemistry
41. (b) 42. (b) 43. (b) 44. (b) 45. (a) 46. (b) 47. (c) 48. (b) 49. (b) 50. (c)
51. (d) 52. (b) 53. (b) 54. (b) 55. (d) 56. (d) 57. (b) 58. (a) 59. (c) 60. (b)
61. (c) 62. (c) 63. (d) 64. (b) 65. (d) 66. (b) 67. (d) 68. (a) 69. (d) 70. (d)
71. (b) 72. (b) 73. (c) 74. (a) 75. (b) 76. (c) 77. (d) 78. (a) 79. (d) 80. (a)

English Proficiency
81. (d) 82. (b) 83. (d) 84. (c) 85. (d) 86. (d) 87. (c) 88. (c) 89. (b) 90. (a)
91. (c) 92. (a) 93. (b) 94. (a) 95. (a)

Logical Reasoning
96. (d) 97. (b) 98. (b) 99. (a) 100. (d) 101. (a) 102. (a) 103. (a) 104. (c) 105. (b)

Mathematics
106. (b) 107. (c) 108. (a) 109. (c) 110. (c) 111. (d) 112. (b) 113. (a) 114. (a) 115. (b)
116. (b) 117. (a) 118. (a) 119. (a) 120. (a) 121. (a) 122. (a) 123. (d) 124. (c) 125. (c)
126. (a) 127. (b) 128. (a) 129. (a) 130. (c) 131. (c) 132. (a) 133. (c) 134. (b) 135. (b)
136. (b) 137. (d) 138. (b) 139. (b) 140. (a) 141. (b) 142. (d) 143. (d) 144. (a) 145. (b)
146. (c) 147. (a) 148. (b) 149. (a) 150. (d)
Practice Set 3
BITSAT
Instructions
1. There are 150 questions in all. The number of questions in each part is as follows Subject No. of Questions
Part I Physics 1-40
Part II Chemistry 41-80
Part III (a) English Proficiency 81-95
(b) Logical Reasoning 96-105
Part IV Mathematics 106-150
2. All questions are multiple choice questions with four options, only one is correct.
3. Each correct answer fetches 3 marks while incorrect answer has a penalty of 1 mark.

PART I
Physics
1. The velocity of water waves may depend on their 4. A train accelerates from rest at a constant rate a for
wavelength l, the density of water r and the distance x1 and time t1. After that is retards to rest at
acceleration due to gravity g. The method of constant rate b for distance x 2 and time t 2. Then, it is
dimensional analysis gives the relation between these found that
quantities as x1 d t1 x1 b t1
a. = = b. = =
a. v 2 = kl-1g -1r -1 b. v 2 = klg x 2 b t2 x 2 a t2
c. v 2 = klrg d. v 2 = kl3g -1r -1 x b t x b t
c. 1 = = 2 d. 1 = = 2
x 2 a t1 x 2 a t1
2. A particle located at x = 0 and t = 0 starts moving along
the positive x-direction with a velocity v that varies as 5. An ant crawls up a hemispherical surface very slowly
v = a x , the displacement of the particle varies with as shown in figure. The coefficient of friction between
time as the ant and the surface is 1/3. If the line joining the
1
a.x µ t b. x µ t c. x µ t 2 d. x µ centre of hemispherical surface to ant makes an angle
t2 a with the vertical, the maximum possible value of a is
3. From an elevated point P , a stone is projected
vertically upwards. When the stone reaches a
distance h below P , its velocity is double of its velocity a
at a height h above P . The greatest height attained by
the stone from the point of projection P is
3 5
a. h b. h
5 3 a. sec-1( 3) b. cot -1( 3)
7 5
c. h d. h c. tan-1( 3) d. cosec-1( 3)
5 7
PRACTICE SET 3 1219
6. A block of weight 5 N is pushed against a vertical wall 12. Assume that a drop of liquid evaporates by decrease
by a force 12 N. The coefficient of friction between the in its surface energy, so that its temperature remains
wall and block is 0.6. The magnitude of the force unchanged. What should be the minimum radius of
exerted by wall on the block is the drop for this to be possible ? The surface tension
is T, density of liquid is r and L is its latent heat of
vaporisation.
12 N a. T /rL b. T /rL c. 2T /rL d. rL / T

13. A liquid is kept in a cylindrical vessel which is rotated


along its axis. The liquid rises at the sides (figure). If
a. 12 N b. 5 N c. 7.2 N d. 13 N the radius of the vessel is 0.05 m and the speed of
7. A vertical spring with force constant k is fixed on a rotation is 2 rad/s, then find the difference in the height
table. A ball of mass m at a height h above the free of liquid at centre and sides.
upper end of spring falls vertically on the springs, so w
that the spring is compressed by a distance d. The net
work done in this process is

k h

m
h
1 1 2 a. 20 cm b. 4 cm c. 2 cm d. 0.2 cm
a. mg (h + d ) + kd 2 b. mg (h + d ) - kd
2 2 14. An ideal gas is expanding such that pT 2 = constant.
1 1 The coefficient of volume expansion of the gas is
c. mg (h - d ) - kd 2 d. mg (h - d ) + kd 2
2 2 1 2 3 4
a. b. c. d.
8. A thin uniform rod of length l and mass m is swinging T T T T
freely about a horizontal axis passing through its end. 15. The ratio of the speed of sound in nitrogen gas to that
Its maximum angular velocity is w. Its centre of mass in helium gas at 300 K is
rises to a maximum height of
1 2 3 6
l 2w2 l 2w2 l 2w2 lw a. b. c. d.
a. b. c. d. 7 7 5 5
6g 3g 2g 6g
16. A point mass oscillates along X-axis according to the
A B

9. Two long metallic strips are joined together by two æ pö


rivets each of radius 0.1cm (see figure). Each rivet can law x = x 0 cosç wt - ÷. If the acceleration of the
è 4ø
with stand a maximum shearing stress of 4 ´ 108 Nm -2. particle is written as a = A cos( wt + d), then
The maximum tangential force which a strip can
a. A = x 0w2, d = p / 4 b. A = x 0w2, d = - p / 4
exert is
c. A = x 0w2, d = 3p / 4 d. A = x 0, d = p / 4
17. The fundamental frequency of a closed end pipe is
480 Hz. What is the fundamental frequency when
th
æ 1ö
ç ÷ length is filled with water?
a. 185 N b. 1885 N c. 855 N d. 1555 N è4ø
10. Two rods P and Q of same length and same diameter a. 120 Hz b. 240 Hz c. 640 Hz d. 960 Hz
having thermal conductivity ratio 2 : 3 joined end to 18. The electric flux for Gaussian surface A that enclose
end. If temperature at one end of P is 100°C and at the charged particles in free space is
one end of Q is 0°C, then temperature of interface is
(given, q1 = - 14nC, q 2 = 78.85 nC,q 3 = - 56 nC)
P Q A
q3
q1
a. 60°C b. 70°C
B
c. 50°C d. 40°C q2
11. A square wire frame of size L is dipped in a liquid. On
taking out, a membrane is formed. If the surface
tension of liquid is T, force acting on the frame will be a. 103 CN-1 m-2 b. 103 Nm2 C-1
a. 2TL b. 4TL c. 8TL d. 10TL c. 6.32 ´ 103 Nm2 C-1 d. 6.32 ´ 103 C N-1 m-2
1220 SELF STUDY GUIDE BITSAT

19. A sphere of 4 cm radius is suspended with in a hollow 25. The oscillating frequency of a cyclotron is 10 MHz. If
sphere of 6 cm radius. The inner sphere is charged to the radius of its dees is 0.5 m, the kinetic energy of a
a potential of 3 esu. When the outer sphere is earthed, proton, which is accelerated by the cyclotron is
then the charge on the inner sphere is a. 10.2 MeV b. 2.55 MeV
c. 20.4 MeV d. 5.1 MeV
26. Two particles A and B of masses m A and mB
4 cm
6 cm respectively and having the same charge are moving
in a plane. A uniform magnetic field exists
perpendicular to this plane. The speeds of particles
are v A and v B respectively and the trajectories are as
shown in figure, then
a. 54 esu b. 0.25 esu c. 30 esu d. 36 esu
A
20. Six identical capacitors are joined in parallel, charged
to a potential difference of 10 V, separated and then
connected in series, i. e. the positive plate of one is B
connected to negative plate of other. Then, potential
difference between free plates is
10
a. 10 V b. 30 V c. 60 V d. V a. mAv A < mBvB
5
b. mAv A > mBvB
21. Kirchhoff’s first law ( Si = 0) and second law ( SiR = SE ),
c. mA < mB and v A < vB
where symbols have their usual meanings, are
respectively on d. mA = mB and v A = vB
a. conservation of charge, conservation of energy 27. A magnetic needle suspended by a silk thread is
b. conservation of charge, conservation of momentum vibrating in the earth’s magnetic field. If the
c. conservation of energy, conservation of charge temperature of the needle is increased by 500° C, then
d. conservation of momentum, conservation of energy a. the time period decreases
22. In the given figure, find the resistance between A and b. the time period remains unchanged
B. Both the circle and the diameter are made of c. the time period increases
uniform wire of resistance 1´ 10-4 W /m. The length AB d. the needle stops vibrating
is 2 m. 28. A circular disc of radius 0.2 m is placed in a uniform
æ 1ö
magnetic field of induction ç ÷ Wb/m 2 in such a way
A B è pø
that its axis makes an angle of 60° with B . The
magnetic flux linked with the disc is
2 a. 0.01 Wb b. 0.02 Wb
a. ´ 10- 4W
3 c. 0.06 Wb d. 0.08 Wb
b. 2p ´ 10-4W
29. A wire of length 50 cm moves with velocity of
c. 14.56 ´ 10-4 W 300 m/min perpendicular to a magnetic field. If the
d. 0.88 ´ 10-4 W emf induced in the wire is 2V, magnitude of field in
tesla is
23. The resistance of series combination of 2 resistors is
S. When they are joined in parallel, the total a.2 b. 5
resistance is P . If S = nP , then minimum possible c. 0.4 d. 0.8
value of n is 30. In an LCR series AC circuit, the voltage across each
a. 4 b. 3 of the components L, C and R is 50 V. The voltage
c. 2 d. 1 across LC combination will be
24. Find R, if power dissipated in the circuit is 150 W. a. 50 V b. 50 2V c. 100 V d. 0 V
2W 31. The refractive index of air is 1.0003. The thickness of
R air column which has one more wavelength of yellow
15V
light (l = 600nm), then the same thickness of
vacuum is
a. 2 W b. 6 W a. 0.02 mm b. 0.2 mm
c. 7 W d. 5 W c. 2 mm d. 2 cm
PRACTICE SET 3 1221
32. In an interference experiment, third bright fringe is 37. The radioactivity of a sample is A 1 at time t1 and A2 at
obtained at a point on the screen with a light of 700 nm. time t 2. If the mean life of the specimen is T, then the
What should be the wavelength of the light source in
number of atoms that have disintegrated in the time
order to obtain 5th bright fringe at the same point ?
interval of (t 2 - t1) is
a. 420 nm b. 750 nm c. 630 nm d. 500 nm
( A1 - A2 )
a. ( A1 - A2 ) b.
33. Two coherent light beams of intensity I and 4I are T
superposed. The maximum and minimum possible c. ( A1 - A2 )T d. ( A1t1 - A1t 2 )
intensities in the resulting beam are
a. 5I and 3I b. 9I and I c. 9I and 3I d. 5I and I 38. The radioactive isotope X with a half-life of 109 year
decays to Y which is stable. A sample of rocks was
34. Two convex lenses of focal lengths 0.3 m and 0.05 m found to contain both the elements X andY in the ratio
are used to make a telescope. Distance kept between 1 : 7. What is the age of the rocks ?
the two in order to obtain image at infinity is a. 2 ´ 109 yr b. 3 ´ 109 yr
a. 0.25 m b. 0.3 m c. 0.35 m d. 0.20 m c. 6 ´ 109 yr d. 7 ´ 109 yr
35. The number of photons of wavelength 660 nm emitted
per second by an electric bulb of 60 W is 39. A common emitter amplifier has a voltage gain of 50,
(Take, 6.6 ´ 10-34 Js) an input impedance of 100W and an output
impedance of 200W. The power gain of the amplifier is
a. 2 ´ 1020 b. 2 ´ 10-20
a. 500 b. 1000
c. 3 ´ 1020 d. 1.5 ´ 1020 c. 1250 d. 100
236
36. When decays in a series by emission of
88 Ra 40. A 600 W carrier is modulated to a depth of 75% by a
3a-particles and one b-particle, isotope X formed as 400 Hz sine wave. Find the total power of antenna.
224 218 220 223
a. 83 X b. 84 X c. 84 X d. 82 X a. 769 W b. 796 W c. 679 W d. 637.5 W

PART II
Chemistry
41. Choose the correct statement from the following reaction. 44. A binary solid ( A +B - ) has a zinc blende structure with
Cl B - ions constituting the lattice and A + ions occupying
alc.NaOH aq. NaOH
C H 3C B 25% tetrahedral holes, then the formula of solid is
A a. AB b. A2B c. AB2 d. AB4
a. B is obtained by elimination reaction 45. Which of the following reactions will not give propane?
b. B is an isomer of ethyl methyl ether, while C is the
Cl
dehydrated compound of B
c. C is obtained by substitution reaction B2H6, ether
d. The molecular formula of B is C3H6 and that of a. H C b. H3C CH3 P+HI
3 CH3COOH
C is C3H8O
O
Cl Mg, ether
42. An inorganic halide P reacts with water to form two CH3 Mg X H 2O
acids Q and R, P also reacts with NaOH to form two CH3 d. H3C
c. H3C H2O
salts S and T which remain in solution. The solution
gives white precipitate with both AgNO3 and BaCl2 46. The molecular formula of certain compound has
solutions respectively. P is a useful reagent. What is P X 4 O6 × If 10 g of X 4 O6 has 5.72 g X , then the atomic
in the reaction? mass of X is
a. AgCl b. NaCl c. SO2Cl2 d. HCl a. 42 b. 30
c. 24 d. 32
43. The vapour density of completely dissociated NH4 Cl
47. 3.15 g oxalic acid [(COOH)2 ×x H2O] was dissolved to
would be
a. half of that of ammonium chloride
make 500 mL solution. On titration, 33.36 mL of this
b. double that of ammonium chloride solution were neutralised by 50 mL N /15 NaOH. The
c. slightly less than half of that of ammonium chloride value of x will be
d. determined by the amount of solid ammonium a. 3 b. 2
chloride used c. 1 d. 4
1222 SELF STUDY GUIDE BITSAT

48. Which of the following represent the given mode of 56. Toluene is nitrated and the resulting product is
hybridisationsp -sp -sp 2-sp 2 from right to reduced with tin and HCl. The product so obtained is
a. H2C== CH¾ C ºº CH diazotised and then heated with cuprous bromide. The
b. H2C== C== C == CH2 final product is
a. mixture of o-and p-dibromobenzene
c. CH 2 b. mixture of o-and p-bromotoluene
H 2C c. mixture of o-and p-bromoaniline
d. HC ºº C ¾ CH2 ¾ C ºº CH d. mixture of o-and m-bromotoluene
49. Which of the following statements is incorrect? 57. In which of the following compounds, the size of cation
a. The degree of dissociation of a weak electrolyte and anion ratio is minimum?
decreases as its concentration decreases a. LiI b. LiF
b. The freezing point of water is depressed by the c. CsF d. CsI
addition of glucose
58. Which of the following is not an antipyretic drug?
c. If two liquids that form an ideal solution, are
mixed, the change in entropy is positive a. Paracetamol b. Aspirin
d. Energy is released when a substance dissolves in c. Phenacetin d. Marijuane
water provided that the hydration energy of the 59. Which of the following represent the heat of formation
substance is more than its lattice energy of CO2(g)?
50. The hardness of water sample which contains 1
a. CO (g ) + O2 (g ) ¾® CO2 (g )
0.001 mole of MgSO4 dissolved per litre of water, is 2
1
a. 150 ppm b. 100 ppm b. C6H6 (l ) + 7 O2 (g ) ¾® 6 CO2 (g ) + 3H2O (l )
c. 120 ppm d. 180 ppm 2
c. C (graphite) + O2 (g ) ¾® CO2 (g )
51. Density of a gas is found to be 5.46 g/dm 3 at 27° C
d. CH4 (g ) + 2O2 (g ) ¾® CO2 (g ) + 2 H2O (l )
and 2 bar pressure. At STP, its density will be
a. 3 g/ cm3 b. 0.3 g/dm3 60. Which of the following factors is of no significance for
c. 3 g/dm3 d. 4 g/dm3 roasting sulphide ores to the oxides and not
subjecting the sulphide ores to carbon reduction
52. On the basis of intermolecular forces, predict the correct directly?
decreasing order of boiling points of the compounds. a. Metal sulphides are thermodynamically more stable
a. CH3OH > H2 > CH4 b. CH3OH > CH4 > H2 than CS2
c. CH4 > CH3OH > H2 d. H2 > CH4 > CH3OH b. Metal sulphides are less stable than the
corresponding oxides
53. Which of the following options does not have isoelectronic c. CO2 is more volatile than CS2
species? d. CO2 is thermodynamically more stable than CS2
a. CN- , N2, C22- b. SO23- , CO23- , NO3-
61. The reaction of acetaldehyde with excess of ethanol in
c. BO33- , CO23- , NO3- d. PO34- , SO24- , ClO-4
the presence of H2SO4 gives the product
54. Actinoids exhibit more number of oxidation states than a. CH3CH(OC2H5 )2
lanthanoids. The main reason for this is b. CH3CH (OH)2
a. more active nature of the actinoids c. Ketal
b. more energy difference between 5f and 6d-orbitals d. C2H5O CH2 CH2OC2H5
than between 4f and 5d-orbitals
c. lesser energy difference between 5f and 6d-orbitals 62. When mercuric iodide is added to the aqueous
than that between 4f and 5d-orbitals solution of KI, the
d. greater metallic character of the lanthanoids than that a. freezing point is raised
of the corresponding actinoids b. freezing point does not change
55. Among the elements Li, K, Ca, S and Kr, which one is c. freezing point is lowered
expected to have the lowest first ionisation enthalpy d. boiling point does not change
and which have the highest first ionisation enthalpy? 63. KMnO4 oxidises NO-2 to NO-3 in basic medium, the
The increasing order of first ionisation enthalpy of the number of moles of NO-2 oxidised by 1 mole of KMnO4 is
elements Li, K, Ca, S and Kr, is
3
a. Li < K < Ca < S < Kr a. b. 2
b. K < Li < Ca < S < Kr 2
4
c. Kr < Ca < S < Li < K c. d. None of these
d. Ca < Li < S < K < Kr 3
PRACTICE SET 3 1223
64. The correct decreasing order of reactivity of the 72. The hydrogen electrode is dipped in a solution of
following alkenes, is pH = 3 at 25° C. The potential of the cell would be
æ 2.303 RT ö
I. 2-butene II. 2-methyl-2-butene ç the value of = 0.059 V ÷
III. 2,3-dimethyl-2-butene IV. ethene è F ø
a. III > I > II > IV b. IV > I > II > III a. - 0.732 V b. 0.059 V
c. IV > II > I > III d. I > III > IV > II c. - 0.177 V d. - 0.059 V
65. The equilibrium constant for the reaction given below 73. During the test of nitrogen by Lassaigne’s method, the
is 2.0 ´ 10-7 at 300 K. prussian blue colour is obtained due to the formation of
PCl 5( g ) s PCl3 ( g ) + Cl2( g ) a. Fe2[ Fe (CN)6] b. Na 4[Fe (CN)6]
The standard entropy change, if DH ° = 28.40 kJ mol-1 c. Fe4[Fe (CN)6] 3 d. Fe3[Fe (CN)6] 4
is 74. On the excess addition of NaoH solution to a solution
a. 33.6 JK -1 b. - 33.6 JK -1 of ZnCl 2 produces
c. 41.2 JK -1 d. - 41.2 JK -1 a. Na 2ZnO2 b. Zn(OH)2
c. ZnO d. None
66. The compound obtained when copper nitrate is
strongly heated, is 75. Which of the following is a 3° amine?
a. copper b. copper nitride a. Allyl amine
c. copper nitrite d. copper oxide b. Propan-2-amine
c. N,N-diethyl butan-1-amine
67. If the half-life of a first order reaction is 60 min, then d. N-methyl ethanamine
how long will it take to consume 90% of the reactant?
76. The geometry of XeF4 is
a. 250 min b. 200 min c. 180 min d. 300 min
a. square planar b. tetrahedral
68. The decreasing order of stability of the following c. trigonal d. trigonal bipyramidal
compound is
+ + + +
77. The vapour density of a mixture containing NO2 and
CH2 CH2 CH2 CH2 N2O4 is 38.3 at 27° C. In 100 g mixture, the mole of
NO2 is
a. 0.321 b. 0.437
c. 0.812 d. 0.617
78. The chief ores of Sn, Fe, Al and Ag and methods for
CH NO2 CH3 NH2 the purification of their ores respectively are given
(I) (II) (III) (IV) below. Choose the incorrect match from the following.
a. Sn-Cassiterite-Gravity separation
a. IV > I > III > II b. II > III > I > IV
b. Fe-Haematite-Magnetic separation
c. IV > III > I > II d. III > IV > I > II
c. Al-Bauxite-Leaching
69. Which of the following yield alkanes, alkenes and d. Ag-Cinnabar-Thermal decomposition
alkynes? 79. Which of the following reactions is non-spontaneous?
a. Kolbe’s reaction a. NO-2 + CH3COOH ¾® CH3COO- + HNO2
b. Wurtz reaction
(in aqueous medium)
c. Sandmeyer’s reaction
d. Williamson’s synthesis b. HCl + CH3COOH ¾® CH3COOH2 + Cl-
(in non-aqueous medium)
70. Sodium aluminium silicate (zeolite) is used for
c. HNO2 + CH3COO- ¾® CH3COOH + NO2-
softening of hard water. This causes
(in aqueous medium)
a. adsorption of Ca 2+ and Mg2+ ions of hard water d. HF + HCl ¾® H+2 Cl + F- (in non-aqueous medium)
replacing Al3+ ions
b. adsorption of Ca 2+ and Mg2+ ions of hard water 80. Choose the correct answer from the following.
replacing Na + ions a. Ni(PPh3 )2 Cl2 has tetrahedral geometry with
c. Both (a) and (b) are true magnetic moment 2.82 M
d. None of the above is true b. [FeCl6] 4- is an inner orbital complex with zero
magnetic moment
71. The molar conductivity for 0.01M acetic acid is 3.907S c. The magnetic moment of [Fe(CN)6] 4- is zero
cm 2 mol-1 and at infinite dilution, it is 390.7. the pH of d. Wavelength of absorption in the visible region for
the solution is [Ni(H2O)6] 2+
a. 2 b. 3 c. 6 d. 4
1224 SELF STUDY GUIDE BITSAT

Part III
a. English Proficiency
Directions (Q. Nos. 81 to 84) Spot the error in the 88. P : was suspended
following sentences and indicate your answer. Q : the officer being corrupt
81. It is a pity (a)/ that even five years old boys (b)/ are R : before his dismissal
engaged in hazardous factories. (c)/ No error (d) S : from serivce
82. I gave him(a)/ two hundred rupees notes (b)/ for The proper sequence should be
a. QPSR b. QPRS
depositing. (c)/ No error (d)
c. RSQP d. RSQP
83. It is not my business (a)/ to give an advice to those
Directions (Q. Nos. 89 to 92) Find the synonym of the
(b)/ who are not sensible enough to deal with their
word written in capital letters in the given sentences.
own problems. (c)/ No error (d)
89. EXAGGERATION of facts would always lead to
84. I don’t think(a)/ it is your house, (b)/ it is someboy’s confusion
else. (c)/ No error (d)
a. Simplification b. Negation
Directions (Q. Nos. 85 to 88) Rearrange the jumbled c. Emancipation d. Amplification
parts of the sentence and then indicate the correct option.
90. After his father’s death,he became INSOLVENT.
85. P : it is a pity that a. Rich b. Poor
Q : by offering a handsome dowry c. Bankrupt d. Nonchalant
R : a number of parents think that 91. He INDUCES human beings to want things they don’t
S : they will able to ensure the happiness of their want.
daughters.
a. Influences b. Dictates
The proper sequence should be c. Persuades d. Appreciates
a. SQRP b. PRSQ c. PSRQ d. PRQS
92. His information is not AUTHENTIC.
86. The common man a. Real b. Reliable c. Believable d. Genuine
P : in nuturing
Q : a more active role
Directions (Q. Nos. 93 to 95) Find the antonym of the
word written in the capital letters.
R : communal harmony
S : should play 93. He finally CONCEDED that he was involved in
The proper sequence should be smuggling
a. Admitted b. Accepted c. Denied d. Concealed
a. PRSQ b. SQPR c. SQRP d. PRQS
87. The doctor 94. The accused emphatically DENIED the charge in the
court.
P : able to find out
Q : what had caused a. Accepted b. Agreed
c. Asserted d. Affirmed
R : the food poisoning
S : had not been 95. The cloth is COARSE of touch.
The proper sequence should be a. Delicate b. Rough c. Painful d. Harsh
a. SPRQ b. PRQS c. PRSQ d. SPQR

b. Logical Reasoning
96. ‘Doctor’ is related to ‘Patient’, in the same way 98. Which figure will best represent the relationship
‘Lawyer’ is related to amongst the three classes doctor, teacher, women ?
a. Customer
b. Criminal
c. Magistrate a. b.
d. Client
97. Find the odd one out.
a. 8-11 b. 1-4 c. 7-10 d. 3-5 c. d.
PRACTICE SET 3 1225
99. Find the missing number. 102. In the question below, figure (X) is embedded in one of
the following. Then, find out the correct alternative.
5 4 3 8 9 4
20 9 24 11 ? 13

a. 26 b. 36
c. 32 d. 117 (X)

100. Find the odd one out.

a. b. c. d.

103. If LOFTY is coded as LPFUY, then DWARF will be


a. b. c. d.
written as
a. DXASF b. DXBSG c. DXATF d. DWBSG
101. Count the number of squares in the following figure.
104. If the day before yesterday was Thursday, when will
Sunday be ?
a.Today b. Two days after today
c. Tomorrow d. Day after Tomorrow
105. Find the next term in the series 5, 16, 51, 158.
a. 4 b. 5 c. 6 d. 7 a. 1452 b. 483 c. 481 d. 1454

PART IV
Mathematics
106. If the median of 25 observations is 45 and if the 1 x4
observations greater than the median are increased
110. If ò x + x 5 dx = f (x ) + C , then the value of ò x + x5
dx
by 4, then the median of the new data is
is
a. 49 b. 41 c. 45 d. 40
a. log x - f (x ) + C b. f (x ) + log x + C
107. The differential equation of rectangular hyperbolas c. f (x ) - log x + C d. None of these
whose axes are asymptotes of the hyperbola
x 2 - y 2 = a 2, is 111. If f (x ) = a log| x | + bx 2 + x has its extremum values at
dy dy x = - 1and x = 2, then
a. y =x b. x = -y
dx dx a. a = 2, b = - 1
dy b. a = 2, b = - 1/ 2
c. x =y d. x dy + y dx = C
dx c. a = - 2, b = 1/ 2
ì 1ü d. None of the above
108. If f (x ) = max í sin x , cos x , ý , then the area of the
î 2 þ 112. Let f (x ) = tan-1{ g (x )}, where g (x ) is monotonically
region bounded by the curves y = f (x ), X-axis, Y-axis p
5p increasing for 0 < x < . Then, f (x ) is
and x = , is 2
3 a. increasing on ( 0, p / 2)
5p 3 5p
a. 2 - 3 + b. 2 + + b. decreasing on ( 0, p / 2)
12 2 12 æ p pö
5p c. increasing on ( 0, p / 4) and decreasing on ç , ÷
c. 2 + 3+ d. None of these è 4 2ø
12
d. None of these
[x] 2x 113. The equation sin x + x cos x = 0 has atleast one root in
109. The value of ò dx is
0 2[ x ] the interval
[x ] a. ( -p / 2, 0)
a. [x ] log 2 b.
log 2 b. ( 0, p )
1 [x ]
c. × d. None of these c. ( -p / 2, p / 2)
2 log 2
1226 SELF STUDY GUIDE BITSAT

d. None of the above ì (x + 2) ( 2x 5 + 3x 4 + 4x 3 + 5x 2 + 6 ü


2 2 ïp : p = ;ï
114. If the tangent at (1, 1) on y = x ( 2 - x ) meets the 124. If A = í x 2 + 2x ý,
curve again at P , then P is ï + ï
î x, p Î Z þ
a.(4, 4) b. ( -1, 2)
c. (9 / 4, 3 / 8) d. None of these
then the number of elements in the set A, is
a. 2 b. 3 c. 4 d. 6
dy
115. If y = | x - x 2 |, then at x = 1is 1 1
dx 125. If ( 21.4)a = ( 0.00214)b = 100, then the value of -
a b
a. -1 is
b. 1
a. 0 b. 1 c. 2 d. 4
c. Does not exist
d. None of the above 1
126. If sin q + cos q = and 0 £ q < p, then tan q equals
116. If f (x ) = | x - 2 | and g (x ) = f {f (x )}, then g ¢(x ) for x > 20 is 5
-4 -3 3 4
a. 1 b. 2 a. b. c. d.
3 4 4 3
c. -1 d. None these
127. If a , b , c are the sides of DABC, right angled at C, then
117. If a and b are the roots of the equation ax 2 + bx + c = 0 2
then lim (1 + ax 2 + bx + c )1/ x - a is æc c ö
the minimum value of ç + ÷ is
x ®a
èa b ø
a ( a - b)
a. e b. e a ( b - a ) a. 0 b. 4 c. 6 d. 8
c. 1 d. None of these 4 2 2
128. The equation sin x - 2 cos x + a = 0 can be solved, if
118. If f (x ) = ax 2 + bx + c satisfies the identity
a. - 3 £ a £ 3 b. - 2 £ a £ 2
f (x + 1) - f (x ) = 8x + 3 for all x Î R . Then, (a , b ) equals
c. -1 £ a £ 1 d. None of these
a. (2, 1) b. ( 4, - 1)
c. ( -1, 4) d. ( -1, 1) 129. Complete solution set of [cot -1 x ] + 2 [tan-1 x ] = 0,
where [ ] denotes the greatest integer function, is
119. The number of times a die must be tossed to obtain a
equal to
6 atleast once with probability exceeding 0.9 is atleast
a. ( 0, cot 1) b. ( 0, tan 1) c. (tan 1, ¥ ) d. (cot 1, tan 1)
a. 13 b. 19
c. 25 d. None of these 130. In DABC, base BC and area of triangle are fixed. The
120. For all n Î N , 3 3n - 26n - 1 is divisible by locus of the centroid of DABC is a straight line that is
a. parallel to side BC
a. 24 b. 64
b. right bisector of side BC
c. 17 d. 676
c. right angle of BC
121. If x = 5 and y = - 2, then x - 2y = 9. The contrapositive d. inclined at an angle sin-1( D /BC ) to side BC
of this proposition is
131. A triangle is formed by the lines x + y = 0, x - y = 0
a. if x - 2y ¹ 9, then x ¹ 5 or y ¹ - 2
and lx + my = 1. If l and m vary subject to the condition
b. if x - 2y = 9, then x ¹ 5 and y ¹ 2 l 2 + m 2 = l , then the locus of its circumcentre is
c. x - 2y = 9 if and only if x = 5 and y = - 2
a. (x 2 - y 2 )2 = x 2 + y 2 b. (x 2 + y 2 )2 = x 2 - y 2
d. None of the above
c. x 2 + y 2 = 4x 2y 2 d. (x 2 - y 2 )2 = (x 2 + y 2 )2
122. The inverse of the function f : R ® R given by
132. If the pairs of lines x 2 + 2xy + ay 2 = 0 and
f (x ) = loga (x + x 2 + 1); (a > 0, a ¹ 1,
) is
ax + 2xy + y 2 = 0 have exactly one line in common,
2
1 x 1 x
a. (a + a - x ) b. (a - a - x ) then the joint equation of the other two lines is given
2 2 by
1 æ a x + a -x ö
c. çç x ÷ d. Not defined a. 3x 2 + 8xy - 3y 2 = 0 b. 3x 2 + 10xy + 3y 2 = 0
2 è a - a - x ÷ø
c. y 2 + 2xy - 3x 2 = 0 d. x 2 + 2xy - 3y 2 = 0
123. The relation R defined on the set A = {1, 2, 3, 4, 5} by 133. If the line x cos q + y sin q = 2 is the equation of a
R = {(a , b ) : | a 2 - b 2 | < 16} is given by transverse common tangent to the circles x 2 + y 2 = 4
a. {(1, 1), ( 2, 1), ( 3, 1), ( 4, 1), ( 2, 3)} and x 2 + y 2 - 6 3x - 6 y + 20 = 0, then the value of q
b. {( 2, 2), ( 3, 2), ( 4, 2), ( 2, 4)} is
c. {( 3, 3), ( 4, 3), (5, 4), ( 3, 4)} a. 5p/ 6 b. 2p/ 3 c. p/3 d. p/6
d. None of the above
PRACTICE SET 3 1227
134. If the parabola y = ax 2 - 6x + b passes through (0, 2) 143. In the quadratic equation ax 2 + bx + c , if D = b 2 - 4ac
and has its tangent at x = 3 / 2 parallel to X-axis, then and a + b , a 2 + b2, a 3 + b3 are in GP, where a, b are
a. a = 2, b = - 2 b. a = 2, b = 2 the roots of ax 2 + bx + c = 0, then
c. a = - 2, b = 2 d. a = - 2, b = - 3
a. D ¹ 0 b. bD = 0
135. An ellipse is sliding along the coordinate axes. If the c. cD = 0 d. D = 0
foci of the ellipse are (1, 1) and (3, 3), then the area of
the director circle of the ellipse (in sq units) is 144. Let z = x + iy be a complex number, where x and y
a. 2p b. 4p c. 6p d. 8p are integers. Then, the area of the rectangle whose
2 2 vertices are the roots of the equation zz 3 + zz 3 = 350,
x y
136. A normal to the hyperbola - = 1 has equal is
4 1
intercepts on the positive X-axis and Y-axis. If this a. 48 sq units
b. 32 sq units
x2 y 2
normal touches the ellipse 2 + 2 = 1, then a 2 + b 2 is c. 40 sq units
a b d. 80 sq units
equal to
145. The largest interval for which x 12 - x 9 + x 4 - x + 1 > 0,
a. 5 b. 25
c. 16 d. None of these is
a. -4 < x £ 0 b. 0 < x < 1
137. India plays two matches each with West Indies and
Australia. In any match, the probabilities of India c. -100 < x < 100 d. - ¥ < x < ¥
getting points 0, 1 and 2 are 0.45, 0.05 and 0.50, 146. The set of all real numbers x for which
respectively. Assuming that the outcomes are x 2 - | x + 2 | + x > 0, is
independent, the probability of India getting atleast
7 points is a. ( - ¥ , - 2)
a. 0.8750 b. 0.0875 c. 0.0625 d. 0.0250 b. ( - ¥ , - 2 ) È ( 2, ¥ )
c. ( - ¥ , - 1) È (1, ¥ )
138. The number of 3 ´ 3 matrices A whose entries are
d. ( 2, ¥ )
éx ù é 1ù
either or 1 and for which the system A êy ú = ê0ú has 147. Let a, b, c be three vectors of magnitude 3 , 1, 2 such
ê ú ê ú
êëz úû êë0úû that a ´ ( a ´ c) + 3b = 0. If q is the angle between a and
c, then cos 2 q is equal to
exactly two distinct solutions, is
3 1
a. 0 b. 29 - 1 a. b.
4 2
c. 168 d. 2 1
c. d. None these
139. A determinant is chosen at random from the set of all 4
determinants of order 2 with elements 0 or 1 only. 148. If resolved part of vector a along the vector b is a1 and
What is the probability that the value of determinant
that perpendicular to b is a2, then a1 ´ a2 is equal to
chosen is positive?
a. 3/16 b. 3/6 ( a ´ b) b ( a ´ b) a
a. b.
c. 13/16 d. 5/16 | b|2 | a |2
140. The coefficient of t 24 in (1 + t 2 )12 (1 + t 12 ) (1 + t 24 ) is ( a × b)(b ´ a) ( a × b)(b ´ a)
c. d.
12
a. C6 + 3 12
b. C6 + 1 | b |2 |b ´ a |
c. 12C6 d. 12C6 + 2 149. The graph of the equation x 2 + y 2 = 0 in the three
141. How many different nine-digit numbers can be formed dimensional space, is
from the number 223355888 by rearranging its digits a. X-axis b. Y-axis
so that the odd digits occupy even positions? c. Z-axis d. xy-plane
a. 16 b. 36 x -1 y - 2 z - 3
c. 60 d. 180 150. If the straight lines = = and
k 2 3
142. If a , b , c , d are positive real numbers such that x - 2 y - 3 z -1
= = intersect at a point, then integer k
a + b + c + d = 2, then M = (a + b ) (c + d ) satisfies the 3 k 2
relation is equal to
a. 0 £ M £ 1 b. 1 £ M £ 2 a. 2 b. - 2
c. 2 £ M £ 3 d. 3 £ M £ 4 c. - 5 d.
Answers
Physics
1. (b) 2. (c) 3. (b) 4. (b) 5. (b) 6. (d) 7. (b) 8. (a) 9. (b) 10. (d)
11. (c) 12. (c) 13. (c) 14. (c) 15. (c) 16. (c) 17. (c) 18. (b) 19. (d) 20. (c)
21. (a) 22. (d) 23. (a) 24. (b) 25. (d) 26. (b) 27. (d) 28. (b) 29. (d) 30. (d)
31. (c) 32. (a) 33. (b) 34. (c) 35. (a) 36. (a) 37. (c) 38. (b) 39. (c) 40. (a)

Chemistry
41. (b) 42. (c) 43. (a) 44. (c) 45. (c) 46. (d) 47. (b) 48. (a) 49. (d) 50. (b)
51. (c) 52. (b) 53. (b) 54. (c) 55. (b) 56. (b) 57. (a) 58. (d) 59. (c) 60. (a)
61. (a) 62. (c) 63. (d) 64. (b) 65. (b) 66. (d) 67. (b) 68. (c) 69. (a) 70. (b)
71. (d) 72. (c) 73. (c) 74. (a) 75. (c) 76. (a) 77. (b) 78. (d) 79. (a) 80. (c)

English Proficiency
81. (b) 82. (b) 83. (b) 84. (c) 85. (b) 86. (b) 87. (d) 88. (a) 89. (d) 90. (c)
91. (c) 92. (d) 93. (c) 94. (a) 95. (a)

Logical Reasoning
96. (d) 97. (d) 98. (d) 99. (b) 100. (d) 101. (b) 102. (d) 103. (a) 104. (c) 105. (c)

Mathematics
106. (c) 107. (b) 108. (b) 109. (b) 110. (a) 111. (b) 112. (a) 113. (b) 114. (c) 115. (c)
116. (a) 117. (a) 118. (b) 119. (a) 120. (d) 121. (a) 122. (b) 123. (d) 124. (c) 125. (c)
126. (a) 127. (d) 128. (b) 129. (d) 130. (a) 131. (a) 132. (b) 133. (d) 134. (b) 135. (d)
136. (d) 137. (b) 138. (a) 139. (a) 140. (d) 141. (c) 142. (a) 143. (c) 144. (a) 145. (d)
146. (b) 147. (a) 148. (c) 149. (c) 150. (c)
Practice Set 4
BITSAT
Instructions
1. There are 150 questions in all. The number of questions in each part is as follows Subject No. of Questions
Part I Physics 1-40
Part II Chemistry 41-80
Part III (a) English Proficiency 81-95
(b) Logical Reasoning 96-105
Part IV Mathematics 106-150
2. All questions are multiple choice questions with four options, only one is correct.
3. Each correct answer fetches 3 marks while incorrect answer has a penalty of 1 mark.

PART I
Physics
1. The coordinates of a moving particle at any time t are 4. A mass m is attached to the end of a rod of length l.
given by x = at 3 and y = bt 3 . The speed of the particle The mass goes along a vertical circular path with the
at time t is given by other end hinged at its centre. What should be the
minimum velocity of the mass at the bottom of the
a. a 2 + b 2 circle, so that the mass completes the circle?
b. t a 2 + b 2 a. 5 gl b. 2 gl
c. t 2 a 2 + b 2 c. 3 gl d. 4 gl
d. 3t 2
a +b
2 2
5. The upper half of an inclined plane with inclination f is
perfectly smooth, while the lower half is rough. A body
2. If power (P ), surface tension (T ) and Planck’s constant
starting from rest at the top will again come to rest at
(h ) are arranged so that the dimensions of time in their the bottom, if the coefficient of friction for the lower
dimensional formula are in ascending order, then half is
which of the following is correct ? a. 2 cos f b. 2 sin f
a. P , T , h b. P , h, T
c. 2 tan f d. tan f
c. T , P , h d. T , h, P
6. A tennis ball of mass m is floating in air by a jet of
3. A girl runs at a speed of 4 m/s to overtake a standing water emerging out of a nozzle. If the water strikes the
bus. When she is 6m behind the door at t = 0, the bus tennis ball with a speed v and just after collision water
moves forward and continues this with an acceleration falls dead, the rate of mass flow of water in the nozzle
of 1.2 ms - 2. The girl reaches the door in time t, then is equal to
a. 4t = 6 + 0.6 t 2 b. 1.2 t 2 = 4t a.
mv
b.
mg
c.
2mg
d.
2mv
c. 4t 2 = 1.2t d. 6 + 4t = 0.2 t 2 g v v g
1230 SELF STUDY GUIDE BITSAT

7. Which of the following is not an example of perfectly 14. A Carnot’s engine operates with source at 127° C and
inelastic collision? sink at 27° C. If the source supplies 40 kJ of heat
a. A bullet fired into a block, if bullet gets embedded in energy, the work done by the energy is
block a. 1 kJ b. 4 kJ
b. Capture of an electron by an atom c. 10 kJ d. 30 kJ
c. A man jumping onto a moving boat
15. The mass of 1 mole of air is 29 ´ 10- 3 kg, then the
d. A ball bearing striking another ball bearing
speed of sound in air at standard temperature and
8. Consider a uniform square plate of side a and mass m. pressure is
The moment of inertia of this plate about an axis
a. 332.5 ms- 1 b. 300 ms- 1
perpendicular to its plane and passing through one of
its corners is c. 280 ms- 1 d. 250 ms- 1
5 ma 2 7 2 16. The equation of wave on a string of linear mass
a. ma 2 b. c. ma 2 d. ma 2
2 12 12 3 density 0.04 kg/m 3 is given by
9. A bimetallic strip consists of metal X and Y . It is é æ t x öù
mounted rigidly at the base as shown in figure. y = 0.02 (m) sin ê2p çç - ÷÷ú
ë è 0.04(s ) 0.50 (m) øû
Metal X has higher coefficient of expansion as
compared to metal Y . When the bimetallic strip is The tension in the string is
placed in a cold bath a. 6.25 N b. 4.0 N
c. 12.5 N d. 0.5 N
17. Two bodies M and N of equal to masses are
X Y suspended from two separate massless springs
of spring constants k1 and k 2, respectively. If the
two bodies oscillate vertically such that their
maximum velocities are equal, the ratio of the
amplitude of vibration of M to that of N is
a. it will bend towards left a. k1 /k 2 b. k 2 /k1
b. it will bend towards right c. k1 /k 2 d. k 2 /k1
c. it will neither bend nor shrink
d. it will not bend but shrink 18. Two identical capacitors, have the same
capacitance C. One of them is charged to potential
10. The excess pressure inside one soap bubble is V1 and the other to V2. The negative end of the
3 times that inside a second bubble, then the ratio of capacitors are connected together. When the
their surface area is positive ends are also connected, the decrease in
a. 9 : 1 b. 1 : 27 c. 1 : 3 d. 1 : 9 the energy of the combined system is
11. Two equal drops are falling through air with terminal 1 1
a. C (V12 - V22 ) b. C (V12 + V22 )
velocity 5 cm/ s. If the drop gets combines, the new 4 4
1 1
terminal velocity will be c. C (V1 - V2 )2 d. C (V1 + V2 )2
a. 5 ´ 2 cm/s b. 5 ´ 4 cm/s 4 4
c. 5 ´ ( 4)1/ 3 cm/s d. 5 ´ ( 4)2/ 3 cm/s 19. A parallel plate capacitor has its capacitance of
100 pF, when the plates of the capacitor are
12. A wire of length L and cross-sectional area A is made separated by a distance of t. Then, a metallic foil of
of a material of Young’s modulus Y . If the wire is thickness t / 3 is introduced between the plates. The
streched by the amount x, the work done is capacitance will then become
YA 2 YA 2
a. x b. x a.100 pF b. 150 pF
L 2L 200 100
YAL AL c. pF d. pF
c. 2 d. Y 3 3
x 2x 2
20. Two charges, each equal to q, are kept at x = - a and
13. A lead bullet of initial temperature 27° C and speed x = + a on X -axis. A particle of mass m and charge
v km/h penetrates into a solid object and melts. If 50% q
of the kinetic energy is used to heat it, the value of v in q 0 = is placed at the origin. If charge q 0 is given a
2
km/h is (for lead, melting point = 600 K, latent heat of
small displacement ( y < < a ) along Y -axis, the net
fusion = 2.5 ´ 104 J/ kg and specific heat = 125 J/kg K)
force acting on the particle is proportional to
a. 1000 b. 1800 1 1
c. 3600 d. 1200 a. y b. - y c. d. -
y y
PRACTICE SET 4 1231
21. An electric cell of emf E is connected across a copper 28. A cylindrical magnet has a length of 5 cm and a
wire of diameter d and length l. The drift velocity of diameter of 1 cm. It has a uniform magnetisation of
electrons in the wire is v d . If the length of the wire is 5.30 ´ 103 A/m 3 . What is its magnetic dipole moment ?
changed to 2l, the new drift velocity of electrons in the a. 1 ´ 10- 2 J/T b. 2.08 ´ 10- 2 J/T
copper wire will be
c. 3.08 ´ 10- 2 J/T d. 1.52 ´ 10- 2 J/T
a. vd b. 2vd
v v 29. A coil of wire of certain radius has 600 turns and a
c. d d. d
2 4 self-inductance of 108 mH. The self-inductance of a
second similar coil of 500 turns will be
22. The resistance of a wire is 5 W at 50° C and 6 W at
100° C, the resistance of the wire at 0° C will be a.74 mH b. 75 mH c. 76 mH d. 77 mH
a. 4 W b. 3 W 30. Two inductors L1 and L2 are connected in parallel and
c. 2 W d. 1W a time varying current flows as shown in figure. The
ratio of currents i1 : i 2 at any time t is
23. An electric current is passed through a circuit
L1
containing two wires of same material connected in
parallell. If the lengths and radii of the wires are in the i i1
4 2 i
ratio of and , then the ratio of the currents passing
3 3 i2 L2
through the wire will be
1 L2 L1 L22 L12
a. 3 b. a. b. c. d.
3 L1 L2 (L1 + L2 )2 (L1 + L2 )2
8 1
c. d. l
9 2 31. The intensity at a point where the path difference is (
6
24. A battery of 10V and internal resistance 0.5 W is l = wavelength of light) is I. If I 0 is the maximum
connected in parallel with a battery of 12 V and intensity, then I / I 0 is equal to
internal resistance 0.8 W. The terminals are connected 3 1 3 1
by an external resistance of 20W. The current flowing a. b. c. d.
2 2 4 2
through the 20W resistance is
a. 0.75 A b. 1.74 A 32. If a thin prism of glass is dipped into water, then
c. 0.53 A d. 1.21 A minimum deviation (w.r.t. air) of light produced by
prism will be [(g m a = 3 / 2) and (a m w = 4 / 3)]
25. A magnetic field of ( 4 ´ 10- 3 k$ )T exerts a force of 1 1 1
( 4 i$ + 3 $j) ´ 10- 10 N on a particle having charge of a.
2
b.
4
c. 2 d.
5
1´ 10- 9 C and moving in xy -plane. Find the velocity of
33. A ray of light passes from vacuum into a medium of
the particle. refractive index m, the angle of incidence is found to
a. ( 75 $i + 100 $j ) ms- 1 be twice the angle of refraction.
b.( 75 $i - 100 $j ) ms- 1 Then, angle of incidence is
c. ( - 75 $i + 100 $j ) ms- 1 æm ö
a. cos- 1 ç ÷ b. sin- 1 (m )
d. ( - 75 $i - 100 $j ) ms- 1 è 2ø
-1 æm ö æm ö
26. A voltmeter has resistance G ohm and range of V c. sin ç ÷ d. 2 cos- 1ç ÷
è 2ø è 2ø
volt. The value of resistance used in series to convert
into a voltmeter of range nV volt is 34. For an optical arrangement shown in figure. Find the
a. nG b. (n - 1) G position of image.
G G
c. d.
n n -1 m1=1 m2=1.33
1 cm
27. The earth’s magnetic field at a certain place has a O C
horizontal component of 0.3 G and total strength
0.5 G. Find angle of dip. 20 cm
-1æ 4ö -1æ 3ö
a. d = tan ç ÷ b. d = tan ç ÷ 40 cm
è 3ø è 4ø
æ 5ö æ 3ö a. + 32 cm b. + 0.6 cm
c. d = tan- 1ç ÷ d. d = tan- 1 ç ÷
è 3ø è5ø c. - 6 cm d. 0.5 cm
1232 SELF STUDY GUIDE BITSAT

35. The energy flux of sunlight reaching the surface of the The effective decay constant of the nuclide is l, where
earth is 1.39 ´ 103 Wm - 2. If the average wavelength a. l = l1 + l 2 b. l = 2 ( l1 + l )
of sunlight is 550 nm, then the number of photons per 1 1 1
c. = + d. l = l1l 2
square metre incident on the earth per second are l l l2
a. 2 ´ 1021 b. 3 ´ 1021
39. A junction diode is connected to a 10 V source and
c. 4 ´ 1021 d. 4 ´ 1019
103 W rheostat figure. The slope of load line on the
27 characteristics curve of diode will be
36. If radius of the Al13 nucleus is estimated to be
3.6 fermi, then the radius of 125 Te52 nucleus will be
nearly 103W
a. 5 fermi b. 4 fermi c. 8 fermi d. 6 fermi
37. The binding energy of an electron in the ground state 10V
of He is equal to 24.6 eV. The energy required to -2 -1
remove both the electrons is a.10 AV b. 10- 3 AV - 1
a. 49.2 eV b. 24.6 eV c. 10- 4 AV - 1 d. 10- 5 AV - 1
c. 38.2 eV d. 79.0 eV
40. If minimum voltage in an AM wave was found to be 2V
38. A radioactive nucleus can decay simultaneously by and maximum voltage 10V. Find % modulation index.
two different processes which have decay constant l1 a. 80% b. 66.67% c. 64.25% d. 76.25%
and l2.

PART II
Chemistry
41. For the reaction, 2A( g ) + B ( g )
3C ( g ) + D ( g ), 2
l 46. The compounds which are functional isomers from the
moles each of A and B were taken into a flask. The following, are
following must always be true when the system O O
attained equilibrium. O
a. [ A] < [B] b. [ A] = [B] c. [B] = [C] d. [ A] > [B] a. and b. O and O

42. The pH at which an acid indicator with K a = 1´ 10- 5 O


changes colour when the indicator is 1´ 10- 3 M, is OH O
c. and d. and
a. 4 b. 3 c. 5 d. 6 O

43. Which of the following reactions does not involve OH


oxidation-reduction? 47. At which of the following four conditions will the
I. 2Cs + 2H2O ¾® 2CsOH + H2 density of nitrogen be the largest?
II. 2CuI2 ¾® 2CuI + I2 a. 273 K and 2 atm b. 546 K and 1 atm
III. NH4Br + KOH ¾® KBr + NH3 + H2O c. 546 K and 2 atm d. STP
IV. 4KCN + Fe(CN)2 ¾® K 4[Fe(CN)6] CH2 ¾ CH¾ CH2 ¾ CH2
a. I and II b. I, III and IV 48. The IUPAC name of ½ ½ ½ is
c. I and III d. III and IV NH2 CH3 NH2
44. 1 L of N / 2 HCl solution was heated in a beaker. When a. 3-methylbutane-1,4-diamine
the volume was reduced to 600 mL, 9.125 g of HCl b. 3-(aminomethyl) butanamine
was lost out. The new normality of solution is c. 2-(aminomethyl) butan-4-amine
approximately d. 2-methylbutane-1, 4-diamine
a. 0.2 b. 0.4 c. 0.8 d. 0.7 49. The vapour pressure of pure water at 25°C is 23 torr.
45. Which of the following is the strongest Bronsted acid? Then, the vapour pressure of 100 g of water to which
NH2 NH2 OH OH 100 g of C6H12O6 has been added, is
a.15.6 torr
b. 21.3 torr
a. b. c. d.
c. 26.7 torr
d. 24.8 torr
PRACTICE SET 4 1233
50. The correct order of magnetic moment (in BM) of the 56. The gold numbers of protective colloids A, B , C and D
following is are 0.04, 0.002, 10 and 25, respectively. The
a. [Fe(CN)6] 4 - > [MnCl4] 2 - > [CoCl4] 2 - protective powers of A, B , C and D are in the order
b. [MnCl4] 2 - > [Fe(CN)6] 4 - > [CoCl4] 2 - a. D > C > A > B b. B > C > A > D
c. [MnCl4] 2 - > [CoCl4] 2 - > [Fe(CN)6] 4 - c. D > C > B > A d. B > A > C > D
d. [Fe(CN)6] 4 - > [CoCl4] 2 - > [MnCl4] 2 - 57. Ammonia, manufactured by the Haber’s process is
usually carried out at about 500°C. If a temperature of
51. What is the product of the following reaction?
about 250 °C was used instead of 500°C, then
(I) Diethyl ether a. no ammonia would be formed at all
+ 2CH3MgBr Product
(ii) H3O+ b. the percentage of ammonia in the equilibrium mixture
O O would be too low
c. a catalyst would be of no use at all at this
a. CH3OCH2CH2CH2CH2 CHCH3 temperature
½
d. the rate of formation of ammonia would be too slow
OH
CH3 58. The edge length of a body-centred cubic unit cell is
½ 390 pm. If the radius of the cation is 150 pm, then, the
b. HOCH2CH2CH2CH2 ¾ C ¾ OH radius of the anion is
½ a. 180.3 pm b. 187.7 pm
CH3
c. 193.4 pm d. 162.4 pm
c. HO ¾ CHCH2 CH2CH2 C H¾ OH
½ ½ 59. For the reaction 2NO2 ¾® 2NO + O2,
CH3 CH3 activation energy is 110 kJ/mol. At 400°C, the rate
d. HOCH2CH2CH2CH2 CHOCH3 constant is 7.8 mol- 1 L s - 1. The value of rate constant
½ at 430°C is
CH3
a. 16 mol- 1 L s- 1 b. 18 mol- 1 L s- 1
52. Which of the following alkyl halide undergo c. 13 mol- 1 L s- 1 d. 19 mol- 1 L s- 1
rearrangement in SN1reaction?
60. Choose the correct statement from the following.
Cl I a. Both CH3 I and CF3 I reacts with Na + [Mn(CO)5] - and
a. b. forms I+ [Mn(CO)5] -
CH3 b. Both CH3 I and CF3 I reacts with Na + [Mn(CO)5] - and
CH3 for [CH3] + [Mn(CO)5] -
c. CH3 I reacts with Na + [ Mn(CO)5] - forms
c. CH3 C CH CH3 d. All of these
[CH3] + [Mn(CO)5] - while CF3I forms I+ [Mn(CO)5] -
CH3 I d. CH3 I reacts with Na + [Mn(CO)5] - forms
I+ [Mn(CO)5] - while CF3 I forms [CF3] + [Mn(CO)5] -
53. The ionic equivalent conductivity of C2O24 - , K + and
61. 6 ´ 1024 atoms of an element weigh 200 g. If this
Na + ions are x , y , z S cm 2 eq- 1 respectively. Then,
element form homodiatomic gas, then the molar mass
L°eq of ( NaOOC ¾ COOK ) is of gas is
y z
a. x + y + z b. x - + a. 30 b. 40
z 2 c. 20 d. 45
y z y z
c. x + + d. x + - 62. The correct decreasing order of the following towards
2 2 2 2
nucleophilic addition reaction is
54. The decreasing order of basic character of the
transition metal monoxides is CHO CHO CHO CHO

a. VO > CrO > TiO > FeO b. TiO > FeO > VO > CrO Cl

c. CrO > VO > FeO > TiO d. TiO > VO > CrO > FeO
Cl
55. At 25°C in the saturated solution of Ag2CrO4 , if
-5
concentration of Ag ion is 1.0 ´ 10 mol/ L, then the
+ Cl
solubility product of Ag2CrO4 at 25°C, is I II III IV

a. 5 ´ 10-3 mol2 L-3 b. 5 ´ 10-15 mol2 L-3 a. IV > III > II > I b. II > III > IV > I
c. 5 ´ 10-16 mol2 L-3 d. 5 ´ 10-14 mol2 L-3 c. I > II > III > IV d. I > IV > III > II
1234 SELF STUDY GUIDE BITSAT

63. D-glucose and D-fructose can be differentiated by 71. The correct decreasing order of melting points of
a. Tollens reagent b. Benedict’s test calcium halides is
c. Br2 /H2O d. Fehling’s solution a. CaF2 > CaBr2 > CaCl2 > CaI2
64. MnO-4 is pink in colour, though Mn is in +7 oxidation b. CaF2 > CaCl2 > CaBr2 > CaI2
state. It is due to c. CaCl2 > CaBr2 > CaI2 > CaF2
a. colour given by oxygen to it d. CaBr2 > CaF2 > CaCl2 > CaI2
b. charge transfer when oxygen gives its electron to Mn 72. The product(s) of the following reaction can be best
making it Mn (VI) coloured
described as
c. charge transfer when Mn gives its electron to oxygen
d. None of the above is correct
S S S S

S
65. Which of the following will not give Hofmann
bromamide reaction?
O O (x) a. b. c. d.
½½ ½½ a. a single enantiomer
a. Ph ¾ C ¾ NH2 b. CH3 ¾ C ¾ NH¾ Br
b. a pair of diastereomers
O O c. a racemic mixture
½½ ½½ d. an achiral molecule
c. CH3 ¾ C ¾ NH2 d. Ph ¾ C ¾ NH¾ Ph
73. When 1 mole of 1,2-dibromopropane is treated with
66. The first IP of lithium is 5.41 eV and electron gain x moles of NaNH2 followed by ethyl bromide gives a
enthalpy of Cl is - 3.61eV. DH in kJ mol- 1 for the pentyne. The value of x is
reaction Li ( g ) + Cl( g ) ¾® Li+ ( g ) + Cl- ( g ), is a. one b. three
a. 173.7 J b. 173.7 kJ c. four d. six
c. 185.4J d. 185.4 kJ 74. Calculate DH 2 - DH1 of the following two reactions:
67. Two longest wavelengths (in nanometre) in the Lyman (i) Propene + H2 ¾® Propane ; DH1
series of the hydrogen spectrum, are (ii) Cyclopropane + H2 ¾® Propane ; DH2
a. 121.5 and 102.6 b. 201.5 and 202.6 a. 2 BEC ¾ C - BEC== C b.2BEC== C - BEC ¾ C
c. 140.0 and 150 d. 6 and 0 c. BEC == C d. 0
68. Two liquids A and B made up of same atoms and both 75. Choose the correct statement from the following.
are diamagnetic. A turns blue litmus to red but B does
a. ClF2- is linear but ClF2+ is V-shaped
not. Also
b. Both ClF2- and ClF2+ are linear
A + Kl + H+ ¾ Starch
¾ ¾® Blue colour
c. Both ClF2- and ClF2+ are V-shaped
B + Kl + H+ ¾ Starch
¾ ¾® No colour d. ClF2+ is linear but ClF2- is V-shaped
The liquids A and B respectively are 76. Excess of KI and dil. H2SO4 were mixed in 50 mL
a. H2O, H2O2 b. H2O2, H2O
H2O2. The I2 liberated requires 20 mL of 0.1 N
c. Na 2SO3, Na 2S2O3 d. Na 2S2O3, Na 2SO3 Na 2S 2O3 . The strength of H2O2 in g/L is
69. Which of the following is not an aromatic compound? a. 0.42 g
b. 0.42 mg
a. + + c. + c. 0.68 g
b. – d.
d. 0.68 mg
77. Light of wavelength l shines on a metal surface with
70. Consider the following reaction, intensity E , and the metal emits X electrons per
CH 3 MgI H O + second of average energy Y . What will happen to X
CH3 ¾ C ºº N ¾¾® A ¾®
3
B and Y , if E is doubled?
Et 2 O
a. X will be doubled andY will become half
What is the major product B? b. X will remain same andY will be doubled
O c. Both X and Y will be doubled
½½ d. X will be doubled butY will remain same
a. CH3 ¾ C ¾ CH3 b. CH3 ¾ CH2 ¾ NH¾ CH3
78. In P4 O6, the number of oxygen atoms bonded to each
N¾ H O P-atom is
½½ ½½ a. 1 b. 2
c. CH3 ¾ C ¾ CH3 d. CH3 ¾ CH2 ¾ C ¾ OH c. 3 d. 4
PRACTICE SET 4 1235
79. The correct increasing order of acidic character of the 80. Which of the following contains greatest number of
following is N-atoms?
O a. 1.0 mole of NH4Cl
O
b. 6.02 ´ 1023 molecules of NO2
Ph C NH2 NH PhNH2
c. 22.4 L nitrogen gas at STP
I II O III d. 500 mL of 2.0 M NH3

a. III < I < II b. II < I < III c. I < III < II d. II < III < I

PART III
a. English Proficiency
Directions (Q. Nos. 81 to 84) Read the passage and 86. P : she gave her old coat
then choose the most suitable option from the given ones. Q : to a begger
It was a very cold evening and so few people were seen out R : the one with the brown fur on it
on the streets. I did not go out myself although it was my S : shivering with cold
habit not to keep indoors after sunset. So. I closed all the The proper sequence should be
doors and windows of my room, took the book which had
a. SQRP b. SPRQ
been lying opened on the table and tried to read it. The cold
c. PRQS d. PSQR
was getting so severe that I started shivering, so I wrapped
myself up with a bigger blanket. But I could not continue 87. S1 : I last visited Kandy almost 10 years ago.
reading the book because I was nearly rendered incapable S6 : Set around low, forested hills. with the Mahavelli
of turning the pages. river flowing nearby, the landscape has an
81. The author’s habit was to instance yet tranquil beauty.
a. read in the evening b. sleep in the evening P : The bomb blasts and ugly face of ethnic conflict
c. go out in the evening d. play in the evening have not robbed the place of its gracious pace
82. If it was not so cold the author would have of life
a. liked to work Q : Theartificial lake, which dominates the landscape
b. liked to sit and look out to the streets of the city, was built by its last king in 1806
c. liked to read R : Located in central Sri Lanka, at an elevation of
d. liked to have a stroll about 1600 ft, Kandy was the last Sinhala outpost
of autonomy, resisting both Protuguese and Dutch
83. The author could not continue reading the book rule before it succumbed to the British in 1818
because
S : Most famous for its Temple of the Tooth, a
a. he did not like to b. he was feeling tired
golden pagoda, elegant and imperaturable city
c. he was feeling very cold d. the lights had gone off
The proper sequence should be
84. There were not many people outside because
a. RPQS b. SPRQ c. QSPR d. PSQR
a. it was a rainy evening
b. it was a cold evening 88. S1 : The status of women in our country is, on the
c. it was a dark evening whole, far from high.
d. there was heavy show outside S6 : Education can lift these moment out of the depths
Directions (Q. Nos. 85 to 88) Rearrange the jumbled up of misery and ignorance into which they have sunk.
parts of the sentence and choose the appropriate option. P : But the plight of women in village is still miserable
85. With an unsteady hand Q : The educated women in cities enjoy equality with
P : on my desk the men folk.
Q : from his pocket R : The moment for the freedom and rights of women
R : he took an envelope has certainly been steadily gaining momentum
S : and threw it S : Their education has been throughly neglected.
The proper sequence should be The proper sequence should be
a. QRPS b. QRSP c. RQSP d. RQSP a. RPQS b. RQPS c. SQPR d. SPQR
1236 SELF STUDY GUIDE BITSAT

Directions (Q. Nos. 89-92) In the sentences given 93. Five year ago today, I am sitting in a small Japanese
below there is an error is one of the parts. Choose that part car driving across Poland toward Berlin.
as your answer. a. was sitting b. sat
89. She misplaced her spectacle (a)/ is now feeling (b)/ c. have been sitting d. No improvement
great difficulty in studying. (c)/ No error (d) 94. Sumit had told me that he hasn’t done it yet.
90. Arabian Nights are (a)/a collection of (b)/ very a. told
interesting episodes of adventure. (c)/No error (d) b. tells
91. I hope to visit (a)/my uncle only next year (b)/during c. was telling
summer vacations. (c)/No error (d) d. No improvement

92. Ration has run out (a)/and the District Magistrate (b)/ 95. The record for the biggest tiger hunt has not been met
has been informed. (c)/No error (d) since 1911 when Lord Hardinge, then Viceroy of India,
shot a tiger measured 11 feet and 6 inches.
Directions (Q. Nos. 93 to 95) The underlined part of the a. improved b. broken
sentence may need an improvement. Choose the option that
c. bettered d. No improvement
suits the improvement in best way.

b. Logical Reasoning
96. In a class, boy stand in a single line. One of the boys in 102. Find out which of the answer figures (a), (b), (c) and
nineteenth in order from both the ends. How many (d) completes the figures matrix.
boys are there in the class?
Problem Figure
a. 27 b. 37 c. 38 d. 39
97. Find out the odd figure

a. c. d. ?
b.

98. Find out next term in the series.


Answer Figures
5, 6, 9, 14, 21, ?
a. 28 b. 30 c. 31 d. 29
99. How many triangles are there in the given figure?

a. b. c. d.

103. The missing in the figure shown below is

a. 5 b. 6 c. 7 d. 8 30 11 ?

100. If the day before yesterday was Friday, what day will 36 22 15 18 44 60
two days after the day after tomorrow be ?
a. 45 b. 54
a. Saturday b. Thursday c. Friday d. Sunday
c. 72 d. 90
101. In the problem figure, which figure will replace the
question mark (?) from the answer figures ? 104. Figure (X) is embedded in one of the four alternative
figures (a), (b), (c) and (d).
Problem Figures
Find the alternative which contains figures (X) as its part.
?
S S S S
S

S
(i) (ii) (iii) (iv)
(x) a. b. c. d.
Answer Figures
105. Find the water image of the word given below
S TOP
S S
a. ^ob b .toq
a. b. c. d. c. tob d. ^op
PART IV
Mathematics
2
106. The set of all values of x satisfying x logx (1 - x ) = 9, is 114. If the circle having radii r1 and r 2 intersect orthogonally,
a. a subset of R containing N then the length of their common chord is
b. a subset of R containing Z (set of all integers) 2r1r2 r12 + r22 r1r2 r12 + r22
c. a finite set containing atleast two elements a. b. c. d.
r12 + r22 2r1r2 r12 + r22 r1r2
d. a finite set
107. If f (x ) = sin6 x + cos 6 x , then range of f (x ) is 115. AB is double ordinate of the parabola y 2 = 4ax .
é1 ù é1 3ù Tangents drawn to the parabola at A and B meet
a. ê , 1ú b. ê ,
4 úû
Y -axis at A1 and B1, respectively. If the area of
ë4 û ë4
trapezium AA1B1B is equal to 12a 2, then the angle
é3 ù
c. ê , 1ú d. None of these subtended by A1B1 at the focus of the parabola is equal
ë4 û
to
108. If a sin x + b cos(x + q ) + b cos(x - q) = d , then the a. 2 tan-1 3 b. tan-1 3 c. 2 tan-1 2 d. tan-1 2
minimum value of | cos q | is
(a - b )m
2 2

a.
1
d 2 - a2 b.
1
d 2 - a2
116. The line y = mx - is normal to the ellipse
2 |b | 2 |a | a 2 + b 2m 2
1
d 2 - a2 x2 y2
c. d. None of the above + = 1 for all values of m belonging to
2 |d | a2 b2
109. The set of values of x in ( 0, p) satisfying the equation a. (0, 1) b. ( 0, ¥ ) c. R d. None
1 + log2 sin x + log2 sin 3x ³ 0, is 117. Let E c
denotes the complement of an event E . If
æ 2p 3p ù æp 2p ö E , F , G are pairwise independent events with P (G ) > 0
a. ç , b. ç , ÷
è 3 4 úû è2 3ø and P (E Ç F Ç G ) = 0, then P (E c Ç F c / G ) equals
æ p ö æ 2p ö æp 2p ö a. P (E c ) + P (F c ) b. P (E c ) - P (F c )
c. ç 0, ÷ È ç , p÷ d. ç , ÷
è 2ø è 3 ø è2 3ø c. P (Ec ) - P (F ) d. P (E ) - P (F c )
110. If cot -1( cos a ) - tan-1( cos a ) = x, then sinx equals 118. Let M and N be two 3 ´ 3 non-singular and
a 2 a
2 skew-symmetric matrices such that MN = NM . If P T
a. tan b. cot
2 2
a denotes the transpose of P, then
c. tan a d. cot M 2N 2(M T N )-1(MN -1)T is equal to
2
a. M 2 b. -N 2 c. -M 2 d. MN
111. In DABC, if P , Q, R divides sides BC , AC and AB
area of DPQR 1 119. Consider the set A of all determinants of order 3 with
respectively in the ratio k : 1 and = ,
area of DABC 3 entries 0 or 1 only. Let B be the subset of A consisting
then k is equal to of all determinants with values -1. Then,
a. C is empty
a. 1/3 b. 2
b. B has as many elements as C
c. 3 d. None of these
c. A = B È C
112. A line is drawn perpendicular to line y = 5x , meeting d. B has twice as many elements as C
the coordinate axes at A and B. If the area of DOAB is
æ 30 ö æ 30 ö æ 30 ö æ 30 ö æ 30 ö æ 30 ö
10 sq units, where O is the origin, then the equation of 120. The value of ç ÷ç ÷ -ç ÷ç ÷ + ç ÷ç ÷
drawn line is è 0 ø è 10 ø è 1 ø è 11 ø è 2 ø è 12 ø
a. 3x - y = 9 b. x + 5y = 10 æ 30 ö æ 30 ö æn ö
-L + ç ÷ ç ÷ where ç ÷ = nCr , is
c. x + 4y = 10 d. x - 4y = 10 è 20 ø è 30 ø èr ø
113. If the slope of one of the lines represented by æ 30ö æ 30ö æ 60ö æ 31ö
a. çç ÷÷ b. çç ÷÷ c. çç ÷÷ d. çç ÷÷
ax + 2hxy + by = 0 is the square of other, then
2 2
è 10 ø è 15 ø è 30ø è10ø
a + b bh 2
+ equals 121. Let S = {1, 2, 3, 4}, then the total number of unordered
h ab
pairs of disjoint subsets of S is equal to
a. 4 b. 6
a. 25 b. 34 c. 42 d. 41
c. 8 d. None of these
1238 SELF STUDY GUIDE BITSAT

122. If a1, a 2, ..., an are positive real numbers whose |2 -x |


132. The critical points of f (x ) = are
product is a fixed number c, then the minimum value x2
of a1 + a 2 + ... + an - 1 + 2an is a. x = 0, 2 b. x = 2, 4
a. n ( 2c )1/n b. (n + 1)c 1/n c. x = 2, - 4 d. None of these
c. 2nc 1/n d. (n + 1)( 2c )1/n
133. The length of the longest interval in which the function
123. For a positive integer n, let 3 sin x - 4 sin3 x is increasing, is
1 1 1 1
a (n ) = 1 + + + + + n . Then, a. p/3 b. p/2 c. 3p/ 2 d. p
2 3 4 L (2 ) - 1
a. a( 200) < 100 b. a( 200) ³ 100 134. If 27a + 9b + 3c + d = 0, the equation
c. a( 200) £ 100 d. a( 200) > 100 4ax 3 + 3bx 2 + 2cx + d = 0 has atleast one real root
124. Let z be a complex number such that the imaginary lying between
part of z is non-zero and a = z 2 + z + 1is real. Then, a a. 0 and 1 b. 1 and 3
cannot take the value c. 0 and 3 d. None
a. -1 b. 1/3 c. 1/2 d. 3/4 135. If the parabolas y = x 2 + ax + b and y = x (c - x ) touch
125. If a and b are the roots of x 2 + px + q = 0 and a 4 , b4 each other at the point (1, 0), then a + b + c equals
are the roots of x 2- rx + s = 0, then the equation of a. 1 b. -1 c. 0 d. None
x 2 - 4qx + 2q 2 - r = 0 has always 136. Let f be a differentiable function satisfying
a. one positive and one negative root [f (x )]n = f (nx ) for all x Î R . Then, f ¢(x ) f (nx ) equals
b. two positive roots a. f (x ) b. 0
c. two negative roots c. f (x ) f ¢(nx ) d. None of these
d. can’t say anything
126. If x satisfies | x - 1| + | x - 2 | + | x - 3 | ³ 6, then 137. If f (x - y ), f (x ) f ( y ) and f (x + y ) are in A.P. for all
x , y Î R and f ( 0) = 0, then
a. 0 £ x £ 4 b. x £ - 2 or x ³ 4
a. f ¢( - 2) = f ¢( 2) b. f ¢( -3) = - f ¢( 3)
c. x £ 0 or x ³ 4 d. None of these
c. f ¢( -2) + f ¢( 2) = 0 d. None of these
127. The standard deviation of some temperature data
(in° C) is 5. If the data were converted into ° F, then [x ]
138. If f (x ) = , x ¹ 0, where [ ] denotes the greatest
the variance would be |x |
a. 81 b. 57 c. 36 d. 25 integer function, then f ¢(1) equals
128. The solution of the differential equation a. -1 b. 1
c. Does not exist d. None of these
xdy - ydx = x + y dx , is 2 2

139. The value of lim { 3 n 2 - n 3 + n}, is


a. x + x 2 + y 2 = Cx 2 b. y - x 2 + y 2 = Cx n ®¥
1 -1 2 -2
c. x - x 2 + y 2 = Cx d. y + x 2 + y 2 = Cx 2 a. b. c. d.
3 3 3 3
129. Let the straight line x = b divide the area enclosed by
140. The domain of definition of
y = (1 - x )2, y = 0 and x = 0 into two parts R1 ( 0 £ x £ b )
1 f (x ) = sin-1{log2(x 2 + 3x + 4 )} is
and R 2 (b £ x £ 1) such that R1 - R 2 = . Then, b equals
4 a. [ -4, - 1/ 4] b. [ -3, - 1/ 3]
a.
3
b.
1
c.
1
d.
1 c. [ -2, - 1] d. None of these
4 2 3 4
100 141. If the range of a random variable X is 0, 1, 2, 3,... with
130. The value of ò [tan-1 x ]dx is æ k + 1ö
0 P (X = k ) = ç K ÷ a for k ³ 0, then a equals
a. 100 b. 100 - tan 1 -1 è 3 ø
c. 100 - tan 1 d. None of these a. 2/3 b. 4/9 c. 8/27 d. 16/81
x -1 142. LetS (k ) = 1 + 3 + 5 + ... + ( 2k - 1) = 3 + k . Then, which
2
131. ò (x + 1) x + x + x3 2
dx equals
one of the following is true?
a. Principle of mathematical induction can be used to
æ x + x + 1ö
2 æ x + x + 1ö
2
a. tan-1çç ÷+C
÷ b. 2 tan-1çç ÷+C
÷
prove the formula
è x ø è x ø b. S (k ) Þ S (k + 1)
æ x 2
+ x + 1ö c. S (k ) Þ
/ S (k + 1)
c. 3 tan-1çç ÷+C
÷ d. None of these
è x ø d. S(1) is correct
PRACTICE SET 4 1239
143. The false statement in the following is 147. A vector which makes equal angles with the vectors
a. p Ù (~ p ) is a contradiction 1 $ 1
{ i - 2$j + 2 k$ ), ( -4 i$ - 3 k$ ), $j, is
b. ( p ® q ) « (~ q ® ~ p ) is a contradiction 3 5
c. ~ (~ p ) « p is tautology a. 5$i + $j + 5k$ b. -5$i + $j + 5k$
d. p Ù(~ p ) is a tautology c. 5$i - $j - 5k$ d. 5$i + $j - 5k$
144. Let f : R ® R be given by f (x ) = [x 2] + [x + 1] - 3, 148. The vectors a = x i$ + (x + 1)$j + (x + 2)k$ ,
where [x ] denotes the greatest integer less than or b = (x + 3 )$i + (x + 4 )$j + (x + 5 )k$ and
equal to x. Then, f (x ) is
c = (x + 6 )$i + (x + 7)$j + (x + 8 ) k$ are coplanar for
a. many-one and onto b. many-one and into
c. one-one and into d. one-one and onto a. all values of x b. x < 0
c. x > 0 d. None of the above
145. The relation on the set A = {x : | x | < 3, x Î Z } is
defined by R = {(x , y ): y = | x |, x ¹ - 1}. Then, the 149. If the direction cosines of a straight line are k , k , k , then
number of elements in the power set of R is a. k > 0 b. 0 < k < 1
a. 32 b. 16 c. 8 d. 64 1 -1
c. k = 1 d. k = or
146. If A = { q : cos q > - 1/ 2, 0 £ q £ p} and 3 3
B = { q : sin q > 1/ 2, p/ 3 £ q £ p}, then x - 2 y -1 z + 2
150. Let the line = = lie in the plane
a. A Ç B = { q : p / 3 £ q £ 2p / 3} 3 -5 2
b. A Ç B = { q : - p / 3 £ q £ 2p / 3} x + 3 y - az + b = 0. Then, ( a, b) equals
c. A È B = { q : - 5p / 6 £ q £ 5p / 6} a. ( 6, - 17) b. ( -6, 7)
d. A È B = { q : 0 £ q £, p / 6 } c. (5, - 15) d. ( -5, 5)

Answers
Physics
1. (d) 2. (a) 3. (a) 4. (d) 5. (c) 6. (b) 7. (d) 8. (d) 9. (a) 10. (a)
11. (c) 12. (b) 13. (b) 14. (c) 15. (a) 16. (a) 17. (b) 18. (c) 19. (b) 20. (a)
21. (c) 22. (a) 23. (b) 24. (c) 25. (c) 26. (b) 27. (a) 28. (b) 29. (b) 30. (a)
31. (c) 32. (b) 33. (d) 34. (a) 35. (c) 36. (d) 37. (d) 38. (a) 39. (b) 40. (b)

Chemistry
41. (a) 42. (c) 43. (d) 44. (b) 45. (d) 46. (a) 47. (a) 48. (d) 49. (b) 50. (c)
51. (b) 52. (d) 53. (c) 54. (d) 55. (c) 56. (d) 57. (d) 58. (b) 59. (b) 60. (c)
61. (b) 62. (b) 63. (c) 64. (b) 65. (d) 66. (b) 67. (a) 68. (b) 69. (a) 70. (a)
71. (b) 72. (b) 73. (b) 74. (a) 75. (a) 76. (c) 77. (d) 78. (c) 79. (a) 80. (c)

English Proficiency
81. (c) 82. (d) 83. (c) 84. (b) 85. (d) 86. (c) 87. (d) 88. (b) 89. (a) 90. (a)
91. (c) 92. (a) 93. (a) 94. (b) 95. (b)

Logical Reasoning
96. (b) 97. (d) 98. (b) 99. (c) 100. (b) 101. (d) 102. (a) 103. (c) 104. (d) 105. (a)

Mathematics
106. (d) 107. (a) 108. (a) 109. (a) 110. (a) 111. (b) 112. (b) 113. (b) 114. (a) 115. (c)
116. (c) 117. (c) 118. (c) 119. (b) 120. (a) 121. (d) 122. (a) 123. (d) 124. (d) 125. (a)
126. (c) 127. (a) 128. (d) 129. (b) 130. (c) 131. (b) 132. (d) 133. (a) 134. (c) 135. (c)
136. (c) 137. (a) 138. (c) 139. (a) 140. (c) 141. (b) 142. (b) 143. (b) 144. (b) 145. (b)
146. (a) 147. (b) 148. (a) 149. (d) 150. (b)
Practice Set 5
BITSAT
Instructions
1. There are 150 questions in all. The number of questions in each part is as follows Subject No. of Questions
Part I Physics 1-40
Part II Chemistry 41-80
Part III (a) English Proficiency 81-95
(b) Logical Reasoning 96-105
Part IV Mathematics 106-150
2. All questions are multiple choice questions with four options, only one is correct.
3. Each correct answer fetches 3 marks while incorrect answer has a penalty of 1 mark.

PART I
Physics
1. Charges + q and -q are placed at points A and B 3. The wavelength of the first line of Lyman series for
respectively, which are a distance 2L apart. C is the hydrogen atom is equal to that of the second line of
mid-point between the points A and B. The work done Balmer series of hydrogen like ion. The atomic
in moving a charge + Q along the semi-circle CRD is number Z of hydrogen like atom is
-q Q -q Q a. 2 b. 3 c. 4 d. 1
a. b.
6pe0L 4pe0L 4. In a Young’s double slit experiment, the angular position
qQ -Q q ( q) of a bright fringe having intensity one forth of the
c. d.
6pe0L 2pe0L maximum intensity is given by

2. In the given figures, which one of the diode shown is


reverse biased? Imax
4
+5 V q
+10 V
R
a. b.
+5 V R

æ l ö æ l ö
a. sin-1 ç ÷ b. sin-1 ç ÷
c. –10 V d. R è 4d ø è3d ø
R
æ l ö æ lö
c. sin-1 ç ÷ d. sin-1 ç ÷
–5 V –10 V è 2d ø èd ø
PRACTICE SET 5 1241
5. A tank of height 33.25 cm is completely filled with a 11. Figure shows an arrangement known as a Helmholtz
liquid of index of refraction 1.33. A concave mirror of coil. It consists of two circular coaxial coils each of 200
focal length 15.0 cm is placed above the tank as turns and radius R = 25.0 cm separated by a distance
shown in figure. An object is placed at the bottom of S = R . The two coils carry equal currents I = 12.2 mA in
the tank on the axis of concave mirror and its image is the same direction. Find the magnitude of the net
formed at a distance 25.0 cm below the surface of the magnetic field at P , mid-way between the coils.
liquid. The focal length of the mirror is Y
I I
15 cm

X
P
25 cm 33.25 cm R

a. 8.7 x 10-6 T b. 6.7 x 10-6 T


a. 10 cm b. 18.3 cm c. 20.3 cm d. 25.0 cm c. 5.7 x 10-6 T d. 10.7 x 10-6 T

6. In a series resonant LCR circuit, the voltage across R 12. A material B has twice the specific resistance of
is 100 V and R = 1kW with C = 2mF. The resonant another material A. A circular wire made of B has
frequency w is 200 rad/s. At resonance the voltage twice the diameter of a wire made of A. Then for the
across L is I
two wires to have the same resistance the ratio A of
a. 4 ´ 10-3 V b. 2 . 5 ´ 10-2 V IB
c. 40 V d. 250V their respective length must be
1 1
7. A 2700 W resistor and a 11
. mF capacitor are connected a. 2 b. 1 c. d.
2 4
in series across a generator (60 Hz, 120 V ). Determine
the average power consumed in the circuit. 13. In a potentiometer experiment, the balancing with a
a. 3 W b. 9 W c. 10 W d. 7 W cell is at length 240 cm on shunting the cell with a
resistance of 2W the balancing length 120 cm. The
8. Needles N1, N 2 and N 3 are made of ferromagnetic, a internal resistance of the cell is
paramagnetic and a diamagnetic substance, a. 1W b. 0 . 5 W c. 4 W d. 2 W
respectively. A magnet when brought close to them will
a. attract all three of them 14. At time t = 0, a battery of 10 V is connected across points A
b. attract N1 and N2 strongly but repel N3 and B shown in figure. If the capacitors have no change
c. attract N1 strongly, N2 weakly and repel N3 weakly initially at what time does the voltage across becomes 4V?
d. attract N1 strongly, but repel N2 and N3 weakly 2 MW 2 mF

9. A long straight wire of radius a carries a steady B


A
current I. The current is uniformly distributed across its
a 2 MW 2 mF
cross-section, the ratio of the magnetic field at and
2 a. 2s b. 3s
2a is c. 4s d. 5s
1 1
a. b. 4 c. 1 d. 15. A thin semicircular ring of radius r as shown in figure as a
4 2
positive charge q distributed uniformly over it . The net field
10. A current carrying closed loop in the form of a right angle E at the centre O is
isosceles DABC is placed in a uniform magnetic field acting j
along AB. If the magnetic force on the arm BC is F, then
force on arm AC is
A
r
i
O
q $ q $j
a. j b.
2p 2e0r 4p 2e0r 2
B C q $jd. - q $j
c. - 2 2
a. 2F b. - 2 F c. - F d. F 4p e0r 2p e0r 2
2
1242 SELF STUDY GUIDE BITSAT

16. The decreasing order of wavelength of infrared, 23. A ball is thrown from a field with a speed of 12.0 m/ s at
microwaves, ultraviolet and gamma rays is an angle of 45° with the horizontal. At what distance
a. infrared, microwaves, ultraviolet, gamma rays will it hit the field again? (Take, g = 10.0 m/ s 2 )
b. microwaves, infrared, ultraviolet, gamma rays a. 10 m b. 14.4 m
c. gamma rays, ultraviolet, infrared, microwaves c. 16.4 m d. 18.4 m
d. microwaves, gamma rays, infrared, ultraviolet
24. With what acceleration a should the box of figure descend,
17. Light with an energy flux of 18 W / cm 2 on a so that the block of mass M exerts a force mg /4 on the
non-reflecting surface at normal incidence. If the floor of the box?
surface has an area of 20 cm 2. Find the average force
exerted on the surface during a 30 min time span a
. ´ 10-6 N
a. 12 b. 2.4 ´ 10-6 N
c. 4.8 ´ 10-6 N d. 6.2 ´ 10-6 N
M
18. An anatomist is viewing heart muscles cells with a
microscope that has two selectable objectives with
refracting powers of 100 and 300 D. When she uses
the 100 D objective, the image of a cell subtends an 2g 3g g 3g
a. b. c. d.
angle of 3 ´ 10-3 rad with the eye. What angle is 5 5 2 4
subtended when she uses the 300 D objective?
25. A block slides down and inclined of 30° with an
a. 9 ´ 10-3 rad b. 3 ´ 10-3 rad acceleration of g/4. Find the kinetic friction coefficient.
c. 2 ´ 10-3 rad d. 10 ´ 10-3 rad 1 1 1 1
a. b. c. d.
2 3 4 3 5 3 6 3
19. A sheet of silver is illuminate by monochromatic
ultraviolet radiations of wavelength 1810 Å. What is 26. A small block of mass 100 g moves with uniform
the maximum energy of the emitted electron? speed in a horizontal circular groove, with vertical side
The threshold wavelength of silver is 2640 Å. walles of radius 25 cm. If the block takes 2.0 s to
a. 9 .16 eV b. 2 .16 eV complete one round, then find the normal contact
c. 3 .16 eV d. 4 .16 eV force by the side wall of the groove.
a. 1 N b. 0 . 25 N
20. Two radioactive materials X 1 and X 2 have decay c. 0 . 4 N d. 2 N
constants 5 l and l, respectively. If initially they have
the same number of nuclei, then the ratio of the 27. A body dropped from a height H reaches the ground
number of nuclei of X 1 to that of X 2 will be 1/e after a with a speed of 1. 2 gH . Calculate the work done by
time air friction.
e 1 1
a. b. l c. l d. a. - 0.28 mgh b. 0.28 mgh
l 2 4l
c. 0.56 mgh d. - 0.56 mgh
21. The period of oscillation of simple pendulum is
l 28. A body of mass 2 . 5 kg is subjected to the force as shown
T = 2p measured value of L is 20.0 cm known to
g in figure. Find the acceleration of the centre of mass.
1 mm accuracy and time for 100 oscillations of the 4N
pendulum is found to be 90 s using a wrist watch of 1s 6N
resolution. What is the accuracy in the determination 60°
of g?
a. 3 % b. 1 % c. 5 % d. 2 % 53°
22. Find the resultant of three vectors OA, OB and OC as 5N 37° 6N
shown in figure. Radius of the circle is R.
C
a. 2 m/s 2 b. 1.6 m/s 2
B
c. 3.2 m/s 2 d. None of these
45°
45° 29. A parallel beam of green light of wavelength 546 nm
A
O passes through a slit of width 0 . 40 mm. the
transmitted light is collected on a screen 40 cm away.
What will be the distance between two first order
minima?
a. R (1 + 2 ) b. R (2 + 2 ) a. 2.6 mm b. 1.8 mm
c. R (3 + 2 ) d. R (4 + 2 ) c. 1.7 mm d. 1.1mm
PRACTICE SET 5 1243
30. A 50 cm long wire of mass 20 g supports a mass of 1. 6 kg 35. The speed of transverse wave giving on a wire having
as shown in figure. Find the fundamental frequency of the a length 50 cm and mass 5.0 g is 80 m/ s. The area of
position of the string between the wall and the pulley. cross-section of the wire is 1. 0 mm 2 and its Young‘s
modulus is 16 ´ 1011 N / m 2 . Find the extension of the
wire over its natural length.
40 cm a. 0.04 mm b. 0.02 mm
10 cm c. 0.06 mm d. 1mm
15 kg
36. A sound wave of wavelength 40 cm travels in air. If the
a. 25 Hz b. 50 Hz difference between the maximum and minimum
c. 75 Hz d. 100 Hz pressures at a given point is 1´ 10-3 N/m 2. What will
be the amplitude of vibration of the particle of the
31. A cylinder is released from rest from the top of an
medium, the bulk modulus of air is 1. 4 ´ 10-10 N/m 2?
incline of inclination q and length l. If the cylinder rolls
a. 4. 4 ´ 10-10 m
without slipping, then what will be its speed when it
reaches the bottom? b. 3.3 ´ 10-10 m
2 4 c. . ´ 10-10 m
11
a. gl sin q b. gl sin q
3 3 d. 2. 2 ´ 10-10 m
6 37. A particle of mass 40 g executes a simple harmonic
c. gl sin q d. None of these
7 motion of amplitude 2.0 cm. If the time period is 0.20 s,
32. The gravitational field in a region is g given by find the total mechanical energy of the system.
F = 10 N ( $i + $j ) kg-1 . What will be work done by an a. 7.9 ´ 10-3 J
external agent to slowly shift a particle of mass 2 kg b. 8.9 ´ 10-3 J
from the point ( 0,0), to a point (5, 4 ) ? c. 9.9 ´ 10-3 J
a. - 180 J b. + 100 J d. 6.9 ´ 10-3 J
c. + 180 J d. - 100 J
38. The sound level at a point is increased by 30 dB. By
33. The liquids shown in figure in the two arms are what factor is the pressure amplitude increased?
mercury (specific gravity = 13.6) and water. If the a. 30 b. 25
difference of heights of the mercury columns is 2 cm. c. 32 d. 27
What will be the height h of the water column?
39. White light is mixture of light of wavelength between
400 nm to 700 nm. If this light goes through water
(m = 133
. ) what will be the limits of the wavelength
2 cm there?
A B a. ( 300 to 525) nm
b. (600 to 700) nm
c. ( 800 to 1000) nm
d. (1200 to 1400) nm

a. 27 cm b. 30 cm c. 40 cm d. 47 cm
40. What will be the moment of inertia of a solid cylinder of
mass M and radius R about a line parallel to the axis
34. A water drop of radius 10-2m is broken into 1000 equal of the cylinder and on the surface of the cylinder?
droplets. Calculate the gain in surface energy. Surface 7 5
tension of water is 0.075 N / m. a. MR 3 b. MR 2
2 2
a. 8 . 5 ´ 10-4 J b. 9 . 5 ´ 10-4 J 7 2
c. MR 2 d. MR 2
c. 10 ´ 10-4 J d. None of these 5 3
1244 SELF STUDY GUIDE BITSAT

PART II
Chemistry
41. In which of the following pairs, the two species are not 47. Which of the following dissolves in water but does not
iso-structural? give any oxyacid solution?
a. CO23 - and NO-3 b. PCl+4 and SiCl4 a. SO2 b. OF2
c. PF3 and BrF5 d. AlF63 - and SF6 c. SCl4 d. SO3

42. Which of the following statements regarding dacron is 48. Heat capacity (C p ) is an extensive property but
correct? specific heat (C ) is an intensive property.
I. Dacron is an example of polyester. The relation between C p and C for 1 mole of water is
II. In the manufacturing of dacron zinc acetate a. + 5.18 JK - 1 b. - 5.18 JK - 1
antimonytrioxide used as a catalyst. c. – 75.3 JK - 1 d.
III. Dacron fibres is increases resistant. + 75.3 JK - 1
IV. It is a polymer of natural rubber.
a. Only IV b. Both IV and I 49. Select the correct statement regarding the compound.
c. I, II and III d. None of these
43. Bombardment of aluminium by a-particle leads to its
artificial disintegration in two ways as shown. Products
X , Y and Z respectively are

4
(ii) 30
27
Al + He P+Y I. The IUPAC name of this compound is 1, 3-epoxy
13 2 15
propane.
II. The functional group of this compound is ether.
(i)
III. Root word in the IUPAC name of this compound is
prop.
30 30
Si +X Si +Z IV. This is a saturated open chain structure.
14 14
a.Only I b. I, II and III
a. proton, neutron, positron c. Only IV d. IV and III
b. neutron, proton, positiron
c. proton, positron, neutron 50. Aniline does not undergo Friedel-Crafts reaction due to
d. positron, neutron, proton a. lone pair nitrogen atoms activate the benzene ring
b. salt formation with aluminium chloride
44. Predict Kroll process in the following reaction, c. meta derivative formation
a. TiCl4(g ) + 2Mg ¾® Ti(s ) + 2MgCl2(l ) d. None of the above
b. Fe2O3 + 3C ¾® 3CO + 2Fe 51. The edges length of a face-centred cubic cell of an
1470 K
c. Al2O3× xH2O (s ) ¾¾® Al2O3 + x H2O (g ) ionic substance is 508 pm. If the radius of the cation is
110 pm, the radius of the anion is
d. Cr2O3 + 2Al ¾® 2Cr + Al2O3
a. 618 pm b. 144 pm
45. The complementary strand of DNA for the given single c. 288 pm d. 398 pm
stranded DNA sequence 52. What is the sweeteners value of the structure in
5¢—A—T—C—A—T—G—C— 3¢ is comparison to cane sugar?
a. 5¢—A—T—C—A—T—G—C— 3¢ CO
b. 5¢—T—A—G—T—A—C—G— 3¢ NH
c. 5¢—G—C—A—T—G—A—T— 3¢ SO2
d. 5¢—C—G—T—A—C—T—A— 3¢ a. 100 b. 500 c. 200 d. 550
46. The IUPAC name of the compound 53. Interstitial compounds are formed when small atoms
is are…… .
a. 2, 8-dimethyl-3,6-decadiene a. trapped inside the crystal lattice of metals
b. 2, 2-dimethylpropane b. chemically very reactive
c. 2, 4, 6, 8 octatetraene c. decomposes in inner orbitals
d. 1, 3, 5, 7 octatetraene d. progressively filled
PRACTICE SET 5 1245
54. C2H5 NH2 C6H5 NHCH3 61. For a sparingly soluble salt ApBq solubility product (LS )
I II
with its solubility (S ) is best represented by
(C2H5 )2 NH and C6H5 NH2 a.LS = S p + q × p p × qq
III
b.LS = S p + q × pq × q p
Above the following which forms H-bonding faster in c.LS = S pq × p p × qq
aqueous medium? d.LS = s pq × ( pq )p + q
a. Only I
b. Only III 62. Sodium has body-centred cubic structure having
c. I, II and III nearest neighbour distance 365.9 pm. The
d. None of these density……… .
55. Identify the reaction upto P and product Q. a. 2 g cm3 b. 1.01 g cm3
NH2 NaNO2+ HX
+ –
N2 X
c. 5 g cm3 d. 3.02 g cm3
273 – 278K 63. RNA contains four bases, the first three bases are
same as in DNA but the fourth one is…… .
H+ COOH
P a. adenine b. cytosine
c. thymine d. uracil
Q
64. Identify A in the reaction,
a. Sandmeyer reaction, benzoic acid CHC6H5 O
b. Fittig reaction, benzoidal chloride (i) O3
A+
c. Friedel-Craft’s reaction benzaldehyde (ii) Zn/H2O
d. Reimer-Tiemann reaction, benzaldehyde
a. C6H 5CHO b. C6H 5CH2CH3
56. Now a days, the pesticide industry has shifted its
c. CH3COCl d. C6H 5COCH3
attention to herbicide such as…… .
a. sodium chlorate 65. Dihydrogen is used in the manufacturing of
b. sodium arsenite a. antiseptic b. disinfectant
c. Both (a) and (b) c. vanaspati fat d. antigens
d. aldrin
66. Thyroxine produced in the thyroid gland is an
57. Ampicillin and amoxycillin are synthetic modification iodinated derivatives of
of
a. acetic acid b. lactic acid
a. amino glycosides b. ofloxacin c. amino acid d. picric acid
c. tetracycline d. penicillins
67. [Fe(CO)4 (PPh3 )2] 2 + [CO(CO)4 (PhO)2] 2 +
58. A sample of wustite is Fe0.93 O1.00. The percentage of I II
iron present in form of Fe (III) is……… . [V(CO)4 (PF3 )2] 2 + [Mn(CO)4 (PCl3 )2] 2 +
a. 15.05% b. 17.5% III IV
c. 19.05% d. 8%
Arrange the following in the decreasing order of
59. An alcohol A is treated with ‘‘Lucas reagent’’ to predict C—O bond length.
whether the alcohol is primary, secondary or tertiary. a. I > II > III > IV b. III > II > I > IV
Which reacts fastest? c. III > IV > II > I d. II > I > III > IV
a.Tertiary alcohol
b. Secondary alcohol 68. Dehydrobromination (—HBr) of the following in increasing
c. Primary alcohol order is
d. None of the above Br

60. Fullerenes are the only pure form of carbon because I. II.
Br
a. layered structured held by van der Waals’ forces
b. they have smooth structure without having ‘dangling’
III. Br
bonds
c. it has 3-dimensional structure
d. extremely hard refractory solid of high melting point a. I < II < III b. III < II < I c. II = III < I d. III < I < II
1246 SELF STUDY GUIDE BITSAT

69. If a thin slice of sugar beet is placed in concentrated Cl Cl


solution of NaCl, then
a. sugar beet will lose water from its cells a. b.
b. sugar beet will absorb water from solution
c. sugar beet will neither absorb nor lose water
d. sugar beet will dissolve in solution CH3 O CH3
70. 0.02 molar solution of NaCl having degree of Cl Cl
dissociation of 90% at 27° C has osmotic pressure
equal to Cl
c. d.
a. 0.94 bar b. 9.4 bar
c. 0.94 bar d. 9.4 ´ 10- 4 bar
NO2
71. Which of the following statements is correct?
a. (n - 1) d subshell has lower energy than ns subshell
76. DU ° of combustion of methane is -x kJ mol- 1.
b. (n - 1) d subshell has higher energy than ns subshell
The value of of DH ° is
c. (n + 1) d subshell has lower energy than nf subshell
a. = DU ° b. > DU °
d. nf subshell has lower energy than (n + s ) s subshell
c. < DU ° d. Equal to zero
h
72. If a = , then the correct expression for the 77. At 500 K, equilibrium constant K C for the following
4 p2me 2
reaction is 5.
calculation of the circumference of the first orbit of
hydrogen atom is 1 1
H2 ( g ) + I2 ( g ) s HI( g )
2 2
a. 4 pha b. 2 pr
What would be the equilibrium constant K C for the
c. 4h 2 pa d. (a) and (c) are correct
reaction?
73. By the addition of 0.035 g of starch the coagulation of 2HI ( g ) s H2( g ) + I2 ( g )
10 cm 3 of gold sol is completely prevented. The gold
a. 0.04 b. 0.4
number of starch is
c. 25 d. 2.5
a. 0.00035 b. 0.35
c. 55 d. 35 78. The reaction,
74. In chain growth polymerisation addition takes place 3ClO- (aq ) ¾® ClO3- (aq ) + 2Cl- (aq ) is an example
head-to-tail when monomer is of
a. oxidation reaction
a. CH2 == CH b. reduction reaction
c. disproportionation reaction
b. CH2 == CH¾ CH== CH2 d. decomposition reaction
c. CH2 == C ¾ C OCH3 79. Gaseous nitric acid has the structure
½ ½½
CH3 O a.planar b. square planar
c. pyramidal d. trigonal bipyramidal
d. CH2 == CH¾ C ºº N
80. p-nitrophenol is less volatile due to
75. The major product (s) of the following reaction is (are)
a. intermolecular hydrogen bonding
Cl b. intramolecular hydrogen bonding
O c. picric acid
Anhyd. AlCl3 d. van der Waals’ force
+ H3C C Cl
PRACTICE SET 5 1247

PART III
a. English Proficiency
Directions (Q. Nos. 81 to 84) In the sentences that are 89. Some people enjoy saving money because
given below, a part of it has been underlined. That part may a. they are able to live a happy life
be improved by one of the options. The part with the best b. saving is a passion for them
improving ability will be your answer. In case, no c. they are able to enjoy the pleasures of life
improvement is needed then “No improvement” is your d. they can entertain others
answer.
90. If a money-box is given to a child, what should
81. If he would have tried, he would have succeeded. accompany it?
a. is tried b. was tried a. Tips on the advantages of saving money
c. had tried d. No improvement b. Guidelines to preserve it
82. I will not go to school, if it shall rain tomorrow. c. instructions on when and how to open it
d. A box of tools to open it
a. it would rain tomorrow b. it will rain tomorrow
c. it rains tomorrow d. No improvement 91. A miserly man of forty does not go to the theatre
because
83. Why the dinosaurs died out is not known.
a. he has no liking for plays
a. it is not known b. the reason is not known
b. he does not want to waste his valuable time
c. that is not known d. No improvement
c. he persuades himself that he is very poor
84. He is quite well now, except a slight cold. d. he is frightened by the darkness of the theatre hall
a. except with a slight cold b. excepting a slight cold
Directions (Q. Nos. 92 and 93) Find the synonym of the
c. except for a slight cold d. No improvement
word given in capital letters.
Directions (Q. Nos. 85 to 88) In the sentences given
92. The vehicle slowed down as they moved up the
below there is an error in one of the parts which will be your
GRADIENT.
answer. If there is no error then ‘No error’ is your answer.
a. Road with sharp bends
85. The table’s wood (a)/ is infested with mite (b)/ and I b. Bumpy, uneven path
am likely to dispose it off. (c)/ No error (d) c. Slope or incline
86. The morale of the army (a)/ was high because the d. Mountainous area
news (b)/ coming from the front are very encouraging. 93. It is DEPLORABLE that millions of people in India are
(c)/ No error (d) below the poverty line.
87. I can’t come to you now (a)/ because a lot of works a. Hateful b. Regretable
(b)/ remains to be done. (c)/ No error (d) c. Undesirable d. Unpleasant
88. A farmer was leading oxes (a)/ to his field for Directions (Q. Nos. 94 and 95) Rearrange the jumbled
ploughing (b)/ early in the morning. (c)/ No error (d) part of the sentence and then choose the correct option.
Directions (Q. Nos. 89 to 95) Read the passage carefully 94. He was a man
and then choose the most suitable option from the given P : even if he had to starve
ones. Q : who would not beg
There are eccentric people who enjoy saving money for no R : borrow or steal
other reason than the pleasure of saving money. It is a S : from anyone
passion like drinking and a hobby like collecting of China.
The proper sequence should be
Does it usually begin with a money-box ? Imagine a painter
drawing the miser ’s progress in a number of scenes, with a. PQRS b. PRQS
the first scene showing a benevolent grandfather holding out c. QRSP d. QPRS
a harmless looking tin money-box to and infant scarcely able 95. P : in the progress of
to walk . Q : universities play a crucial role
The gift should always be accompanied by a box of tools. As R : our civilisation
a young man the infant has grown into a miser. By the age of S : in the present age
forty he has substantial bank account. But he persuades
himself that he is so poor that he never goes to the theater, The proper sequence should be
never invites a friend to dinner. Buy by sixty, he is a rich man a. SQPR b. QRSP
and is convinced that he is all but a pauper. c. QRPS d. SQRP
1248 SELF STUDY GUIDE BITSAT

b. Logical Reasoning
96. If STUDENT is coded as RUTEDOS, which word
would be coded as RDGPKBQ ?
a. SHERBET b. SHINGLE
c. SHACKLE d. SCHOLAR
a. b. c d.
97. Find the next term in the series.
1, 2, 2, 4, 8, ? 101. Examine the following figure.
a. 8 b. 9 c. 32 d. 16
98. Which of the following figures represents village,
district, state ?

a. b.
Which one of the following figures has the above
c. d. figure embedded in it ?

99. Which of the following diagrams represents humans,


females and girls ?
a. b. c d.
a. b. 102. ‘Flower’ is related to ‘Petal’ in the same way as ‘Book’
is related to
c. d. a. Pages b. Content
c. Author d. Library
100. From amongst the four alternative figures (a), (b), (c) 103. Find the odd one out.
and (d), select the one showing the unfolded position a. Cheese b. Wine c. Milk d. Curd
of Z.
104. Find the odd one out.
a. Moon b. Planet c. Sky d. Star
105. Find the odd one out.
a. 42-4 b. 36-6 c. 32-2 d. 15-5
X Y Z

PART IV
Mathematics
106. If ò f (x ) dx = 2 {f (x )} 3 + C , then f (x ) is 109. The abscissae of the point on the curve xy = (a + x )2,
x the normal at which cuts off numerically equal
a. b. x 3 intercepts from the axes of coordinates, is
2
1 a -a
c. d.
x a. b. a c. 2a d.
3 2 2
x
107. The interval in which 2x 3 + 5 increase less rapidly 110. If 4x + 3 | y | = 5 y , then y as a function of x is
2
than 9x - 12x , is a. differentiable at x = 0 b. continuous at x = 0
dy
a. ( - ¥ , 1) b. (1, 2) c. = 2 for all x d. None of these
dx
c. ( 2, ¥ ) d. None of these 1

108. The number of real roots of the equation 111. If a × lim x 1– x


+ b = e -1; a ³ 1, b ³ 0, then
e x - 1 + x - 2 = 0, is x ®1

a. a = 1, b = e -1 b. a = 2, b = e -1
a. 1 b. 2
c. 3 d. 4 c. a = - 1, b = e -1 d. a = 1, b = 0
PRACTICE SET 5 1249
112. If e x + ef ( x ) = e, then the domain of definition of f (x ) is 124. In DABC, medians AD and CE are drawn. If
a. ( - ¥ , 1) b. ( - ¥ , 0) c. (1, ¥ ) d. None AD = 5, ÐDAC = p/ 8 and ÐACE = p/ 4, then the area
of DABC (in sq units) is equal to
113. A coin is tossed n times. The probability that head will 25 10 25 25
a. b. c. d.
turn up an odd number of times, is 9 3 18 3
-1
1 x +1 x -1 2x -1
a. b. c. d. 125. The locus of the foot of the perpendicular drawn from
2 2x 2x 2x either focus on a variable tangent to the hyperbola
114. For n Î N , 3 2n + 2 - 23 n - 9 is divisible by 16 y 2 - 9x 2 = 1, is
a. 3 b. 9 c. 64 d. 81 a. x 2 + y 2 = 9 b. x 2 + y 2 = 1/9
c. x 2 + y 2 = 7 /144 d. x 2 + y 2 = 1/16
115. If p , q , r have truth values T, F, T respectively, then
which of the following is true? 126. The length of the sides of the square which can be
a. ( p ® q ) Ù r b. ( p ® q ) Ù ~ r made by four perpendicular tangents to the ellipse
c. ( p Ù q ) Ù ( p Ú r ) d. q ® ( p Ù r ) x 2 2y 2
+ = 1, is
7 11
116. Let A = {x ÎR : x £ 1} and f : A ® A be defined as
a. 10 units b. 8 units c. 6 units d. 5 units
f (x ) = x ( 2 - x ). Then, f -1(x ) is
127. The set of points on the axis of the parabola
a. 1 + 1 - x b. 1 - 1 - x c. 1 - x d. 1 ± 1 - x (x - 1)2 = 8( y + 2) from where three distinct normals
117. The relation R = {(1, 3 ), ( 3, 5 )} is defined on the set with can be drawn to the parabola, is the set (h, k ) of points
minimum number of elements of natural numbers. The satisfying
minimum number of elements to be included in R, so a. h > 2 b. h > 1
that R is an equivalence relation, is c. k > 2 d. None of these
a. 5 b. 6 c. 9 d. 8
128. If the chords of contact of tangents from three points
118. If S is the set of squares and R is the set of rectangles, A, B and C to the circle x 2 + y 2 = a 2 are concurrent,
then (S È R ) - (S Ç R ) is then A, B and C will
a. S
a. be concyclic
b. R
b. be collinear
c. set of squares but not rectangles
c. form the vertices of triangle
d. set of rectangles but not squares
d. None of the above
119. If x1 and x 2 are the roots of the equation e 2 × x ln x = x 3
129. The condition that one of the straight lines given by
with x1 > x 2, then the equation ax 2 + 2hxy + by 2 = 0 may coincide with
a. x1 = 2x 2 b. x1 = x 22 c. 2x1 = x 22 d. x12 = x 23
one of those given by the equation
120. If 2 sec 2 A - sec 4 A - 2 cosec 2A + cosec 4 A = 15 / 4, a ¢x 2 + 2h¢xy + b ¢y 2 = 0, is
then tan A is equal to a. (ab ¢ - a ¢b )2 = 4(ha ¢ - h ¢a ) (bh ¢ - b ¢h )
1 1 1
a. ± b. ± c. ± 2 d. ± 2 b. (ab ¢ - a ¢b )2 = (ha ¢ - h ¢a ) (bh ¢ - b ¢h )
2 2 2
c. (ha ¢ - h ¢a )2 = 4(ab ¢ - a ¢b ) (bh ¢ - b ¢h )
121. The range of R for which the inequality d. (bh ¢ - b ¢h )2 = 4(ab ¢ - a ¢b ) (ha ¢ - h ¢a )
k cos 2 x - k cos x + 1 ³ 0, " x Î ( - ¥, ¥ ), is
130. If P (1 + t 2, 2 + t 2 ) is any point on the line, then the
1 1 1
a. k < - b. - £ k £ 4 c. k > 4 d. £k £5 range of the value of t for which the point P lies
2 2 2
between the parallel lines x + 2y = 1 and 2x + 4 y = 15
122. The number of ordered pairs which satisfy the is
equation x 2 + 2x sin (xy ) + 1 = 0, y Î[ 0, 2p], are a. - 4 2 / 3 < t < 5 2 / 3 b. 0 < t < 5 2 / 6
a. 1 b. 2 c. 3 d. 0 c. 4 2 < t < 0 d. None of these
é a -b qù
123. The value of 2 tan-1 ê tan ú is 131. One Indian and four American men and their wives
ë a +b 2û are to be seated randomly around a circular table.
æ a cos q + b ö æ a + b cos q ö Then, the conditional probability that the Indian man is
a. cos-1çç ÷÷ b. cos-1çç ÷÷ seated adjacent to his wife, given that each American
è a + b cos q ø è a cos q + b ø man is seated adjacent to his wife, is
æ a cos q ö æ b cos q ö a. 1/2 b. 1/3
c. cos-1çç ÷÷ d. cos-1 çç ÷÷
è a + b cos q ø è a cos q + b ø c. 2/5 d. 1/5
1250 SELF STUDY GUIDE BITSAT

132. Let p be an odd prime number and Tp be the following 141. Let a = x $i + x 2$j + 2k$ , b = - 3 $i + $j + k$ and the
set of 2 ´ 2 matrices : $ $ $
c = ( 3x + 11)i + (x - 9 )j - 3k be three vectors. Then,
ì éa b ù ü the angle between a and b is acute angle and between
Tp = í A = ê ú : a , b , c Î { 0, 1L p - 1} ý c and a is obtuse, if x lies in
î ëc a û þ a. ( - ¥ , 1) È ( 2, 3) b. ( - ¥ , 1]
The number of A in Tp such that A is either symmetric c. [ 2, 3] d. None of these
or skew-symmetric or both and det(A) is divisible by p,
is 142. The mean of the numbers a , b , 8, 5, 10 is 6 and the
a. ( p - 1) 2
b. 2( p - 1) 2
c. ( p - 1) + 1 d. 2p - 1 variance is 6.80. Then, which one of the following
gives possible values of a and b?
133. Let a , b , c be the real numbers. Then, the system of a. a = 1, b = 6 b. a = 3, b = 4 c. a = 0, b = 7 d. a = 5, b = 2
x2 y2 z2 x2 y2 z2
equations in + - = 1, 2 - 2 + 2 = 1 143. The degree of the differential equation
a2 b2 c2 a b c 2 3
dy 1 æ dy ö 1 æ dy ö
x2 y 2 z2 x = 1+ + ç ÷ + ç ÷ + ... , is
and - 2 + 2 + 2 = 1, has dx 2 ! è dx ø 3 ! è dx ø
a b c
a. 3 b. 1
a. no solution b. unique solution
c. Not defined d. None of these
c. infinitely many solutions d. finitely many solutions
144. The area bounded by y = xe | x | and lines | x | = 1, y = 0,
134. If the coefficients of three consecutive terms of
is
(1 + x )n + 5 are in the ratio 5 : 10 : 14, then n equals a. 4 sq units b. 6 sq units c. 1 sq units d. 2 sq units
a. 3 b. 4 c. 5 d. 6 q

135. Consider the set of eight vectors


145. If òp / 2 sin x dx = sin 2 q, then the value of q satisfying

V = {a i$ + b$j + c k$ ; a , b , c Î { - 1, 1}}. If three 0 < q < p, is


non-coplanar vectors can be chosen from V in 2p a. 3p/ 2 b. p/6 c. 5p/ 6 d. p/2
ways, then p is 4n k (k + 1)
a. 4 b. 5 c. 6 d. 7 146. Let Sn = å ( - 1) 2 k 2. Then, Sn can take value(s)
2 k =1
æ pö tan a
136. If a Î ç 0, ÷, then x 2 + x + is always greater a. 1056 b. 1088 c. 1120 d. 1333
è 2ø x2 + x 2 2
147. If the function f (x ) = ( 4 sin x - 1) (x - x + 1), n Î N , has
n
than or equal to a local maximum at x = p/6, then n
a. 2 tan a b. 1 c. 2 d. sec2 a a. can be any odd natural number
1 b. can only be an odd prime number
137. Let complex numbers a and lie on circles c. can be any even natural number
a
d. can only be a multiple of 4
(x - x 0 )2 + ( y - y 0 )2 = r 2 and (x - x 0 )2 + ( y - y 0 )2 = 4r 2,
respectively. If z 0 = x 0 + iy 0 satisfies the equation 148. In three dimensional space, the equation x 2 - x - 2 = 0
2 | z 0 |2 = r 2 + 2, then | a | equals represents
a. a pair of straight lines
a. 1/ 2 b. 1/ 2 c. 1/ 7 d. 1/3
b. a set containing two distinct points
138. Let a , b , c be real numbers and a ¹ 0. If a is the root of c. a pair of parallel planes
a 2x 2 + bx + c = 0 and b is the root of a 2x 2 - bx - c = 0; d. None of the above
0 < a < b, then the equation a 2x 2 + 2bx + 2c = 0 has a 149. If the sum of the slopes of the normal from a point P to
root g that always satisfies the hyperbola xy = c 2 is equal to l; l Î R + , then the
a+b b locus of point P is
a. g = b. g = a + c. g = a d. a < g < b
2 2 a. x 2 = lc 2 b. y 2 = lc 2 c. xy = lc 2 d. None
2 2 2
139. If a + b + c = 1, then ab + bc + ca lies in the interval x + 1 y + 2 z +1
150. Consider the lines L1: = = and
é1 ù é 1 ù é 1ù 3 1 2
a. ê , 2ú b. [ - 1, 2] c. ê - , 1ú d. ê - 1, ú
ë2 û ë 2 û ë 2 û x -2 y + 2 z -3
L2 : = = . The unit vector
140. If a, b, c are non-coplanar vectors, then 1 2 3
a × (b ´ c) b × ( a ´ c) perpendicular to both L1 and L2 is
+ is equal to 1 1
( c ´ a) × b c × ( a ´ b) a. ( - $i + 7$j + 5k$ ) b. ( - $i - 7$j + 5 k$ )
99 5 3
a. 0 b. 1 1 1
c. 2 d. None of these c. ( - $i + 7$j + 5 k$ ) d. ( 7$i - 7$j - k$ )
5 3 99
Answers
Physics
1. (a) 2. (a) 3. (a) 4. (b) 5. (d) 6. (d) 7. (a) 8. (c) 9. (c) 10. (c)
11. (a) 12. (c) 13. (d) 14. (a) 15. (d) 16. (b) 17. (a) 18. (a) 19. (b) 20. (d)
21. (a) 22. (a) 23. (b) 24. (d) 25. (a) 26. (b) 27. (a) 28. (b) 29. (d) 30. (a)
31. (b) 32. (a) 33. (a) 34. (a) 35. (b) 36. (d) 37. (a) 38. (c) 39. (a) 40. (c)

Chemistry
41. (c) 42. (c) 43. (a) 44. (a) 45. (a) 46. (d) 47. (b) 48. (d) 49. (b) 50. (c)
51. (b) 52. (d) 53. (a) 54. (a) 55. (a) 56. (c) 57. (d) 58. (a) 59. (a) 60. (b)
61. (a) 62. (b) 63. (d) 64. (a) 65. (c) 66. (c) 67. (c) 68. (a) 69. (a) 70. (a)
71. (b) 72. (d) 73. (d) 74. (d) 75. (b) 76. (c) 77. (a) 78. (c) 79. (a) 80. (a)

English Proficiency
81. (c) 82. (c) 83. (d) 84. (c) 85. (a) 86. (c) 87. (b) 88. (a) 89. (b) 90. (d)

91. (c) 92. (c) 93. (b) 94. (c) 95. (a)

Logical Reasoning
96. (d) 97. (d) 98. (b) 99. (d) 100. (b) 101. (c) 102. (a) 103. (b) 104. (c) 105. (a)

Mathematics
106. (d) 107. (b) 108. (a) 109. (a) 110. (b) 111. (d) 112. (a) 113. (a) 114. (c) 115. (d)
116. (b) 117. (c) 118. (d) 119. (b) 120. (a) 121. (b) 122. (b) 123. (a) 124. (d) 125. (d)
126. (d) 127. (c) 128. (b) 129. (a) 130. (a) 131. (c) 132. (d) 133. (b) 134. (d) 135. (b)
136. (a) 137. (c) 138. (d) 139. (c) 140. (a) 141. (a) 142. (b) 143. (b) 144. (d) 145. (d)
146. (a) 147. (c) 148. (c) 149. (a) 150. (b)
Solved Paper 2017
BITSAT
Instructions
1. There are 150 questions in all. The number of questions in each part is as given below. No. of Questions
Part I Physics 1-40
Part II Chemistry 41-80
Part III (a) English Proficiency 81-95
(b) Logical Reasoning 96-105
Part IV Mathematics 106-150
2. All questions are multiple choice questions with four options, only one is correct.
3. Each correct answer fetches 3 marks while incorrect answer has a penalty of 1 mark.

PART I
Physics
1. If temperature of a black body increases from 300 K to 5. At what angle q to the horizontal should an object is
900 K, then the rate of energy radiation increases by projected so that the maximum height reached is equal
a. 81 b. 3 c. 9 d. 2 to the horizontal range?
a. tan- 1 ( 2) b. tan- 1 ( 4)
2. A whistle of frequency 500 Hz tied to the end of a string
æ 2ö
of length 1.2 m revolves at 400 rev/min. A listener c. tan- 1 ç ÷ d. tan- 1 ( 3)
standing some distance away in the plane of rotation of è 3ø
whistle hears frequencies in the range.
6. A body of mass 1 kg is executing simple harmonic
(Speed of sound = 340 m/s)
motion. Its displacement y (cm) at t seconds is given by
a. 436 to 574 b. 426 to 586 æ pö
c. 426 to 574 d. 436 to 586 y = 6 sin ç 100 t + ÷.
è 4 ø
3. The focal length of a thin convex lens for red and blue Its maximum kinetic energy is
rays are 100 cm and 96.8 cm respectively. Then, the a. 6 J b. 18 J
dispersive power of the material of the lens is c. 24 J d. 36 J
a. 0.968 b. 0.98
c. 0.0325 d. 0.325 7. A positive charge q is projected in magnetic field of
mv
4. Two metal plates having a potential difference of 800 V width with velocity v . Then, the time taken by
2 qB
are 2 cm apart. It is found that a particle of mass
charged particle to emerge from the magnetic field is
1.96 ´ 10- 15 kg remains suspended in the region
m pm
between the plates. The charge on the particle must be a. b.
(e = elementary charge). 2 qB 4qB
a. 2e b. 3e pm pm
c. d.
c. 6e d. 8e 2qB 2 × qB
2 SELF STUDY GUIDE BITSAT
8. In Young’s double slit experiment, the slits are 2 mm 14. The de-Broglie wavelength of a proton (charge
apart and are illuminated by photons of two = 1.6 ´ 10- 19 C, m = 1.6 ´ 10- 27 kg) accelerated
wavelengths l1 = 12000 Å and l2 = 10000 Å. At what through a potential difference of 1 kV is
minimum distance from the common central bright a. 600 Å b. 0.9 ´ 10- 12 m
fringe on the screen 2m from the slit will a bright fringe
c. 7 Å d. 0.9 nm
from one interference pattern coincide with a bright
fringe from the other? 15. An ice-berg of density 900 kgm - 3 is floating in water of
a. 8 mm b. 6 mm c. 4 mm d. 3 mm density 1000 kgm - 3 . The percentage of volume of
ice-berg outside the water is
9. Two blocks A and B are placed one over the other on a
smooth horizontal surface. The maximum horizontal a. 20% b. 35%
force that can be applied on lower block B, so that A and c. 10% d. 11%
B move without separation is 49 N. 16. The total energy of an electron in the first excited state
The coefficient of friction between A and B is of hydrogen is about - 3.4 eV. Its kinetic energy in this
state is
3kg B
a. - 3.4 eV b. - 6.8 eV
7kg A
c. 6.8 eV d. 3.4 eV
a. 0.2 b. 0.3 17. A common emitter amplifier has a voltage gain of 50, an
c. 0.5 d. 0.8 input impedance of 100W and an output impedance of
10. An aeroplane is flying in a horizontal direction with a 200W. The power gain of the amplifier is
velocity u and at a height of 2000 m. When it is vertically a. 500 b. 1000
below a point A on the ground a food packet is released c. 1250 d. 50
from it. The packet strikes the ground at point B.
18. The horizontal range and maximum height attained by
If AB = 3 km and g =10 m/s 2, then the value of u is
a projectile are R and H respectively. If a constant
a. 54 km/h b. 540 km/h g
horizontal acceleration a = is imparted to the
c. 150 km/h d. 300 km/h 4
projectile due to wind, then its horizontal range and
11. A conducting circular loop is placed in a uniform
maximum height will be
magnetic field, B = 0.025 T with its plane perpendicular
H
to the loop. The radius of the loop is made to shrink at a a. (R + H ),
constant rate of 1 mm/s. The induced emf when the 2
radius is 2 cm, is æ Hö
b. çR + ÷ , 2H
a. 2 pmV b. pmV è 2ø
p c. (R + 2H ), H
c. mV d. 2 mV
2 d. (R + H ), H

12. A mild steel wire of length 2L and cross-sectional area A 19. A balloon is filled at 27° C and 1 atm pressure by 500 m 3
is stretched, well with in the elastic limit, horizontally He. At - 3° C and 0.5 atm pressure, the volume of He
between two pillars as shown in figure. A mass m is will be
suspended from the mid-point of the wire strain in the a. 700 m 3 b. 900 m 3
wire is c. 1000 m 3 d. 500 m 3
2L
x
20. The ratio of intensity at the centre of a bright fringe to
the intensity at a point distance one-fourth of the
m distance between two successive bright fringes will be
a. 4 b. 3
2 c. 2 d. 1
x x
a. b.
2 L2 L 21. A rectangular block of mass m and area of
x2
x 2 cross-section A floats in a liquid of density r. If it is given
c. d. a vertical displacement from equilibrium, it undergoes
L 2L
oscillation with a time period T. Then
13. The resistance of a wire at 20° C is 20W and 500 °C is 1
a. T µ r b. T µ
60 W. At which temperature, its resistance will be 25W? A
a. 50 °C b. 60 °C 1 1
c. T µ d. T µ
c. 70 °C d. 80 °C r m
SOLVED PAPER 2017 3
22. Three charges are placed at the three vertices of an 26. In the given circuit diagram,
equilateral triangle of side ‘a’ as shown in the figure.
E = 5 V, r = 1W, R 2 = 4W, R1 = R 3 = 1W and C = 3 mF
The force experienced by the charge placed at the
vertex A in a direction normal to BC is
C R1 C
A
+Q
R2=4W

C R3 C

–Q +Q
B C Er=1W
2 2
Q Q
a. b. - Then, what will be the numerical value of charge on
4pe0a 2 4pe0a 2 each plates of the capacitor.
Q2 a. 24 mC b. 12 mC
c. zero d.
2pe0a 2 c. 6 mC d. 3 mC

23. A load of mass m falls from a height h on the scale pan 27. A block A of mass 100 kg rests on another block B of
hung from a spring as shown. If the spring constant is k mass 200 kg and is tied to a wall as shown in the figure.
and the mass of the scale pan is zero and the mass m The coefficient of friction between A and B is 0.2 and
does not bounce relative to the pan, then the amplitude that between B and ground is 0.3. The minimum force
of vibration is required to move the block B is (g = 10 ms - 2)

k
A
=100 kg
Wall F
B
m =200 kg
h S
mg mg 2hk
a. b. 1+ a. 900 N b. 200 N
k k mg
c. 1100 N d. 700 N
mg mg 1 + 2hk
c. + d. None of these 28. A uniform rod of length l and mass m is free to rotate in a
k k mg
vertical plane about A. The rod initially in horizontal
24. The activity of a radioactive sample is measured as N 0 position is released. The initial angular acceleration of
N0 ml 2
counts per minute at t = 0 and counts per minute at the rod is (Moment of inertia of rod about A is )
e 3
t = 5 min. The time (in minutes) at which the activity
l
reduces to half its value is
2 5 A B
a. loge b.
5 loge 2
c. 5 log10 2 d. 5 loge 2 3g 2l
a. b.
2l 3g
25. A plano-convex lens fits exactly into a plano-concave 3g l
lens. Their plane surfaces are parallel to each other. If c. 2 d. mg
2l 2
lenses are made of different materials of refractive
indices m 1 and m 2, R is the radius of curvature of the 29. Monochromatic radiation of wavelength l is incident on
curved surface of the lenses, then the focal length of a hydrogen sample in ground state. Hydrogen atom
combination is absorbs a friction of light and subsequently emits
a.
R
b.
R radiations of six different wavelengths. The wavelength
2 (m 1 + m 2 ) 2 (m 1 - m 2 ) l is
R 2R a. 97.5 nm b. 121.6 nm
c. d.
(m 1 - m 2 ) m1 - m 2 c. 110.3 nm d. 45.2 nm
4 SELF STUDY GUIDE BITSAT
30. A coil in the shape of an equilateral triangle of side l is 35. Two simple pendulums of lengths 5m and 20m
suspended between the pole pieces of a permanent respectively are given small displacement in one
magnet such that B is in plane of the coil. If due to a direction at the same time they will again be in the same
current i in the triangle, a torque t rests on it, the side l of sense when the pendulum of shorter length has
the triangle is completed n oscillations. Then, n is
1
a. 5 b. 1 c. 2 d. 3
æ t ö2 2 ætö
a. 2 çç ÷
÷ b. ç ÷
è 3 Bi ø 3 è Bi ø 36. A parallel plate capacitor with air between the plates
1 has a capacitance of 9 pF. The separation between the
æ t ö2 1 ætö plates is d. The space between the plates is now filled
c. 2 ç ÷ d. ç ÷ d
è Bi ø 3 è Bi ø with two dielectrics constant K 1 = 3 and thickness
3
31. Work done in increasing the size of a soap bubble from while the other one has dielectric constant K 2 = 6 and
radius of (3 to 5) cm is nearly (surface tension of soap 2d
thickness . Capacitance of the capacitor is now
solution = 0.03 Nm -1) 3
a. 0.2 p mJ b. 2p mJ c. 0.4 mJ d. 4p mJ a. 1.8 pF b. 45 pF
c. 40.5 pF d. 20.25 pF
32. The velocity of a projectile at the initial point A is
( 2$i + 3 $j ) m/s. Its velocity (in m/s) at point B is 37. A particle moving along X -axis has acceleration f , at
æ t ö
time t given by f = f0ç 1 - ÷, where f0 and T are
è Tø
constants. The particle at t = 0 and the instant when
f = 0, the particle’s velocity v X is
1
a. f0T b. f0T 2
2
1
c. f0T 2 d. f0T
2

B
X
38. A geostationary satellite orbits around the earth in a
A circular orbit of radius 36000 km. Then, the time period
of a sky satellite orbiting a few 100 km above the earth’s
a. - 2$i - 3$j b. - 2$i + 3$j surface (R = 64000 km) will approximately be
1
c. 2$i - 3$j d. 2$i + 3$j a. h b. 1h c. 2 h d. 4 h
2
33. In the circuit shown, the heat produced in 5W resistor is 39. A transverse wave propagating on a stretched string of
10 cal s - 1. The heat produced per sec in 4W resistor linear density 3 ´ 10- 4 kgm - 1 is represented by the
will be
equation
4W 6W
y = 0.2 sin(1.5x + 60 t )
where, x in metres and t is in seconds. The tension in
the string (in Newton) is
a. 0.24 b. 0.48
5W
c. 1.20 d. 1.80
a. 1 cal b. 2 cal c. 3 cal d. 4 cal 40. What is the magnetic field at the centre of arc in the
34. An a-particle after passing through potential difference figure below?
ofV volt collides with a nucleus. If the atomic number of i
the nucleus is Z , then distance of closest approach is i i
Z Form ¥ To ¥
a. 14.4× D
V 90° r
Z
b. 14.4 m O
V
V m 0 2i m 0 2i é pù
c. 14.4 m a. × [ 2 + p] b. × 2+ ú
Z 4p r 4p r êë 4û
V m i m0 i é pù
d. 14.4 D c. 0 × [ 2 + p] d. × 2+ ú
Z 4p r 4p r êë 4û
PART II
Chemistry
41. 4g of copper was dissolved in conc. HNO3 . The copper 48. Which of the following reactions does not involved
nitrate thus obtained gave 5g of its oxide on strong absorption energy?
heating the equivalent weight of copper is I. O( g )+ e - ¾® O - ( g )
a. 23 b. 32 II. S( g )+ e - ¾® S - ( g )
c. 12 d. 20
III. O - ( g )+ e - ¾® O 2- ( g )
42. Choose the incorrect statement. IV. Cl( g )+ e - ¾® Cl - ( g )
a. Sodium borohydride reacts very slowly with cold water
a. Only II b. I and III
b. Sodium borohydride reacts violently with cold water to c. I, II and III d. I, II and IV
give H2(g )
c. Solubility of sodium borohydride in water at 25°C is
49. The most reactive amine towards reactions with dil. HCl
10.05 g/ml is
NH2
d. Melting point of sodium borohydride is 500°C H 3C
43. The orbital angular momentum of an electron is 2 S a. CH3 NH2 b. NH c. (CH3)3.N d.
orbital is H 3C
h
a. b. zero 50. Blocks of magnesium are fixed to the bottom of a
4p ship to
h h
c. d. 2 . a. block hole in the ship
2p 2p
b. acidity of sea water
44. Which of the following are isoelectronic species? c. make the ship lighter
+ d. prevent the action of water and salt
I. C H3 II. NH-2
51. On electrolysis of water, a total of 1 mole of gases is
III. NH+4 IV. NH3 evolved. The amount of water decomposed is
1 2
a. I, II and III b. II, III and IV a. 1 mol b. 2 mol c. mol d. mol
3 3
c. I, II and IV d. II and I
45. Two elements A and B have electronegativities 1.2 and 52. How many moles of Fe2+ ions are formed when excess
3.0 respectively. The nature of bond between A and B of iron react with 500 mL O. 4NHCl, under inert
atmosphere?
would be
(assume no change in volume)
a. ionic b. covalent
a. 0.4 b. 0.1 c. 0.2 d. 0.8
c. co-ordinate d. metallic
53. Sodium sulphate is soluble in water but barium
46. In the compound,
sulphate is insoluble because
CH2 == CH¾ C ºº CH a. hydration energy of Na 2SO4 is more than of its lattice
(I) (II) (III) energy

The most acidic hydrogen atom is b. lattice energy of BaSO4 is more than its hydration
a. Only I b. Only II energy
c. Only III d. All are equally acidic c. Both (a) and (b)
d. None of the above
47. Reductive ozonolysis of (CH3 )2 C ==
= C(CH3 )2
followed by hydrolysis gives 54. In the reaction,
a. only one type of ketone KCN LiAlH 4
b. only one type of aldehyde CH3 × CH2 × Cl ¾® ( A ) ¾¾® (P ) end product (P ) is
c. two types of ketone a. CH3CH2 × NO2 b. CH3CH2CH2NO2
d. two types of aldehyde
c. CH3CH2NH2 d. CH3 × CH2 × CH2NH2

55. Which of the following reactions is an example of


calcination process?
6 SELF STUDY GUIDE BITSAT
a. 2Ag + 2HCl + [O] ¾® 2AgCl + H2O 65. Which of the following is an incorrect set of quantum
b. 2Zn + O2 ¾® 2ZnO members?
c. 2ZnS + 3O2 ¾® 2ZnO + 2SO2 a. n = 2, l = 0, m = 0 b. n = 1, l = 0, m = 0
D c. n = 3, l = 3, m = 0 d. n = 2, l = 1, m = 1
d. MgCO3 ¾® MgO + CO2
66. The most acidic oxide for nitrogen is
56. For an endothermic reaction, where DH represent the
a. NO2 b. N2O
enthalpy of reaction in kJ/mol, the minimum value for
energy of activation (for forward reaction) will be c. NO d. N2O5
a. less than DH b. zero 67. Which of the following show maximum bond-order?
c. more than DH d. equal to DH
a. O2 b. O-2
57. Which of the following metal is leached by cyanide c. O+2 d. O2-
2
process
a. Ag b. Na c. Al d. Cu 68. Which of the following show an increase in entropy?

58. Which of the following is a diamagnetic complex I. Boiling of water


a. [Co(NH3 )6] 3+
b. [NiCl4] 2- II. Melting of ice
2- III. Freezing of water
c. [CuCl4] d. [Fe(H2O)6] 3+
IV. Formation of hydrogen gas from water
59. Neoprene is a a. (I) and (II) b. Only (III)
a. monomer of rubber b. synthetic rubber c. (I), (II) and (IV) d. (III) and (IV)
c. a natural rubber d. vulcanised rubber 69. BF3 is an acid, according to
60. Among the following, which have highest melting point? a. Lewis
a. Ionic solids b. Pseudo solids b. Arrhenius
c. Molecular solids d. Amorphous solids c. Bronsted and Lowery
d. All of the above
61. The night-blindness is developed due to deficiency of
vitamin 70. For the reaction
a. B6 b. C c. B12 d. A N2O4 ( g ) ¾® 2NO2( g )
62. The transfer RNA anticodon for the messenger RNA a. DH > DE b. DH < DE
codon G - C - A is c. DH = DE d. DH = 0
a. C - G - U b. C - C - U
c. U - C - C d. G - U - C 71. Which of the following elements mostly form covalent
compounds?
63. 0.765 g of an acid gives 0.535 g of CO 2 and 0.138 g of a. Cs b. Rb
H 2O. Then, the ratio of percentage of carbon and c. K d. Li
hydrogen is 72. When aqueous solutions of borax is acidified with HCl,
a. 19 : 2 b. 18 : 11 we get
c. 20 : 17 d. 1 : 7
a. B2H6 b. H3BO3
64. Maximum pK b value is of c. B2O3 d. All of these

NH×CH3 73. Which of the following compound does not follow


Huckel’s rule?
a. b. (CH3CH2)2NH +
a. b.
sAA
c. (CH3)2NH d. NH
c. d.
r
SOLVED PAPER 2017 7
1 77. Consider the following radioactive decays
74. A graph is ploted between log K virsus for calculation
T -a
of activation energy (E a ). The correct plot is I. 92 U ¾® 90Th and

-a
II. 90 Th ¾® 88Ra
log k
a. In which case group of parent and daughter elements
remains unchanged.
a. In (I)
1/T b. In (II)
c. Both in (I) and (II)
d. None of the above
-(X)
b. log k
78. Chlorobenzene ¾ Reaction
¾ ¾ ¾¾ ® Phenol
-(Y )
¾ Reaction
¾ ¾ ¾¾ ® Salicylaldehyde.
1/T The reactions(s) ‘X ’ and ‘Y ’ respectively are
a. Fries rearrangement and Kolbe
b. Cumene and Reimer-Tiemann
c. Dow and Reimer-Tiemann
c. log k d. Dow and Sandmeyer.
79. Which of the following has largest number of moles?
a. 8 g of oxygen atoms
1/T
b. 16 g of oxygen gas
c. 14 g of nitrogen gas (N2 )
d. All have same number of moles
d.
log k 80. One moles each of four ideal gases are kept as follows.
I. 5 L of gas (A) at 2 atm pressure
1/T II. 2.5 L of gas (B) at 2 atm pressure
75. The hybridisation of Fe is K 3 Fe(CN)6 is III. 1.25 L of gas (C) at 2 atm pressure
IV. 2.5 L of gas (D) at 2.5 atm pressure
a. sp 3 b. dsp 3 c. sp 3d 2 d. d 2sp 3
Which of the above gases is kept at highest
76. Which of the following shows maximum magnetic temperature.
moment?
a. Gas (A) b. Gas (B)
a. Mg2+ b. Ti3+ c. V 3+ d. Fe2+
c. Gas (C) d. Gas (D)
Part III
a. English Proficiency
Directions (Q. Nos. 81-83) In the following questions the a. dilemma b. deadlock c. exhaustion d. settlement
sentences may or may not be grammatically correct. Find out
Directions (Q. Nos. 91-95) Read the passage given below
which part of a sentence has an error and mark that part. If
and answer the questions that follow.
there is no error mark part ‘d’ as your answer.
A pioneering scheme has been started recently in
81. Along the northern frontier of India is seen Southampton of England’s south coast to educate tourists who
a b have been convicted of drunken driving.
the Himalayas mighty in their splendour. No error The penalty for drunken driving might be the loss of the
c d driving licence and a heavy fine. But under the new scheme,
convicted drivers do not pay the fine. Instead they have to
82. The father with the son were mysteriously missing attend eight training sessions; one a week organised by the
a b local authority probation service.
from the house. No error Designed to demonstrate the damage alcohol can do, the
c d scheme was devised by senior probation officer John Cook. He
said that about a quarter of the people who came to him had a
83. It is not advisable to take heavy luggages drinking problem, and had not realised how much they were
a b drinking. One way of getting the message across was to make
while on journey these days. No error the drivers pour out their usual ration of alcohol and then
measure it. Almost everyone poured out not a single measure,
c d
but a double atleast, an example of how easy it is to have more
Directions (Q. Nos. 84-85) Fill in the blanks with suitable than just one drink and to encourage other people to do the
preposition from the alternatives given under each sentence. same. The instructors on the course are giving clinical
evidence of the effects of alcohol on the body and brain. The
84. The problem of communal harmony cannot be glossed
sober truth is that drinking badly affects driving skills,
.............. by the government.
although the drinker might like to believe otherwise.
a. at b. on c. over d. for
91. The Southampton scheme requires convicted drivers
85. She could not muster .............. courage to stand a. to pay a heavy fine
against the maltreatment. b. to attend eight driving sessions-one a week
a. to b. up c. about d. on c. to undergo a probation service
Directions (Q. Nos. 86-88) The following sentences d. to surrender their driving licence
consist of a word or a phrase which is written in italicised 92. John Cook devised the scheme
letters. Each sentence is followed by four words or phrases. a. as a demonstration technique for driving
Select the word or the phrase which is closest to the opposite b. to deny the harmful effects of alcohol
in meaning of the italicised word or phrase. c. to show that Southampton was concerned about drivers
86. Philosophers say that the world is an illusion. d. to prove that alcohol does influence driving
a. a fact b. a reality c. an actuality d. a truth 93. The problem with a quarter of the people who went to
87. She used to disparage her neighbours every now and John Cook was that they
then. a. did not want to stop drinking
a. please b. praise c. belittle d. denigrate b. were unaware of the fact that they could get drunk
c. would not admit that they had a drinking problem
88. The momentum of the movement slackened in course d. did not know how much they were drinking
of time.
a. stopped b. quickened c. multiplied d. recovered 94. Most drivers start off with atleast
a. a double measure b. a single measure
Directions (Q. Nos. 89-90) In the following sentences, a c. a little less than a single measure
word or a phrase is written in italicised letters. For each d. two doubles
italicised part four words/phrases are listed below each
sentence. Choose the word nearest in meaning to the 95. The truth is that alcohol
italicised word/phrase. a. does not affect the body but only the brain
b. affects only the brain
89. The opposition criticised the ruling party for the
c. affects the body and the brain
deteriorating law and order situation in the state.
d. has no effect on the body or the brain
a. disrupting b. worsening c. crumbling d. eroding
90. The two opposing parties have reached stalemate.
SOLVED PAPER 2017 9
b. Logical Reasoning
96. ‘Shoes’ is related to ‘Leather’, in the same way as Question Figures
‘Rubber’ is related to
a. Plastic b. Polythene
c. Latex d. Chappal
X Y Z
97. Find the odd one from the following options
Answer Figures
a. 81 : 243 b. 25 : 75
c. 64 : 192 d. 16 : 64
98. Complete the series by replacing ‘?’ mark.
(a) (b) (c) (d)
4, 11, 30, 67, 128, ?
a. 219 b. 228 103. Choose the answer figure which completes the
c. 237 d. 240 problem figure matrix.
99. Lakshmi is elder than Meenu. Leela is elder than Question Figures
Meenu but younger than Lakshmi. Latha is younger
than both Meenu and Hari but Hari is younger than
Meenu. Who is the youngest? Answer Figures
a. Lakshmi b. Meenu
c. Leela d. Latha ?
100. In the following question a part of problem figure is (a) (b) (c) (d)
missing. Find out from the given answer figures a , b , c
and d that can replace the question mark (?) to 104. In the following question, one or more dots are place in
complete the figure. the figure marked as ( A ). This figure is followed by four
alternatives marked as a , b , c and d. One out of these
Question Figure Answer Figure
four options contains region(s) common to circle,
square and triangles, similar to that marked by the dot
in figures (A). Find that figure
Question Figure
?
(a) (b) (c) (d)

101. In the following question, five figures are given. Out of (A)
them find the three figures that can be joined to form a Answer Figures
square.

(A) (B) (C) (D) (E) (a) (b) (c) (d)

a. ACD b. BCD 105. How many different triangles are there in the figures
c. BDE d. CDE shown below?
102. The three problem figures marked X , Y and Z show the
manner in which a piece of paper is folded step by step
and then cut. From the answer figures a , b , c and d
select the one showing the unfolded pattern of the a. 28 b. 24
paper after the cut. c. 20 d. 16
Part IV
Mathematics
106. The coefficient of x 5 in the expansion of 115. The number of times the digit 5 will be written when
(1 + x ) + (1 + x )22 + ... + (1 + x )30 is
21
listing the integers from 1 to 1000, is
a. 51
C5 b. C59 a. 271 b. 272
31 21 30 20 c. 300 d. None of these
c. C6 - C6 d. C5 + C5
116. Let A and B be two sets such that A Ç X = B Ç X = f
107. If z = a + ib satisfies arg (z - 1) = arg (z + 3i ), then
and A È X = B È X for same set X . Then,
(a - 1) : b =
a. A = B b. A = X
a. 2 : 1 b. 1 : 3
c. B = X d. A È B = X
c. - 1: 3 d. None of these
117. Let A = [ -1, 1] and f : A ® A be defined as f (x ) = x | x | for
108. If p and p¢ denote the lengths of the perpendicular from
all x Î A, then f (x ) is
a focus and the centre of an ellipse with semi-major axis
of length a, respectively, on a tangent to the ellipse and a. many-one and into function
r denotes the focal distance of the point, then b. one-one and into function
a. ap = rp ¢ b. rp = ap ¢ c. many-one and into function
d. one-one and onto function
c. ap = rp ¢ + 1 d. ap ¢ + rp = 1
10 n 118. The general solution of sin x - 3 sin 2x + sin 3x = cos x
Cr
109. The value of år . n
Cr
is equal to - 3 cos 2x + cos 3x is
r =1 -1 p np p
a. np + b. +
a. 5 ( 2n - 9 ) b. 10 n 8 2 8
n np p 3
c. 9 (n - 4) d. None of these c. ( -1) + d. 2np + cos-1
2 8 2
110. The numbers 3 2 sin 2a - 1, 14 and 3 4 - 2 sin 2a form first
119. Two equal sides of an isosceles triangle are
three terms of an A.P., its fifth term is
7x - y + 3 = 0 and x + y - 3 = 0 and its third side
a. - 25 b. - 12 c. 40 d. 53 passes through the point (1, - 10). Find the equation of
the third side
111. For the equation 3x 2 + px + 3 = 0, p > 0, if one of the
a. x - 3y = - 31 b. x - 3y = 31
roots is square of the other, then p is equal to
1 2 c. x + 3y = 31 d. x + 3y = - 31
a. b. 1 c. 3 d.
2 3
120. If two distinct chords drawn from the point ( p , q ) on the
112. If a = log2 3, b = log2 5 and c = log7 2, then log140 63 in circle x 2 + y 2 = px + q y (where pq ¹ 0 ) are bisected
terms of a , b , c is by the X -axis, then
2ac + 1 2ac + 1 a. p 2 = q 2 b. p 2 = 8q 2
a. b.
2c + abc + 1 2a + c + a c. p 2 < 8q 2 d. p 2 > 8q 2
2ac + 1
c. d. None of these
2c + ab + a 121. The length of perpendicular drawn from the point
4 - x y 1- z
(2, 3, 4) to line = = , is
113. If cos (x - y ), cos x and cos (x + y ) are in HP, then 2 6 3
cos x sec ( y / 2) is equal to 3 2
a. 101 b. 101
a. ± 2 b. ± 1/ 2 7 7
c. ± 2 d. None of these 2 3
c. 103 d. 103
7 7
114. Let A = {1, 2, 3, 4, 5} and R be a relation defined by
122. The image of the point (1, 6 , 3 ) in the line
R = {(x , y ) : x , y Î A, x + y = 5}. Then, R is
x y -1 z - 2
a. reflexive and symmetric but not transitive = = is
1 2 3
b. an equivalence relation
a. (- 1, 0, 7) b. ( - 1, 0, - 7)
c. symmetric but neither reflexive nor transitive
d. neither reflexive nor symmetric but transitive c. (1, 0, 7) d. ( 2 , 0, 7)
SOLVED PAPER 2017 11
123. The distances of the point (1, - 5 , 9 ) from the plane a -b log a - log b
a. b.
x + y + z = 5 measured along a straight line x = y = z is 1 + ab 1 + log a log b
2 3 k, then the value of k is a+b log a + log b
c. d.
1 - ab 1 - log a log b
a. 5 b. 6
c. 3 d. 4 n
ò [x ] dx , where [x ] and {x } are integral
132. The expression n0
124. lim sin [ p n 2 + 1] is equal to
n ®¥ ò0 {x } dx
a. ¥ b. 0 and fractional part of x and n Î N , is equal to
c. does not exist d. None of these 1 1
a. b.
ì ex , x £0 n -1 n
125. If f (x ) = í , then
c. n d. n - 1
î | 1 - x |, x >0
a. f (x ) is differentiable at x = 0 133. The maximum value of f (x ) = x + sin 2x , x Î[ 0, 2p] is
b. f (x ) is continuous at x = 0, 1 p 3p 3p
a. b. 2p c. d.
c. f (x ) is differentiable at x = 1 2 4 2
d. None of the above p
134. The area under the curve y = | cos x - sin x |, 0 £ x £
126. If a function f : R ® R satisfy the equation 2
and above X -axis, is
f (x + y ) = f (x ) + f ( y ), " x , y and the function f (x ) is
continuous at x = 0, then a. 2 2 b. 2 2 - 2
a. f (x ) is continuous for all positive real values of x c. 2 2 + 2 d. 0
b. f (x ) is continuous for all x dy
135. The solution of = cos (x + y ) + sin (x + y ), is
c. f (x ) = 0 for all x dx
d. None of the above é æx + y öù
a. log ê1 + tan ç ÷ú + C = 0
-1
2x - sin x ë è 2 øû
127. The value of f ( 0), so that the function f (x ) = é æx + y öù
2x + tan-1 x b. log ê1 + tan ç ÷ú = x + C
ë è 2 øû
is continuous at each point in its domain, is
1 1 é æx + y öù
a. b. - c. log ê1 - tan ç ÷ú = x + C
3 3 ë è 2 øû
2 2 d. None of the above
c. d. -
3 3
136. The area enclosed by the curves y = x 3 and y = x is
128. Consider the greatest integer function, defined by 5 5
f (x ) = [x ], 0 £ x < 2. Then, a. sq units b. sq units
3 4
a. f is derivable at x = 1 b. f is not derivable at x = 1 5 12
c. sq units d. sq units
c. f is derivable at x = 2 d. None of these 12 5
æ 2 ö
129. Let f (x ) = - 2x 3 + 21x 2 - 60x + 41, then 1+ n
137. If lim çç an - ÷ = b , where a is finite number, then
a. f (x ) is decreasing in ( - ¥ , 1)
x ®¥
è 1 + n ÷ø
b. f (x ) is decreasing in ( - ¥ , 2) a. a = 2 b. a = 0
c. f (x ) is increasing in ( - ¥ , 1) c. b = 1 d. b = - 1
d. f (x ) is increasing in ( - ¥ , 2)
138. If the papers of 4 students can be checked by anyone of
130. Rolle’s theorem is not applicable for the function the 7 teachers, then the probability that all the 4 papers
are checked by exactly 2 teachers, is equal to
f (x ) = | x | in the interval [ - 1, 1] because
12 6
a. f ¢(1) does not exist a. b.
49 49
b. f ¢ ( - 1) does not exist 9 15
c. d.
c. f (x ) is discontinuous at x = 0 49 49
d. f ¢( 0) does not exist

131. If the curve y = a x and y = b x intersect at angle a, then


tan a is equal to
12 SELF STUDY GUIDE BITSAT
139. If equation (10x - 5 )2 + (10y - 4 )2 = l2 ( 3x + 4 y - 1)2 3p
145. If sin-1 x + sin-1 y + sin-1 z = and f (1) = 2,
represents a hyperbola, then 2
a. - 2 < l < 2 b. l > 2 f ( p + q ) = f ( p ) . f (q ), " p , q Î R , then
(x + y + z )
c. l < - 2 or l > 2 d. 0 < l < 2 x f ( 1) + y f ( 2) + z f ( 3 ) - f ( 1) is equal to
x + y f ( 2) + z f ( 3 )
140. Let a$ and b$ be two non-collinear unit vectors. If a. 0 b. 1
u = a$ - ( a$ × b$ ) b$ and v = a$ ´ b,
$ then | v | is equal to c. 2 d. 3
a. | u | 2 2+ 4 2+ 4 +6
146. The value of + + + .... is
b. | u | + | v . a$ | 1! 2! 3!
c. 2 | v | a. e b. 2e
d. | v | + u . ( a$ + b$ ) c. 3e d. None of these
147. If z1 , z 2 and z 3 represent the vertices of an equilateral
141. If the variance of the observations x1, x 2, ......., xn is s2,
triangle such that | z1| = | z 2| = | z 3 |, then
then the variance of ax1, ax 2, ......, axn , a ¹ 0 is
a. z1 + z 2 = z 3 b. z1 + z 2 + z 3 = 0
a. s2 b. as2
1
s2 c. z1 z 2 = d. z1 - z 2 = z 3 - z 2
c. a 2 s2 d. 2 z3
a
142. Coefficient of variation of two distributions are 50 and ( x )5 dx æ x9 ö
148. If ò = l log ç 9 ÷ + C , then a + l equal to
60 and their arithmetic means are 30 and 25,
7
( x) +x 6 ç x + 1÷
è ø
respectively. Difference of their standard deviation is a. 2 b. > 2
a. 0 b. 1
c. < 2 d. > 3
c. 1.3 d. 2.5
143. The maximum value of z = 9x + 13 y subject to 149. Line joining the points ( 0, 3 ) and (5, - 2) is a tangent to
ax
constraints 2x + 3 y £ 18, 2x + y £ 10, x ³ 0, y ³ 0 is the curve y = , then
a. 130 b. 81 1+ x
c. 79 d. 99 a. a = 1 ± 3 b. a Î f
c. a = - 1 ± 3 d. a = - 2 ± 2 3
144. A coin is tossed 7 times. Each time a man calls head.
The probability that he wins the toss atleast 4 occasions 150. The shortest distance between the parabolas y 2 = 4x
is
1 5 and y 2 = 2x - 6 is
a. b.
4 8 a. 2 b. 5
1 1 c. 3 d. None of these
c. d.
2 6
Answers
Physics
1. (a) 2. (d) 3. (c) 4. (b) 5. (b) 6. (b) 7. (b) 8. (b) 9. (c) 10. (b)
11. (b) 12. (a) 13. (d) 14. (b) 15. (c) 16. (d) 17. (c) 18. (d) 19. (b) 20. (c)
21. (b) 22. (c) 23. (b) 24. (d) 25. (c) 26. (c) 27. (c) 28. (a) 29. (a) 30. (a)
31. (d) 32. (c) 33. (b) 34. (a) 35. (c) 36. (c) 37. (d) 38. (c) 39. (b) 40. (b)

Chemistry
41. (b) 42. (b) 43. (b) 44. (b) 45. (a) 46. (c) 47. (a) 48. (d) 49. (b) 50. (d)
51. (d) 52. (b) 53. (c) 54. (d) 55. (d) 56. (c) 57. (a) 58. (a) 59. (b) 60. (a)
61. (d) 62. (a) 63. (a) 64. (d) 65. (c) 66. (d) 67. (c) 68. (c) 69. (a) 70. (a)
71. (d) 72. (b) 73. (d) 74. (b) 75. (d) 76. (d) 77. (a) 78. (c) 79. (d) 80. (d)

English Proficiency
81. (b) 82. (b) 83. (b) 84. (c) 85. (b) 86. (b) 87. (b) 88. (b) 89. (b) 90. (b)
91. (b) 92. (d) 93. (d) 94. (a) 95. (c)

Logical Reasoning
96. (c) 97. (d) 98. (a) 99. (d) 100. (a) 101. (a) 102. (b) 103. (c) 104. (c) 105. (a)

Mathematics
106. (c) 107. (b) 108. (a) 109. (a) 110. (d) 111. (c) 112. (d) 113. (a) 114. (c) 115. (c)
116. (a) 117. (d) 118. (b) 119. (b) 120. (d) 121. (a) 122. (c) 123. (a) 124. (b) 125. (b)
126. (b) 127. (a) 128. (b) 129. (b) 130. (d) 131. (b) 132. (d) 133. (b) 134. (b) 135. (b)
136. (c) 137. (c) 138. (b) 139. (c) 140. (a) 141. (c) 142. (a) 143. (c) 144. (c) 145. (c)
146. (c) 147. (b) 148. (b) 149. (b) 150. (b)
SOLUTIONS
Physics
1. (a) According to Stefan’s law r =
E
= sAeT 4 qE = mg
t mg
Þ q=
So, we can write as E
4 1.96 ´ 10- 15 ´ 9.8
E2 æT2 ö = coulomb
=ç ÷ 4 ´ 104
E1 çè T1 ÷ø
1.96 ´ 10- 15 ´ 9.8
E2 æ 900 ö
4
E2 = elementary charges
=ç ÷ Þ = ( 3 )4 4 ´ 104 ´ 1.6 ´ 10- 19
E1 è 300 ø E1
= 3e
E2
E2 = 81E1 Þ = 81 5. (b) Given, H = R
E1
u 2 sin2 q u 2 sin 2q
2. (d) ns = rw = 1.2 ´ 2pf [Qw = 2pf ] or =
2g g
æ 400 ö
= 1.2 ´ 2 ´ 3.14 ´ ç ÷ = 50 m/s 2u 2 sin q cos q
è 60 ø =
g
v
n min = ×n or tan q = 4
v + vs
Þ q = tan- 1( 4)
340
= ´ 500
340 + 50 6. (b) Given, a = 6 cm = 6 ´ 10-2 m
= 436 Hz w = 100 rad/s, m = 1kg
v 1
n max = ×n K max = mw2a 2
v - vs 2
340 1
= ´ 500 = ´ 1 ´ (100)2 ´ ( 6 ´ 10- 2 )2
340 - 50 2
= 586 Hz = 18 J

3. (c) We have, fr - fb = wfy 7. (b) From triangle ABC


fr - fb A
w=
fy q

Here, fr = 100 cm
fb = 96.8 cm
C Mv
fy = fb ´ fr
B Ö2qB
= 96.8 ´ 100 Mv
O qB
= 98 cm
\ Dispersive power mv
f -f l 2 qB
w= r b sin q = =
fy H mv
100 - 96.8 qB
=
98 1
=
3.2 2
= = 0.0325
98 Þ sin q = sin 45°
4. (b) V = 800 V p
= sin
V 800 4
E= = V/m 2pm
d 2 ´ 10- 2 Time period for angle 2p is T =
qB
= 4 ´ 104 V/m
SOLVED PAPER 2017 15
\ Time by charged particle to emerge from magnetic field \Magnetic flux f = B × A = B × pr 2
pm df
is . Induced emf | e | =
4qB dt
8. (b) Given, l1 = 12000 Å dr
= B p 2r ×
and l 2 = 10000 Å dt
D = 2 cm = 0.025 ´ p ´ 2 ´ 2 ´ 10- 2 ´ 1 ´ 10- 3
d = 2 mm = 2 ´ 10- 3 m = pm V
We have, 12. (a) Increase in length
l1 n 2 12000 6
= = = L L
l 2 n1 10000 5
B C
n lD 90–q D 90–q
as X = 1 1 (n1 = 5) X
d q q
5 ´ 12000 ´ 10- 10 ´ 2 O
=
2 ´ 1 0- 3
DL = BO + OC - (BC )
= 5 ´ 1.2 ´ 104 ´ 10- 10 ´ 103 = 6 mm = 2BO - 2BD
9. (c) Equation of motion of lower block is Þ DL = 2BO - 2L
1
F = m(M A + MB ) × g
= 2 [L2 + x 2] 2 - 2 L
F
m = 1
(M A + MB )g é x2ù2
or DL = 2 L ê1 + 2 ú - 2L [Q using Bionomial theorem]
49 1 L û
= = = 0.5 ë
10 ´ 9.8 2
é 1 x2 ù
Þ DL » 2L ê1 + 2
- 1ú
ë 2 L û
10. (b) Given R = 3 km = 3000 m
u x2
= [Qx << L]
L
DL
\ Strain =
2L
200 cm

x2
x2
= L = 2
2L 2L
A B
3 km 13. (d) We have, Rt = R0(1 + at )
2h Here, 20 = R0(1 + 20a)
Range, R = u
g 60 = R0(1 + 500 a)
g 10 60 R0(1 + 500a)
Þ u =R or u = 3000 ´ Þ =
2h 2 ´ 2000 20 R0(1 + 20a)
1 1+ 500a
= 3000 ´ 3=
20 1 + 20a
= 150 m/s Þ 3(1 + 20a) = 1 + 500a
18 Þ 3 + 60a = 1 + 500a
= 150 ´ km/h
5 Þ 440a = 2
= 540 km/h 2 1
Þ a= = I° C
11. (b) Here, magnetic field, B = 0.025 T 440 220
16 SELF STUDY GUIDE BITSAT
Also given Rt = 25° C, t = ? Also power gain = b 2 ´ impedance gain
æ 20 ö 200
R0 ç1 + ÷ = ( 25)2 ´ = 1250
20 è 220 ø 100
Now, =
25 æ t ö
R0 ç1 + ÷ 2U y
è 220 ø 18. (d) T =
g
æ t ö æ 20 ö
4 ç1 + ÷ = 5 ç1 + ÷ U y2
è 220 ø è 220 ø H=
2g
On solving,
t = 80° C and R = U x ´ T

14. (b) de-Broglie wavelength, When a horizontal acceleration is also given to the projectile
h h h U y , T and H will remains unchanged while range will
l= = = become
p 2 mE 2 mqV
1
R ¢ = U x ´ T + aT 2
6.6 ´ 10- 34 2
Þ l=
2 ´ 1.6 ´ 10- 27 ´ 1.6 ´ 10- 19 ´ 1000 2
1 g æç 4U y ö÷ é gù
- 34 =R + =R +H êëQ a = 4 úû
6.6 ´ 10 2 4 çè g ÷ø
2
Þ l= = 0.9 ´ 10- 12 m
7.16 ´ 10- 22
PV
1 1 PV
15. (c) Let the volume of ice-berg isV and its density is r. If this 19. (b) We have, = 2 2
T1 T2
ice-berg floats in water with volumeVin inside it, then
PVT
1 1 2
V insg = Vrg or V2 =
P2T1
s = density of water
1 ´ 500 ´ (273 - 3) 1 ´ 500 ´ 270
ærö = =
Þ V in = ç ÷ × V 0.5 ´ (273 + 27) 0.5 ´ 300
è sø
V2 = 900 m3
Þ Vout = V - V in
é s - rù 1000 - 900 20. (c) Intensity at centre of bright fringe,
=ê V= V
ë s úû 1000 I 0 = I + I + 2 II × cos 0
V I 0 = 2I + 2I [Q cos 0° = 1]
=
10 = 4I
Þ Vout / V = 0.1 = 10% P
Similarly Intensity at point
16. (d) We have, Kinetic energy of electron, 4
Ze 2 2p p
K = (with phase difference = = )
8pe0 × r 4 2
p
also, Potential energy of electron, I ¢ = I + I + 2 II cos
2
1 Ze 2
U =- × = 2I + 2 II ´ 0
4pe0 r
= 2I
\Total energy I 0 4I
E =K +U \ = =2
I ¢ 2I
2 2
Ze Ze
= - 21. (b) Let block is displaced through x m, then weight of
8pe0 × r 4pe0 × r
displaced water or upthrust (upwards)
Ze 2 = - Axrg
E=- or K = - E = - ( - 3.4)
8pe0r
Where, A is area of a cross-section of the block and r is
= 3.4 eV density. This must be equal to force (= ma) applied where m
17. (c) Voltage gain = b ´ impedance gain is mass of the block and a is acceleration.
200 \ ma = - Axrg
Þ 50 = b ´
100 Arg
or a=- x = - w2x
Þ b = 25 m
SOLVED PAPER 2017 17
This is equation of SHM. Taking log on both sides, we get
The time period of oscillation, 5 1
log 1 - log e = log
2p m T 2
T = = 2p 5
w Arg Þ - 1 = ( - log 2)
T
1
Þ T µ Þ T = 5 loge 2
A
Now, let t ¢ be the time after which activity reduces to half
22. (c) Resultant force
æ 1 ö æ 1 ö t ¢ / 5 loge 2
é -1ù ç ÷ =ç ÷
F ¢ = F 2 + F 2 + 2FF cos 120° = F êQ cos120° = ú è 2ø è 2ø
ë 2û
Þ t ¢ = 5 loge 2
F
25. (c) Focal length of combination
F¢ +Q 1 1 1
120° A = + ...(i)
F
f f1 f2

F¢¢ Using lens maker’s formula


1 æ 1 1ö (m - 1)
= (m 2 - 1) çç - ÷÷ = - 2
f2 è-R ¥ø R
B C
–Q +Q 1 æ1 1 ö m1 - 1
= (m 1 - 1) çç - ÷=
Now, from figure f1 è ¥ - R ÷ø R

F = F ¢2 + F ¢¢2 + 2F ¢ F ¢¢ cos 90°


1 (m - 1)
Now, the force normal to BC at vertex A is or =
f1 R
F ¢¢ = F 2 - F ¢2 = 0 (QF ¢ = F ) 1 (m - 1)
=- 2
f2 R
23. (b) From the conservation principle,
1
mgh = kX 02 - mgX 0
2
where, X 0 is maximum elongation in spring. 45°
1 45°
Þ kX 02 - mgX 0 - mgh = 0
2
2mg 2mg
Þ X 02 - X0 - h=0
k k
mg æ 2mg ö
2
2mg Putting these values in Eq (i), we get
2 ± ç ÷ + 4´ h 1 (m 1 - 1) (m 2 - 1)
2 è k ø k Þ = -
X0 = f R R
2
1 (m 1 - 1 - m 2 + 1)
Amplitude = elongation in spring for lowest extreme position Þ =
f R
- elongation in spring for equilibrium position 1 m1 - m 2
2hk Þ =
mg é mg ù f R
= X 0 - X1 = 1+ êëQ X1 = k úû
k mg R
f =
24. (d) Fraction remains after n half-lives (m 1 - m 2 )
t
N æ 1ö
n 26. (c) Current will flow only through the branch containing
æ 1ö T
= ç ÷ =ç ÷ resistance R2
N0 è 2 ø è 2ø
E 5
5 \ i= = = 1A
N0 N0 æ 1ö T R2 + r 4 + 1
Given, N= Þ =ç ÷
e eN0 è 2 ø Potential difference across R2
5 = 1´ 4 = 4 V
1 æ 1ö T
Þ =ç ÷ If q be the charge on each plate of capacitor, then
e è 2ø
q q
+ =4
c c
18 SELF STUDY GUIDE BITSAT
2q Substituting the given values in expression for torque, we
or =4
c get
2q 3 2
or =4 t =i ´ l B sin 90°
3 ´ 10- 6 4
q = 6 mC 3 2
t= il B (Q sin 90° = 1)
27. (c) Friction force between A and B and between block B and 4
1
surface S will oppose F æ t ö2
\ F = FAB + FBS = m ABM Ag + m BS (mA + mB )g Here, l = 2 çç ÷
÷
è 3 Bi ø
= 0.2 ´ 100 ´ 10 + 0.3(100 + 200) ´ 10
= 200 + 900 = 1100 N 31. (d) Work done = Change in surface energy
r2 = 5 cm
28. (a) The moment of inertia of the uniform rod about an axis
through one end and perpendicular to its length is r1 = 3 cm

ml 2 Þ W = 2T ´ 4p(r22 - r12 )
=
3 = 2 ´ 0.03 ´ 4p [(5)2 - (3)2] ´ 10- 4
where, m is the mass and l its length. = 0.4p mJ
Torque (t = I × a) acting on centre of gravity of rod is 32. (c) From the figure, the X-component remains unchanged
given by while theY-component is reverse. Then, the velocity at point
t = mg
I B is ( 2i$ - 3$j) m/s.
2
33. (b) H = l 2 Rt
ml 2 l
also, × a = mg 2
3 2 H1 æ I1 ö æR ö
\ = çç ÷÷ ´ çç 1 ÷÷
3g H2 è I 2 ø è R2 ø
a=
2l
é ù
29. (a) As H-atom emits 6 spectral lines
n(n - 1)
=6 10 ê 1 ú 5
or =ê ú´
2 H2 ê æ 1 ö ú 4
ç ÷
\ n=4 êë è 2 ø úû
DE = E4 - E1 = 13.6 - 0.85 H2 = 2 cal
= 12.75 eV
34. (a) Given KE of a-particle = 2 eV
1242 nm
l= ( Ze ) (e )
12.75 r =
4pe0 × ( KE)
= 97.5 nm
2Ze ´ 9 ´ 109
30. (a) Torque acting on equilateral triangle in a magnetic field =
2V
B is
. ´ 10-19 ´ 9 ´ 109
2 ´ Z ´ 16
t = i AB sin q Þ r =
2V
L
Z
= 14.4 × Å
V
B l 35. (c) According to question
n1T1 = n 2T2
5 20
Þ n × 2p = (n - 1) 2p
g g
M O N n
=2 Þ n=2
n -1
Normal
36. (c) The two capacitors formed are in parallel, hence
Area of triangle LMN, capacitance of the combination
3 2 C1C2
A= l and q = 90° C= ...(i)
2 C1 + C2
SOLVED PAPER 2017 19
K1e0A æ e Aö Simiarly T22 µ r23
where, C1 = çQ C = 0 ÷ ...(ii)
d è d ø
\ T12 µ ( 36000)3
3
K 2e0A T22 µ (6400 + h )3
C2 = ...(iii)
2d Therefore,
3 3
æ 6400 + h ö
3K1e0A K 2e0A × 3 (T2 )2 = T12 ç ÷
´ è 36000 ø
Ceq = d 2d 3
3K1e0A 3K 2e0A é 6400 ù
+ > T12 ê
d 2d ë 3600 úû
9K1K 2e02A 2 d é 8ù
3
= ´ > ( 24)2 ê ú
2d 2 e0A ´ 18 ë 45 û
It is given 24 ´ 8 ´ 8
\ T2 >
45 ´ 45
2d/3 d/3
K1 K2 > 1.8 h
So, T2 @ 2 h
e0A 39. (b) Equation of wave,
= 9 pF
d y = 0.2 sin [1.5x + 60t ]
Using given values Comparing with standard equation, we get
Ceq = 40 . 5 pF y = A sin [kx + wt ]
2p
37. (d) Acceleration k = 1.5 =
l
dv æ tö
f = = f0 ç 1 + ÷ 2p
dt è Tø and w = 60 =
T
æ tö w
or dv = f0 ç1 - ÷ × dt ...(i) \Velocity of wave, v =
è Tø k
Integrating Eq. (i) on both sides, we get 60
= = 40 m/s
f0 t 2 1.5
v = f0t - × +C ...(ii)
T 2 T
Velocity of wave in stretched string,v = ,
After applying boundary conditions m
v = 0 at t = 0 where, m is the linear density,T is tension in the string.
We get, C=0 So, T = v 2 m = ( 40)2 ´ 3 ´ 10- 4
2
f0 t = 0.48 N
Þ v = f0t - × ...(iii)
T 2
40. (b) Distance of straight path from O
æ tö
As f = f0 ç1 - ÷ r
è Tø =
2
When f0 = 0, t = T
Hence, B = B1 + B2
Substituting t = T in Eq. (iii), then velocity m 2i m 2pi æ 1 ö
f0 T 2 1 = 0× + 0 ç ÷
vX = f0T - × = f0T 4p r 4p r è 4 ø
T 2 2 2
38. (c) By Kepler’s law of planetary motion, m 0 2i é pù
= × 2+ ú
2
T µr , 3
hence T12 µ r13 4p r êë 4û
20 SELF STUDY GUIDE BITSAT

Chemistry
41. (b) Q 4g of copper gave 5g of its oxide means, 48. (d) Reaction (I), (II) and (IV) are of exothermic nature, thus
1 g of oxygen combine with 4g of copper. do not involve absorption energy.
Q Eq. wt of oxygen = 8, 49. (b) More be the ability of N-atom of ¾ NH2 group to donate
Therefore, lone pair of electrons to HCl (dil.), more be the reactivity of
8g of oxygen combine with = 4 ´ 8 g of copper = 32 g amine.
Hence, In (b) two alkyl groups are attached to the ¾ NH2 (having
Equivalent weight of copper = 32 electron releasing nature) which increases the electron
donating ability of N-atom of ¾ NH2.
42. (b) Option (b) is the incorrect statement as reaction of Hence is most reactive.
NaBH4 with cold water is very slow. It is not violent. All other
are correct statements. Note ( i ) T h o u g h ( c ) i . e . (CH3 ) 3 N h a s t h r e e a l k y l g r o u p s a t t a c h e d w i t h t h e
N -a to m , b u t
43. (b) Q for s -subshell. d u e t o s t e r ic -e ffe c t , N -a t o m c a n n o t a b le t o d o n a t e lo n e p a ir o f
l=0 e le c t r o n s .
Hence, orbital angular momentum of an electron is 2s (ii) In (d ), d u e t o r e s o n a n c e , lo n e p a ir o f e le c t r o n s g e t d e lo c a lis e d .
orbital = zero. 50. (d) Block of magnesium prevents the action of water and
é h ù salt of iron (of ship) by sacrificing itself.
êëQ orbital angular momentum = l (l + 1) . 2p úû
This is method of protection of ship (i.e. of iron), known as
+
44. (b) I. Number of electrons in C H3 = 6 + 3 - 1 = 8 sacrificial protection,

II. Number of electrons in NH-2 = 7 + 2 + 1 = 10 51. (d) Q Only one mole of total gases are evolved and H2O
+
III. Number of electrons in NH = 7 + 4 - 1 = 10
4 decomposes as
IV. Number of electrons in NH3 = 7 + 3 = 10 decomposition 1
H2O ¾ ¾ ¾ ¾ ¾ ¾
¾® H2 + O2
2
Since, species with same number of electrons are called
1 3
isoelectronic species, Hence, IInd, IIIrd and IVth are i.e. total moles = 1 + = moles by one mole of H2O
isoelectronic. 2 2
Now,
45. (a) Q Difference of electronegativity between (A) and (B) 3
= 3 . 0 - 1. 2 = 1. 8 and, a bond having electronegativity Q moles of gases are evolved by = 1mole of H2O
2
difference greater than 1.65 is of ionic nature. 1 2
\ 1 mole of gases are evolved by = =
Hence, bond between (A) and (B) would be of ionic nature. 3/2 3

46. (c) The hydrogen, attached with sp-hybrid C-atom is most 2


Hence, 1 mole of total gases are evolved by mole of H2O.
acidic because of highest electronegativity of sp hybrid 3
carbon atom, H atom becomes relatively more acidic. 52. (b) Q Reaction between Fe and HCl is as follows
47. (a) Reductive ozonolysis of (CH3 )2 £ CH(CH3 )2 followed by Fe + 2HCl ¾® FeCl2 + H2 on reaction
( + 2)
hydrolysis gives only one type of ketones because of n (moles of Fe) = n (moles of HCl)
symmetry across the C == C and bonding of two same alkyl
groups (CH3 ) with both the carbons bonded through double n( Fe) = Normality (HCl) ´ Volume of HCl
bond. 0.4 ´ 500
n( Fe) = = 0. 2 moles
CH3 CH3 (i) Ozonolysis 1000
C C As each Fe consumers 2 electrons to change to Fe2+ ion.
(ii) hydrolysis
CH3 CH3 0. 2
\ Number of Fe2+ ions produced = = 0.1 mole
CH3 CH3 2
C O+O C
CH3
53. (c) For dissolution hydration energy must be more higher
CH3
(aceton) (-ve) value than of lattice energy. Thus, (c) is the correct
answer.
SOLVED PAPER 2017 21
54. (d) In the given reaction the end product is (d). It is obtained 12 ´ 70
\ 70 g of CO2 gives, C =
as follows. 44
CH3 CH2 × Cl ¾ KCN
¾¾® CH3 CH2CN ¾ LiAlH
¾¾ 4
® = 19.09 g
(Reduction)
(A )
\ % of carbon = 19.00
CH3 CH2CH2 × NH2 Similarly,
55. (d) Conversion of carbonate are into its oxide (by heating in (i) for hydrogen
absence of air) is known as calcination. % of H2O
\(d) is the correct answer i.e. 0.138 ´ 100
Þ
0.765
D
MgCO3 ¾® MgO + CO2(g ) = 18.03 = 18.00
56. (c) We know that, DH = Ef - Eb % of Hydrogen
2 ´ 18
where, Eb & Ef = activation energy for backward and forward Þ = 2.00
18
respectively.
Hence ratio of carbon and hydrogen is 19 : 2
Thus, Ef = DH + Eb , means Ef > DH.
64. (d) Least basic has maximum value of pKb .
57. (a) Ag is leached by cyanide process by making the
Since, in (d), lone pair of electrons over N-atom are
complex using NaCN/KCN as follows
delocalised to two benzene rings, thus is least basic.
Ag2S + 4NaCN ¾® 2Na[Ag(CN)2] + Na 2S
Soluble
65. (c) Q for (l ) It can be upto (n - 1) and
58. (a) Among the given option (a) is a diamagnetic complex. Thus, m = ± l, i.e. + l, o, - l
(a)[Co(NH3 )6]3+ Þ Co (27)Þ Co 3+ = 3d 6 4s 0. Thus, has no for (a) n = 2, l = 0, m = 0 (correct),
unpaired electrons and is a diamagnetic complex.
for (b) n = 1, l = 0, m = 0 (correct),
(b) [NiCl 4 ] 2- Þ Ni (28) Þ Ni 2+ = 3d 8 4s 0
(c) n = 3, l = 3, (not correct),
Thus, has two unpaired electrons is paramagnetic.
(d) n = 2 , l = 1, m = 1(correct),
(c) [CuCl 4 ]2- Þ Cu (29)Þ Cu 2+ = 3d 9 4s 0
Thus, has one unpaired electrons and is paramagnetic.
66. (d) More higher be the ( + ) ve oxidation number of N-atom in
its oxide more acidic be the nature of oxide. Thus N2O5 is
(d) [Fe(H2O)6]3+Þ Fe (26) Þ Fe3 + = 3d 5 4s 0
most acidic oxide.
Thus, has five unpaired electrons and is paramagnetic.
Nb - Na
59. (b) Neoprene is a synthetic rubber. It is a polymer of 67. (c) Q Bond order ( BO) =
2
chloroprene (2-chlorobuta,1, 3 diene).
Thus,
60. (a) Ionic solids have highest melting point due to strong 10 - 6
for O 2, BO = =2
electrostatic force of attraction. 2
10 - 7
61. (d) Night blindness is developed due to deficiency of for O -2 , BO = + 15
.
2
vitamin-A.
10 - 5
for O +2 , BO = = 2.5
62. (a) The transfer RNA anticodon for messanger RNA codon 2
G - C - A is 10 - 8
for O 22- , BO = = 10
.
C - G - U, because in anticodon of RNA, 2
G replaced by C Hence maximum value for BO is for O+2 .
C replaced by G
68. (c) Option (c) is the correct option as in (i), (ii) and (iv) the
A replaced by U state changes towards more random state, thus entropy
63. (a) % for CO2 increases.

Q 0.765 g of acid gives CO2 = 0.535 g 69. (a) BF3 is a Lewis acid, as it accepts lone pair of electrons.
0.535 ´ 100
\ 100 g of acid gives CO2 = It is not an Arrhenius acid because it does not furnish H+ ion
0.765
in its aqueous solution.
% of CO2 = 70.00
It is also not a Bronsted Lowry acid as if does not give H+
% for carbon
ions in any solvent.
Q 44 g of CO2 gives, C = 12 g
22 SELF STUDY GUIDE BITSAT
70. (a) Q DH = DE + DngRT Number of unpaired electrons in V 3 + = 2
and Dn (for the given reaction)= 2 - 1 = + 1 Number of unpaired electrons in Fe2+ = 4
Thus, Fe2+ show maximum value of magnetic moment (m ).
\ DH > DE
also, m = n(n + 2) . BM(n = Number of unpaired
71. (d) First member of 1st group i.e. lithium, mostly form electrons).
covalent compounds due to its small size and comparatively
high I.E. (with respect to other members of its group). 77. (a) Q Elements with atomic Number 90 to 103 belongs to
same group, i.e. III-B 3rd group (i.e. actinoids)
72. (b) When aqueous solution of borax (Na 2B4 O7 ) is acidified
78. (c)
with HCl, we get H3BO 4 and NaCl as follows
Cl
Na 2B4 O7 + 2HCl + 5H2O ¾® 2NaCl + 4H3 BO3 (i) NaOH (ii)H+
Borox Boric acid (X)=Dows process

73. (d) An aromatic compound can follow Huckel’s rule if it has OH


(i) CHCl3+NaOHCal (ii) H+
( 4n + 2)p-electrons Salicylaldehyde
(Y)=Reimer tiemann
where, n = integer 0, 1, 2, 3, ... etc. Phenol
Thus,
10
(a) n = 0; 4 ´ 0 + 2 = 2 p 79. (d) Qn = (actual mass) (molar mass)
m
\ follow Huckel’s rule
for
(b) n = 1; Number of p-electrons = 6 w 8 1
\ follows Huckels rules. (a) n = = = mole
m 16 2
(c) n = 1; Number of p-electrons = 6 16 1
(b) n = = mole
\ follows Huckel’s rule. 32 2
(d) n = 1, Number of p-electrons = 8 14 1
(c) n = = mole
( 4n + 2 = 4 ´ 1 + 2 = 6) 28 2
but this compound has 8 electrons which distorts the plane (d) Thus, all have same number of moles.
and hence does not follows Huckel’s rule. 80. (d) Q For ideals gas
74. (b) According to Arrhenius equation’s pV = nRT
Ea 1 pV
log K = log A - or T =
2.303 R T nR
Q n = 1 = constant
on comparing it with the equation of st. line y = mx + c plot
(b) is the correct answer. R = 0.083 = constant
\ T µ pV
75. (d) Q CN- is a strong ligand and from six Coordinats bonds
Thus,
with Fe-elements. Thus Fe show d 2sp 3 hybrisation in
K 3 Fe(CN)6. The gas that shows highest product of p ´ V , has highest
temperature.
76. (d) More be the number of unpaired electrons ® more be for (A) p ´ V = 1 ´ 5 = 5
the magnetic moment. (B) p ´ V = 2 ´ 2 . 5 = 5
and (C) p ´ V = 2 ´ 1. 25 = 2 .5
Number of unpaired electrons in Mg2+ = 0
(D) p ´ V = 2 . 5 ´ 2 . 5 = 6 . 25
Number of unpaired electrons in Ti3 + = 1
Hence gas (D) show highest temperature.
SOLVED PAPER 2017 23
English Proficiency 89. (b) Deteriorating means becoming progressively worse
worsening.
81. (b) 'Are’ should be used in place of ‘is’. 90. (b) Stalemate means a situation in which neither group
82. (b) ‘Was’ should be used in place of were. involved in an argument or an act can win or get an
advantage and no action can be taken.
83. (b) Luggage should be used in place of luggages.
91. (b) The answer is clear from the second paragraph of the
84. (c) Gloss over something is used. passage.
85. (b) Preposition up is used with muster. 92. (d) The answer is clear from the sentence Designed to
86. (b) IIlusion means a false idea or belief; a deceptive demonstrate the damage alcohol can do ……
appearance or impression. Its antonym will be reality 93. (d) The answer is clear from the sentence …… who came to
87. (b) Disparage means to criticise someone in a way that him …… had not realised how much they were drinking
shows that you do not respect or value him. Its antonym will 94. (a) The answer is clear from the sentence ‘Almost everyone
be praise poured out not a single measure, but a double at least …… .
88. (b) Slackened means reduce or decrease in speed or 95. (c) The answer is clear from the first sentence of the last
intensity. Its antonym will be quickened. paragraph.

Logical Reasoning
96. (c) ‘Leather’ is a raw material used to make ‘Shoes’. Same Meenu > Hari > Latha …(iii)
as, ‘Rubber’ is made using ‘Latex’ which is a raw material. After arranging the equations,
97. (d) Here, 81 ´ 3 = 243 Lakshmi > Leela > Meenu >Hari > Latha
25 ´ 3 = 75 \Latha is the youngest.
64 ´ 3 = 192 100. (a) Here, all the three equal parts have same design. So, we
but 16 ´ 4 = 64 can obtain the answer figure for the missing portion by
We can see that in all options second number is three rotating question figure by 90° clockwise.
times the first number except 16 : 64.
Hence, option(a) is the right answer.
Hence, 16 : 64 is odd one.
101. (a) Out of five given figures, we can take figure (A), (C) and
98. (a) The sequence to given series is as follows (D) to make a square.
4 11 30 67 128 219
102. (b) After falding, paper step, by step and then cut as shown
+7 +19 +37 +61 +91 in the question, it will show as an option (b).
+12 +18 +24 +30 103. (c) Moving row wise the number of horizontal lines
+6 +6 +6 increases by one in each step continuing the process, we
will get figure of option (c) which will replace the question
Hence, 219 will come in place of question mark‘?’. mark‘?’.
99. (d) From given information, we have 104. (c) Required figure given in option (c).
Lakshmi > Meenu …(i)
105. (a) There are 28 triangles in the given figure.
Lakshmi > Leela > Meenu …(ii)

Mathematics
106. (c) (1 + x )21 + (1 + x )22 + ...... + (1 + x )30 \ Coefficient of x 5 in the given expression
1
= (1 + x )21 [1 + (1 + x )1 + .... + (1 + x )9] = Coefficient of x 5 in [(1 + x )31 - (1 + x )21]
x
é (1 + x )10 - 1ù
= (1 + x )21 ê ú = Coefficient of x 6 in [(1 + x )31 - (1 + x )21]
ë (1 + x ) - 1 û
= 31C6 - 21C6
1
= [(1 + x )31 - (1 + x )21]
x
24 SELF STUDY GUIDE BITSAT
107. (b) We have, 1
Þ sin 2a = Þ 2a = 30° [Q sin 30° = 1/ 2]
arg ( z - 1) = arg ( z + 3 i ) 2
Þ arg ((a - 1) + ib ) = arg (a + (b + 3) i ) Thus, the first three terms of the AP are 1, 14, 27.
æ b ö Hence, its fifth term a 5 = a1 + (5 - 1) d
b + 3ö
Þ tan-1 çç ÷÷ = tan-1 æç ÷ = 1 + 4 ´ 13 = 1 + 52 = 53
è a - 1ø è a ø
b b+3 111. (c) Let a, a 2 be the roots of 3x 2 + px + 3
Þ =
a -1 a \ Product of the roots,
Þ ab = (a - 1) (b + 3) a × a2 = 1 Þ a3 = 1
2
Þ 3 (a - 1) = b Þ a = 1, w, w
Þ (a - 1) : b = 1: 3 Again, a + a2 = -
p
3
x2 y2
108. (a) Let the ellipse be 2 + 2 = 1 p
a b Þ 1+ 1= - (if a = 1)
3
x cos q y sin q
and let + =1 …(i) Þ p=-6
a b
be a tangent to it at point (a cos q, b sin q). Then, p = length But p > 0
of the perpendicular from S (ae, 0) on Eq. (i) \ a = 1is not possible.
If a = w, then a + a 2 = w + w2 = - 1
e cos q - 1 p
Þ p= \ - 1= - Þ p=3
3
cos2 q sin2 q
+ Again, if a = w2, then
a2 b2
a + a 2 = w2 + w4 = w2 + w = - 1
p ¢ = length of the perpendicular from O ( 0, 0) on Eq. (i)
p
\ - 1= -
3
1
Þ p¢ = Þ p=3
cos q sin2 q
2
+ 112. (d) We have,
a2 b2
and r = ae cos q - a log140 63 = log22 ´ 5 ´ 7 ( 3 ´ 3 ´ 7)
log2 ( 3 ´ 3 ´ 7) log2 3 + log2 3 + log2 7
Clearly, rp ¢ = ap = =
log2 ( 22 ´ 5 ´ 7) 2 log2 2 + log2 5 + log2 7
109. (a) We have, 1
2a +
10 n
Cr 10
= c = 2ac + 1
år . n
= å (n - r + 1) 1 2c + bc + 1
r =1 Cr - 1 r = 1 2+b +
c
10 n 10
Cr
Þ år .
r =1
n
= å {(n + 1) - r }
Cr - 1 r = 1 113. (a) Given that, cos (x - y ), cos x , cos (x + y ) are in HP.
2 cos (x - y ) cos (x + y )
10 n
Cr 10
Then, cos x =
Þ år .
r =1
n
Cr - 1
= 10 (n + 1) - å r
r =1
cos (x - y ) + cos (x + y )
2 (cos2 x - sin2 y )
10
C n
Þ cos x =
Þ å r . n C r = 10 (n + 1) - 55
r =1
2 cos x cos y
r -1
Þ cos2 x cos y = cos2 x - sin2 y
= 10n - 45 = 5 ( 2n - 9 )
Þ cos2 x (cos y - 1) = - sin2 y
2 sin 2a -1 4 - 2 sin 2a
110. (d) Since, 3 , 14 and 3 are in AP. Therefore, Þ cos2 x (1 - cos y ) = 1 - cos2 y
2 ´ 14 = 32 sin 2a -1
+ 34 - 2 sin 2a Þ cos2 x (1 - cos y ) = (1 - cos y ) (1 + cos y )
a 34 Þ cos2 x = 1 + cos y Þ cos2 x = 2 cos2 y / 2
Þ 28 = + , where a = 32 sin 2a
3 a Þ cos2 x sec2 (y / 2) = 2
Þ a 2 - 84a + 243 = 0 Þ cos x sec (y / 2) = ± 2
Þ (a - 81) (a - 3) = 0
114. (c) We first write the elements of the set R.
Þ a = 81, a = 3
i.e. R = {(1, 4), (4, 1), (2, 3), (3, 2)}
Þ 32 sin 2a = 34 or 32 sin 2a = 3
Since, (1, 1) ÏR Þ R is not reflexive.
Þ 2 sin 2a = 1 [Q 2 sin 2 a ¹ 4]
SOLVED PAPER 2017 25
Now as, (1, 4) ÎR Þ ( 4, 1) ÎR and ( 2, 3) ÎR Þ (sin 2x - cos 2x ) ( 2 cos x - 3) = 0
Þ ( 3, 2) ÎR Þ sin 2x = cos 2x [Q cos x ¹ 3 / 2]
So, R is symmetric æp ö
Þ 2x = 2xp ± ç - 2x ÷
Now, (1, 4) ÎR and ( 4, 1) ÎR è 2 ø
Þ
/ (1, 1) ÎR np p
Þ x = + [Q neglect - ve sign]
Thus, R is not transitive. 2 8
Hence, R is symmetric but neither reflexive nor transitive. 119. (b) The third side is parallel to a bisector of the angle
115. (c) Any number between 1 to 999 is of the form abc when between equal sides.
0 £ a, b, c £ 9. Let us first count the number in which 5 The bisectors are 7x - y + 3 = ± 5 (x + y - 3)
occurs exactly once. Þ 2x - 6y + 18 = 0
Since, 5 can occur at one place in 1 ´3 C1 ´ 9 ´ 9 = 243 or 12x + 4y - 12 = 0
ways, next 5 can occur in exactly two places in 3 C2 ´ 9 = 27. Þ x - 3y + 9 = 0
Lastly, 5 can occur in all three digits in only one way. or 3x + y - 3 = 0
Hence, the number of times 5 occurs Let the third side be x - 3y = k or 3x + y = L
= 1 ´ 243 + 27 ´ 2 + 1 ´ 3 It passes through (1, - 10).
= 243 + 54 + 3 = 300 k = 31, L = - 7
116. (a) Given, that, A È X = B È X Hence, required lines are x - 3y = 31, 3x + y = - 7

Þ A Ç ( A È X ) = A Ç (B È X ) 120. (d) Let (t , m ) be the other end of the chord drawn from the
Þ (A Ç A) È (A Ç X ) = (A Ç B) È (A Ç X ) point ( p, q ) on the circle
[Using distributive law] x 2 + y 2 = px + q y
Þ A È f = ( A Ç B ) È f [Q A Ç X = f] æt + p m + q ö
Their mid-point is ç , ÷
Þ A = A ÇB …(i) è 2 2 ø
Again, consider A È X = B È X Since, mid-point lies on X-axis i.e. y = 0
Þ B Ç ( A È X ) = B Ç (B È X ) m+q=0 …(i)
Þ (B Ç A ) È (B Ç X ) = (B Ç B ) È (B Ç X ) Also, (t , m ) lies on the circle.
Þ (B Ç A ) È f = B È f [ B Ç X = f] \ t 2 + m 2 - pt - qm = 0 …(ii)
Þ A ÇB = B …(ii) From Eqs. (i) and (ii), we get t 2 - pt + 2q 2 = 0
Thus from Eq. (i) and (ii), we get Which is quadratic in t such that, Discriminant > 0
A =B Þ p 2 - 8q 2 > 0 Þ p 2 > 8q 2
ì x 2, x ³0 121. (a) Let P be the foot of the perpendicular drawn from
117. (d) f (x ) = x | x | = í
î- x , x < 0
2
A ( 2, 3, 4) to the given line l.
Y x - 4 y z -1
Then, = =
-2 6 -3
Now, any point on the line l is given by

X¢ X x = 4 - 2l, y = 6l, z = 1 - 3l
The coordinates of P are ( 4 - 2l, 6l, 1 - 3l )
x=–1 The direction ratios of AP are
x=1
Y¢ ( 4 - 2l - 2, 6l - 3, 1 - 3 l - 4)
i.e. ( 2 - 2l, 6l - 3, - 3 - 3l )
Since, - 1 £ x £ 1, therefore - 1 £ f (x ) £ 1
And the direction ratios of l are - 2, 6 and - 3.
\ Function is one-one and onto.
Given, AP ^ l
118. (b) We have, \ - 2 ( 2 - 2l ) + 6 ( 6l - 3) - 3 ( - 3 - 3l) = 0
sin x - 3 sin 2x + sin 3x = cos x - 3 cos 2x + cos 3x Þ l=
13
Þ sin x + sin 3x - 3 sin 2x = cos x + cos 3x - 3 cos 2x 49
Þ 2 sin 2x cos x - 3 sin 2x - 2 cos 2x cos x + 3 cos 2x = 0 \ AP 2 = ( 4 - 2l - 2)2 + ( 6l - 3)2 + (1 - 3l - 4)2
Þ sin 2x ( 2 cos x - 3) - cos 2x ( 2 cos x - 3) = 0 = 22 - 26l + 49 l2
26 SELF STUDY GUIDE BITSAT
13 ì ex, x £0
Put l= , we get
49 ï
125. (b) f (x ) = í1 - x , 0 < x £ 1
909 3 ïx - 1, x >1
AP 2 = Þ AP = 101 î
49 7
f ( 0 + h ) - f ( 0) é h - 1ù
Rf ¢ ( 0) = lim = lim - ê = -1
122. (c) Let P(1, 6, 3) be the given point and L P (1, 6, 3) h ®0 h h ®0
ë h úû
be the foot of perpendicular from P to f ( 0 - h ) - f ( 0) e-h - 1
the given line. The coordinates of a Lf ¢ ( 0) = lim = lim =1
L
h ®0 -h h ®0 -h
general point on the given line are
x - 0 y -1 z - 2 So, it is not differentiable at x = 0.
= = = l (say)
1 2 3 Similarly, it is not differentiable at x = 1but it is continuous at
i.e. x = l Q x = 0 and 1.
y = 2l + 1, z = 3l + 2 126. (b) Q lim f (x ) = f ( 0)
x ®0
If the coordinates of L are ( l, 2l + 1, 3l + 2), then the
Let a be any point
direction ratios of PL are ( l - 1, 2l - 5, 3l - 1).
Now, at x = a, lim f (x ) = lim f (a + h )
x ®a h ®0
Since, the direction ratios of given line which is
perpendicular to PL, are 1, 2 and 3. Therefore, = lim f (a ) + lim f (h )
h ®0 h ®0
( l - 1) 1 + ( 2l - 5) 2 + ( 3l - 1) 3 = 0, which gives l = 1. = f (a ) + f ( 0) = f (a + 0) = f (a )
Hence, coordinates of L are (1, 3, 5). \ f (x ) is continuous at x = a, where a is any arbitrary
Let Q (x1, y1, z1 ) be the image of P(1, 6, 3) in the given line. point.
Then, L is the mid-point of PQ. Hence, f (x ) is continuous for all x.
x +1 y +6 z +3
Therefore, 1 = 1, 1 = 3, 1 =5
2 2 2 127. (a) Since, x , sin-1 x , tan-1x are continuous functions, so the
Þ x1 = 1, y = 0, z = 7 function f is clearly continuous at each point of its domain
Hence, the image of (1, 6, 3) in the given line is (1, 0, 7). except possibly at x = 0.
So, for f to be continuous at x = 0
123. (a) Given equation of plane is x + y + z = 5.
sin-1 x
2-
The distance measured along the line x = y = z . x 1
f ( 0) = lim f (x ) = lim =
Direction ratios of the given line is (1, 1, 1). x ®0 x ®0 tan-1 x 3
2+
So, the equation of line PQ is x
x -1 y + 5 z -9 é sin -1 x tan -1 x ù
= = =l Q lim
ê x ®0 = 1, lim = 1ú
1 1 1 x ®0
ë x x û
Þ x = l + 1, y = l - 5, z = l + 9
128. (b) The given function is
lies on the plane x + y + z = 5
ì0, 0 £ x < 1
\ (l + 1) - ( - l + 5) + ( l + 9 ) = 5 f (x ) = í
Þ l = - 10 î 1, 1 £ x < 2
f (x ) - f (1) 0 -1
The coordinate of Q is ( - 9, - 15, - 1) and the coordinate of P Now, LHD = lim- = lim-1
x ®1 x -1 x ®1 x - 1
is (1, - 5, 9 )
Which does not exist
PQ = (10)2 + (10)2 + (10)2 = 10 3
\ f is not derivable at x = 1.
\ 2 3k = 10 3 Þ k = 5
129. (b) Given, f (x ) = - 2x 3 + 21x 2 - 60x + 41 …(i)
ì 1/ 2
ü
124. (b) lim sin ínp æç1+ 2 ö÷
1 On differentiating Eq. (i) w.r.t. x, we get
ý
î è n ø
n ®¥
þ f ¢ (x ) = - 6x 2 + 42x - 60
ì æ 1 1 öü = - 6(x 2 - 7x + 10) = - 6 (x - 2) (x - 5)
= lim sin ínp ç1 + 2
- 4
+ ....÷ ý
n ®¥
î è 2n 8n øþ If x < 2, f ¢ (x ) < 0, i.e., f (x ) is decreasing, then
ì p p ü f ¢ (x ) < 0 for x < 2
= lim sin ínp + - + ....ý
n ®¥
î 2n 8n 3 þ
130. (d) Rolle’s theorem is not applicable for the function
æ 1 1 ö f (x ) = | x | in [ - 1, 1]
= lim ( -1)n sin p ç - 3
+ ....÷ = 0
n ®¥
è 2n 8n ø
Qf ¢( 0) does not exist.
SOLVED PAPER 2017 27
131. (b) Clearly, the point of intersection of curves is ( 0, 1). = [sin x + cos x ] p0 / 4 + [ - cos x - sin x ] pp//24
Now, slope of tangent of first curve = ( 2 - 1) - (1 - 2 ) = 2 2 - 2
dy dy dv
m1 = = a x log a 135. (b) Put x + y = v and 1 + =
dx dx dx
æ dy ö Therefore, the differential equation reduces to
Þ ç ÷ = m1 = log a
è dx ø ( 0, 1) dv v v v
= (1 + cos v ) + sin v = 2 cos2 + 2 sin cos
Slope of tangent of second curve, dx 2 2 2
dy 2 v æ tanv ö
m2 = = b x log b = 2 cos ç1 + ÷
dx 2è 2 ø
æ dy ö v
sec2
Þ m2 = ç ÷ = log b
è dx ø ( 0, 1) 2
Þ ò é æv ö ù
dv = ò dx
m - m2 log a - log b 2 ê1 + tan ç ÷ ú
\ tan a = 1 = ë è 2øû
1 + m1m2 1 + log a log b
é æx + y öù
n \ log ê1 + tan ç ÷ú = x + C
132. (d) We have, ò0 [x ] dx ë è 2 øû
1 2 3 n
= ò 0 dx + ò 1dx + ò 2 dx + .... + ò (n - 1) dx 136. (c) Since, the intersection of two curves
0 1 2 n -1
y = x 3 and y = x are x = 0 and x = 1.
= 1( 2 - 1) + 2 ( 3 - 2) + 3 ( 4 - 3) + ...... + (n - 1) {n - (n - 1)}
n (n - 1)
= 1 + 2 + 3 + .....+ (n - 1) = Y
2 y=x3
n n n
and ò {x } dx = ò (x - [x ]) dx = y=Ö3
0 0 2
n

\
ò [x ] dx
0
=n -1

O
X
n
ò {x } dx
0

133. (b) Given, f (x ) = x + sin 2x



Þ f ¢ (x ) = 1 + 2 cos 2x
1
For maximum or minimum value, f ¢ (x ) = 0 1
é x 4 2x 3 / 2 ù
\ A= ò (x - x ) dx = ê -
3

Þ 1 + 2 cos 2x = 0 Þ cos 2x = -
1
0 ë 4 3 úû 0
2
2p é 1 2ù 5
Þ 2x = 2np ± = ê - ú = sq units
3 ë 4 3 û 12
p
Þ x = np ± an (1 + n ) - (1 + n 2 )
3 137. (c) lim
n ®¥ 1+ n
4p 2p 5p
Þ x = , ,
3 3 3 (a - 1) n 2+ an - 1
= lim = ¥,
æ 2p ö æ 4p ö æ 5p ö n ®¥ n+1
Find f ( 0), f ç ÷, f ç ÷, f ç ÷, f ( 2p )
è 3ø è 3ø è 3ø If a - 1 ¹ 0 limit does not exist and if a - 1 = 0, then
æ 2p ö 2p an - 1
Þ f ( 0) = 0, f ç ÷ = - 0.8 lim =a =b
n ®¥ n + 1
è 3ø 3
æ 4p ö 4p æ 5p ö 5p Þ a =b =1
Þ fç ÷= + 0.8, f ç ÷ = - 0.8
è 3ø 3 è 3ø 3
Þ f ( 2p) = 2p + 0 = 2p 138. (b) Total ways in which papers can be checked is equal to
\ Maximum value of f (x ) = 2p. 7 4 . Now, two teachers who have to check all the papers can
p/2
be selected in 7C2 ways and papers can be checked by
134. (b) Required area = ò | cos x - sin x | dx
0
them is ( 2 4 - 2) favourable ways.
p/ 4
Thus, required probability
p/2
7
C2 ( 24 - 2) 6
= ò ( cos x - sin x ) dx + ò (sin x - cos x ) dx = =
0 p/4 74 49
28 SELF STUDY GUIDE BITSAT
139. (c) Given, equation of hyperbola is Also, z = 9x +13y

(10x - 5) + (10y - 4) = l ( 3x + 4y - 1) can be rewritten


2 2 2 2

as D(0, 10)
2 2 2x+y=10
æ 1ö æ 2ö
çx - ÷ + çy - ÷ (0, 6) C
è 2ø è 5ø l B(3, 4)
=
3x + 4y - 1 2 2x+3y=18
5 (9, 0)
PS O (0, 0)
This is of the form of =e A(5, 0)
PM
Where, P is any point on the hyperbola and S is a focus and At O ( 0, 0), z = 0
M is the point of directrix. At A (5, 0), z = 45
l At B ( 3, 4), z = 27 + 52 = 79
Here, > 1 Þ | l | > 2 (Q e > 1)
2 At C ( 0, 6), z = 78
Þ l < - 2 or l > 2 \ Maximum value of z is 79.

140. (a) v = a$ ´ b$ = | a | | b | sin q n$ = sin q n$ 144. (c) The man has to win atleast 4 times.
Where, q is the angle between a and b. [Q| a | = 1 = | b| ] \ Required probability
4 3 5 2 6 7
\ [V ] = sin q æ 1ö æ 1ö æ 1ö æ 1ö æ 1ö æ 1ö æ 1ö
= 7C4 ç ÷ ç ÷ + 7C5 ç ÷ ç ÷ + 7C6 ç ÷ ç ÷ + 7C7 ç ÷
$)b
Now, u = a$ - ( a$ . b $ = a$ - (cos q) b
$ è 2ø è 2ø è 2ø è 2ø è 2ø è 2ø è 2ø
1 64 1
Þ | u |2 = ( a$ - (cos q) b$ ) . ( a$ - cos q) b$ ) = ( 7C4 + 7C5 + 7C6 + 7C7 ) ´ 7 = 7 =
2 2 2
= a 2 + cos2 q b 2 - 2 cos q ab [| a|2 = | b|2 = | a|| b| = 1]
-p p -p p
= 1 + cos2 q - 2 cos2 q 145. (c) Q £ sin-1 x £ , £ sin-1 y £
2 2 2 2
= 1 - cos2 q = sin2 q
-p p
\ | u | = sin q and £ sin-1 z £
2 2
Thus |v | = | u |
Given that,
1 3p
141. (c) Variance = S(x - x )2 s2 sin-1 x + sin-1 y + sin-1 z =
n 2
1
New variance = å( ax - ax )2 Which is possible only when
n p
1 sin-1 x = sin-1 y = sin-1 z =
= a 2 å(x - x )2 = a 2s2 2
n
Þ x =y = z =1
142. (a) Given, coefficient of variation C1 = 50 Put p = q = 1
and coefficient of variation C2 = 60. Then, f ( 2) = f (1) f (1) = 2 ´ 2 = 4
We have, x1 = 30 and x 2 = 25 and put p = 1, q = 2
s then, f ( 3) = f (1) f ( 2)
Q C = ´ 100
x = 2 × 22 = 8
s x + y+ z
50 = 1 ´ 100 \ x f (1) + y f ( 2) + z f ( 3 ) -
30 x f (1) + y f ( 2) + z f ( 3 )
Þ s1 = 15 3
= 1+ 1+ 1- = 3 - 1= 2
s 1+ 1+ 1
and 60 = 2 ´ 100
25
Þ s2 = 15 146. (c) Here,
n(n + 1) n - 1 + 2 1 2
\ Required difference s1 - s2 = 15 - 15 = 0 Tn = = = +
n! (n - 1)! (n - 2)! (n - 1)!
143. (c) Given, constraints are 2x + 3y £ 18, 2x + y £ 10, ¥ ¥
1 ¥
1
x ³ 0, y ³ 0.
\ S = åT
n =1
n = å (n - 2)! + 2 å (n - 1)!
n =1 n =1

The feasible region is OABCO.


= e + 2e = 3e
SOLVED PAPER 2017 29
147. (b) Since, | z1| = | z 2| = | z 3 | -2 5
Þ l= and a =
5 2
\ 0 is the circumcentre of an equilateral DABC.
\ a + l = 2 .1 > 2
x1 + x 2 + x 3 y + y2 + y3
\ =0= 1
3 3 149. (b) Equation of line joining the points ( 0, 3) and (5, - 2) is
where, z1 = x1 + iy1 y = 3 - x.
x + x2 + x3 æy + y2 + y3 ö If this line is tangent to y =
ax
Þ 1 + iç 1 ÷=0 , then
3 è 3 ø a(x + 1)
Þ (x1 + iy1 ) + (x 2+ iy 2 ) + (x 3 + iy 3 ) = 0 ( 3 - x ) (x - 11) = ax should have equal roots.
z1 + z 2 + z 3 = 0 Thus, (a - 2)2 + 12 = 0
dx Þ No value of a Þ a Î f
148. (b) Let I = ò
( x ) + ( x )7
2
150. (b) Shortest distance between two curves occured along
dx the common normal.

æ 1 ö \ Normal to y 2 = 4x at (m 2, 2m ) is
( x )7 çç 5
+ 1÷÷
è( x ) ø y + mx - 2m - m 3 = 0
1 æ m2 ö
Put 1 + =t Normal to y 2 = 2(x - 3) at çç + 3, m ÷÷ is
( x )5 è 2 ø
5 1 m3
Þ - . dx = dt y + m (x - 3) - m - =0
2 ( x )7 2
2 dt - 2 Both normals are same, if
\ I =-
5 ò t
=
5
log t + C
1
-2m - m 3 = - 4m - m 3 Þ m = 0, 2
-2 é 1 ù 2
= log ê1 + +C
5 ë ( x )5 úû So, points will be ( 4, 4) and (5, 2) or (4, - 4) and (5, - 2).
2 æ x + 1ö
5/ 2
Hence, shortest distance will be
=- log çç ÷÷ + C
5 è x
5/ 2
ø (1 + 4) = 5
Solved Paper 2018
BITSAT
Instructions
1. There are 150 questions in all. The number of questions in each part is as given below. No. of Questions
Part I Physics 1-40
Part II Chemistry 41-80
Part III (a) English Proficiency 81-95
(b) Logical Reasoning 96-105
Part IV Mathematics 106-150
2. All questions are multiple choice questions with four options, only one is correct.
3. Each correct answer fetches 3 marks while incorrect answer has a penalty of 1 mark.

PART I
Physics
1. Two inductors L1 and L2 are connected in parallel and a 3. Steam at 100°C is passed into 1.1 kg of water
time varying current flows as shown in figure. The ratio contained in a calorimeter of water equivalent to 0.2 kg
i at 15°C till the temperature of the calorimeter and its
of current 1 at any time t is contents rises to 80°C. The mass of steam condensed
i2
(in kg) is
i1 L1
a. 0.130 b. 0.065 c. 0.260 d. 0.135

i i 4. Dimension of which base quantity corresponds to that


Gh
of =?
c3
i2 L2 a. Time b. Length
c. Mass d. Temperature
L2 L1 L22 L12
a. b. c. d.
L1 L2 (L1 + L2 )2 (L1 + L2 )2 5. A reservoir is at 827°C and Carnot’s engine takes a
thousand kilocalories of heat from it and exhausts it to a
2. A parallel plate capacitor sink at 27°C. What is the amount of work and the
C with plates of unit area efficiency of the engine?
C
and separation d is filled a. 2.7 ´ 105 cal,70.70% b. 2.72 ´ 105 cal, 72.72%
d R
with a liquid of dielectric d/3
constant K = 2, the level c. 2.70 ´ 105 cal, 80.70% d. 3.70 ´ 105 cal, 70.70%
d
of liquid is , initially. 6. A train moves towards stationary observer with speed
3 34 m/s. The train blows whistle and its frequency is
Suppose, the liquid level decreases at a constant registered by the observer as f1. If the train’s speed is
speed v , the time constant as a function of time is reduced to 17 m/s, the frequency registered is f2. If the
6 e0R (15d + 9vt ) e0 R f
a. b. speed of sound is 340 m/s, then the ratio 1 is
5d + 3vt 2d 3 - 3dvt - gv 2t 2 f2
6 e0R (15d - 9vt ) e0 R 19 18
c. d. a. b. c. 2 d. 1/2
5d - 3vt 2d 3 + 3dvt - 9v 2t 2 18 19
2 SELF STUDY GUIDE BITSAT
7. An object of mass 5 kg is projected with a velocity b. always constant and is equal to twice the ratio of mass
20 ms -1 at an angle 60°, to the horizontal. At the highest to the charge of electron to the principal quantum
point of its path, the projectile explodes and breaks up number n
into two fragments of masses 1 kg and 4 kg. The c. proportional to the principal quantum number n
fragments separate horizontally after the explosion, 1
d. proportional to 2
which releases internal energy such that the KE of the n
system at the highest point is doubled. The separation
between the two fragments when they reach the 13. Three rods of identical A(T)
ground is cross-sectional area and made
a. 52.25 m b. 44.25 m from the same metal, form the
c. 65.32 m d. 78.76 m
sides of an isosceles triangle
ABC right angled at B as shown
8. An automobile moving with a speed of 36 km/h reaches in figure. The point A and B are 90°
an upward inclined road of angle 30°, its engine maintained at temperature T B (Ö2T) C
becomes switch off. If the coefficient of friction is 0.1, and 2 T respectively, in the
then how much distance will automobile move before
steady state. Now, assuming that only heat conduction
coming to rest?
takes place. The temperature of point C will be
a. 12.53 m b. 21.42 m
T T 3T 3T
c. 15.43 m d. 8.53 m a. b. c. d.
2+1 2 -1 2+1 ( 2 + 1)
9. In the circuit given below, the value of resistance X ,
when the potential difference between the points B and 14. A resistor R, an inductor
D is zero, will be L, a capacitor C and
voltmeters V1, V2 and V3 V1 V2
B are connected to an
V3
6W X oscillator in the circuit as
shown in the adjoining
~
15 W
8W 3W diagram. When the
e = e0 sin wt
A C frequency of the
oscillation is increased, then at the resonant frequency,
15 W 6W 4W
the voltmeter reading is zero in the case of
4W
a. voltmeter V1 only b. voltmeter V2 only
6W
c. voltmeter V3 only d. All the three voltmeters
D
15. In Young’s double slit experiment, intensity at a point is
æ 1ö
a. 9 W b. 8 W c. 4 W d. 6 W
ç ÷ of the maximum intensity. Angular position of this
è4ø
10. A block of wood floats in water with ( 4 / 5 ) th of its point is
volume submerged. If the same block just floats in a æ lö æ l ö
a. sin-1ç ÷ b. sin-1ç ÷
liquid, the density of liquid in (kg m -3 ) is èd ø è 2d ø
a. 1250 b. 600 c. 400 d. 800 -1 æ l ö æ l ö
c. sin ç ÷ d. sin-1ç ÷
è 3d ø è 4d ø
11. As current i flowing through the i
loop as shown in figure. The 16. The bob of simple pendulum is a spherical hollow ball
magnetic field at the centre O is O 2r filled with water. A plugged hole near the bottom of the
7m 0i r oscillating bob get suddenly unplugged. During
a. acting downwards
12 r observation, till water is coming out, the time period of
5m 0i i oscillation would
b. acting upwards
12 r a. first increase and then decrease to the original value
7m 0i b. first decrease and then increase to the original value
c. acting upwards c. remain unchanged
12 r
5m 0i d. increase towards a saturation value
d. acting downwards
12 r 17. At a certain temperature, the number density of charge
carriers in a semiconductor is n. When an electric field is
12. The ratio of angular momentum L to the atomic dipole applied to it, the charge carriers drift with an average
moment m l for hydrogen like atoms and ions is speed v . If the temperature of the semiconductor is
a. always constant and is equal to the ratio of mass to the raised.
charge of electron
SOLVED PAPER 2018 3
a. n will increase butv will decrease 25. Helium gas goes through a cycle ABCDA (consisting of
b. n will decrease butv will increase two isochoric and two isobaric lines) as shown in figure.
c. Both n and v will increase The efficiency of this cycle is approximately
d. Both n and v will decrease
18. Consider the acceleration, velocity and displacement of
2p0 B C
a tennis ball as its falls to the ground and bounces back.
Directions of which of these change in the process?
a. Velocity only
p0 D
b. Displacement and velocity A
c. Acceleration, velocity and displacement
d. Displacement and acceleration
O
Vo 2Vo
19. A convex lens shown in the figure is made up of
two types of transparent materials. A point a. 15.4% b. 9.1% c. 10.5% d. 12.5%
sources of light is placed on its principal axis. If
reflections from the boundaries between layers 26. The frequency and the intensity of incident beam of light
are ignored, the lens will form falling on the surface of a photoelectric material is
a. only one image increased by a factor of two. This will
b. two images a. increase the maximum kinetic energy of the ejected
c. infinite images photoelectrons by a factor of more than two and would
d. no image at all increase the photoelectric current by a factor of two.
b. increase the maximum kinetic energy of the photo
20. If the time period is doubled, then the angular electrons and would increase the photoelectric current
momentum of the body will (provided the moment of both by a factor of two.
inertia of the body is constant)
c. increase the maximum kinetic energy of the
a. remain constant b. quadruple photoelectrons by a factor of two and will have no effect
c. become half d. double on the magnitude of the photoelectric current produced.
21. Breaking stress of a steel wire is p and the density of d. not produce any effect on the kinetic energy of the
steel is r. The greatest length of steel wire that can emitted photoelectrons but will increase the
hang vertically without breaking is photoelectric current by a factor of two.
p p 27. A copper rod of mass m slides
a. b.
rg 2rg under gravity on two smooth B
2p parallel rails l distance apart
c. d. None of these
rg and set an angle q to the VT
horizontal. At the bottom, the
22. A stone is projected with velocity 2 gh , so that it just rails are joined by a resistance R
clears two walls of equal height h, at distance of 2h from R in figure. There is a uniform q i
each other. The time interval of passing between the magnetic field B prependicular
two walls is to the plane of the rails. The terminal velocity of rod is
h 2h h 2h mg R tan q mg R cot q
a. b. c. 2 d. a. b.
g g g g B 2l 2 B 2l 2
mg R sin q mg R cos q
c. d.
23. An object takes n times as much time to slide down a B 2l 2 B 2l 2
45° rough inclined plane as it takes to slide down a
perfectly smooth 45° inclined plane. The coefficient of 28. An asteroid of mass m is approaching earth, initially at a
kinetic friction between the rough plane and the distance 10 Re with speedv i . It hits earth with a speedv f
object is (Re and M e are radius and mass of earth), then
1 1 2Gm æ 1ö
a. n 2 - 1 b. 1 - c. n 2 + 1 d. 1 + a. vf2 = vi2 + ç1 + ÷
n2 n2 Re è 10 ø

24. Some amount of a radioactive substance (half-life 2G Me æ 1ö


b. vf2 = vi2 + ç1 + ÷
= 10 days) is spread inside a room and consequently Re è 10 ø
the level of radiation becomes 50 times the permissible 2GMe æ 1ö
level for normal occupancy of the room. After how many c. vf2 = vi2 + ç1 - ÷
Re è 10 ø
days will the room be safe for occupation?
2Gm æ 1ö
a. 20 days b. 34.8 days d. vf2 = vi2 + ç1 - ÷
Re è 10 ø
c. 56.4 days d. 62.9 days
4 SELF STUDY GUIDE BITSAT
29. When photon of energy 4.0 eV strikes the surface of a 33. In the given circuit,
metal A, the ejected photoelectrons have maximum D3
kinetic energyTA eV and de-Broglie wavelength lA . The 5W
maximum kinetic energy of photoelectrons liberated
from another metal B by photon of energy 4.50 eV is D1
TB = (TA - 1.50) eV. If the de-Broglie wavelength of
these photoelectrons lB = 2lA , then choose the correct 10 W
statement(s). 20 W D2 5W
a. The work function of A is 1.50 eV
b. The work function of B is 4.0 eV
c. TA = 3.2 eV 10 V
d. All of the above The current through battery is
30. A pulley of radius 2 m is rotated about its axis by a force a. 0.5 A b. 1 A c. 1.5 A d. 2.5 A
2 34. A trolley having mass of 200 kg moves with uniform
= ( 20t - 5t ) newton (where t is measured in seconds)
applied tangentially. If the moment of inertia of the speed of 36 kmh-1 on a frictionless track. A child of
pulley about its axis of rotation is 10 kg m2 , the number mass 20 kg runs on the trolley from one end to the other
of rotation made by the pulley before its direction of (10 m away) with a speed of 4 ms -1 relative to the trolley
motion is reversed is in a direction opposite to its motion and ultimately jumps
a. more than 3 but less than 6
out of the trolley. With how much velocity has the trolley
moved from the time the child begins to run?
b. more than 6 but less than 9
c. more than 9 a. 10.36 ms-1 b. 11.36 ms-1
d. less than 3 c. 12.36 ms-1 d. 14.40 ms-1
35. A gas has molar heat capacity C = 37.55 J mol-1K -1, in
31. A planoconvex lens has thickness of 4 cm. When
this process pT = constant. The number of degree of
placed on a horizontal table, with the curved surface in
contact with it, the apparent depth of the bottom most freedom of the molecule of gas is
point of the lens is found to be 3 cm. If the lens is a. 2 b. 3 c. 5 d. 7
inverted such that the plane face is in contact with the 36. If x , v and a denote the displacement, the velocity and
table, the apparent depth of the centre of the plane face the acceleration of a particle executing SHM of time
25
is found to be cm. The focal length of the lens is period T. Then, which of the following does not change
8 with time?
(assume thickness of lens to be negligible) aT aT
a. b. aT + 2pv c. d. a 2T 2 + 4p 2v 2
a. 85 cm b. 59 cm c. 75 cm d. 7.5 cm x v
32. A spherically symmetric gravitational system of 37. To increase the current sensitivity of a moving coil
particles has mass density galvanometer by 50%, its resistance is increased, so
that new resistance becomes twice its initial resistance,
ì r for r £ R
r =í 0 by what factor does its voltage sensitivity change?
î 0 for r > R a. Increases by 15% b. Decreases by 15%
where, r 0 is a constant. A test mass can undergo c. Increases by 25% d. Decreases by 25%
circular motion under the influence of the gravitational 38. A ball is dropped vertically from a height d above the
field of particles. Its speed v as a function of distance ground. It hits the ground and bounces up vertically to a
r ( 0 < r < ¥ ) from the centre of the system is height d / 2. Neglecting subsequent motion and air
represented by resistance, its velocity v varies with height h above the
ground as
v v v v
a. b. d
a. b. d
h h
O O
R r r

v v v v
c. d. d
c. d.
d h h

O r O r
R R
SOLVED PAPER 2018 5
39. In the given circuit, it is observe that the current I is b. R1R4 = R2R3
independent of the value of resistance R 6. Then, the c. R1R2R5 = R3R4R6
resistance value must satisfy
d. R1R3 = R2R4 = R5R6
R6
40. If a drop of liquid breaks into smaller droplets, it results
in lowering of temperature of the droplets. Let a drop of
I R1 R3
radius R, break into N small droplets each of radius r ,
R5
then decrease (drop) in temperature Q¢ (given, specific
heat of liquid drop = S and surface tension =T)
R2 R4 3T é 1 1 ù 2T é 1 1 ù
a. - b. - -
rS êë r R úû rS êë r R úû
2R é 1 1ù 3T é 1 1ù
1 1 1 1 c. - d. -
a. + = × rS êë R r úû rS êë R r úû
R5 R6 R1 + R2 R3 + R4

PART II
Chemistry
41. A metallic element has a cubic lattice. Each edge of the b. Al2O3 is mixed with CaF2 which lowers the melting point
. g cm -3 . The
unit cell is 2Å and the density of metal is 25 of the mixture and brings conductivity.
unit cell in 200 g of metal are c. Al3+ is reduced at the cathode to form Al.
a. 1´ 1024 b. 1´ 1022 d. Na 3AlF6 serves as the electrolyte.
20
c. 1´ 10 d. 1´ 1025
47. Which test among the following is not used for the
42. Four gases P, Q, R and S have almost same values of distinction among 1°, 2° and 3° aliphatic amine.
‘b’ but their ‘a’ values (a and b are van der Waals’ a. Hinsberg's reagent test b. Carbylamine reaction
constant) are in the order Q < R < S < P . At a particular c. Azo dye test d. Action with nitrous acid
temperature, among the four gases, the most easily
liquifiable one is 48. The incorrect statement about carbonate (CO2–
3 ) ion is,
a. P b. Q c. R d. S a. It has planar structure
é 0.51 ´ 10 -10 ù b. It has one coordinate bond
43. If ê ú metre is the radius of smallest electron c. It has three resonating structure
ë 4 û d. Hydrolysis of CO2-3 ion gives basic solution
orbit in hydrogen like atom, then the atom is
a. hydrogen atom b. He+ 49. Under the same reaction conditions, initial
c. Li2+ d. Be3+ concentration of 1.386 mol dm -3 of a substance
becomes half in 40 s and 20 s through first order and
44. Which of the following is the correct order for the æk ö
wavelength if absorption in the visible region. zero order kinetics, respectively. Ratio çç 1 ÷÷ of the rate
a. [Ni(NO2 )6] 4- < [Ni(NH3 )6] 2+ < [Ni(H2O)6] 2+ è k0 ø
of constants for first order (k1) and zero order (k 0 ) of the
b. [Ni(NO2 )6] 4- < [Ni(H2O)6] 2+ < [Ni(NH3 )6] 2+
reaction is
c. [Ni(H2O)6] 2+ < [Ni(NH3 )6] 2+ < [Ni(NO2 )6] 4- a. 0.5 mol-1dm3 b. 1.0 mol dm-3
2+ 2+ 4-
d. [Ni(NH3 )6] < [Ni(H2O)6] < [Ni(NO2 )6] c. 1.5 mol dm-3 d. 2.0 mol-1dm3
45. The empirical formula and molecular mass of a 50. Which substance has a dipole moment?
compound are CH2O and 180g respectively. The a. CCl4 b. CH2Cl2
molecular formula of the compound will be
c. C2Cl2 d. C2Cl4
a. C9H18O9 b. CH2O
c. C6H12O6 d. C2H4O2 51. Enthalpy of combustion of methane and ethane are
-210 kcal/mol and -368 kcal/mol respectively. The
46. In the context of the Hall-Heroult process for the enthalpy of combustion of decane is
extraction of Al, which of the following statement is a. - 1582 kcal b. - 1632 kcal
incorrect.
c. - 1700 kcal d. - 1480 kcal
a. CO and CO2 are produced in the process.
6 SELF STUDY GUIDE BITSAT
52. The correct sequence of reagents for the following a. The equivalent weight of MeCHO is 22.
conversion will be –
b. Three moles of O H are required in the reaction.
O c. MeCHO acts as an oxidising agent.
d. [Ag(NH3 )2] + gets reduced.
C CH CH3
60. Which of the following graph is correctly represented
according to Freundlich isotherm.
a. O3 / Red P, AlCl3, MeCOOH
b. H2SO4 + HgSO4, H2O / Heat

log x/m

log x/m
c. O3 / Zn - AcOH, H2SO4 + HgSO4 / H2O / Heat
-
a. b.
d. CH3COOH, H2O2 + OH / H2O

53. In an atom, an electron is moving with a speed of


log p log p
600 m/s with an accuracy of 0.005%. Certainly with
which the position of the electron can be located is
(h = 6.6 ´ 10-34 kg m 2s -1 , mass of electron,

log x/m

log x/m
em = 9.1´ 10 - 31 kg) c. d.
a. 1.52 ´ 10-4 m b. 5.10 ´ 10-3 m
c. 1.92 ´ 10-3 m d. 3.84 ´ 10-3 m
log p log p
54. Which of the following diatomic molecules would be
stabilised by the removal of an electron? 61. pH of a saturated solution of Ba(OH)2 is 12. The value
a. C2 b. CN of its K sp is
c. N2 d. O2 a. 3.3 ´ 10-7 M b. 5.0 ´ 10-7 M
-6
55. 0.5F of electricity is through 500 mL of copper sulphate c. 4.0 ´ 10 M d. 5.0 ´ 10-6 M
solution. The amount of copper which can be deposited
62. An unsaturated hydrocarbon ‘X ’ gives white precipitate
will be
with Tollen's reagent. If X is gaseous in nature, the
a. 63.5g b. 31.75g molecular formula of X is
c. 15.80g d. unpredictable a. C3H6 b. C2H4
56. Consider the following sequence of reactions. c. C2H2 d. C4H8

Z ¾ PCl
¾¾ 5
® X ¾ Alc.
¾ ¾¾ KOH
® Y ¾ Conc.
¾ ¾H¾ 2 SO4
¾® Z 63. The rate of a reaction triples when temperature
2H 2 O/ D
changes from 20°C to 50°C. The energy of activation for
‘Z ’ is the reaction is (R = 8.314 JK –1 mol-1)
a. CH3CH2CH2OH b. (CH)2 COH a. 181327
. J mol-1
c. CH3 — C H — CH3 d. None of these b. 428.141J mol-1
| c. 32.4321kJ mol-1
OH
d. 28.8118 kJ mol-1
57. An equilibrium mixture at 300K contains N2O4 and NO2
at 0.28 and 1.1 atm pressure, respectively. If the 64. Which of the following compound will give blood red
volume of the container is doubled, the new equilibrium colour while doing the Lassaigne's test for N.
pressure of these two gases are respectively. a. (NH2 )C == O b. H2N (C6H4 )SO3H
a. 0.064 atm and 0.095 atm c. C6H5SO3H d. CHCl3
b. 0.64 atm and 0.095 atm
c. 0.095 atm and 0.632 atm 65. For a reaction, A + B 2+ ¾® B + A 2+ , at 25°C
d. 0.095 atm and 0.64 atm E ° = 0.2955 V. The value of K eq is
58. Which among the following actinoids does not have a. 10 b. 1010
stable electronic configuration c. -10 d. 10-10
a. Protactinium b. Nobelium 66. Which of the following is the correct order of stability of
c. Americium d. Lawrencium conformations for NH2 ¾ CH2 ¾ CH2 ¾ OH.
59. Which of the following statement is incorrect regarding a. gauche > eclipsed > anti
the equation b. gauche > anti > eclipsed

c. eclipsed > gauche > anti
MeCHO + [Ag(NH3 )2] + + O H ¾® MeCOO- + Ag d. anti > eclipsed > gauche
SOLVED PAPER 2018 7
67. When H2S gas is passed into a mixture of Mn2+ , Ni2+ , a. chromate ions reduced
Cu2+ and Hg2+ ion in an acidified aqueous solution, the b. chromate ions are oxidised
precipitates formed are c. monocentric complex is converted into dicentric
complex
a. CuS and HgS
d. oxygen gets removed from chromate ions
b. MnS and CuS
c. MnS and NiS 74. Valence electrons in the element A are 3 and that in
d. NiS and HgS element B and 6. Most probable compound formed
from A and B is
68. A mixture of bromo trichloride and hydrogen is
subjected to silent electric discharge to form x and HCl. a. A2B b. AB2 c. A6B3 d. A2B3
x is mixed with NH3 and heated to 200°C to form y . The 75. In non-reducing disaccharide, the reducing group of
formula of y is monosaccharides i.e. aldehydic or ketonic group are
a. B2O3 b. B3N3H6 bounded.
c. H3BO3 d. B2H6 Which of the following disaccharide is a non-reducing
sugar?
69. Which of the following reactions increases the
production of dihydrogen from synthesis gas? CH2OH CH2OH
1270K O H O
a. CH4(g ) + H2O(g) ¾¾®
Ni
CO (g ) + 3H2(g ) H H H
a. H H
1270 K OH H OH H
b. C(s ) + H2O(g) ¾¾® CO (g ) + H2(g ) HO O OH
673 K
c. CO (g ) + H2O(g) ¾¾¾® CO2 (g ) + H2(g ) H OH H OH
Catalyst

1270K CH2OH
d. C2H6 + 2H2O ¾¾® 2CO + 5H2
Ni O
H H
70. When the heat of a reaction at constant pressure is b. H CH2OH H
-3 OH H O
-25
. ´ 10 cal and entropy change for the reaction is HO O H HO
7.4 cal deg -1, it is predicted that the reaction at 25°C is CH2OH
a. revesible b. spontaneous H OH
OH H
c. non-spontaneous d. irreversible
CH2OH CH2OH
71. The addition of HBr to 1-butene gives a mixture of O O
HO HO H
products x , y and z.
c. H H
Br C2H5 OH H O OH H
H H H
C C CH3 CH2 CH2 CH2Br
H OH H OH
H5C2 CH3 H CH3 CH2OH CH2OH
H Br
(X) (Y) (Z ) HO O
H
O
OH
H
The mixture consists of d. OH H O OH H
a. x and y as major and z as minor products HO H H
b. y as major, x and z as minor products
H OH H OH
c. y as minor, x and z as major products
d. x and y as minor and z as major products 76. Which of the following 0.1 M aqueous solution will have
lowest freezing point?
72. The correct statement about silicone is a. Potassium sulphate b. Sodium chloride
a. They are ketones with silyl group (SiH3 ) similar to alkyl, c. Urea d. Glucose
(SiH3 )2 CO. 77. A penicillin is a member of a family of drugs that have a
b. They are synthetic polymer containing repeated R2SiO2 a. four membered cyclic amide fused to a five membered
units. thiazole ring.
c. They are formed by hydrolysis of R2SiCl2. b. three membered cyclic amide fused to a five-membered
d. All of the above thiazole ring
c. four-membered cyclic amide fused to have a four
73. When dil. sulphuric acid reacts with aqueous solution of membered thiazole ring.
potassium chromate, the colour changes from yellow to d. five-membered cyclic amide fused to have a five
orange. This shows that membered thiazole ring
8 SELF STUDY GUIDE BITSAT
78. Which of the following property of alkaline earth metal 80. Match the polymer given in column I with correct
increases with their atomic number? monomer of column II and choose the correct option.
a. electronegativity Column I Column II
b. solubility of their hydroxides in water
c. solubility of their sulphate in water A. Neoprene I. Isoprene
d. ionisation energy B. Natural rubber II. Tetrafluoro ethane
79. Which of the following expression is correct for the rate C. Teflon III. Chloroprene
of reaction given below ? D. Acrilan IV. Acrylnitrite
5Br - (aq ) + BrO3- (aq ) + 6H+ (aq ) ¾® 3Br2(aq ) + 3H2O(l )
D[Br - ] 5 D[H+ ] D[Br - ] 6 D[H+ ] Codes
a. = b. = A B C D A B C D
Dt 6 Dt Dt 5 Dt
a. IV III II I b. I II III IV
D[Br - ] 5D[H+ ] D[Br - ] 6D[H+ ]
c. = d. = c. III I II IV d. II IV I II
Dt Dt Dt Dt

Part III
a. English Proficiency
Directions (Q. Nos. 81-83) In the following questions, the Directions (Q. Nos. 89-90) Choose the word nearest in
sentences may or may not be grammatically correct. Find out meaning to the underlined word.
which part of a sentence has an error and mark that part. If
there is no error mark part ‘d’ as your answer. 89. Before I could make out anything he had spoken again.
a. find out b. apprehend
81. The captain along with his team (a)/are practising very c. explain d. reveal
hard (b)/ for the forthcoming match. (c)/ No error (d).
90. He wrote a scathing review of the prize winning novel.
82. I am going (a)/to have this certificate (b)/attest by the
a. biased b. scornful
director. (c)/No error (d).
c. unbalanced d. subjective
83. He is (a)/ having many (b)/friends here.(c)/No error…(d).
Directions (Q. Nos. 91-95) Read the passage given below
Directions (Q. Nos. 84-85) Fill in the blanks with suitable and answer the questions that follow.
preposition from the alternatives given under each sentence. The megalomaniac differs from the narcissist by the fact that he
84. Is not learning superior ................... wealth? wishes to be powerful rather than charming and seeks to be
feared rather than loved. To this type belong many lunatics and
a. than b. from
most of the great men in history. Love of power, like vanity, is
c. by d. to
a strong element in normal human nature and as such is to be
85. He could not cope .................... the heavy workload. accepted; it becomes deplorable only when it is excessive or
a. in with b. up with associated with an insufficient sense of reality. Where this
c. up d. with occurs, it makes a man unhappy or foolish, if not both. The
lunatic who thinks he is crowned head may be, in a sense,
Directions (Q. Nos. 86-88) Select the word or the phrase
happy, but his happiness is not of a kind that any sane person
which is closest to the opposite in meaning of the italicized
word or phrase.
would envy. Alexander the Great was psychologically of the
same type as the lunatic, though he possessed the talent to
86. He was in a dejected mood. achieve the lunatic’s dream.
a. jubiliant b. rejected He could not, however, achieve his own dream, which
c. irritable d. romantic enlarged his scope as his achievement grew. When it became
87. The attack on the freedom of the press is a retrograde clear that he was the greatest conqueror known to fame, he
step. decided that he was a God. Was he a happy man ? His
a. progressive b. stubborn drunkenness, his furious rages, his indifference to women and
c. punitive d. aggressive
his claim to divinity, suggest that he was not. There is no
ultimate satisfaction in the cultivation of one element of
88. We should not belittle the value of small things. human nature at the expense of all the others, nor in viewing
a. extal b. praise all the world as raw material for the magnificence of one’s own
c. inflate d. expand ego.
SOLVED PAPER 2018 9
91. What is the difference between an ordinary 93. In ‘‘Where this occurs it makes a man..... if not both,’’
megalomaniac and a megalomaniac like Alexander the ‘this’ refers to
Great? a. vanity b. lunacy
a. The ordinary megalomaniac does not have excessive c. love of power d. excessive lover of power
desire for power which Alexander the Great had
94. Which among the following is the reason for
b. The ordinary megalomaniac does not have the talent to unhappiness?
realise his wish which Alexander the Great had
a. Dealing with the raw material of the world
c. The ordinary megalomaniac is a lunatic while Alexander b. Realising one’s dream as a megalomaniac
the Great was not a lunatic c. The nurturing of only one element in human nature
d. The ordinary megalomaniac is not great while d. Being indifferent towards women
Alexander the Great was great
95. Why has love of power to be accepted?
92. How does a megalomaniac differ from a narcissist? a. Because it can become unreal
a. By wishing to be charming and feared b. Because it is an excess in human nature
b. By wishing to be loved and not feared c. Because it is a part of human nature
c. By wishing to be powerful and not feared d. Because it is vanity in human nature
d. By wishing to be powerful and feared

b. Logical Reasoning
96. ‘Umpire’ is related to ‘Match’ in the same way as ‘Judge’ 101. Find out which of the figure (a), (b), (c) and (d) can be
is related to formed from the pieces given in question figures.
a. Bar council b. Lawyer Questions figure
c. Judgement d. Lawsuit
97. Find the odd one from the following options.
a. 9, 49 b. 13, 121
c. 10, 61 d. 7, 25
98. Complete the series by replacing question mark ‘?’. Answer figures
10, 9, 16, 45, 176, ?
a. 815 b. 222
c. 555 d. 875
99. In a cricket team, Dhoni is taller than Virat but not as tall
a. b. c. d.
as Raina, Rohit is shorter than Dhoni but taller than
Shikhar. Who among them is the shortest? 102. In this question, a piece of paper is folded and then cut
a. Dhoni as shown below. The dotted lines shown are the portion
b. Virat which have been folded.
c. Shikhar The curve arrow shows the directions of folding. And
d. Cannot be determined the number of scissors beneath the figure show the
number of portions cut. From the given responses,
100. Identify the missing part of the question figure and indicate how it will appear when opened. The opening is
select it from given answer figures. in the same order as folding.
Question figures Question figures

Answer figures Answer figures

a. b. c. d. a. b. c. d.
10 SELF STUDY GUIDE BITSAT
103. Which of the answer figures (a), (b), (c) or (d) completes Question figure
the figure matrix ?
Questions figures

Answer figures

a. b.
?

Answer figures

c. d.

a. b. c. d.
105. How many triangles are there in the following figure?
104. In the following question, one or more dots are place in
the question figure.
This figure is followed by four alternatives marked as
(a), (b), (c) and (d). One out of these four options
contains region(s) common to circle, square and
triangles, similar to that marked by the dot in question
figure. Find that figure. a. 16 b. 20 c. 12 d. 22

Part IV
Mathematics
æ 1ö
n
½ sin q cos q 0 ½
106. The coefficient of x -n in (1 + x )n ç 1 + ÷ is
è xø 110. If x 2 =½- cos q sin q 1 ½, then the value of
½ ½
a. 0 b. 1 c. 2n
d. 2n ½ sin q cos q 2 ½
3p
20 4x 2 + x sin + 5 is
æ 1 ö 2
107. The greatest term in the expansion of 3 ç 1 + ÷ is
è 3ø a. 13 - 2 b. 13 + 2
26840 24840 c. 2 - 13 d. Both (a) and (b)
a. b.
9 9
25840 111. If a , b , c are in GP and log a - log 2b ,log 2b - log 3c and
c. d. None of these
9 log 3c - log a are in AP, then a , b and c are the lengths
of the sides of a triangle, which is
108. The nth roots of unity are in
a. equilateral b. right angled
a. AP b. GP
c. acute-angled d. obtuse angled
c. HP d. None of these
n
n(n + 1)(n + 2)(n + 3 )
éa b c ù 112. If å tr = , where tr denotes the r th
109. If P = êb c a ú, abc = P T P = I , then the value of r =1 8
ê ú n
1
êëc a b úû term of a series, then lim
n ®¥
åt is
a 3 + b 3 + c 3 is r =1r
1 1 1
a. 2 b. 1 c. 0 d. 5 a. b. c. d. 1
8 4 2
SOLVED PAPER 2018 11
113. Which of the following statement is a tautology? 121. The area of the triangle formed by joining the origin to
a. ( p Ú q ) Ú (~ p ) the point of intersection of the line x 5 + 2y = 3 5 and
b. (~ q Ù p ) Ú ( p Ú ~ p ) circle x 2 + y 2 = 10 is
c. Both (a) and (b) a. 3 b. 4
d. None of the above c. 5 d. 6
114. If a parallelogram is cut by two sets of m lines parallel to 122. Radius of the largest circle which passes through the
its sides, then the number of parallelogram thus focus of the parabola y 2 = 4x and contained in it, is
formed, is
a. 8 b. 4
a. m C2 ´ mC2 b. 2(m + 2C2 )
c. 2 d. 5
c. (m + 2C2 )2 d. None of these
x2 æ pöy2
2 123. A tangent drawn to hyperbola -
= 1 at P ç ÷
115. The inverse of the function f (x ) = loga (x + x + 1) a b 2
è6 ø
2

(where, a < 0, a ¹ 1) is forms a triangle of area 3a 2 square units, with


1 x coordinate axes. If the eccentricity of hyperbola is e,
a. (a - a - x ) b. not defined for all x
2 then the value of e 2 - 9 is
c. defined for x > 0 d. None of these a. 9 b. 10
¥
2n c. 11 d. 8
116. The value of S = å tan-1 n 4 + n 2 + 2 is equal to 124. If the sum of squares of distances of a point from the
n =1
p planes x + y + z = 0, x - z = 0 and x - 2y + z = 0 is p 2,
a. b. p then locus of the point is
2
p a. x 2 + z 2 = p 2
c. d. None of these
4 b. x 2 + 2xy + y 2 + z 2 = p 2
117. Equation sin x + cos(t + x ) + cos(t - x ) = 2 has real c. x + y + z = p 2
solution, then sint can be d. x 2 + y 2 + z 2 = p 2
1 1
a. b. (x + 1) (z + 2)
2 5 125. Line = y - 1= is perpendicular to
3 -3 l -4
c. d. 2x + 2y - 8z + 5 = 0, then l is
4 4
a. 1 b. -4
118. A line makes angles a, b, g with the coordinate axes. If c. -5 d. -3
p
a + b = , then (cos a + cos b + cos g )2 is equal to 126. OPQR is a square and M , N are the middle points of the
2
sides PQ and QR respectively, then the ratio of the
a. 1 + cos 2a b. 1 - sin 2a
areas of the square and D OMN is
c. 1 + sin 2a d. None of these a. 4 : 1 b. 2 : 1
c. 8 : 3 d. 4 : 3
119. Straight lines 3x + 4 y = 5 and 4x - 3 y = 15 intersect at
the point A. If point B and C are chosen on these two 127. The line passing through the extremity A of the major
lines such that AB = AC , then the possible equation of axis and extremity B of the minor axis of the ellipse
the line BC passing through the point (1, 2) is x 2 + 9 y 2 = 9 meets its auxiliary circle at the point M .
a. x + 7y + 13 = 0 or 7x + y + 9 = 0 Then, the area of the triangle with vertices at A, M and
b. x + 7y + 13 = 0 or 7x + 2y + 7 = 0 the origin O is
31 29
c. x - 7y + 13 = 0 or 7x + y - 9 = 0 a. b.
10 10
d. None of the above 21 27
c. d.
120. Normals drawn to y 2 = 4ax at the points where it is 10 10
intersected by the line y = mx + c intersected at P . 128. If e1 and e2 are the eccentricities of a hyperbola
Coordinates of foot of the another normal drawn to the
parabola from the point ‘P ’ is 3x 2 - 3 y 2 = 25 and its conjugate, then
æ a 2a ö 9 6a a. e12 + e 22 = 2
a. ç 2 , - ÷ b. ,-
èm mø m m b. e12 + e 22 = 4
æ 4a 4a ö c. e1 + e 2 = 4
c. (am 2, - 2am ) d. ç 2 , - ÷
è m mø d. e1 + e 2 = 2
12 SELF STUDY GUIDE BITSAT
129. Let f :R ® R be a function satisfying cos 3x x4 1
c. - + ex + + x +1
f (x + y ) = f (x ) + 2y 2 + kxy for all x , y Î R . If f (1) = 2 and 9 12 3
f ( 2) = 8, then f (x ) is equal to d. None of the above
a. 2x 2 b. 6x - 4 137. For which interval the given function
2
c. x + 3x - 2 d. -x 2
+ 9x - 6 f (x ) = - 2x 3 - 9x 2 - 12x + 1 is decreasing?
a. ( -2, ¥ ),
130. If the planes r ( 2i$ - l$j + 3k$ ) = 0 and r × ( l $i + 5 $j - k$ ) = 5
b. ( -2, - 1)
are perpendicular to each other, then the value of l2 + l
c. ( -¥ , - 1)
is
d. ( -¥ , - 2) or ( -1, ¥ )
a. 0 b. 2
c. 1 d. 3 138. If q is the angle between the vectors 4(i - k ) and
131. Solution of the differential equation i$ + j$ + k$ , then (sin q + cos q ) equals to
dy 1
= sin(x + y ) + cos(x + y ) is equal to a. 0 b. c. 1 d. 2
dx 2
æ x +yö
a. logç 2 + sec ÷ =x + C 139. In a DABC, D , E , F are the mid-points of the sides BC,
è 2 ø ¾®
b. log(1 + tan(x + y )) = x + C CA and AB respectively, the vector AD is equal to
¾® ¾® ¾® ¾®
æ x +yö a. BE + CF b. BE - CF
c. logç1 + tan ÷ =y + C
è 2 ø ¾® ¾® ¾® ¾®
æ x +yö c. CF - BE d. - BE - CF
d. logç1 + tan ÷ =x + C
è 2 ø 140. The arithmetic mean of a set of observation is X . If each
1 3 observation is divided by a and increased by 10, then
132. The value of a, so that lim (e ax - e x - x ) = , is
2 the mean of the new series is
x ®0 x 2
X X + 10
a. 1 b. 0 a. b.
c. 4 d. 2 a a
X + 10a
133. An inverted conical flask is being filled with water at the c. d. aX + 10
3 a
rate of 3 cm /sec. The height of the flask is 10 cm and
the radius of the base is 5 cm. How fast is the water 141. If h is the altitude of a parallelopiped determined by the
level rising when the level is 4 cm? vectors a, b, c and the base is taken to be the
4 3 parallelogram determined by a and b where
a. p cm/sec b. cm/sec
3 4p a = $i + $j + k$ , b = 2$i + 4 $j - k$ and c = $i + $j + 3k$ , then the
3p 4 value of 19h 2 is
c. cm/sec d. cm/sec
4 3p a. 19 b. 16
134. The equation of the curve whose slope at any point is c. 8 d. None of these
¾®
equal to y + 2x and which passes through the origin is
142. The mean and variance of a Binomial distribution ( BD)
a. y = 2(x - 1) b. y = 2(e - x - 1)
x
¾®
c. y = 2(e - 1)x
d. y = 2(e x x - 1) for 3 trials is 2.7, then the BD is given by
a. ( 0.2 + 0.8)5 b. ( 0.3 + 0.7)5
ìï p 1 c. ( 0.4 + 0.6)5 d. None of these
135. Let f (x ) = í x sin x , x ¹ 0, then f (x ) is continuous but
ïî 0, x = 0 dx
143. Let P (x ) = ò -x
,
not differentiable at x = 0, if e + 8e
x
+ 4e -3 x
a. p < 0 b. p = 0
dx
c. 0 < p £ 1 d. p ³ 1 Q(x ) = ò and R (x ) = P (x ) - 2Q(x ).
e 3x
+ 8e x + 4e - x
136. The solution of the differential equation
1 æ B + 2e - x ö
d 2y 2 If R (x ) = Aç ÷ + K , then the value of ( A, B , C )
= sin 3x + e + x when y 1 ( 0) = 1 and y ( 0) = 0, is 2 çè ÷
x
C ø
dx 2
sin 3x x4 1 is
a. - + ex + + x -1 a. (tan-1, 2, e x ) b. (tan-1, e x , 2)
9 12 3
sin 3x x4 1 æ 1 1ö æ 1 1ö
b. - + ex + + x c. ç tan-1, , x ÷ d. ç tan-1, x , ÷
9 12 3 è 2 e ø è e 2ø
SOLVED PAPER 2018 13
1
144. The value of ò cot -1 (1 - x + x 2 )dx is 147. In a test an examiner either guesses or copies or knows
0 the answer to a multiple choice question with 4 choices.
a. log 2 1
The probability that he/she makes a guess is . The
p 3
b. - log 2
2 1
probability that he/she copies the answer is . If the
p 6
c. + log 2
2 probability that the answer is correct, given that he/she
d. - log 2 1
copied, it is , then the probability that he/she knows
8
145. The area of the region included between the curves the answer to a question given that he/she correctly
x 2 + y 2 = a 2 and | x | + | y | = a (a > 0), is answered it, is
2ö 27 26
æ a. b.
a. ç p - ÷a 2 sq units 29 29
è 3ø
25 24
æ2 ö c. d.
b. ç - p ÷a 2 sq units 29 29
è3 ø
2 2 148. If p : 4 is an even prime number, q : 6 is a divisor of 12
c. pa sq units and r : the HCF of 4 and 6 is 2, then which of the
3
following is correct?
æ 2ö
d. ç p + ÷a 2 sq units a. ( p Ù q ) b. ( p Ú q ) Ù ~ r
è 3ø
c. ~ (q Ù r ) Ú p d. ~ p Ú (q Ù r )
146. Let A and B are two independent events. If the
1 149. The maximum value of Z = 4x + 2y subject to
probability that both A and B occur together is and the
6 constraints 2x + 3 y £ 18, x + y ³ 10 and x , y ³ 0 is
1 a. 20 b. 36
probability that neither of them occurs is , then the
3 c. 40 d. None of these
probability of occurrence of A is
1 1
150. The coordinates of the point at which minimum value of
a. 0 or 1 b. or Z = 7x - 8 y subject to constraints x + y - 20 £ 0, y ³ 5,
2 3 x ³ 0, y ³ 0 is attained, is
1 1 1 1
c. or d. or a. (20, 0) b. (15, 5)
2 4 3 4 c. (0, 5) d. (0, 20)
Answers
Physics
1. (a) 2. (a) 3. (a) 4. (b) 5. (b) 6. (a) 7. (b) 8. (d) 9. (b) 10. (d)
11. (d) 12. (a) 13. (c) 14. (b) 15. (c) 16. (a) 17. (a) 18. (b) 19. (b) 20. (c)
21. (a) 22. (c) 23. (b) 24. (c) 25. (a) 26. (a) 27. (c) 28. (c) 29. (b) 30. (a)
31. (c) 32. (c) 33. (c) 34. (a) 35. (c) 36. (a) 37. (d) 38. (a) 39. (b) 40. (d)

Chemistry
41. (d) 42. (a) 43. (d) 44. (a) 45. (c) 46. (d) 47. (c) 48. (b) 49. (a) 50. (b)
51. (b) 52. (b) 53. (c) 54. (d) 55. (c) 56. (c) 57. (d) 58. (a) 59. (c) 60. (a)
61. (b) 62. (c) 63. (d) 64. (b) 65. (b) 66. (b) 67. (a) 68. (b) 69. (c) 70. (b)
71. (a) 72. (c) 73. (c) 74. (d) 75. (b) 76. (a) 77. (a) 78. (b) 79. (a) 80. (c)

English Proficiency
81. (a) 82. (c) 83. (b) 84. (d) 85. (d) 86. (a) 87. (a) 88. (a) 89. (b) 90. (b)
91. (b) 92. (d) 93. (d) 94. (c) 95. (c)

Logical Reasoning
96. (d) 97. (c) 98. (d) 99. (d) 100. (c) 101. (b) 102. (a) 103. (c) 104. (b) 105. (b)

Mathematics
106. (b) 107. (c) 108. (b) 109. (a) 110. (d) 111. (d) 112. (c) 113. (c) 114. (c) 115. (a)
116. (c) 117. (a) 118. (c) 119. (c) 120. (d) 121. (c) 122. (b) 123. (d) 124. (d) 125. (a)
126. (c) 127. (d) 128. (b) 129. (a) 130. (a) 131. (d) 132. (d) 133. (b) 134. (b) 135. (c)
136. (a) 137. (d) 138. (c) 139. (d) 140. (c) 141. (c) 142. (b) 143. (b) 144. (b) 145. (a)
146. (b) 147. (d) 148. (d) 149. (d) 150. (d)
SOLUTIONS
Physics
1. (a) As the inductors are in parallel, induced emf across the 4. (b) As, gravitational constant,
two inductors is the same, i.e. [G] = [ M-1L3T -2]
e1 = e 2
Planck’s constant, [h] = [ ML2T -1]
æ di1 ö æ di ö
L1ç ÷ = L2 ç 2 ÷ Hence, [G] [h] = [M-1L3T -2] [ML2T -1]
è ø
dt è dt ø
On integrating both sides, we get = [M0L5T -3]
di di Velocity of light, [c ] = [LT -1]
L1ò 1 = L2 ò 2
dt dt Now,
L1ii = L2i2 é Gh ù
1/ 2
[L5T -3]1/2
êë c 3 úû =
Þ
i1 L2
= [L3T -3]1/2
i2 L1
= [L2]1/2 = [L]
2. (a) We have, the time constant, t = RC ¢ …(i) Hence, [L] = length
C1C2
Now, C ¢ = 5. (b) Given, Q = 106 cal
C1 + C2
T1 = 827°C = ( 827 + 273) = 1100 K
æ Ae0 ö æ KA e0 ö
çç ÷ç ÷ T2 = 27°C = ( 27 + 273) = 300 K
d - x ÷ø è x ø
=è Q1 Q 2
A e0 KA e0 As, =
+ T1 T2
d -x x
T2 300
KAe0 \ Q2 = Q1 = ´ 106
\ C¢ = T1 1100
x + K (d - x )
= 2.72 ´ 105 cal
Putting the value of C ¢ in Eq. (i), we get
R K A e0 é d ù Efficiency of the engine,
t= êëQ x = 3 - vt úû
d æ d ö æ T ö
- vt + k çd - + vt ÷ h = çç1 - 2 ÷÷ ´ 100
3 è 3 ø è T1 ø
Given, A = 1and K = 2 æ 300 ö
h = ç1 - ÷ ´ 100
3 ´ 2 e0 ´ R 6 R e0 è 1100 ø
\ t= =
d - 3vt + 6d - 2d + 6vt 5d + 3vt = 72.72%
3. (a) According to principle of calorimetry, 6. (a) According to Doppler’s effect, the approximate
heat gained = heat lost frequency heard by the stationary observer,
Heat is lost by steam in two stages n=
v
n0
(i) Change of state from steam at 100°C to water at 100°C is v - vs
m ´ 540. Case (i) vs = 34 m/s
(ii) To change water at 100°C to water at 80°C is where, v = speed of sound in air,
m ´ 1 ´ (100 - 80), where m is the mass of the steam us = speed of source
condensed. and v 0 = frequency of the source.
Total heat lost by steam is 340
\ n1 = n0
m ´ 540 + m ´ 20 = m (540 + 20) = 560 m 340 - 34
Heat gained by calorimeter and its contents is 340
= n0 … (i)
. + 0.2) ´ ( 80 - 15) = 1.12 ´ 65 cal
(11 306
Þ 560 m = 1.12 ´ 65 Case (ii) vs = 17 m/s
1.12 ´ 65 340 340
Þ m= = 0.130 kg \ n2 = n0 = n0 … (ii)
560 340 - 17 323
16 SELF STUDY GUIDE BITSAT
From Eqs. (i) and (ii), we get 1
(mg sin q + mR ) ´ s = mu 2
n1 340 / 306 323 19 2
\ = Þ »
n 2 340 / 323 306 18 1
(mg sin q + m mg cos q) ´s = mu 2
2
7. (b) Given, m = 5 kg, v = 20 ms-1, q = 60° 1
mu 2
Vertical component of velocity,v y = v sin 60° 2 u2
s = =
3 mg(sin q + m cos q) 2g (sin q + m cos q)
= 20 ´ = 10 3 ms-1
2 10 ´ 10
= = 8.53 m
Time taken to reach the highest point = Time taken to reach 2 ´ 10 ´ (sin 30° + 0.1 cos 30° )
the ground from highest point. 8X
9. (b) P = 15 + 6 = 21W, Q = +3
v sin q v y 10 3 8+X
t= = = = 177
. s
g g 9.8 6 ´6 4 ´4
R = 15 + = 18 W, S = 4 + = 6W
If the highest point, m splits up into two parts of masses 6+ 6 4+ 4
m1 = 1 kg and m2 = 4 kg. P R PS 21 ´ 6
As, = , so Q = = = 7W
If their velocities v1 and v 2 respectively, then applying the Q S R 18
principle of conservation of linear momentum, we get 8X
3+ = 7 Þ X = 8W
m1v1 + m2v 2 = mv cos q 8+X
1
v1 + v 2 = 5 ´ 20 ´ [Q q = 60° ] 10. (d) Let V be the volume of the block. When block floats in
2
æ4 ö
v1 + 4v 2 = 5 ´ 10 = 50 … (i) water, then Vr block g = ç V ÷ r water g
è5 ø
1
Initial KE = m (v cos q)2 4
2 or r block = r water …(i)
5
1
= ´ 5 ´ (10)2 = 250 J When block floats in liquid.
2
Final KE = 2 (initial KE) = 2 ´ 250 = 500 J Vr block g = Vr liquid g
1 1 r block = r liquid
\ m1v12 + m2v 22 = 500 4
2 2 r liquid = r water [from Eq. (i)]
1 1 5
or ´ 1 ´ v1 + ´ 4 ´ v 22 = 500
2
4
2 2 = ´ 103 kg m-3
or v12 + 4v 22 = 1000 … (ii) 5
= 800 kg m-3
Solving Eqs. (i) and (ii), we get
v1 = 30 m/s, v 2 = 5 m/s p
11. (d) The angle, subtended by arc DE at O is and FC at
Hence, the separation between the two fragments 2
3p
= (v1 - v 2 ) ´ t = ( 30 - 5) ´ 177
. m = 44.25 m O= the effective magnetic field at O is
2
8. (d) Given, initial speed, u = 36 km/h B = BDE + BFC
36 ´ 1000
= = 10 ms-1 l
60 ´ 60

R O 2r D
m
r C
B

F i
s in q
mg
q E
q
A C m0i p m 0i æ 3p ö
= ´ + ´ç ÷
mg mg cos q 4p 3r 2 4pr è 2 ø
m 0i é 1 ù
q = 30°, m = 0.1, s = ? = + 3ú
8r êë 3 û
Here, work done in moving up the inclined road
5m 0i
= KE of the vehicle = acting downwards
12 r
1
(mg sin q + F )s = mu 2
2
SOLVED PAPER 2018 17
12. (a) The electron moving with a speed v in the circular Bohr Putting the value of Dx in Eq. (i), we get
orbit of radius r constitutes a current of magnitude. l
sin q =
3d
æ l ö
v q = sin-1ç ÷
e–
è 3d ø
r 16. (a) The bob filled completely with water has its centre of
mass at its centre. The time period of oscillation is
l
mi
T = 2p
g
e As the water starts coming out of the bob, its centre of mass
i= , where T is the orbital period of the electron.
T shifts vertically downward as a result effective length of the
T = 2p r / v pendulum increases and hence its period also increases.
ev When the bob is empty, again its centre of mass appears at
i= its centre and as a result, the period of oscillation again
2pr
reaches to its original value.
By definition of atomic dipole moment m l , its magnitude is
given by 17. (a) On raising the temperature of the semiconductor,
covalent bonds start breaking up and thus more charge
ev evr
m l = iA = × pr 2 = carriers release. This will increase the value of n and as a
2pr 2
result the rate of collision of charge carriers will increase.
\ Angular momentum, L = mvr This will decrease the drift speedv.
L 2mvr 2m
\ = = = constant 18. (b) When a tennis ball falls on the ground and bounces
ml evr e
back, its velocity and displacement changes in reverse
13. (c) Let T0 be the temperature of point C and x be the length direction while acceleration remains unchanged.
of rod AB or BC.
19. (b) Since, the lens is made up of two kinds of transparent
Then, CA = x 2 + x 2 = 2 x material, it has two refractive indices for the incident beam
of light. Hence, there will be two focal lengths of the lens and
At steady state, the rate of heat flowing from B to C = rate of
therefore two images will be observed.
heat flowing from C to A.
KA( 2 T - T0 ) KA (T0 - T ) 20. (c) As we know,
So, = L = Iw
x 2x
2p
2 ( 2 T - T0 ) = T0 - T Þ L =I ´
T
3T 1
By solving, T0 = Lµ
( 2 + 1) T
1 Given, T2 = 2T
14. (b) At resonance, XL = XC or wL =
wC L1 2T
Hence, =
L2 T
Voltage across the series L-C combination,
L1
V2 = i (XL - XC ) = 0 Þ L2 =
2
æ fö
15. (c) I = I max cos2 ç ÷ Maximum weight
è 2ø 21. (a) Maximum stress =
Cross- sectional area
I max f
= I max cos2 Now, maximum weight of steel wire
4 2
= Volume ´ Density ´ g
f 1
cos = = A lrg
2 2
f p where, l is the maximum length of steel wire that can hang
= vertically without breaking, r is the density of steel and A is
2 3
2p æ 2p ö the cross-sectional area of steel wire.
Þ f= = ç ÷ ´ Dx … (i) A lrg
3 è l ø \ Maximum stress, p = = lrg
A
where, Dx = d sin q p
\ l=
rg
18 SELF STUDY GUIDE BITSAT
22. (c) 2h = ut 2s 2s
and t¢ = =
2h a g sin q - m k g cos q
ux = [Q u = u x , t = Dt ] … (i)
Dt
t¢ g sin q
1 2 Now, =n =
h = uy t - gt t g sin q - m k g cos q
2
sin 45°
y =
uy sin 45° - m k cos 45°
v 1
=
1- mk
h 1 1
x Þ = 1- mk Þ mk = 1- 2
ux n2 n
2h
2 24. (c) Since, the initial activity is 50 times the activity for safe
gt - 2u y t + 2h = 0
occupancy, therefore R0 = 50 R, where R = l N.
2u y + 4u y2 - 8gh Since, R µ N
t1 = n t /T
2g R N æ 1ö æ 1ö
= =ç ÷ =ç ÷
2u y - 4u y2 - 8gh R0 N0 è 2 ø è 2ø
t2 = t /10
2g æ 1ö 1
or ç ÷ =
è 2ø 50
4 u y2 - 8gh
Dt = t1 - t 2 = t
g ( 2)t /10 = 50 Þ log10 2 = log10 50
10
g 2 ( D t )2 10 log10 50 10 ´ 1699 .
u y2 = + 2gh Þ t= = = 56.4 days
4 2
log10 0 .301
u x2 + u y2 = u 2 = ( 2 gh )2
25. (a) Helium is monoatomic gas, for which
4h 2 g 2 ( D t )2 3 5
2
+ + 2gh = 4gh CV = R, Cp = R
(Dt ) 4 2 2
g2 Work done by the gas in one complete cycle W = area
( Dt )4 - 2gh ( Dt )2 + 4h 2 = 0
4 ABCDA = p 0V0
2gh ± 4g 2h 2 - 4g 2h 2 4h From A to B,
( Dt )2 = =
g2 / 2 g Heat given to gas = nCV DT
h æ3 ö 3 3
Dt = 2 = 1 ´ ç R ÷ ´ DT = V0 ( Dp ) = V0p 0
g è2 ø 2 2
1 2 From B to C, heat given to gas = nCp DT
23. (b) s = ut + at , a = g sin q, u = 0
2 æ5 ö 5
= 1 ´ ç R ÷ ´ DT = ( 2p 0 )DV = 5p 0V0
1 è2 ø 2
Then, s = 0 + at 2
2 Work done by the gas / cycle
Efficiency of cycle =
2s 2s Total heat given to gas / cycle
or t= = [for smooth plane]
a g sin q p 0V0 2
= =
3
q p 0V0 + 5p 0V0 13
c os 2
R
u xg 2
B
Efficiency (%) = ´ 100 = 15.4%
13
26. (a) Using Einstein’s photoelectric equations,
g sin q g cos q
g hn = f + eVs
q
A C eVs = hn - f … (i)
Thus, when the frequency of incident light radiation
For the rough plane, the effective value of acceleration
(photons) n increases to 2n, the stopping potential Vs
along the incline is
changes to Vs¢.
a ¢ = g sin q - m k g cos q
eVs¢ = h ( 2n ) - f … (ii)
SOLVED PAPER 2018 19
On dividing Eq. (ii) by Eq. (ii), we get Similarly, lB =
h
Vs¢ 2hn - f 2hn - 2f + f 2mTB
= =
Vs hn - f hn - f lA T T - 1.5 æ 1.5 ö
1/ 2
\ = B = A = çç1 - ÷
=
2(hn - f) + f
=2+
f lB TA TA è TA ÷ø
hn - f hn - f 2
æ 1ö 1.5
Vs¢ ç ÷ = 1-
\ > 2 Þ Vs¢ > 2Vs è 2ø TA
Vs
On solving, TA = 2.0 eV
27. (c) Terminal velocity of the rod is attained when magnetic So, fA = 4 - TA = 4 - 2 = 2.0 eV
force on the rod (Bil) balances the component of weight of fB = 6 - TA = 6 - 2 = 4.0 eV
the rod (mg sin q), as in figure.
30. (a) We have,
in q Torque = r ´ F = Ia
mg s
Fm So, 2 ( 20t - 5t 2 ) = 10a
a = 4t - t 2
dw
q But a = , so
dt
i
dw
So, Bil = mg sin q = 4t - t 2
dt
æe ö dw = ( 4t - t 2 ) dt
B ç ÷ l = mg sin q
èR ø
t3
B le Integrating, w = 2t 2 -
= mg sin q 3
R
B l (B lvT ) w will be zero at t = 6 s.
= mg sin q dq t3
R So, w= = 2t 2 -
mg sin qR dt 3
vT =
B 2l 2 æ t3 ö
or d q = çç 2t 2 - ÷÷ dt
è 3ø
28. (c) Initial energy of the asteroid is
1 GMem Again, integrating both sides
Ei = Ki + Ui = mvi2 -
2 10 Re 2t 3 t 4
q= -
Final energy of the asteroid, 3 12
1 GMem Since, t = 6 s
Ef = mvf2 -
2 Re 2 ´ 63 64
so, q= - = 36
According to law of conservation of energy, Ei = Ef 3 12
1 GMem 1 GMem q 36
mvi2 - = mvf2 - Number of turns, n = = = 5.73
2 10Re 2 Re 2p 2p
2 GMe 2GMe So, option (a) is right.
vf2 - = vi2 -
Re 10Re 31. (c)
2GMe æ 1ö
Þ vf2 = vi2 + ç1 - ÷
Re è 10 ø
(a) (b)
29. (b) From Einstein photoelectric equation, In first case refraction of the rays, takes place from a plane
E = f0 + KEmax d
surface, so we can use d app = actual
For metal A 4 = fA + TA … (i) m
For metal B 4.5 = fB + (TA - 1.5) … (ii) 4 4
3= Þ m =
m 3
From Eqs. (i) and (ii), we get
fB - fA = 2 where, d app and d actual = apparent and real depth,
respectively.
Now, according to de-Broglie hypothesis,
h h Now, in second case, as from Fig. (b), refraction takes place
lA = = from a spherical surface, so
mv 2mTA
20 SELF STUDY GUIDE BITSAT
m 2 m1 m 2 - m1 Now, the equivalent resistance of the circuit is
- =
v u R (5 + 5) ´ 20 20
Req = + W
1 4/3 1- 4 / 3 (5 + 5) + 20 3
or - =
( - 25 / 8) ( -4) -R 10v
Current through battery, I = = 1.5 A
1 1 8 1 20
or = - = W
3R 3 25 75 3
R = 25 cm 34. (a) Since, no external force is acting on the system, we can
Now, using Lens’ Makers formula, to calculate focal length apply conservation of linear momentum.
1 æ 1 1ö 36 ´ 1000
= (m - 1) çç - ÷÷ Speed of 200 kg trolley= = 10 ms-1
60 ´ 60
f è 1
R R 2ø

1 æ4 ö æ1 1 ö 1 If u be the initial velocity of trolley,vb be the absolute velocity


= ç - 1÷ ç - ÷=
f è3 ø è ¥ ( - 25 ) ø 75 of the boy after the beginning of journey of the boy, their
relative velocity is 4.
f = 75 cm
So, v ¢ - vb = 4 Þ vb = (v ¢ - 4)
32. (c) When r £ R, then force on the test mass m at the surface
Now, applying law of conservation of momentum,
of the sphere = mg
Momentum before the boy begins to run
Force on the test mass at distance r from the centre of = Momentum after the beginning of boy’s running
sphere is
Þ 220 ´ 10 = 200 v ¢ + 20(v ¢ - 4)
2200 = 220v ¢ - 80 Þ 220v ¢ = 2280
2280
v¢ = = 10.36 ms-1
R 220
O r
35. (c) Given, C = 37.55 J mol -1 K -1
Also, pT = constant (K ) … (i)
According to ideal gas equation, pV = RT
RT
If r < R, then F =
GMm r
× =
GMm
×r Þ p= … (ii)
R2 R R3 V
Putting the value of p in Eq. (i), we get
mv 2 GMm
\ = ×r RT RT 2
r R3 ´T = K Þ V =
V K
\ v ´r
GMm On differentiating above equation both sides, we get
If r > R, then F = dV 2RT
r2 = … (iii)
dT K
Mv 2 GMm
\ = 2 T 1
r r But = [from Eq. (i)]
K p
1
\ v´ Hence, Eq. (iii) becomes
r
dV 2R
Hence, option (c) is correct. =
dT p
33. (c) In the given circuit, diode D1 is reverse biased, so it will pdV
not conduct but D2, D3 are forward biased, so they will So, C = CV +
dT
conduct, hence corresponding equivalent circuit. For the p ´ 2R
given circuit is or C = CV + = CV + 2R
p
D3
5W or CV = C - 2R … (iv)
nR
As CV = , where n = number of degrees of freedom.
2
20 W
5W Putting the value of CV in Eq. (iv), we get
D2 nR
= C - 2R
2
2 (C - 2R ) 2 (37.55 - 2 ´ 8.3)
n= = = 5.048 = ~
-5
10 V R 8.3
SOLVED PAPER 2018 21
36. (a) We know, instantaneous displacement, Initially, velocity is downwards (- ve), after collision, it
x = r sin wt reverses the direction with smaller magnitude and velocity
dx is upwards (+ ve). So, graph (a) satisfies these conditions.
Q Instantaneous velocity,v = = rw cos wt
dt Also,
Q Instantaneous acceleration, When t = 0, h = d
dv Velocity increases downwards ( 0 ® A )
a= = - rw2 sin wt = - w2x
dt
When t = 1, velocity reverses its direction ( A ¢ ® B )
aT - w2x ´ T - 4p 2
So, = = - w2T = ´T 39. (b) From the given circuit, we can say that I is independent
x x T2
of resistance R5, so no current flows through R5. This
- 4p 2 required that the junction of R1 and R2 is at the same
= = constant
T potential as the junction of R3 and R4 (\ Wheatstone bridge
aT - w2r sin wt ´ T condition)
Þ = = - wT tan wT
V w r cos wt R1 R3
So, =
2p R2 R4
=- ´ T tan wt = not constant
T or R1R4 = R3 R2
2 2 2 2
Similarly, aT + 2pv and a T + 4p v is also not constant, 40. (d) Since, volume remains unchanged, during this
i.e. both are function of t. phenomenon, so
50 4 4
37. (d) New, current sensitivity, Is¢ = Is + Is pR 3 = N ´ pr 3
100 3 3
150 3 R3
= Is = Is N=
100 2 r3
New resistance, R ¢ = 2R Now, change in surface area = 4pR 2 - N 4pr 2
Is = 4p (R 2 - Nr 2 )
Initial voltage sensitivity,Vs =
R
Energy released ( DU ) = T ´ change in surface area
Now, new voltage sensitivity,
= T ´ 4p [R 2 - Nr 2]
3
Is
Is¢ 3
¢
Vs = = 2 = Vs Here, all this energy released is at the cost of lowering the
R ¢ 2R 4 4
temperature and mass of the big drop of liquid = pR 2r.
% decrease in voltage sensitivity 3
Vs - Vs¢ Now, change in temperature,
= ´ 100
Vs DU
Dq =
æ V¢ö ms
= çç1 - s ÷÷ ´ 100 T ´ 4p(R 2 - Nr 2 )
è Vs ø =
æ4 3 ö
æ 3ö ç pR r ÷ S
= ç1 - ÷ ´ 100 = 25% è3 ø
è 4ø
3T æ 1 Nr 2 ö
= ç - ÷
38. (a) For the uniformly accelerated/decelerated motion, rS çR R3 ÷
è ø
v 2 = u 2 ± 2gh
3T æ 1 R3 ´r 2 ö
= ç - ÷
So, from this equation, we can sayv-h graph is parabola. rS çR r 3 ´R3 ÷
è ø
v A¢ At t = 0 3T æ 1 1ö
= ç - ÷
h=d rS è R r ø
d
o
B o h

A
22 SELF STUDY GUIDE BITSAT

Chemistry
41. (d) Edge length of the unit cell = 2 Å = 2 ´ 10-8 cm 47. (c) Azo dye test is not used for the distinction in 1°, 2° and 3°
Volume of unit cell = ( 2 ´ 10-8 )3 = 8 ´ 10-24 cm 3 aliphatic amines. The other tests can be easily used to
Mass of one unit cell = volume × density distinguish between 1°, 2° and 3° amines.

= 8 ´ 10-24 ´ 2.5 Test Primary amine Secondary amine Tertiary


amine
Number of unit cell in 200g of metal
i Hinsberg's Forms a Forms a No reaction
mass of the metal
= reagent test sulphonamide sulphonamide
mass of one unit cell soluble in alkali insoluble in alkali
200
= = 1´ 1025 ii Carbylamine Forms a No reaction No reaction
8 ´ 10-24 ´ 2.5 reaction carbylamine with
unpleasant smell
42. (a) Higher the values of ‘a’ more will be the tendency to get
iii Reaction Forms primary Forms nitrosamine Forms nitrite
liquefy. Value of a is highest for gas P. Thus, it is the most
with nitrous alcohol and evolves which gives salt
liquefiable gas among the given gases. acid nitrogen gas with Libermann's
effervescence nitroso amine
43. (d) Radius of hydrogen like atom,
reaction
n2
rn = ro
Z 48. (b) Resonating structure of carbonate CO2-
3 ion are
Where, ro = 0.51´ 10-10m O O– O–
-10
0.51´ 10 m
and rn = C C C
4 O– O– O– O– O O
At ground state, n = 1 I II III
Hence, the atom is Be3+ [Q Z = 4] Thus, there is no coordinate bond present in CO2-
3 .
Hence, the second statement is incorrect.
44. (a) When central metal ion is same (here Ni2+ ), the
absorption of colour depends on the ligand. According to 49. (a) For first order reaction,
spectrochemical series, various ligands are as follows ln2
t1/ 2 = = 40 s …(i)
- - -
I < Br < Cl < NO3- -
< F < H2O k1

< NH3 < NO2- < CN- < CO [ A] 0


For zero order reaction, t1/ 2 = = 20 s …(ii)
2k 0
Thus, H2O is the weakest ligand among these, therefore the
absorbed energy will be lowest in [Ni(H2O)6] 2+ , so it will Divide eqn. (i) by (ii), we get
1 [ A] 0 k1
absorb highest wavelength (red light). = ´
2 2k 0 ln2
1
Eµ k1 ln 0.693
l = 2 = = 0.5
k 0 [ A] 0 1386
.
\ The order of increasing wavelength is
[Ni(NO2 )6] 4- < [Ni(NH3 )6] 2+ < [Ni(H2O)6] 2+ 50. (b) CH2Cl2 is similar to CH4 (tetrahedral) but it has three
different bond angles.
45. (c) Empirical formula mass = CH2O = 12 + 2 + 1´ 16 = 30
HCH, HCCl, ClCCl, which are close to but not equal to
Molecular mass = 180
109.5°. Since, electronegativity of Cl > C > H the bond
Molecular Mass 180 dipole moments do not cancel and the molecule has a
n= = =6
Empirical formula mass 30 dipole moment.
Q Molecular formula = n ´ empirical formula
Cl
\ Molecular formula = 6 ´ CH2O = C6H12O6
+
46. (d) (i) In Hall-Heroult process for extraction of Al, carbon C
H Cl
anode is oxidised to CO and CO2. H
(ii) When Al2O3 is mixed with CaF2, it lowers the melting
point of the mixture and brings conductivity. 51. (b) Given, CH4 + 2O2 ¾® CO2 + 2H2O ;
(iii) Al 3+
is reduced at cathode to form Al. DH = - 210 kcal / mol …(i)
(iv) Al2O3 serves as the electrolyte, undergoing the redox 7
C2H6 + O2 ¾® 2CO2 + 3H2O ;
process. Na 3AlF6 although is an electrolyte but serves as 2
a solvent, not electrolyte. DH = - 368 kcal /mol …(ii)
SOLVED PAPER 2018 23
On subtracting Eq. (i) from Eq (ii), we get 5 PCl Alc. KOH
3 56. (c) CH3 — CH— CH3 ¾¾® CH3 — CH— CH3 ¾¾¾¾
CH2 + O2 ¾® CO2 + H2O ; ½ ½ ½
2 OH Cl ½
2-propanol 2-chloropropane ¯
DH = - 158 kcal / mol
(z) (x )
\ Enthalpy of combustion of one CH2 unit H2 SO 4 / D
= - 158 kcal / mol CH3 — CH— CH3 ¬¾¾¾¾¾ CH3 — CH==CH2
conc. H2 SO 4 Propene
½
DHcomb(C10H22 ) = DHcomb(CH4 ) +9 ´ DHcomb(CH2 ) OH (y)
2-propanol
= -210 + (9 ´ - 158) (z)
= -1632 kcal
57. (d) N2O4(g ) = 2NO (g ) 2
52. (b) R — C ºº CH is converted to ketone by catalytic 0. 28 1.1

hydration with reagents. Pressure at equilibrium


2
O p NO (1.1)2
Kp = 2
= = 4.32 atm
p N2O4 0.28
(i) H2SO4+HgSO4
C CH CH3
(ii) H2O,Heat
If volume of the container is doubled, the pressure will
reduced to half
53. (c) By Heisenberg's uncertainty principle, N2O4 2N2O=
h é 0.028 ù é 11
. ù
DmDv = New pressure, ê - p ú ê + 2p ú
4p ë 2 û ë2 û
2
Dv = 0.005%or 600 m/s é 11. ù
êë 2 + 2p úû
600 ´ 0.005 Kp = = 4.32
= = 0.03 é 0.28 ù
100 - p
êë 2 úû
6.6 ´ 10-34
Dx ´ 9.1´ 10-31 ´ 0.03 = On solving, we get
4 ´ 3.14
p = 0.045
6.6 ´ 10-34
Hence, Dx = \ p N2O4 = 0.14 - 0.045 = 0.095 atm
4 ´ 3.14 ´ 0.03 ´ 9.1´ 10-31
p NO2 = 0.55 + (2 ´ 0.045) = 0.64 atm
. ´ 10-3 m
= 192
58. (a) Protoactinium (Pa) is the element that does not have
54. (d) In O2 molecule, two electrons are present in antibonding stable electronic configuration.
orbitals. Protoactinium (Pa) = [Rn] 5f 2 6d 17s 2 (Atomic number = 91)
O2 ( 8 + 8 = 16) = s1s 2, s *1s 2, s2s 2s * 2s 2, Nobelium (No) = [Rn] 5f 14 6d 0 7s 2 (Atomic number = 102)
s2 2pz 2, p 2px 2 = p 2py 2, p * 2px = p * 2py 1 Lawrencium (Lr) = [Rn] 5f 14 6d 17s 2 (Atomic number = 103)

Removal of one electron from the O2 molecule gives O+2 in Amercium (Am) = [Rn] 5f 7 6d 0 7s 2 (Atomic number = 95)

which the number of antibonding electrons is one less and 59. (c) Oxidation :
hence, BO increases. Thus, removal of one electron from -
O2 stabilises the molecule. CH3COO- + 3 O H ¾® CH3COO- + 2e - + 2H2O
55. (c) Cu2+ +2e - ¾® Cu …(i) Reduction : [Ag(NH3 )2] + + e - ¾® Ag + NH3] ´ 2
According to Faraday's first law, when an electric current is - -
CH3CHO + 2 [ Ag( NH3 )2] + + 3 O H ¾® CH3O O + 2H2O
passed through an electrolyte, amount of substance
deposited is directly proportional to the quantity of electric Molecular mass (M) of CH3CO = 44 g mol-1
charge passed through the electrolyte. M 44
Equivalent weight (Ew) = = = 22 g mol-1
Q 1 mol of 63.5g of Cu is obtained by passing 2F of n - factor 2
electricity. 60. (a) According to Freundlich adsorption isotherm,
63.5 1
Mass of Cu obtained by passing 1F of electricity = g
2 x / m = kpn
\ Mass of Cu ontained by passing 0.5F of electricity where, x = mass of gas adsorbed on mass ‘m’ of the
63 .5 absorbent at pressure p.
= ´ 0.5 = 15. 80 g
2 k and n = constants.
24 SELF STUDY GUIDE BITSAT
Taking logarithm on both sides, we get 0.0591
E° = log k eq (at 25°C)
1 n
log x / m = log k + log p
n 0.0591
Þ 0.2955 = log k eq
1 2
This is equation of a straight line with slope of and k eq = 1010
n
intercept of log k .
66. (b) H N
NH2 N NH2
61. (b) Given, pH of Ba(OH)2 = 12 H
H H H O H
Q pH + pOH = 14
\ pOH = 14 - pH = 14 - 12 = 2 H
H H H H H H
Now, pOH = - log [OH- ] H
OH H OH
Þ 2 = - log [OH-] Anti Gauche Eclipsed
[OH- ] = 10-2
Since, gauche form is stabilised by intermolecular hydrogen
Ba(OH)2(s ) = Ba 2+
+ 2OH- bonding, hence it is more stable than anti.
S mol-1 S 2S
- -2 67. (a) In acidic medium, H2S is very feebly ionised giving very
[OH ] = 2S = 10
small concentration of sulphide ion for precipitation.
\ S = 10-1 Therefore, the most insoluble salts CuS and HgS are
K sp = [Ba 2+ ] [OH- ] 2 precipitated.
Þ [10-1] [10-2] 2 = 0.5 ´ 10-6 = 5 ´ 10-7 M 68. (b) When a mixture of BCl3 and H2 is subjected to silent
electric discharge, diborane and HCl are formed. Diborane
62. (c) With Tollen's reagent (ammoniacal AgNO3), a white
reacts with NH3 at 200°C to give borazine (inorganic
precipitate of silver salt is obtained benzene).
H — C ºº C — H + 2AgNO3 + 2NH4OH
Electric
2BCl3 + 6H2 ¾¾¾® B2H6 + 6HCl
¾® Ag — C ºº C — Ag + 2NH4NO3 + H2O discharge
(X )
Silver acetylide (white ppt.)
200 °C
63. (d) Arrhenius equation is given by, 3B2H6 + 6NH3 ¾¾® 2B3N3H6 + 12H2
1 2 Boarzine ( y )
K Ea éT2 - T1 ù
log10 2 = ê ú 69. (b) The process of producing syn gas or synthesis gas from
K1 2.303 ´ R ë TT 1 2 û
coal is called ‘coal gasification’?
K2
Given, = 3;R = - 8.314 JK -1mol-1 1270 K
K1 C(s ) + H2O(g ) ¾¾® CO(g ) + H2(g )
Coal Ni 1442443
Steam
T1 = 20 + 273 = 293 K Synthesis gas

and T2 = 50 + 273 = 323 K The production of hydrogen can be increased by reaching


Substituting the given values in Arrhenius equation, carbon monoxide of the syn gas with steam in the presence
Ea é 323 - 293 ù of iron chromate as a catalyst at 673 K.
log10 3 = ê ú
8.314 ´ 2.303 ë 323 ´ 293 û FeCrO4 , 673 K
CO(g ) + H2O(g ) ¾¾¾¾® CO2(g ) + H2(g )
2.303 ´ 8.314 ´ 323 ´ 293 ´ 0.477
Ea =
30 CO2 is removed by scrubbing with a solution of sodium
-1 arsenite.
= 28811.8 J mol
= 28.8118 kJ mol-1 70. (b) Heat at constant pressure means enthalpy. i.e.
64. (b) Compounds containing both N and S give blood red DH = 2.5 ´ 103 cal
colour in Lassaigne’s test due to the formation of Fe(SCN)2. DS = 7.4 cal deg-1
Thus, H2N (C6H4 )SO3H gives blood red colour in
Lassaigne's test of nitrogen. T = 298 K
DG = DH - TDS
65. (b) Given that,
A + B 2+ ¾® B + A 2+ , E° = 0.2955 V = 2.5 ´ 103 - 298 ´ 7.4

Using Nernst equation, = 4705 cal


2.303nRT Hence, the process is spontaneous.
E° = log k eq
nF
SOLVED PAPER 2018 25
71. (a) The alkene is unsymmetrical, hence will follow 75. (b) CH2OH
Markownikoff’s rule to give major product. O H
H
CH3 ¾ CH2 ¾ CH == CH2 + H¾ Br ¾® H CH2OH H
But -1-ene O
Br OH H
O H HO
½ HO
CH3 ¾ CH2 ¾ CHBr ¾ CH3 + C2H5 ¾ C ¾ CH3 CH2OH
(x ) * H OH Glycosidic
½ a-D-glucose linkage
OH H
H b-D-fructose
(y )

+ CH3 ¾ CH2 ¾ CH2 ¾ CH2Br This structure represents sucrose in which a-D-glucose and
( z ) (Minor product) b-D fructose is attached to each other by C1 - C2 glycosidic
linkage. Since reducing groups of glucose and fructose are
Since, y contains, a chiral carbon, it exists in two involved in glycosidic bond formation, this is considered as
enantiomers (x and y) which are mirror images of each non-reducing sugar.
other.
76. (a) Potassium sulphate (K 2SO4 ), i = 3
Mirror
Br Br Sodium chloride (NaCl), i = 2
Urea, i = 1
C C
Glucose, i = 1
H5C2 CH3 CH3 C2H5
H H Greater the value of i, greater lowering in freezing point and
hence, lower will be the freezing temperature.
Therefore, K 2SO4 solution has the lowest freezing point.
72. (c) Silicons are synthetic organo-silicon polymers
containing repeated R2SiO units. Since the empirical 77. (a) A penicillin is a member of a family of drugs that have a
formula is same as that of a ketone (R2CO), the name silicon four membered cyclic amide fused to a five membered
has been given to these materials. They can be formed by thiazole ring. It is narrow spectrum antibiotic
hydrolysis of dichlorosilanes O H
(R2SiCl2) S
R C N CH3
R2SiCl2 + 2H2O ¾® R2Si(OH)2 + 2HCl H
N CH3
R R O
H
½ ½
nHO — Si— OH + HO — Si— OH ¾® COOH
½ ½ (Penicillin)
R R
é R R ù R R 78. (b) The lattice energy of the hydroxides of alkaline earth
ê ½ ½ ú - nH2 O ½ ½ metal decreases more rapidly than their hydration energy
ê O — Si— O — Si— O ú ¬¾¾ HO — Si— O — Si— OH leading to more negative value of DHsol down the group.
ê ½ ½ ú ½ ½
êë R R úû R R More negative is DHsol, more is the solubility of compounds.
Hence the solubility of hydroxides of alkaline earth metal
73. (c) On acidification of potassium chromate solution, yellow increases with their atomic number.
colour changes to orange colour due to formation of
79. (a) The given chemical reaction is
dichromate which suggests that monocentric complex is
converted into dicentric complex. 5Br - (aq ) + BrO-3(aq ) + 6H+ (aq ) ¾® 3Br2(aq ) + 3H2O (l )
2K 2CrO4 + H2SO4 ¾® K 2Cr2O7 + K 2SO4 + H2O Rate law expression for the given reaction can be written as
(Dil.)
1 D[Br] - D[BrO3- ]
- =-
74. (d) Valence electrons in A = 3 5 Dt Dt

Valence electron in B = 6 1 D[H+ ] 1 D[Br2]


- =+
6 Dt 3 Dt
Thus, A is electropositive and B is electronegative, A can
D[Br - ] 5D[BrO3- ] 5 D[H+ ]
loose three electrons and B can gain two electrons to attain Þ - =- =-
Dt Dt 6 Dt
stable configuration
D[Br - ] 5 D[H+ ]
Hence, A exist as A 3+ and B as B 2- Þ =
Dt 6 Dt
\ Compound formed is A2B3.
26 SELF STUDY GUIDE BITSAT
80. (c) A-III, B-I, C-II, D-IV Polymerisation
Cl CF2 CF2 — CF2 —CF2 —
Tetrafluroethane Teflon n
Zieglar-Natta
CH2 C—CH CH2
Catalyst Polymerisation
Chloroprene
Cl n CH2 CH CH2 — CH —
Neoprene
CN CN n
CH3 CH3 Acryl nitrile Acrilan
Polymerisation
nCH2 C—CH CH2 — CH— C CHCH2 —
n
Isoprene Natural rubber

English Proficiency
81. (a) Use ‘is’ in place of ‘are’ as when two subjects are joined 85. (d) ‘Cope’ always agree with preposition ‘with’ and use of
‘along with’, then helping verb is used according to first any other auxiliary term is improper.
subject. 91. (b) The ordinary megalomaniac does not have the talent to
82. (c) Use ‘attested’ in place of ‘attest’ as sentence is in realise his wish which Alexander the Great had
passive voice. 92. (d) By wishing to be powerful and feared.
83. (b) Use ‘making’ in place of ‘having’ as ‘have’ is not used in 93. (d) Excessive lover of power
progressive sense. 94. (c) The nurturing of only one element in human nature
84. (d) ‘Superior’ is followed by ‘to’ preposition. 95. (c) Because it is a part of human nature

Logical Reasoning
96. (d) ‘Umpire’ is required to give decision in ‘Match’. 101. (b) Options (b) can be formed from the pieces given in
problem figures.
Likewise ‘Judge’ is required to give decision in a ‘Lawsuit’.
102. (a) After folding and cutting the paper, answer figure (a) will
97. (c) As, 9 - 2 = 7 ® 72 = 49 appear.
13 - 2 = 11® 112 = 121 103. (c) The third figure is each row comprises of parts which are
10 - 2 = 8 ® 82 ¹ 61 not common in the first two figures.
104. (b) Correct answer figure is (b).
7 - 2 = 5 ® 52 = 25
105. (b) The figure in the question may be labelled as shown
Thus, option (c) is odd.
below
98. (d) The pattern is as follows G
A B
10 9 16 45 176 875 J
I
O
E F
×1–1 ×2–2 ×3–3 ×4–4 ×5–5
L K
99. (d) The data is inadequate because it is not given that who is D C
taller between Virat and Shikhar. Hence, answer cannot be H
determined. There are nine triangle in the upper half of the figure
100. (c) Option (c) will complete the given pattern as follow. ABFE = DAEI , DAIG, DAEG, DGEO , DGBJ , DBFJ , DGBF ,
DGOF and DGEF .
Similarly, there are nine triangles in the lower half figure,
i.e. EFCD.
There are two more triangles DEGH and DFGH.
Hence, there are a total of 20 triangles.
SOLVED PAPER 2018 27
Mathematics
æ 1ö é
n

n
½ sin q cos q 0½
106. (b) (1 + x )n ç1 + ÷ = ê( 2 + x ) + ú
è xø ë xû 110. (d) Given, x = ½- cos q sin q 1½
2
½ ½
æ 1ö 1 ½ sin q cos q 2½
= nC0( 2 + x )n × ç ÷ + ¼ + nCn × n
èx ø x ½ sin q cos q 0½
\ Coefficient of x -n
= Cn = 1
n On expanding½- cos q sin q 1½along C3, we get
½ ½
r ½ sin q cos q 2½
æ 1 ö
107. (c) Tr = 3 ×20 Cr ç ÷ ½ sin q cos q½ sin q cos q½
½ + 2½
+1
è 3ø x 2 = 0 - 1½ ½ ½
r -1 ½ sin q cos q½ ½ cos q sin q ½
-
æ 1 ö
and Tr = 3 ×20 Cr - 1ç ÷ = - 1(sin q cos q - sin q cos q) + 2 (sin2 q + cos2 q)
è 3ø
Tr + 1 20 - r + 1 æ 1 ö = - 1´ 0 + 2 ´1
Now, = ç ÷ Þ x2 = 2 Þ x = ± 2
Tr r è 3ø
Since, Tr ³ Tr If x = 2, then
+1
Þ 20 - r + 1 ³ 3r 3p
4x 2 + x sin + 5 = 4 ´ ( 2 )2 - 2 + 5
2
21 21
Þ r £ = = 8 - 2 + 5 = (13 - 2 )
3 + 1 2.73
3p
Þ r £ 7.692 Þ r = 7 If x = - 2, then 4x 2 + x sin +5
7 2
20 æ 1 ö 25840
\ The greatest term is T8 = 3 × C7 ç ÷ = = 4 ´ ( - 2 )2 + 2+5
è 3ø 9
=8+ 2 + 5 = (13 + 2)
108. (b) Let x = 1
= cos 0° + i sin 0° 111. (d) Given, a, b, c are in GP.

= cos 2rp + i sin 2rp = ei 2rp \ b 2 = ac


and 2(log 2b - log 3c ) = log a - log 2b + log 3c - log a
Þ x 1/n = ei ( 2rp)/n; r = 0, 1, 2, ¼
2 pi 4 pi
Þ b 2 = ac and 2b = 3c
Then, the roots are 1, e n ,e n , ¼… 4a 2a
Þ c =
and b =
9 3
which are clearly on GP with common ratio e 2pi /n.
5a 10a
éa b c ù Q a+b= > c, b + c = >a
3 9
109. (a) Given, P = êb c a ú 13a
ê ú and c +a= > b.
êëc a b úû 9
éa b c ù \ a, b, c are the sides of a triangle.
\ P = êb c a ú
T Also, a is the greatest side
ê ú
êëc a b úû b2 + c 2 - a2 29
\ cos A = =- <0
2bc 48
Also given, P T P = I
\ D ABC is an obtuse angled triangle.
\ | P T P | = | I| = 1 Hence, option (d) is correct.
2
½a b c ½ ½a b c ½ n
n(n + 1) (n + 2)(n + 3)
or ½b c a ½ = 1 Þ ½b c a ½ = ± 1 112. (c) Given, å tr =
8
= Sn (say)
r =1
½ ½ ½ ½
½c a b ½ ½c a b ½ n -1
(n - 1) n (n + 1)(n + 2)
3 3 3
\ å tr =
8
= Sn - 1
or 3abc - a - b - c = ± 1 r =1

or 3
3 m 1= a + b + c 3 3 n(n + 1) (n + 2)
Now, tn = Sn - Sn - 1 =
2
\ a 3 + b 3 + c 3 = 2 or 4 n
1 n
2
\ Option (a) is correct.
\ lim
n ®¥ r =1
å = lim å
tr n ® ¥ r = 1 n(n + 1) (n + 2)
28 SELF STUDY GUIDE BITSAT
n æ 1 1 ö 2n
= lim å çç n(n + 1) - (n + 1) (n + 2) ÷÷ = tan-1
n ® ¥ r = 1è ø 1 + (n + n 2 + 1)
4

næ 1 1 ö (n 2 + n + 1) - (n 2 - n + 1)
= - lim å çç - ÷ = tan-1
n ® ¥ r = 1è (n + 1)(n + 2 ) n (n + 1) ÷ø 1 + (n 2 + n + 1) (n 2 - n + 1)
un = tan-1 (n 2 + n + 1) - tan-1 (n 2 - n + 1)…(iii)
æ 1 1ö
= - lim çç - ÷÷ On putting n = 1, 2, 3, ¼¼ successively in Eq. (iii), we get
n ® ¥ è (n + 1) (n + 2 ) 2ø
u1 = tan-1 3 - tan-1 1
æ 1ö 1
= - ç0 - ÷ = u 2 = tan-1 7 - tan-1 3
è 2ø 2
Hence, option (c) is correct. u 3 = tan-1 13 - tan-1 7
…………………………
113. (c) Truth table
un = tan-1(n 2 + n + 1) - tan-1(n 2 - n + 1)
p q ~ p ~q p Ú q ~q Ù p p Ú ~ p (p Ú q ) Ú (~ p ) (~ q Ù p ) Ú (p Ú ~ p )
On adding vertically, we get
T T F F T F T T T ¥

T F F T T T T T T å un = tan-1 (n 2 + n + 1) - tan-1 1
n =1
F T T F T F T T T ¥
F F T T F F T T T S = lim å un
n ®¥ n =1
[from Eq. (i)]

Hence, option (c) is correct.


= lim tan-1(n 2 + n + 1) - tan-1 1
114. (c) The two sets of m parallel lines along with two sets of two n ®¥

parallel lines of the given parallelogram will form two sets of p p p


= - =
(m + 2) parallel lines. Each parallelogram is formed by 2 4 4
choosing two parallel lines from each of the above
117. (a) Given, sin x + cos(t + x ) + cos(t - x ) = 2
\ Total number of parallelograms
Þ sin x + 2 cos t × cos x = 2
= m + 2C2 ´ m + 2C2 = (m + 2C2 )2
For real solution
Hence, option (c) is correct.
1 + 4 cos2 t ³ 2
115. (a) Let f (x ) = y , then 3
Þ cos2 t ³
ay = x + x 2 + 1 4
1 1 1
Þ a -y = Þ - £ sin t £
2 2
x + x2 + 1
Hence, option (a) is correct.
-y x - x2 + 1
Þ a = [rationalising] 118. (c) We have,
-1
cos2 a + cos2 b + cos2 g = 1
\ a y - a - y = 2x
1 æp ö é pù
Þ x = (a y - a - y ) Þ cos2 a + cos2 ç - a ÷ + cos2 g = 1 êgiven, a + b = ú
2 è2 ø ë 2û
1 Þ cos2 a + sin2 a + cos2 g = 1
\ f -1(y ) = (a y - a - y ) [Q f (x ) = y Þ x = f -1(y )]
2
Þ 1 + cos2 g = 1
1
Þ f -1(x ) = (a x - a - x ) [replacing y by x]
2 Þ cos2 g = 0
Hence, option (a) is correct. Þ cos g = 0
¥
2n \ (cos a + cos b + cos g ) = (cos a + sin a )2
2
116. (c) Given, S = å tan-1 n 4 + n 2 + 2 ...(i)
n =1 = 1 + 2 sin a × cos a = 1 + sin 2 a
4 2 2 2
Let n + n + 1 = [(n ) + 1 + 2(n ) (1)] - n2 2 2 Hence, option (c) is correct.

= (n 2 + 1)2 - n 2 119. (c) The given straight lines are 3x + 4y = 5 and 4x - 3y = 15.
Clearly, these straight lines are perpendicular to each other
= (n 2 + n + 1)(n 2 - n + 1) …(ii)
(m1m2 = - 1) and intersect at A. Now, B and C are points on
æ 2n ö these lines such that AB = AC and BC passes through
Let un = tan-1 çç 4 2
÷
÷
è n + n + 2 ø (1, 2).
SOLVED PAPER 2018 29
From figure it is clear that ÐB = ÐC = 45°

4x – 3y – 15 = 0 Q
C
L
(1, 2)
O
45° 3x + 4y – 5 = 0 P
Ö10
A B
Ö5x + 2y = 3Ö5

Let slope of BC be m. Then,


3
m+ Radius of the given circle = 10 = OQ = OP
tan 45° = 4
3 PQ = 2QL
1-
m
4 = 2 OQ 2 - OL2
4m + 3
Þ ± 1= = 2 10 - 5 = 2 5
4 - 3m
4m + 3 = ± ( 4 - 3m ) 1
Thus, area of D OPQ = ´ PQ ´ OL
4m + 3 = 4 - 3m 2
1
or 4m + 3 = - 4 + 3m = ´ 2 5 ´ 5 = 5 sq units
2
1
m= Hence, option (c) is correct.
7
or m=-7 122. (b) Let r be the radius of the largest circle passing through
Hence, equation of BC is the focus (1, 0) of y 2 = 4x
1 Y
y -2= (x - 1)
7
y 2 = 4x
or y - 2 = - 7(x - 1)
Þ 7y - 14 = x - 1
(1, 0)
or y - 2 = - 7x + 7 X¢ X
(1+r, 0)
Þ x - 7y + 13 = 0
or 7x + y - 9 = 0
Hence, option (c) is correct.

120. (d) Let y = mx + c , intersect y 2 = 4ax at A(at12, 2at1) and
B(at 22, 2at 2 ) Clearly, centre of the circle will be on X-axis and its
coordinates are (1 + r , 0).
Then,
2 The equation of the circle, is (x - 1 - r )2 + y 2 = r 2.
=m
t1 + t 2
It touches y 2 = 4x . Therefore, the equation
2
Þ t1 + t 2 = 2 2
(x - r - 1) + 4x = r must have equal roots
m
Let the foot of another normal be C(at 32, 2at 3 ). \ 4(1 - r )2 - 4( 2r + 1) = 0

Then, t1 + t 2 + t 3 = 0 Þ r =4
2 Hence, option (b) is correct.
t 3 = (t1 + t 2 ) = -
m æ pö æ p pö æ 2a b ö
123. (d) The point P ç ÷ is ça sec , b tan ÷ or P ç , ÷
æ 4a -4a ö è 6ø è 6 6ø è 3 3ø
Thus, other foot is ç 2 , ÷.
èm m ø x y
\ Equation of tangent at P is - =1
Hence, option (d) is correct. 3a 3b
121. (c) Length of perpendicular from origin to the line 2
x 5 + 2y = 3 5 is 1 3a
\ Area of the triangle = ´ ´ 3b = 3a 2
2 2
3 5 3 5
OL = = = 5 b
2
( 5) + 2 2 9 \ =4
a
30 SELF STUDY GUIDE BITSAT
b2 127. (d) Equation of auxiliary circle is
\ e2 = 1+ = 17
a2 x2 + y2 =9 …(i)
2
Now, e - 9 = 17 - 9 = 8
Y
Hence, option (d) is correct.
–12 , 9 M
124. (d) Let distances of a point p (x , y , z ) from the planes 5 5 B(0,1)
x + y + z = 0, x - z = 0 and x - 2y + z = 0 are
x + y + z x -z x - 2y + z X¢ X
N O A(3, 0)
, and respectively, then
3 2 6
the sum of the squares of distances, is as
2 2 2
æx + y + z ö æx - z ö æ x - 2y + z ö 2 Y¢
Þ ç ÷ +ç ÷ +ç ÷ =p
è 3 ø è 2 ø è 6 ø x y
\Equation of AM is + =1 …(ii)
Þ 2(x + y + z )2 + 3(x - z )2 + (x - 2y + z )2 = 6p 2 3 1
Þ 2x 2 + 2y 2 + 2z 2 + 4xy + 4yz + 4zx + 3x 2 On solving Eqs. (i) and (ii), we get
2 2 2 2
+ 3z - 6xz + x + 4y + z - 4xy æ 12 9 ö
M ç- , ÷
- 4yz + 2xz = 6p 2 è 5 5ø
Þ 6x 2 + 6y + 6z 2 = 6p 2 1 27
Now, area of D AOM = OA ´ MN = sq units
2 10
Þ x 2 + y 2 + z 2 = p2
Hence, option (d) is correct.
Hence, option (d) is correct.
128. (b) Given equation can be written as
125. (a) When line 25
x2 -y2 =
x - x1 y - y1 z - z1 3
= =
a1 b1 c1 b2
\ e1 = 1 +
is perpendicular to plane ax + by + cz + d = 0, then a2
a1 -b1 c1 = 1+ 1= 2
= =
a b c The equation of conjugate hyperbola is
l 1 -4 25
\ = = -x2 + y2 =
2 2 -8 3
Þ l =1 b2
Hence, option (a) is correct. \ e2 = 1 + = 1+ 1 = 2
a2
126. (c) Taking the coordinates of vertices O , P , Q , R as \ e12 + e12 = ( 2 )2 + ( 2 )2 = 4
( 0, 0), (a, 0), (a, a ), ( 0, a ), respectively.
Hence, option (b) is correct.
æ aö æa ö
\The coordinates of M is ça, ÷ and N is ç , a ÷ . 129. (a) We have,
è 2ø è2 ø
f (x + y ) = f (x ) + 2y 2 + kxy for all x , y ÎR
N(a/2, a)
R(0, a) f (x + y ) - f (x )
Q(a, a) Þ = 2y + kx for all x ÎR
y
f (x + y ) - f (x )
Þ lim = lim ( 2y + kx )
M(a, a/2) y ®0 y y ®0

Þ f ¢ (x ) = kx for all x Î R
kx 2
O(0, 0) P(a, 0) Þ f (x ) = + C for all x Î R [by integration]
2
0 0 1 But, f (1) = 2 and f ( 2) = 8.
1 3a 2
\ Area of D OMN = a a/2 1 = k
2 8 \ 2 = + C and 8 = 2k + C
a/2 a 1 2
and area of the square = a 2 k = 4 and C = 0

\ The required ratio is 8 : 3. Hence, f (x ) = 2x 2 for all x Î R


Hence, option (c) is correct. So, option (a) is correct.
SOLVED PAPER 2018 31
130. (a) Since, given planes are perpendicular to each other, i.e. From similar DOMP and DONQ, we have
its normal are perpendicular OM PM
=
\ 2( l ) - l(5) + 3( -1) = 0 ON QN
Þ - 3l - 3 = 0 Þ l = -1 h r
Þ =
\ l2 + l = ( -1)2 - 1 = 0 10 5
5h
Þ r =
Hence, option (a) is correct. 10
dv dy h
131. (d) Put x + y = v and = 1+ in the given differential Þ r =
dx dx 2
2
equation. 1 2 1 æh ö
We have, V = pr h = p ç ÷ h
dv 3 3 è 2ø
\ = 1 + sinv + cosv
dx
ph 3
dv Þ V =
Þ = dx 12
v v v
2 cos2 + 2 sin cos On differentiating both sides, we get
2 2 2
1 dV p dh ph 2 dh
sec 2v = × 3h 2 =
2 2 dv = dx dt 12 dt 4 dt
Þ
v dV
1 + tan Given, = 3 cm 3/sec when h = 4 cm, so we get
2 dt
æ x +yö p ´ 42 dh
Þ log ç1 + tan ÷ = x + C [by integration] 3=
è 2 ø 4 dt
Hence, option (d) is correct. dh 3
Þ = cm/sec
dt 4p
132. (d) Since, the numerator tends to ¥ as x ® 0,
3
1 ax Hence, the water level is rising at cm/sec.
so lim (e - e x - x ) 4p
x® 0 x 2
\Option (b) is correct.
1 ( a e ax - e x - 1)
= lim
2 x® 0 x 134. (b) We have,
dy
For last limit to exist we must have, = y + 2x
dx
lim ( a e ax - e x - 1) = 0
x® 0 dy
Þ - y = 2x
\ a - 1- 1= 0 Þ a=2 dx
For a = 2 the last limit and equal to Now, IF = e - ò 1dx = e - x
1 ( 2 e 2 x - e x - 1) y × e - x = ò 2x e - x dx + k , k be the constant of integration
= lim
2 x® 0 x
òe dx - ò 1× ( -e -x ) dx ] + k
-x
= 2 [x
1 2x 3
= lim ( 4e - e x ) =
2 x® 0 2 [using integration by parts]
Hence, option (d) is correct. Þ y × e - x = - 2xe -x - 2e -x + k …(i)

133. (b) Let depth of water at time t be h and the radius of the As curve (i) passes through (0, 0)
base of water level be r. \ 0=0-2+k
Þ k =2
5 cm
N Q Thus, the curve is
ye - x = - 2 xe - x - 2e - x + 2
\ y = 2 (e x - x - 1)

r 10 cm Hence, option (b) is correct.


M P
h 135. (c) f ( 0) = 0
For f (x ) to be continuous at x = 0
lim f (x ) = 0
O x® 0

1
\ lim x p sin =0
x® 0 x
32 SELF STUDY GUIDE BITSAT
This possible only when p > 0 …(i) 4 + 0 -4
= =0
f (h ) - f ( 0) | a | | b|
f ¢ ( 0) = lim
h® 0 h cos q = cos 90° Þ q = 90°
1 \ sin q + cos q = cos 90° + sin 90° = 0 + 1= 1
h p sin - 0
= lim h Hence, option (c) is correct.
h® 0 h
1 139. (d) A
= lim h p - 1 sin
h® 0 h 2
F E
Þ f ¢( 0) will exist only when p > 0 G

\ f (x ) will not be differentiable if 1


p £1 …(ii) B C
D
From Eqs. (i) and (ii), for f (x ) to be not differentiable but ¾® ¾®
continuous at x = 0, possible values of p are given by \ AD = 3 GD
0 < p £ 1. Hence option (c) is correct. 1 ¾® ¾®
= 3 × (GB + GC )
2
136. (a) Integrating the given differential equation, we have
3 æ 2 ¾® 2 ¾®ö
dy - cos 3x x3 = ç EB + FC ÷
= + ex + + C1 2 è3 3 ø
dx 3 3
¾® ¾®
But y1( 0) = 1 = - BE - CF
æ 1ö Hence, option (d) is correct.
So, 1 = ç - ÷ + 1 + C1
è 3ø
140. (c) Let x1, x 2, ¼, xn be n observations.
1
Þ C1 = 1
3 Then, x = S xi
n
dy - cos 3x x3 1
\ = + ex + + x
dx 3 3 3 Let yi = i + 10
a
Again integrating, we get 1 1 1 1
å y = æç S xi ö÷ø + n (10 n )
n

sin 3x x4 1 Then,
y =- + ex + + x + C2 n i =1 i a èn
9 12 3
1 X + 10a
But y( 0) = 0, so 0 = 0 + 1 + C2 Þ x new = X + 10 =
a a
Þ C2 = - 1
Hence option (c) is correct.
sin 3x x4 1
Thus, y =- + ex + + x -1 141. (c) Volume of the parallelopiped = |[ a b c]|
9 12 3
Hence, option (a) correct. Þ (area of the base parallelogram) ´ h = |[ a b c]|
Þ | a ´ b | h = |[ a b c]|
137. (d) Given, f (x ) = - 2x 3 - 9x 2 - 12x + 1 1 1 1
Þ f ¢ (x ) = - 6x 2 - 18x - 12 Þ $ $ $
|-5i + 3j + 2k | h = 2 4 -1
To be decreasing f ¢ (x ) < 0 1 1 3
Þ 2
- 6x - 18x - 12 < 0 4
Þ 38 h = 4 Þ h =
2 38
Þ x + 3x + 2 > 0
Þ 38h 2 = 42 Þ 19h 2 = 8
Þ (x + 2)(x + 1) > 0
Hence, option (c) is correct.
Therefore, either x < - 2 or x > - 1
142. (b) Mean (m ) for BD = np
Þ x Î ( - 1, ¥ ) or ( - ¥ , - 2)
and variance ( s2 ) for BD = npq
Hence, option (d) is correct.
Given, np + npq = 2.7 and n = 3
138. (c) Let a = 4$i - 4k$ and b = $i + $j + k$ 2.7
\ np(1 + q ) = 2.7 Þ p(1 + q ) =
Let q be the angle between a and b 3
a ×b Þ p (1 + q ) = 0.9
\ cos q =
| a | | b| Þ (1 - q )(1 + q ) = 0.9
[Q for BD probability standard deviation (SD), p + q = 1]
( 4$i - 4k$ ) × ( $i + $j + k$ )
= Þ 1 - q 2 = 0 .9
| a | | b|
SOLVED PAPER 2018 33
Þ q 2 = 0 . 1 Þ q = ± 0.3 145. (a) Given curves are
\ q = 0.3 |x| + |y| = a …(i)
[Qq = - 0.3 can’t possible as 0 £ q £ 1]
and x 2 + y 2 = a2 …(ii)
Þ p = 0.7 and q = 0.3
Y
\ B.D. = ( 0.3 + 0.7)5
B
Hence, option (b) is correct. C
143. (b) We have,
dx dx X¢ X
R(x ) = ò - 2ò
O A
-x -3 x
e + 8e
x
+ 4e e 3x
+ 8e x + 4e - x
2x
e (e - 2)
x
Þ R(x ) = ò dx Y¢
e + 8e 2x + 4
4x

Now, required areas = 4 [shaded area in the first quadrant]


On substituting
é pa 2 a ù
ex = t Þ e x dx = dt , = 4ê - ò ( a - x )2 dx ú
ë 4 0
û
we get
[from Equ. (i) and (ii)]
(t 2 - 2) dt (1 - 2t -2 ) dt
R(t ) = ò =ò é pa 2 ù
4 2
t + 8t + 4 (t + 2t -1)2 + 4 = 4ê
a
- ò (a + x - 2 a x ) dx ú
ë 4 0
û
1 æ t + 2t -1 ö
= tan-1 çç ÷+K
2 ÷ø
a
2 è pa2 é x2 4 ù
=4 - 4 êax + - a x 3/ 2 ú
æ e x + 2e - x ö 4 ë 2 3 û0
1
Þ R(x ) = tan-1 çç ÷+K
÷
2 è 2 ø æ 2ö
= ç p - ÷ a 2 sq units
è 3ø
Hence, ( A, B, C ) = (tan-1, e x , 2)
Hence, option (a) is correct.
\Option (b) is correct.
1 1
146. (b) P ( A Ç B ) = and P ( AC Ç BC ) =
-1 2 -1 æ
ç
1 ö
÷ 6 3
144. (b) cot (1 - x + x ) = tan ç1- x + x 2 ÷
è ø 1
Now, P ( A È B )C = P ( AC Ç BC ) =
æ 1 ö 3
= tan-1 çç ÷÷ 1
è 1 - x (1 - x ) ø Þ 1 - P(A È B) =
3
æ x + (1 - x ) ö 2
= tan-1 çç ÷÷ Þ P(A È B) =
è 1 - x (1 - x ) ø 3
Þ cot -1(1 - x + x 2 ) = tan-1 x - tan-1(1 - x ) But P ( A È B ) = P ( A ) + P (B ) - P ( A Ç B )
5
1 1 1 Þ P ( A ) + P (B ) = …(i)
\ ò 0 cot -1(1 - x + x 2 ) dx = ò tan-1 x dx - ò tan-1 (1 - x ) dx 6
0 0
1 1 Q A and B are independent events
= ò tan-1 x dx + ò0 tan
-1
x dx
0 \ P ( A Ç B ) = P ( A ) P (B )
éQ af (x ) dx = - a f (a - x ) dx ù 1
ëê ò0 ò0 Þ P ( A ) P (B ) =
ûú 6
1 [P ( A ) - P (B )] 2 = [P ( A ) + P (B )] 2 - 4 P ( A ) P (B )
= 2 ò tan-1 x dx
0 25 4 1
= - =
On evaluating by integration by parts, we have 36 6 36
ì ü 1
= 2 í[tan-1 x × x ]10 - ò
1 x
dx ý Þ P ( A ) - P (B ) = ± …(ii)
0 1+ x 2 6
î þ
On solving Eqs. (i) and (ii), we get
ïì p é 1 ù ïü
1
1 1
= 2 í - ê ln (1 + x 2 )ú ý P ( A ) = or
îï 4 ë 2 û 0 þï 2 3
Hence, option (b) is correct.
ép 1 ù p
= 2 ê - log 2ú = - log 2
ë4 2 û 2 147. (d) Let A be the event that the examinee gives the correct
1 p answer.
Hence, ò cot -1(1 - x + x 2 ) dx = - log 2
0 2 Let G, C, K stand for guessing, copying and knowing,
So, option (b) is correct. respectively.
34 SELF STUDY GUIDE BITSAT
1 1 149. (d) From the figure, it is clear that there is no common area.
Given, P (C ) = , P (G ) =
6 3 So, we cannot find maximum value of z.
æAö 1
and Pç ÷ = Y
èC ø 8
Þ P (C ) + P (G ) + P (K ) = 1
1 1
Þ + + P (K ) = 1
6 3 (0, 6)
1
\ P (K ) =
2 X¢ X
O (9, 0)
æAö
Also, P ç ÷ = 1, for if the examinee knows, he/she will
èK ø
æAö 1
correctly answer it and P ç ÷ = , since there are four
èGø 4
choices. Y¢

Now, total probability Hence, option (d) is correct.


æAö æAö æAö 150. (d) Feasible region is ABCA and Z = 7x - 8y
P ( A ) = P (G ) P ç ÷ + P (C ) P ç ÷ + P (K ) P ç ÷
èGø èC ø èK ø
Y
1 1 1 1 1
= × + × + ×1
3 4 6 8 2 C(0, 20)
1 1 1 4 + 1 + 24 29
= + + = =
12 48 2 48 48
y=5 B(15, 5)
æAö
P (K ) P ç ÷ A(0, 5)
æK ö èK ø
Now, Pç ÷ = X¢
O x+
X
èAø P(A) y=
1 20
×1
2 1 48 24
= = × = Y¢
29 2 29 29
48 at A( 0, 5), Z = 7 ´ 0 - 8 ´ 5 = - 40
Hence, option (d) is correct. at B (15, 5), Z = 7(15) - 8(5) = 65
148. (d) Since, p : 4 is an even prime number, at C( 0, 20), Z = 7( 0) - 8( 20) = - 160
q : 6 is a divisor of 12 and r : the HCF of 4 and 6 is 2. Hence, the minimum value of Z is attained at point (0, 20).
So, ~ p Ú (q Ù r ) is correct. So, option (d) is correct.
Hence, option (d) is correct.
Solved Paper 2019
BITSAT
Instructions
1. There are 150 questions in all. The number of questions in each part is as given below. No. of Questions
Part I Physics 1-40
Part II Chemistry 41-80
Part III a. English Proficiency 81-95
b. Logical Reasoning 96-105
Part IV Mathematics 106-150
2. All questions are multiple choice questions with four options, only one is correct.
3. Each correct answer fetches 3 marks while incorrect answer has a penalty of 1 mark.

PART I
Physics
1. A radioactive sample of half life 23.1 days is 5. An ideal monoatomic gas at 300 K expands
disintegrating continuously. The percentage decay of adiabatically to twice its volume. The final temperature
its in 15th to 16th days will be of gas is
2/ 3
[Take, e 0. 03 = 103
. ] æ 1ö
a. 300 2 b. 300 3 c. 300ç ÷ d. 300( 2)2/ 3
a. 5% b. 1% c. 2.9% d. 3.5% è 2ø
2. Two seperate soap bubbles of radii 3 ´ 10-3 m and 6. Two sound producing sources A and B are moving
2 ´ 10-3 m respectively, formed of same liquid towards and away from a stationary observer with
(surface tension 6.5 ´ 10-2 N/m) come together to form same speed respectively. If frequency of sound
a double bubble. The radius of interface of double produced by both sources are equal as 400 Hz, then
bubble is speed of sources (approximately) when observer
a. 6 ´ 10-3 m b. 4 ´ 10-3 m detects 4 beats per second, is
. ´ 10
c. 15 -3
m d. 0.66 ´ 10-3 m [Given, speed of sound = 340 m/s]
a. 1.7 m/s b. 3.4 m/s c. 2.4 m/s d. 1 m/s
3. An electron revolves in a circular orbit of radius r with
7. Pressure versus temperature graph of an ideal gas is
angular speed w. The magnetic field at the centre of
shown in the given figure. Density of gas at point A is
electron orbit is
m ew m ew r 0, then density of gas at point B will be
a. 0 b. 0 P
pr 4pr
m 0e 2w m 0ew B
c. d. 2P
4pr 2pr
4. A steel wire of cross-sectional area 4 cm 2 has elastic
limit of 2.2 ´ 108 N/m 2. The maximum upward P A
acceleration that can be given to a 1000 kg elevator
T
supported by this steel wire if the stress is to exceed T0 3T0
one-fourth of the elastic limit is [Take, g = 10 m/s 2] a. 2 r 0 b.
2
r0 c.
3
r0 d. 3 r 0
a. 10 m/s 2
b. 9 m/s 2
c. 15 m/s 2
d. 12 m/s 2 3 2
2 SELF STUDY GUIDE BITSAT

8. A rubber cord has a cross-sectional area 10-6 m 2 and 15. The magnetic field at the point O in the following
total unstretched length 0.1 m. It is stretched to current carrying square loop at the centre will be
0.125 m and then released to project a particle of 2A
mass 5.0 g. The velocity of projection is
[Given, Young’s modulus of rubber, Y = 5 ´ 108 N/m 2]
a. 45 m/s b. 30 m/s c. 25 m/s d. 15 m/s O
9. A body is moving unidirectionally under the influence
of a sources of constant power. The square of its
displacement in time t is proportional to
4 cm
a. t 3 b.t 2 c. t d. t
a. 12 ´ 10-5 T b. 4 ´ 10-5 T
10. Two electrons are moving with speed of 5 ´ 105 m/s c. 4 2 ´ 10 -5
T d. 2 2 ´ 10-5 T
parallel to each other, then the electrostatic and
magnetic force between them is 16. The value of current I as shown in the given circuit
a. 5 ´ 105 b. 2 . 5 ´ 104 diagram is
5 3 6W
c. 3 . 6 ´ 10 d. 4 . 4 ´ 10
10W B 15W 12W C
11. In the following circuit diagram, the current through
A D
battery is
D1 11W 20W

D2 I
7W 10V
a. 2 A b. 1.5 A c. 0.9 A d. 0.2 A
17. A semicircular wire of radius r rotates in uniform
D3 5W magnetic field B about its diameter with angular
velocity w. If the total resistance of the circuit is R, then
the mean power generated per period of rotation is
I
20 V 3W ( pBr 2w)2 pBrw2 ( pBrw)2
a. b. c. d. 0
8R 8R 8R
a. 4 A b. 3 A
c. 1 A d. 2 A 18. A convex lens of focal length 25 cm produces images
of the same magnification 2, when an object is kept at
12. A semicircular disc of mass M and radius R is free to two positions x1 and x 2 (x1 > x 2 ) from the lens. The ratio
rotate about its diameter. The moment of inertia of of x 2 and x1 is
semicircular disc about a line perpendicular to its
a. 2 : 1 b. 1 : 3 c. 1 : 2 d. 3 : 1
plane through centre is
3 MR 2 19. A particle X of mass m and initial velocity u collide with
a. MR 2 b. 3m
4 2 another particle Y of mass which is at rest, The
MR 2 MR 2 4
c. d. collision is head on and perfectly elastic. The ratio of
3 4
de-Broglie wavelengths lY and lX after the collision is
13. 12 eV energy is given to electron in third orbit of a. 4 : 3 b. 2 : 32 c. 3 : 4 d. 3 : 32
H-atom, then final energy of electron when it ionise
from third orbit, is m0
20. The dimensional formula of is
a. 5.25 eV b. 10.49 eV e0
c. 12 eV d. 11.15 eV a. [ML2T -3A2] b. [M0LT -1A0]
14. In Young’s double slit experiment, the distance c. [ML2T -3A-2] d. [M-1L-2T 3A2]
between slits and screen is 2 m and distance
between slits is 0.25 mm. A light of wavelength
21. A body of mass m is moving in a straight line with
momentum p. Starting at time t = 0, a force F = at acts
800 nm is used to find fringes on the screen. If
in the same direction on the moving particle during time
screen moves with a speed of 5 m/s, then first
interval of T. So that its momentum changes from p to
maxima will move with a speed of
2p. The value of T is
a. 16 mm/s b. 8 mm/s
2p p 2p 2p
c. 20 m/s d. 50 mm/s a. b. c. 2 d.
a a a a
SOLVED PAPER 2019 3
22. When a maximum force of 3 N is applied on a body 28. The escape velocity of the earth surface is v e . A body
kept on rough inclined plane of shown in the figure, is projected with velocity 3v e . With what constant
then body remains stationary. The maximum external velocity will it move in the inter planetary space?
force up the inclined plane that does not move the a. ve b. 3ve c. 2 ve d. 2 2 ve
block is 12 N. The coefficient of static friction between
the block and the plane is [Take, g = 10 m/s 2] 29. The time period of a bob performing simple harmonic
4
12 N ´ 103 kg/m 3 ,
motion in water is 2s. If density of bob is
3
then time period of bob performing simple harmonic
motion in air will be
3N a. 3s b. 4s c. 2s d. 1s
30º
30. A uniform solid cylindrical roller of mass m is being
3 1 3 3 5 pulled on horizontal surface with force F parallel to the
a. b. c. d.
5 3 5 3 3 surface applied at its centre. If the acceleration of the
cylinder is a and it is rolling without slipping, then the
23. In the shown situation, if middle portion of the lens is value of F is
painted black, then 3 ma 5 ma
a. 2 ma b. 3 ma c. d.
Object Lens 2 2
31. A projectile is given an initial velocity of ( $i + 3 $j) m/s,
2F F where $i is along the ground and $j is along the vertical.
Then, the equation of the path of projectile is
[Take, g = 10 m/s 2]
a. complete image of the object will form with low intensity
b. complete image of the object will form with high intensity a. y = 3 x - 5x 2 b. y = 3 x + 5x 2
c. incomplete image of the object will form with low c. x = 3 y + 5x 2 d. x 2 = y 2 + 3
intensity
d. incomplete image of the object will form with high 32. The force on a body of mass 1 kg is ( 20$i + 10$j) N. If is
intensity starts from rest, then the position of the body at time
t = 2 s, is
24. If sound travels in air with the speed of 340 m/s, then a. - 20$i - 40$j b. 20$i - 40$j
number of tones present in an open organ pipe of
length 2 m and its maximum frequency 1200 Hz, are c. 40$i - 20$j d. 40$i + 20$j
a. 17 b. 11 c. 9 d. 14
33. A cylinder of mass 2 kg is released from rest from the
25. A proton is revolving on a circular path of radius 2 mm top of an inclined plane of inclination 30° and length
with frequency 10 Hz. Magnetic dipole moment 1m. If the cylinder rolls without slipping, then its speed
associated with proton is when it reaches the bottom, is [Take, g = 10 m/s 2]
a. 2 ´ 10-24 A-m 2 b. 4 ´ 10-24 A-m 2 20 20
a. m/s b. m/s
c. 3 ´ 10-20 A-m 2 d. 6 ´ 10-20A-m 2 3 3
10 10
26. When a capacitor is fully charged as shown in the c. m/s d. m/s
3 3
following figure, then current drawn from the cell is
7 kW 8 kW 34. Frequency of oscillation of a body is 5 Hz when a
force F1 is applied and 12 Hz when another force F2 is
applied. If both forces F1 and F2 are applied together,
4 kW then frequency of oscillation of the body will be
9 kW C
a. 13 Hz b. 169 Hz
10 V c. 62 Hz d. 52 Hz
pV
a. 1 mA b. 0.5 mA c. 2 mA d. 5 mA 35. The figure shows the graph of versus p for
T
27. A charged capacitor of capacitance C is discharging 2 ´ 10-4 kg of hydrogen gas at two different
through a resistor of resistance R. temperatures, where p, V and T represents pressure,
At what time the charge on the capacitor is equal to volume and temperature respectively.
one half of its initial value?
RC RC 1 pV
a. b. c. RC loge 2 d. loge 2 Then, the value of , where the curve meet on the
2 loge 2 RC T
vertical axis, is
4 SELF STUDY GUIDE BITSAT

T2
T1
38. A carbon resistor of (56 ± 5.6 ) kW is to be marked with
pV/T(JK–1)
rings of different colours for its identification. The colour
code sequence will be
a. green, blue, orange, silver
b. blue, yellow, orange, silver
c. green, red, orange, silver
P
d. red, green, blue, gold
R 2R R R
a. b. c. d.
5 3 10 15 39. Two monochromatic light waves are travelling with
same frequency and constant phase different. If both
36. An ammeter of resistance 1.5 W can measure waves interfere, then
currents upto 1 A. The value of shunt resistance to
a. there is a gain of energy
measure current upto 4 A is
b. there is a loss of energy
a. 0.5 W b. 1 W c. 1.5 W d. 0.2 W
c. the energy is redistributed and the distribution remains
37. In the following figures, which represents the constant in time
variation of the electric field with distance from the d. the energy is redistributed and the distribution changes
centre of a uniformly charged non-conducting spheres with time
of radius R? 40. In a coil of resistance 50 W, the induced current
developed by changing magnetic flux through it, is
shown in figure as a function of time. The magnitude of
E E
1/r 2 1/r 2 change in flux through the coil is
a. b.

I(A)
r r 8P

E E
1/r 2 1/r 2 1/r 2 S
c. d. O 0.2 t (s)

a. 10 Wb b. 20 Wb
r R r c. 32 Wb d. 40 Wb

PART II
Chemistry
41. In a compound, atoms of elements Y form ccp lattice 45. The heat of reaction for
and those of element X occupy 2/3 rd of tetrahedral C18H8(s )12 O2 ( g ) ¾® 10CO2( g ) + 4 H2O(l ) at constant
voids. The formula of the compound can be volume is - 1228.2 kcal at 25°C. The heat of reaction at
a. X 4Y3 b. X 2Y3 c. X 2Y d. X 3Y4 constant pressure and same temperature is
a. - 1228.2 kcal b. - 1229.3 kcal
42. The density of a certain gas at 30°C and 768 torr is c. - 1232.9 kcal d. -1242.6 kcal
1.35 kg/m 3 , then density of the gas at STP is
a. 1.48 kg/m 3 b. 1.27 kg/m 3 46. Given the hypothetical reaction mechanism
c. 1.35 kg/m 3 d. 1.00 kg/m 3
A ¾¾I ® B ¾¾
II III
® C ¾¾ ® D ¾ IV
¾® E and the rate as
43. For the octahedral complex, which of the following
d-electron configuration will give maximum value of Species formed Rate of its formation
CFSE B 0.002 mol/h per mole of A
a. high spin, d 6 b. low spin, d 4 C 0.030 mol/h per mole of B
c. low spin, d 5 d. high spin, d 7 D 0.011 mol/h per mole of C
44. The wavelength corresponding to maximum energy E 0.420 mol/h per mole of D
for hydrogen is 91.2 nm. The corresponding
wavelength for He+ ion is The rate determining step is
a. 2.28 nm b. 22.8 nm c. 182.4 nm d. 364.8 nm a. step I b. step II c. step III d. step IV
SOLVED PAPER 2019 5
47. For the reaction C6H5. NH2 ¾ H¾
2SO4
¾
¾® H2N. C6H4(SO3H) 56. Thermosetting polymer bakelite is formed by the
180 °C (Para)
reaction of phenol with
the true statement about the product is a. CH3. CH2. CHO b. CH3. CHO
a. it does not exist as Zwitter ion c. H. CHO d. HCOOH
b. ¾ NH2 group displays a powerful basic character
57. The drug that is antipyretic as well as analgesic is
c. it does not act as inner salt a. chloroquine
d. ¾ SO3H, diminishes the basic character of ¾ NH2 b. penicillin
c. paracetamol
48. The hybridisation of phosphorous in PO34 - is
d. chloropromazine hydrochloride
a. sp b. sp 2 c. sp 3 d. sp 3d
58. The correct statement about the following
49. Which of the following lanthanoid is diamagnetic? disaccharide is–
Atomic no’s CH2OH
Sm = 62, Ce = 58, Eu = 63, Yb = 70
H O H H2COH O
H
a. Sm2+ b. Eu2+ c. Yb2+ d. Ce2+
H
OH H H OH
50. Which of the following is/are aromatic alcohol?
OH OCH2CH2O CH2OH
OH CH2 OH CH2 OH OH
H OH OH H
(i) (ii)

(I) (II) (III)


CH3 (IV) a. Ring (I) is pyranose with a-glycosidic link
CH3 b. Ring (I) is furanose with a-glycosidic link
a. I,II,III,IV b. II and III c. I and IV d. Only I c. Ring (II) is furanose with a-glycosidic link
d. Ring (II) is pyranose with a-glycosidic link
51. Among the following substituted silanes, the one
which will give rise to cross linked silicone polymer on 59. If one strand of DNA has sequence ATCGTATG, the
hydrolysis is? sequence in the complementary strand would be
a. R4 Si b. R SiCl3 a. TAGCTTAC b. TCACATAC
c. R2SiCl d. R3 SiCl c. TAGCATAC d. TACGATAC
60. Which of the following salt would give SO2 with hot
52. For the chemical reaction
and dil H2SO4 and also decolourises Br2 water?
2O3 ¾® 3O2, the reaction proceeds as follows
a. Na 2SO3 b. NaHSO4 c. Na 2SO4 d. Na 2S
O3 c O2 + O (Fast)
61. In the reaction (X ) and (Y ) are respectively.
O + O3 ¾® 2O2, (Slow)
CH3
CCl3
the rate law expression should be given as Cl2 CHO
a. r = k [ O3] 2[ O2] -1 b. r = k [ O3] 2 X
FeCl3
Y
H2SO4
Cl CH Cl
c. r = k [ O3] [ O2] d. unpridictable
Cl CHO Cl
53. An aqueous solution freezes at 272.4 K, while pure
. K kg mol -1, and
water freezes at 273 K, given K f = 186 a.
,
b.
,
K b = 0512
. - k kg mol -1; the molality of solution and
Cl Cl
boiling point of solution respectively will be– Cl
a. 0.322 and 373.16 K b. 0.222 and 273.15 K Cl
c. d.
c. 0.413 and 400 K d. 0.5 and 300.73 K , , CHO

54. The order of acidic strength of boron trihalides


62. Which of the following reactions is an example of use
a. BF3 < BCl3 < BBr3 < BI3 b. BI3 < BBr3 < BCl3 < BF3 of water gas in the synthesis of other compounds?
c. BCl3 < BBr3 < BI3 < BF3 d. BBr3 < BCl3 < BF3 < BI3 1270 K
a. CH4(g ) + H2O (g ) ¾ ¾ ¾
¾® CO (g ) + H2(g )
Ni
673 K
55. The correct order of mobility of alkali metal ions in b. CO(g ) + H2O (g ) ¾ ¾¾® CO2 (g ) + H2(g )
Catalyst
aqueous solution is—
1270 K
+ +
a. Li > Na > K > Rb + + + +
b. Na > K > Rb > Li + + c. CnH2n + 2 + n . H2O (g ) ¾ ¾ ¾
¾® n . CO + ( 2n + 1) H2
Ni
c. K + > Rb+ > Na + > Li+ d. Rb+ > K + > Na + > Li+ d. CO (g ) + 2H2 (g ) ¾ Cobalt
¾¾ ¾® CH3 × OH (l )
Catalyst
6 SELF STUDY GUIDE BITSAT

63. Which of the following compounds, on reaction with 71. Spin only magnetic moment of Mn4 + ion is nearly
NaOH and Na 2 O2 gives yellow colour? a. 3 BM b. 6 BM
a. Zn(OH)2 b. Al(OH)3 c. Cr(OH)3 d. CaCO3 c. 4 BM d. 5 BM

64. For the reaction– 72. 0.45 g acid of molecular mass 90 was neutralise by 20
ml of 0.5 N KOH. The basicity of the acid is
O O
– H 2O – a. 2 b. 4 c. 1 d. 3
R—C +NCl R—C +Z
Z NCl 73. 4 moles of A are mixed with 4 moles of B. At
equilibrium for the reaction A + B c C + D , 2 moles
rate of reaction is faster, when Z is–
a. Cl b. NH2 c. OC2H5 d. OCOCH3 of C and D are formed, the equilibrium constant for the
reaction will be,
65. E 1, E 2 and E 3 are the emf of the following three a. 1 b. 1/2
galvanic cells respectively c. 4 d. 1/4

(i) Zn(s )| Zn2+ ( 01


. M )| | Cu2+ (1M )| Cu(s ) 74. What time in sec is required for depositing all the silver
present in 125 ml of 1 M AgNO3 solution by passing a
(ii) Zn(s )| Zn2+ (1M )| | Cu2+ (1M )| Cu(s ) current of 241.25 A (Given IF= 96500 C)
(iii) Zn(s )| Zn2+ (1M )| | Cu2+ ( 0.1M )| Cu(s ) a. 10 b. 50
c. 100 d. 1000
Which of the following is true?
a. E2 > E1 > E3 b. E1 > E2 > E3 75. In a homonuclear molecule, which of the following set
of orbitals is degenerated?
c. E3 > E1 > E2 d. E3 > E2 > E1
a. s1s and s 2s b. p 2p x and p 2p y
* *
66. Sodium nitroprusside, when added to an alkaline c. p 2p x and s 2p z d. s2p x and s* 2p z
solution of sulphide ions, produce a
a. red colouration b. blue colouration
76. If the photon of wavelength 150 pm striks an atom and
c. purple colouration d. brown colouration
one of its inner bond electron is ejected out with a
velocity of 1. 5 ´ 107 m/s, what is the energy with which
67. For a reverse reaction, A c
B , which one of the it is bond to the nucleus?
following statement is wrong from the given energy. a. 1. 2 ´ 102 eV b. 2.15 ´ 103 eV
Profile diagram? c. 7.6 ´ 103 eV d. 8.12 ´ 103 eV
a. Activation energy of forward reaction is greater than 77. Identify compound (C ) in the following reaction
backward reaction
/D
b. The forward reaction is endothermic C7H8 ¾ 3Cl
¾2¾ ¾ ®( A ) ¾ Br
¾2 /¾
Fe
¾® (B ) ¾ Zn/
¾ HCl
¾
¾® (C )
c. The threshold energy is less than that of activation a. o-bromotoluene b. m-bromotoluene
energy c. p-bromotoluene
d. The energy of activation of forward reaction is equal
d. 3-bromo, 2, 4, 6 trichlorotoluene
to the sum of heat of reaction and the energy of
activation of backward reaction 78. In a compound C, H and N are present in 9 : 1 : 3.5 by
weight. If molecular weight of compound is 108, the
68. Which of the following reactants is used for the molecular formula of compound is
preparation of ethyl benzene, where anhyd. AlCl3 is a a. C2H6N2 b. C3H4N
catalyst?
c. C6H8N2 d. C9H12N3
a. CH3. CH2. OH + C6H6 b. CH3 ¾ CH== CH2 + C6H6
c. H2C == CH2 + C6H6 d. CH3 ¾ CH3 + C6H6 79. Which of the following compounds contain all the
carbon atoms in the same hybridisation state
69. Given pH of a solution ‘A’ is ‘3’ and it is mixed with a. H¾ C ºº C ¾ C ºº C ¾ H b. CH3 ¾ C ºº C ¾ CH3
another solution ‘B’ having pH ‘2’. If both solutions
c. CH2 == C == CH2 d. CH2 == CH¾ C ºº CH
are mixed, the resultant pH of the solution will be
a. 3.2 b. 1.9 c. 3.4 d. 3.5 80. The final step for extraction of copper from copper
70. In Freundlich adsorption isotherm, the value of 1/ n is pyrite in bessemer converter involves the reaction
a. between 0 and 1 in all cases a. Cu2S + 2Cu2O ¾® 6Cu + SO2
b. between 2 and 4 in all cases b. 4Cu2O + FeS ¾® 8Cu + FeSO4
c. always 1 in case of physical adsorption c. 2Cu2O + FeS ¾® 4Cu + Fe + SO2
d. always 1 in use of chemical adsorption d. Cu2S + 2FeO ¾® 2Cu + Fe + SO2
Part III
a. English Proficiency
Directions (Q. Nos 81-83) In the following questions, the For India, already struggling to cope with a drought, the
sentences may or may not be grammatically correct. Find out implications of this are devastating and further crop failure
which part of a sentence has an error and mark that part. If is will amount to a life and death question for many Indians. The
no error mark part ‘d’ as your answer. increase in premature deaths will have adverse social and
economic consequences and a rise in morbidities will place
81. Many a man a. / have come to India from Bangladesh
an unbearable burden on our crumbling health system. And
b. / to live here permanently. c./ No error d. there is no one to blame but ourselves.
82. How long it takes a./ to travel from Chennai to Trichy Both official and corporate India has always been allergic to
b. / by train? c./ No error d. any mention of clean technology. Most mechanical two
83. One should respect a./ the religions of others b./ as wheelers roll off the assembly line without proper pollution
much as his own. c./ No error d. control system.

Directions (Q. Nos. 84 and 85) Fill in the blanks with suitable Little effort is made for R & D on simple technologies,
preposition from the alternatives given under each sentence. which could make a difference to people’s lives and the
environment. However, while there is no denying that
84. The master assured her ………… success in the South Asia must clean up its act, skeptics might question
examination. the timing of the haze report.
a. of b. in c. for d. with
91. Both official and corporate India is allergic to
85. Why do we always have to submit ……… his a. failure of monsoon
authority? b. poverty and inequality
a. under b. with c. for d. to c. slowdown in industrial production
Directions (Q. Nos. 86-88) Select the word or the phrase d. mention of clean technology
that is closest to the opposite in meaning of the italicised or
92. Which, according to the passage, is a life and death
phrase. question to many Indians?
86. The officer made an audacious remark. a. Increase in respiratory diseases
a. Mute b. Visible c. Cultivated d. Timid b. Use of clean technology
c. Thick blanket of pollution over the region
87. They put forward cogent arguments for their
d. Failure in crops
membership.
a. Logical b. Weighty c. Dissuasive d. Persuasive 93. Choose the word which is similar in meaning to the
word ‘profligacy’ as used in the passage.
88. His administration was tainted by scandal
a. wastefulness b. conservation
a. Construct b. Clear c. Purify d. Repair
c. upliftment d. criticalness
Directions (Q. Nos. 89 and 90) Choose the word nearest
meaning to the underlined word. 94. According to the passage, India cannot tolerate any
further
89. Her mother is vivacious and elegant. a. crop failure
a. Imaginary b. Lively c. Perceptible d. Languid b. deterioration of healthcare system
c. increase in respiratory diseases
90. Sporadic fighting broke out in the office.
d. proliferation of nuclear devices
a. Timely b. Scattered c. Frequent d. Irrelevant
Directions (Q. 91-95) Read the passage given below and 95. What could be the reason behind timing of the haze
answer the questions that follow. report just before the Johannesburg meet, as
indicated in the passage?
Radically changing monsoon patterns, reduction in the a. United Nations is working hand-in-glove with US
winter rice harvest and a quantum increase in respiratory b. Organises the forthcoming meet to teach a lesson to
diseases-all part of the environmental doomsday scenario the US
which is reportedly playing out in South Asia. According to a c. Drawing attentions of the world towards devastating
United Nations Environment Programme report, a deadly effects of environment degradation
3-km deep blanket of pollution comprising a fearsome d. US wants to use it as a handle against the developing
cocktail of ash, acids, aerosols and other particles has countries in the forthcoming meet
enveloped this region.
8 SELF STUDY GUIDE BITSAT

b. Logical Reasoning
96. College’ is related to ‘Student’ in the same way as Problem Figures
‘Hospital’ is related to
a. Doctor b. Nurse c. Medicine d. Patient
97. Choose the odd one.
a. 27 b. 35 X Y Z
c. 18 d. 9
Answer Figures
98. Complete the series by replacing questions mark “?”
1, 2, 2, 4, 3, 8, 7, 10, ?
a. 9 b. 8
c. 11 d. 13
(a) (b) (c) (d)
99. Mohan is taller than Shyam but shorter than Ramesh.
Ramesh is taller than Rajat but shorter than Gautam. 103. Which of the answer figure (a), (b), (c) or (d) completes
If Shyam is taller than Rajat, then who is the shortest the figures matrix?
among all?
Problem Figure Answer Figures
a. Gautam b. Rajat
c. Shyam d. Ramesh
100. In the following question, complete the missing + + + +
segment by selecting the appropriate figure from the + + + (a) (b) (c) (d)
given alternatives, a., b., c. and d.. + +

Problem Figure Answer Figures ?

104. In the following question, some dots are placed in the


? figure marked as (A). The figure is followed by four
(a) (b) (c) (d) alternatives marked as (a), (b), (c) and (d). One out of
these four options contains region(s) common to the
101. In the question given below, find out which of the circle, square, triangle, similar to that marked by the
figures can be formed from the pieces given in the dot in figure (A). Choose the correct option.
problem figure.
Question Figure Answer Figures
Problem Figure Answer Figures

(a) (b) (c) (d) (A) (a) (b) (c) (d)

102. In the following question, a set of three figures (X), 105. How many triangles are there in the given figure?
(Y) and (Z) have been given, showing a sequence in
which paper is folded and finally cut from a particular
section. These figures are followed by a set of
answer figures marked a., b., c. and d. showing the
design which the paper actually acquires when it is
a. 28 b. 24
unfolded. You need to select the answer figure which
c. 25 d. 26
is closest to the unfolded piece of paper.
Part IV
Mathematics
x2 y2 114. The equation of plane containing line x - y = 1, z = 1
106. If the length of the major axis of the ellipse 2
+ =1
a b2 and parallel to
x z
– = 1, y = 3 is
is three times the length of minor axis, its 2 3
eccentricity is a. 3x + 3y - 2z = 1 b. 3x - 3y - 2z = 1
1 1 2 2 2 c. 3x + 3y + 2z = 1 d. 3x + 3y + 2z = - 1
a. b. c. d.
3 3 3 3 40 39
n æ ö
115. If (1 + x + x 2 )20 = S ar × x r , then S ( - 1)r × ar × ar +1
2m r =0 r =0
107. S tan-1çç 4 2
÷ is equal to
÷
m =1
èm + m + 2ø equal to
a. 79 b. 239 × 78
C39 c. 339 × 78
C39 d. 0
-1 æ
n +n ö -1 æ
n -n ö
2 2
a. tan ç 2ç ÷ b. tan ç 2 ç ÷
÷ ÷ 4 sin2 x cos x - cos x + sin x
èn + n + 2ø èn - n + 2ø 116. The value of lim is
æn2 + n + 2ö x ®
3p sin x + cos x
c. tan-1çç 2
÷
÷ d. None of these 4
è n +n ø equal to
20 a8 a. - 1 b. 0 c. 1 d. None of these
108. Let ( 2x 2 + 3x + 4 )10 = S ar x r , then the value of , is
r =0 a12 117. If a, b, c are non-coplaner vectors such that b ´ c = a;
c ´ a = b; a ´ b = c, then which of the following is not TRUE?
a. 2 b. 4 c. 8 d. 16
a. | a| - | b| = 0 b. | a| = | b | = | c| = 2
109. Let ABC be an acute-angled triangle and let D be the c. [ a b c] = 1 d. | a| | b| | c| = 1
tan B
mid-point of BC. If AB = AD , then equal
tan C p
118. The value of ‘l’ for which the loci arg z = and
a. 2 b. 3 6
c. 2 d. 3 | z - 2 3 i | = l on the argand plane touch each other is
1 1 a. 3 b. 4 c. 5 d. 6
110. Suppose the limit L = lim
n ®¥
n ò0 (1 + x 2 )n dx exists and 119. The angle between the lines whose direction cosine
1 satisfy the equations l + m + n = 0 and l 2 = m 2 + n 2 is
is larger than , then
2 p p
a. b.
1 3 4
a. < L < 2 b. 2 < L < 3 c. 3 < L < 4 d. L ³ 4
2 p p
c. d.
6 2
111. In the real number system, the equation
x + 3 - 4 x - 1 + x + 8 - 6 x - 1 = 1has 120. The locus of the foot of perpendicular drawn from
the centre of the ellipse x 2 + 3 y 2 = 6 on any tangent
a. No solution
b. Exactly two distinct solution to it is
c. Exactly four distinct solution a. (x 2 - y 2 )2 = 6x 2 + 2y 2 b. (x 2 - y 2 )2 = 6x 2 - 2y 2
d. Infinitely many solution c. (x 2 + y 2 )2 = 6x 2 + 2y 2 d. (x 2 + y 2 )2 = 6x 2 - 2y 2

112. If system of equation ax + y + z = a , x + by + z = b 121. If f and g are differentiable function in [0, 1] satisfying
and x + y + cz = c is inconsistent, then which of the f ( 0) = 2 = g (1), g( 0) = 0 and f (1) = 6, then for some
following is correct? c Î( 0, 1)
a. abc - a - b - c + 2 = 0 a. 2f ¢ (c ) = g(c ) b. 2f ¢ (c ) = 3g ¢ (c )
b. abc - a - b - c + 3 = 0, a = 1 c. f ¢ (c ) = g ¢ (c ) d. f ¢ (c ) = 2g ¢ (c )
c. abc - a - b - c + 3 = 0 1
æ 1ö x + x
d. abc - a - b - c + 2 = 0,a ¹ 1, b ¹ 1, c ¹ 1 122. The integral ò ç 1 + x – ÷e dx is equal to
è xø
113. A die is thrown 7 times. What is the probability that an x +
1
x +
1

even number turns up at least 4 times? a. (x - 1) e x +C b. x e x +C


1 31 51 35 x +
1
x +
1
a. b. c. d.
2 64 128 128 c. (x + 1) e x +C d. - xe x +C
10 SELF STUDY GUIDE BITSAT

123. If the coefficient of x 3 and x 4 in the expansion of 132. The least value of the function
(1 + ax + bx 2 )(1 - 2x )18 in powers of x are both zero, x é5 p 4 p ù
then (a , b ) is equal to f (x ) = ò ( 3 sin x + 4 cos x )dx on the interval ê , is
0
ë 4 3 úû
æ 251ö æ 251ö æ 272 ö æ 272 ö
a. ç16, ÷ b. ç14, ÷ c. ç14, ÷ d. ç16, ÷ 3 3 5-4 3 7-4 3 9-4 3
è 3 ø è 3 ø è 3 ø è 3 ø a. - b. c. d.
2 2 2 2 2
124. An equation of plane parallel to plane
x - 2y + 2z - 5 = 0 and at a unit distance from the 133. If z1and z1 represents adjacent vertices of a regular
origin is Im (z1)
polygon of n sides and if = 2 - 1, then n is
a. x - 2y + 2z = 3 b. x - 2y + 2z + 1 = 0 Re (z1)
c. x - 2y + 2z - 1 = 0 d. x - 2y + 2z + 5 = 0 equal to
a. 8 b. 16 c. 18 d. 24
125. Three numbers are chosen at random without
replacement from {1, 2, 3, ……, 8}. The probability that 134. In the expansion of (1 + x + x 3 +x 4 )10, the coefficient of
their minimum is 3, given that their maximum is 6, is x 4 is
3 1 1 2 a. 235 b. 310 c. 285 d. 325
a. b. c. d.
8 5 4 5
135. A person writes letter to six friends and addresses the
126. The number of real numbers l for which the equality corresponding envelopes. Let x be the numbers of
sin ( la ) cos ( la ) ways so that at least two of the letters are in wrong
- = l - 1, holds for all real a which
sin a cos a envelopes and y be the numbers of ways so that all the
are not integral multiple of p / 2 is letters are in wrong envelopes. Then x - y =
a. 1 b. 2 a. 719 b. 265 c. 454 d. 720
c. 3 d. Infinite 136. If x = log5 3 + log7 5 + log9 7, then
127. Suppose a parabola y = ax 2 + bx + c has two x a. x >
3
b. x >
1
c. x >
3
d. x > 3 2
3 3
intercepts, one positive and one negative, and its 2 2 2
vertex is ( 2, - 2), then which of the following is true?
137. Let p and q be roots of the equation x 2 - 2x + A = 0 and
a. ab > 0 b. bc > 0
let r and s be the roots of the equation x 2 - 18x + B = 0,
c. ac > 0 d. a + b + c > 0
If p < q < r < s are in AP, then A and B are
128. The larger of two angles made with the X-axis of a a. - 3, - 77 b. 3, - 77 c. - 3, 77 d. 3, 77
straight line drawn through (1, 2) so that it intersects
the line x + y = 4 at a paint distant 6 / 3 from the 138. Let a , b , c Î R + and the system of equations
point (1, 2) is (1 - a ) x + y + z = 0, x + (1 - b )y + z = 0 and
a. 60° b. 75° x + y + (1 - c )z = 0 has infinitely many solutions, the
minimum value of ‘abc ’ is
c. 105° d. None of these
a. 3 3 b. 9 c. 27 d. 3
129. The point ([P + 1], [P ]) (where, [x ] is the greatest
139. If A is an 3 ´ 3 non- singular matrix, such that
integer function) lying inside the region bounded by
the circle x 2 + y 2 - 2x - 15 = 0 and AA¢ = A¢ A and B = A –1A¢, then BB¢ equals to
x 2 + y 2 - 2x - 7 = 0, then a. (B -1)¢ b. I + B c. I d. B -1
a. P Î [ - 1, 2) - { 0,1} b. P Î [ - 1, 0) È ( 0, 1) È (1, 2] 140. If sin-1 x + tan-1 x = p / 2,then 2x 2 + 1 =
c. P Î ( - 1, 2) d. None of these 5 -1 5+1
a. 5 b. c. 2 d.
dy 2 2
130. Solution of the equation = e x - y (e x - e y ) is
dx 141. Number of solution of the equation | cos x | = 2 [x ] are
x x
-x
a. e y = e x - 1 + ce –e b. e y = - 1 + ce –e (where | x |, [x ] are modulus and greatest integer
c. e x + e y = ce –e
x
d. None of these function respectively).
a. 0 b. 2 c. 1 d. infinitely many
131. Area bounded by two branches of the curve 4 5
( y – x )2 = x 3 and x = 1is 142. Let cos ( a + b) = and let sin( a – b) = , where
5 13
5 3 p
a. sq unit b. sq unit 0 £ a, b £ , then tan 2a =
4 2 4
1 4 20 25 56 19
c. sq unit d. sq unit a. b. c. d.
4 5 7 16 33 2
SOLVED PAPER 2019 11
1
143. Cards are drawn one by one at random from a well
shuffled pack of 52 cards until 2 aces are obtained for
147. If f ¢ (x ) = f (x ) + ò0 f (x )dx , f ( 0) = 1, thenf (x ) =
the first time. The probability that 18 draws are 2e x 1- e ex 1+ e
a. + b. +
required for this is 3 -e 3 -e 3 - e 1- e
a. 3/34 b. 17/455 3e x 1+ e 3e x 1- e
561 c. + d. +
c. d. None of these 2 - e 1- e 2 -e 3 + e
15925
148. The distance between the origin and the normal to
144. If a = $i + 2$j + 3k$ , b = - i$ + 2$j + k$ , c = 3 i$ + $j and a + pb
curve y = e 2x + x 2 at x = 0 is
is normal to c, then p is equal to 2 2 1 1
a. 0 b. 1 a. b. c. d.
3 5 3 5
c. 5 d. 3
x -1 y - 2 z - 4
145. Let f (x ) = x [x ], x Ï Z , ([×] denotes greatest integer 149. Reflection of the line = = in the plane
-1 3 1
function), then f ¢ (x ) is equal to x + y + z = 7 is
a. 2x b. [x ] x -1 y -2 z -4 x -1 y - 2 z - 4
a. = = b. = =
c. 2[x ] d. 1 3 1 1 -3 -1 1
x -1 y -2 z -4 x -1 y - 2 z - 4
146. The difference of maximum and minimum values of c. = = d. = =
f (x ) = x 2e - x is -3 1 –1 3 1 1

a. e b. 1/ e 150. If x , y , z Î R , x + y + z = 5, x 2 + y 2 + z 2 = 9, then
1 1
c. 1 - d. 1 + length of interval in which x lies is
e e a. 8/3 b. 4/3 c. 2/3 d. 1/3

Answers
Physics
1. (c) 2. (a) 3. (b) 4. (d) 5. (c) 6. (a) 7. (b) 8. (c) 9. (a) 10. (c)
11. (d) 12. (b) 13. (b) 14. (a) 15. (c) 16. (c) 17. (a) 18. (b) 19. (d) 20. (c)
21. (a) 22. (d) 23. (a) 24. (d) 25. (a) 26. (b) 27. (c) 28. (d) 29. (d) 30. (c)
31. (a) 32. (d) 33. (b) 34. (a) 35. (c) 36. (a) 37. (c) 38. (a) 39. (c) 40. (d)

Chemistry
41. (a) 42. (a) 43. (c) 44. (b) 45. (b) 46. (a) 47. (d) 48. (c) 49. (c) 50. (b)
51. (b) 52. (a) 53. (a) 54. (a) 55. (d) 56. (c) 57. (c) 58. (a) 59. (c) 60. (a)
61. (b) 62. (d) 63. (c) 64. (a) 65. (b) 66. (c) 67. (c) 68. (c) 69. (b) 70. (a)
71. (c) 72. (a) 73. (a) 74. (b) 75. (b) 76. (c) 77. (b) 78. (c) 79. (a) 80. (a)

English Proficiency
81. (b) 82. (a) 83. (c) 84. (a) 85. (d) 86. (d) 87. (c) 88. (b) 89. (b) 90. (b)
91. (d) 92. (d) 93. (a) 94. (a) 95. (d)

Logical Reasoning
96. (d) 97. (b) 98. (c) 99. (b) 100. (b) 101. (b) 102. (d) 103. (a) 104. (b) 105. (a)

Mathematics
106. (d) 107. (a) 108. (b) 109. (d) 110. (a) 111. (d) 112. (d) 113. (a) 114. (b) 115. (d)
116. (a) 117. (b) 118. (a) 119. (a) 120. (c) 121. (d) 122. (b) 123. (d) 124. (a) 125. (b)
126. (b) 127. (b) 128. (b) 129. (d) 130. (a) 131. (d) 132. (d) 133. (a) 134. (b) 135. (c)
136. (c) 137. (c) 138. (c) 139. (c) 140. (a) 141. (a) 142. (c) 143. (c) 144. (c) 145. (b)
146. (b) 147. (a) 148. (b) 149. (c) 150. (b)
Solutions
Physics
1. (c) Given, half life (T1/ 2 ) = 23.1 days where, w = angular velocity of electron
If N0 be the initial amount, then active nuclei Current produced due to movement of electron,
at t = 15 days e e ew
I= = =
T 2p 2p
N1 = N0 e -15l … (i)
w
Active nuclei at t = 16 days On substituting the value of I in Eq. (i), we get
N2 = N0 e -16l …(ii) m ew
B= 0.
Disintegration constant, 2r 2p
0.693 0.693 m ew
l= = = 0.03 Þ B= 0
T1/ 2 23.1 4pr

\% decay in 15th to 16th days 4. (d) Given, Y = 2.2 ´ 108 m/s 2


- 15 l -16 l
N1 - N2 N e - N0 e Area of cross-section of steel wire,
= ´ 100 = 0 ´ 100
N1 N0 e -15l A = 4 cm2 = 4 ´ 10-4m2
= (1 - e - l ) ´ 100 = (1 - e -0. 03 ) ´ 100 Mass of elevator, m = 1000 kg
æ 1 ö æ 1 ö The maximum tension in the steel wire that can support
= ç1 - 0. 03 ÷ ´ 100 = ç1 - ÷ ´ 100 = 2.9%
è e ø è . ø
103 elevator is given by
1
2. (a) Excess pressure inside first bubble of radius r1, T = ´ stress ´ area of cross - section
4
4T
p1 = p 0 + 1
= ´ 2.2 ´ 108 ´ 4 ´ 10-4
r1 4
Excess pressure inside second bubble of radius r2, = 2.2 ´ 104 N/m 2
4T
p2 = p0 + If f be the maximum upward acceleration of the elevator,
r2
then
Excess pressure inside double bubble, Þ T = m(g + f )
p = p 2 - p1 2.2 ´ 104 = 1000 (10 + f )
ær - r ö
p = 4T çç 1 2 ÷÷ Þ f = 12 m/s 2
è r1r2 ø
5. (c) For an ideal monoatomic gas, the heat capacity
If R be the radius of double bubble, then
Ti , Tf = initial and final temperatures of gas
4T
p= 5
R ratio, g =
3
æ r - r ö 4T
or 4T çç 1 2 ÷÷ = In an adiabatic process,
è r1r2 ø R
TV g - 1 = constant
rr 3 ´ 10-3 ´ 2 ´ 10-3
Þ R = 12 = = 6 ´ 10-3 m g -1 5
-1
r1 - r2 ( 3 - 2) ´ 10-3 g- 1 g- 1 æV ö æ 1ö 3
i.e. Tf Vf = TV
i i Þ Tf = Ti çç i ÷÷ = 300 ç ÷
èVf ø è 2ø
R = 6 ´ 10-3 m
2/ 3
æ 1ö
3. (b) Magnetic field at the centre of circular loop, = 300 ç ÷
è 2ø
m I
B= 0 … (i)
2r 6. (a) Given, frequency of sound produced by sources A
and B, fA = fB = 400 Hz
Period of revolution of electron, where w = angular
velocity of electron Speed of sound in air, v = 340 m/s
2p Apparent frequency heard by the observer when source
T =
w A is moving towards him,
SOLVED PAPER 2019 13
v 340 9. (a) By work-energy theorem,
fA ¢ = ´ fA = ´ 400
v - vS 340 - vS 1
W = P ´ t = mv 2
Apparent frequency heard by the observer when sourceB 2
is moving away from him, Pt
Þ v2 = 2
v 340 m
fB ¢ = ´ fB = ´ 400 1/ 2
v + vS 340 + vS æ 2Pt ö
v =ç ÷
Given, Beats = fA ¢ - fB ¢ = 4 è m ø
1/ 2
340 340 ds æ 2Pt ö
´ 400 - ´ 400 = 4 or =ç ÷
340 - vS 340 + vS dt è m ø
34000 ´ 2vS æ 2Pt ö
1/ 2
Þ =1
3402 - vS2 ò ds = ò çè m ÷ø dt
34000 ´ 2vS 1/ 2
=1 [QvS2 < < 3402] æ 2P ö 2 3/ 2
3402 s =ç ÷ . t
èm ø 3
Þ vS = 1.7 m/s
i.e. s µ t 3/ 2
7. (b) Given, density of gas at point A, r A = r 0
s 2 µt3
General equation for an ideal gas
10. (c) Given, speed of electrons,v = 5 ´ 105 m/s
pV = nRT
m v v
Þ pV = . RT [m ® mass, M ® molecular mass]
M
m R
Þ p = . .T r
V M
æR ö é mù
Þ p =r ç ÷T ê where, density r = ú … (i)
è ø
M ë Vû e e
Applying Eq. (i) at point A and B, respectively.
æR ö Electrostatic force between two electrons,
p = r A ç ÷ T0 … (ii)
èM ø e2
Fe = k . … (i)
æR ö r2
2p = rB ç ÷ 3T0 … (iii)
èM ø Magnetic field produced due to moving electron with
From Eqs. (ii) and (iii), we get velocity v,
1 rA 1 m ev
= . B= 0. 2 … (ii)
2 rB 3 4p r

2 2 Magnetic force on proton


Þ rB = rA = r0 m 0 ev
3 3 Fm = B ev = . . ev (from Eq. (ii))
4p r 2
8. (c) Force constant K of rubber is given by
m 0 e 2v 2
YA 5 ´ 108 ´ 10-6 Fm = . … (iii)
K = = = 5 ´ 103 N/m 4p r 2
l 0.1
\ From Eqs. (i) and (iii), we have
Now, from conservation of energy
Fe ke 2 / r 2 k . 4p
elastic potential energy of cord = kinetic energy of particle = =
Fm m 0e 2v 2 m 0v 2
1 1
K ( Dl) = mv 2
2
i.e. 4p. r 2
2 2
9 ´ 109 ´ 4 ´ 3.14
K = = 3.6 ´ 105
Þ v = . Dl 4p ´ 10-7 (5 ´ 105 )2
m
5 ´ 103 11. (d) From the figure, It is clear that diodes D1 and D3 are
= ( 0.125 - 0.1) reverse biased, therefore they will not conduct resistance
5 ´ 10-3
11W and 5W will be not in use but diode D2 is in forward
= 103 ´ 0.025 = 25 m/s biased. Therefore, current flows through diode D2 is same
as current through battery, according to the circuit shown
after redrawing,
14 SELF STUDY GUIDE BITSAT

20 15. (c) Given, current through the square loop is 2A.


\ Current, I = =2A
7+ 3 B C

p 7W
n

2A 45º
O
20V 3W 45º
+ –

2 cm
12. (b) The moment of inertia of disc of radius R about its
axis is A D
4 cm
2
M ¢R
I= I = 2A
2
The given situation is shown in the figure. Magnetic field
where, M ¢ = mass of complete disc
due to wire AB at the centre O is
= 2M m I
BAB = 0 . [sin 45° + sin 45° ]
2MR 2 4p r
\ I=
2 2 é 1 1 ù
= 10-7 ´ -2 ê
+
I = MR 2 … (i) 2 ´ 10 ë 2 2 úû
If I1 be the moment of inertia of semicircular disc about its = 2 ´ 10-5 T [Downward to the plane of square loop]
axis, then disc may be assumed as combination of two
semi-circular parts, Since, magnetic field due to side AB, BC, CD and DA is
same in magnitude and direction. Hence, net magnetic
I MR 2 field at O
Thus, I = I1 + I1 Þ I1 = =
2 2
B = 4 ´ BAB = 4 2 ´ 10-5 T
13. (b) Given, total energy given to electron in third orbit, P R
Wheatstone bridge in balanced condition maintains =
E = 12 eV Q S
-13.6 i.e. no current flows through the galvanometer (i.e. ig = 0)
Energy of third orbit of H-atom, E ¢ =
n2
16. (c) The given circuit can be redrawn as
-13.6
= = - 1. 51 eV B
32
10W 20W
Ionisation energy of electron of third orbit, E ¢¢ = - E ¢
A C
= - ( - 151
. eV ) Þ E¢¢ = 151
. eV 15W

Hence, final energy of electron when it comes out of 6W 12W


H-atom from third orbit
D
EF = E - E ¢ ¢
= 12 - 1. 51 = 10.49 eV I 10V
-4
14. (a) Given, D = 2 m, d = 0.25 mm = 2.5 ´ 10 cm RAB RAD 1
Since, = = , hence, it is a balanced Wheatstone
dD RBC RDC 2
Velocity of screen, = 5 m/s
dt bridge, therefore current flowing in arm BD = 0
db \ Req = (10 + 20)||( 6 + 12) = 30 || 18
Speed of first maxima = =?
dt 30 ´ 18
= = 1125
. W
Dl 30 + 18
In Young’s double slit experiment, fringe width b =
d V 10
On differentiating with respect to t, we get \ I= = = 0.89 A ~
- 0.9 A
Req 1125
.
db l dD
= .
dt d dt 17. (a) The flux linked with coil (made of wire of radius r) of
-9 area A and magnetic field B, is given by
800 ´ 10
= ×5 æ pr 2 ö
2 . 5 ´ 10-4 f = BA cos q = B çç ÷ cos wt
÷
è 2 ø
= 0.16 ´ 10-3 m/s = 16 mm/s
SOLVED PAPER 2019 15
é pr 2 ù vx =
u
and u y =
8u
pBr 2 êQ A = ú
f= cos wt 7 7
2 ê and q = 2
wt úû
ë ly h / mv y 3 v 3 u/7 3
\ = = . x = . =
By Faraday’s law of electromagnetic induction, induced lx h / 3 m
.v x 4 v y 4 8u / 7 32
df 4
emf, e = -
dt 20. (c) We know that, velocity of light c is terms of Î0 and m 0 is
Hence, after differentiating, given by
d æ pBr 2 ö pBr 2w 1 1
e=- ç cos w t ÷= sin wt c = Þc2 =
dt ç 2 ÷ 2 m 0e0 m 0 e0
è ø
(where, R = resistance in the circuit) 1 m0 1 m0 m0
c2 = . Þc = Þ =c m0 … (i)
e 2 2 2 4 2
p B r w sin wt 2 m 20 e0 m0 e0 e0
\ Power, P = =
R 4R t [ML2T -2]
2 2 4 2 Dimension of magnetic field, B = =
pB r w NIA [A] [L2]
\ Mean power, P = . sin2 wt
4R
= [MA-1T -2]
p 2B 2r 4w2 1 é 2 1ù
= . êëQ sin wt = 2 úû Again, B = m 0n I
4R 2
B [MA-1 T -2]
( pBr 2w)2 \ m0 = = = [MLA-2T -2]
= nI [L-1] [A]
8R
m 0 = [MLA-2T -2] … (ii)
18. (b) For the given convex lens, focal length, f = 25 cm
and C = [LT -1] … (ii)
Magnification of lens in term of u and f is given by
f 25 From Eqs. (i), (ii) and (iii), we get
m= = … (i)
f + u 25 + u m0
= [LT -1] [MLA-2T -2] = [ML2T -3A-2]
Î0
When the image is real, then m = - 2 and u = - x1
25 75 21. (a) Given, F = at
\From Eq. (i), - 2 = Þ x1 =
25 - x1 2 when, t = 0, then linear momentum = p
Again, when the image is virtual, then m = 2 and u = - x 2 when, t = T , then linear momentum = 2p
25 25 According to Newton’s law of motion,
\ From Eq. (i), 2 = Þ x2 =
25 - x 2 2 dp
applied force, F =
Hence, the ratio of x 2 and x1 is dt
x 2 25 / 2 1 or, dp = Fdt
\ = =
x1 75 / 2 3 dp = at dt
2p T
19. (d) For perfectly elastic collision, or, ò dp = a ò t dt
total momentum before collision p 0

= total momentum after collision T2


( 2p - p ) = a .
3m 3m 2
i.e. m ×u + ´ 0 = mv x + .v y
4 4 2p 2p
Þ =T 2 ÞT =
Þ 4u = 4v x + 3 v y … (i) a a
Coefficient of restitution, 22. (d) Block does not move upto a maximum applied force of
3 N down the inclined plane.
Velocity of seperation (v y - v x ) after collision Equation of motion is 3 + mg sin 30° = F
e=
Velocity of approach (u x - u y ) before collision 3 + mg sin 30° = m mg cos 30° … (i)
vy -vx é Q ux = v ù R
f
Þ 1= ê and u = v ú
u-0 ë y û
u =vy -vx … (ii) 3+mg sin 30º
30º mg cos 30º
Solving Eqs. (i) and (ii), we have mg
30º
16 SELF STUDY GUIDE BITSAT

Similarly, block also does not move upto a maximum 27. (c) The charge on the capacitor at time t in a discharging
applied force of 12 N up the plane. circuit is given by q = q 0e -t /t
Now, equation of motion is where, t = RC = time constant
mg sin 30° + F = 12 \ q = q 0 e -t /RC
or, mg sin 30° + m mg cos 30° = 12 … (ii) q0
when, q=
Solving Eqs. (i) and (ii), we have 2
9 15 q0
mg sin 30° = and m mg cos 30° = then = q 0 e -t /RC
2 2 2
Hence, dividing 1
= e -t /RC
1 9/2 3 5 2
\ tan 30° = = Þm =
m 15 / 2 2 3 3 æ 1ö
Taking natural log on both side loge ç ÷ = loge e -t /RC
è 2ø
23. (a) If middle portion of lens is painted black, then less
number of refracted rays will intersect, therefore intensity t
- loge 2 = - loge e Þ t = RC loge 2
of the corresponding image of object will be less. Though, RC
the rays coming from object refracted with small portion of
28. (d) If v be the velocity in the inter planetary space andUi , Uf
lens, but complete image of object will be formed but of
lesser intensity. initial and final PE

24. (d) Given, speed of sound,v = 340 m/s Then, by the law of conservation of energy,
Ui + Ki = Uf + Kf
Length of open organ pipe, l = 2 m
- GMem 1 1
Frequency, f = 1200 Hz + m ( 3ve )2 = 0 + mv 2
Re 2 2
For open organ pipe,
- GMe 9 2 1 2
v 340 + ve = v
Fundamental frequency, f0 = = = 85 Re 2 2
2l 2 ´ 2
-2GMe
\Number of tones present in the open organ pipe Þ + 9Ve2 = v 2
Re
f 1200
= = = 14.11~ 14 - 2 gRe2
f0 85 Þ + 9ve2 = v 2 [Q GMe = gRe2]
Re
25. (a) Given, charge on proton, q = e
- 2gRe + 9 ´ 2 = gRe = v 2, [Qve2 = 2g Re ]
-19
= 16
. ´ 10 C
16 gRe = v 2
-3
Radius, r = 2 mm = 2 ´ 10 cm
8.2 gRe = v 2
Frequency, f = 10 Hz
Þ v = 8 . 2g Re = 2 2 ve
Current associated by proton is given by
4
I=
q
[ T ® time period] 29. (d) Given, density of bob, r = ´ 103 kg/m3
t 3

é 1ù Density of water, s = 103 kg/m 3


=qf êëQT = f úû If g ¢ be gravitational acceleration in water then,
-19 -18
= 16
. ´ 10 ´ 10 = 16
. ´ 10 A æ ö
æ sö ç 103 ÷ g
\ Magnetic dipole moment, M = IA g ¢ = g ç1 - ÷ = g ç1 - ÷=
è rø çç 4
´ 103 ÷÷ 4
. ´ 10-18 ´ 3.14 ´ ( 2 ´ 10-3 )2
= I × pr 2 = 16 è 3 ø
= 2 ´ 10-24 A-m 2 l
As, Tair = 2p ,
g
26. (b) When capacitor is fully charged, then it acts like a
open switch, therefore 8 kW resistor has no use. So, rest l æ gö
Similarly, Twater = 2p [Twater = 2s] çg ¢ = ÷
of the resistances 4 kW, 7kW and 9 kW will be in series. g¢ è 4ø
Hence, current drawn from cell
l l
10 10 2 = 2p = 2p . 2
I= = = 0 . 5 mA g/4 g
( 4 + 7 + 9 ) ´ 103 20 ´ 103
2 = 2.Tair Þ Tair = 1s
SOLVED PAPER 2019 17
30. (c) By Newton’s law of motion 1
=0+ ´ 20 ´ 22
F - fs = ma … (i) 2
= 40 m
a \Displacement of body in y-direction in 2s.
m 1
F y = uy t + ay t 2
2
fs 1
= 0 + ´ 10 ´ 22
2
and torque, t = Ia = 20 m
where, a = tangential component of acceleration \Position of the body at t = 2 s
I = moment of inertia r = x$i + y$j
Þ fR = Ia
r = 40$i + 20$j
MR 2 a é MR 2 aù
fR = . ê\I = and, a = ú
2 R ë 2 Rû 33. (b) Potential energy of cylinder at top position
ma B
Þ f =
2
1m
\ F = f + ma
ma 3ma
= + ma =
2 2
30º
31. (a) Velocity of projectile at time t is given by C A

v = u - gt j U = mg( AB )
v = $i + 3 $j - 10t $j é AB ù
= 2 ´ 10 [1. sin 30° ] êQ sin 30° = BC ú
\ r = ò v dt ê\ AB = 1 sin 30°ú
ë û
= ò ( $i + 3 $j - 10 t $j ) dt = 10J

t 2 If v is the linear speed of cylinder when it reaches at


= t $i +
3 t $j - 10 $j bottom, then at bottom, total potential energy of cylinder is
2
converted into kinetic energy,
$ $
=t i + 3 t j -5t j2$
i.e. K =U
r = t $i + ( 3t - 5t 2 ) $j 1 2 1
Iw + mv 2 = 10
2 2
or x$i + y$j = t $i + ( 3t - 5t 2 ) $j
1 mr 2 v 2 1 é vù
On comparing, x =t … (i) . . + mv 2 = 10 êëQ w = r úû
2 2 r2 2
and y = 3 t - 5t 2 … (ii) 3
Þ mv 2 = 10
From Eqs (i) and (ii), we get 4
y = 3 x - 5x 2 40 40 20
v = = = m/s
3m 3 ´2 3
32. (d) F = 20$i + 10$j
34. (a) According to the question,
\ Fx = 20 N and Fy = 10 N
F1 = - k1x and F2 = - k 2x
\Acceleration,
Fx 20 \Frequency of oscillation
ax = =
m 1 1 k1
n1 =
a x = 20 m/s 2 2p m

Fy 1 k1
and ay = =
10
= 10 m/s 2 5= Þ k1 = 100 p 2 m
m 1 2p m

\Displacement of body in x-direction in 2s. 1 k2


Similarly, 12 =
1 2p m
x = ux t + ax t 2
2 Þ k 2 = 576 p 2 m
18 SELF STUDY GUIDE BITSAT

Now, F = F1 + F2 = - (k1 + k 2 ) x 1
i.e. Eµ … (i)
\ Frequency of oscillation, r2
The electric field intensity at the surface (r = R ),
1 k1 + k 2 1 100p 2m + 576p 2m
n= = 1 q 1
2p m 2p m E= ÞE µ 2
4pe0 R 2 R
Þ n = 13 Hz
where, R being the radius of sphere,
35. (c) Number of moles in 2 ´ 10-4 kg The electric field intensity inside the sphere is
Mass of hydrogen in gram qr
m = E= Þ E µr
Molecules mass 4pe0R 3
2 ´ 10-4 ´ 103
= At the centre of sphere, r = 0, \E = 0
2
38. (a) Given, R = (56 ± 5.6) kW
= 0.1 mole
\ pV = mRT = 56 kW ± 10% of 56 kW
pV R = 56 ´ 103 ± 10% W
= mR = 0.1 ´ R =
T 10 As per the colour code for carbon resistors, the colour
assigned to numbers
36. (a) Given, for an ammeter, Ig = 1A, Rg = 15
. W
5 ® Green
Total current, I = 4 A 6 ® Blue
If Rs be the shunt resistance, then 3 ® Orange
according to the question circuit diagram is given, For 10% accuracy or tolerance, the colour is silver.
(Rs || Rg , hence have equal potential difference) Hence, the bands of colours of carbon resistor in sequence
are green, blue, orange and silver.
Rs
39. (c) When two waves with same frequency and constant
phase difference interfere to each other, then after
(I–Ig) interference, intensity of resultant wave is distributed such
that it is maximum at some points [constructive interference
I or bright fringes] and minimum at another points.
Rg
[destructive interference or dark fringes].
where, IgRg = (I - Ig ) Rs Hence, energy is distributed and remains constant [in the
form of bright and dark fringes]
Ig 1
Rs = ´ Rg = ´ 1. 5 = 0 . 5 W
I - Ig 4 -1 40. (d) According to the given graph (I - t ) induced current (I ) is
obtained from the slope.
37. (c) The electric field intensity at a point lying outside the 1 df
sphere (non-conducting) is Hence, I =
R dt
1 q 8 ´ 0.2
E= . Þ df = I dt . R = area of triangle POS ´ R = ´ 50 = 40 Wb
4pe0 r 2 2

Chemistry
41. (a) Let atoms of Y in ccp structure = 100 . kg/m 3
42. (a) Given, density (d1) at 30°C (T1) = 135
Then, number of tetrahedral voids = 2 ´ 100 To find density (d 2 ) at S.T.P, (i.e at 760 torr and 273 K (T2 )
2 temperature), we use formula
Q Number of X atoms = rd of tetrahedral voids
3 d 2 p 2 . T1
=
2 400 d1 p1T2
\Number of X atoms = ´ 200 =
3 3 where, p1 = 768 torr (Pressure at 30°C, i.e. at 303 K)
X 400 400 4
Thus, = = = p 2 = 760 torr (Pressure at 273 K)
Y 3 ´ 100 300 3
d ´ P2 ´ T1 1. 35 ´ 760 ´ 303
\ d2 = 1 = . kg/m 3
= 148
Hence, formula of compound can be = X 4Y3. P1 ´ T2 768 ´ 273
SOLVED PAPER 2019 19
43. (c) D 0 = ( - 0.4 ´ nt 2g ) + ( 0.6 ´ neg ) 47. (d) NH2 NH2

where, n = number of electrons in nt 2g and neg . H2SO4


180°C
CFSE of given d-electrons configuration are as follows
a. For high spin, d 6 ; (Aniline) SO3H
D 0 (CFSE) = - 0.4 ´ 4 + 0.6 ´ 2 = - 0.4 (Sulphonic acid)

b. For low spin, d 4; Sulphanilic acid exist as dipolar ion (zwitter ion); act as
D 0(CFSE) = - 0.4 ´ 4 + 0.6 ´ 0 = 1. 6 inner salt and SO3H group diminishes the basic character
of ¾ NH2.
c. For low spin d 5
D 0 (CFSE) = - 0.4 ´ 5 + 0.6 ´ 0 = - 2.0 48. (c) The structure of PO3-
4 is as follows

d. For high spin, d 7 O–

D 0 (CFSE) = - 0.4 ´ 5 + 0.6 ´ 2 = - 0.8 – P


O O–
Neglecting -ve sign (as it indicates that energy is
O
released.
Hence, maximum D 0 (CFSE) value is shown by low spin As phosphorous central atom forms four (4) sigma bonds
d 5 configuration. and has no lone pair of electrons. It uses four hybrid
orbital and thus, hybridisation of p-atom is sp 3.
44. (b) According to Rydberg’s equation
49. (c) Electronic configuration of the given species are
1 é 1 1ù
n= = RZ 2 ê 2 - 2ú a. Sm2+ ( Z = 62) = [Xe] 4f 6 b. Eu2+ ( Z = 63) = [Xe] 4f 7
l ë nL nH û
c. Yb2+ ( Z = 70) = [Xe] 4f 14 d. Ce2+ (Z = 58) = [Xe] 4f 15d 1
where,
On the basis of above electronic configuration, only
n = wave number
Yb2+ ( Z = 70) has no unpaired electrons i.e. has fully-filled
l = wavelength orbitals. Thus it is dimagnetic in nature.
R = Rydberg constant (= 109677 cm-1)
50. (b) Alcohol in which OH group is directly bonded to
Z = atomic number benzene are aromatic alcohols.
nL = lower energy state OH
nH = higher energy state. is an aromatic alcohol,
1 as ¾ OH group is directly bonded with the
i.e. µZ2 benzene ring.
l
b.
l ( He+ ) Z 2( H) CH2OH
Thus, = 2
l ( H) Z ( He+ )
To find l( He) = ?
Given, l( H) = 912
. nm
It is not an aromatic alcohol because OH is not bonded
Z ( H) = 1
directly with the benzene group.
Z (He) = 2
c.
1 ´ 912
. CH2OH
\ l ( He+ ) = = 22.8 nm
4
45. (b) C18H8(s ) + 12O2(g ) ¾® 10CO2(g ) + 4H2O(l )
CH3
We known that DH = DE + DngRT
It is also not an aromatic alcohol because ¾ OH group is
Dng = 10 - 12 = - 2 not directly bonded with the benzene ring.
DH = - 1228. 2 ´ 103 + ( -2) ´ 2 ´ 298
d.
OH
= - 1229393 Ca
= - 1229.39 kcal
46. (a) Among the formation of B, C, D and E, the slowest step
is the rate determining step. As formation of B, i.e. 0.002 CH3
mol/h per mole of A is the slowest step. Hence, step-(I) is
the rate determining step. It is an aromatic alcohol because ¾ OH is directly bonded
with the benzene ring.
20 SELF STUDY GUIDE BITSAT

51. (b) RSiCl3 gives cross-linked silicon polymer on The pp-pp back bonding is shown maximum by BF3, as the
hydrolysis. size of B and F are small and comparatively same. Due to
Cl OH this effect tendency of accepting lone-pair of electrons of
R—Si—Cl
H 2O
R—Si—OH
Polymerisation boron decreases as size of halogen increases. The order of
size of halogens are F < Cl < Br < I. Thus, acidic nature is in
Cl OH
order
Si Si BF3 < BCl3 < BBr3 < BI3

O O 55. (d) Key idea Mobility of alkali ions in aqueous solution µ


size of ion
R—Si—O—Si—R
1
O O n µ .
size of hydrated ion
Si Si
The mobility of alkali ions is inversely proportional to the
size of ion in hydrated state, because smaller the ion more
52. (a) Let k be the rate constant of given reaction it will get hydrated and becomes larger in size, thus its ionic
k mobility decreases.
2O3 ¾® 3O2
Hence, correct order of mobility is
From the slowest step r = k ¢[ O] [ O3] … (i)
Rb+ > K + > Na + > Li+
To eliminate [ O], (From fast step)
[ O ] [ O] 56. (c) Bakelite is a thermosetting plastic which is formed by
k eq = 2 the reaction of phenol with H.CHO in the presence of
[ O3 ]
H2SO4 (conc).
k eq. [ O3]
[ O] = The reaction occurs as follows:
[ O2 ]
OH OH OH
Now, substituting value of [ O] in Eq (i)
H2SO4(Conc) H2C— —CH2— —CH2
k ¢ k eq.[ O3] [ O3] n.HCHO+n.
r = D
[ O2]
CH2 CH2
Let k ¢ k eq = k
2 -1
Thus, r = k [ O3] [ O2] Bakelite

53. (a) Depression in freezing point is given as DTf = Kf


A cross-linked condensation polymer (at o-and p-position)
DTf
Molality of solution (m ) = bakelite is thus produced.
kf
57. (c) Paracetamol is the drug, that can be used as an
0.6
or m= = 0.322 m antipyretic as well as an analgesic, i.e. to reduce the fever
1. 86 and gives relieve from pain.Its structure is as follows
Elevation in boiling point of solution is given as OH
DTb = kb ´ m
DTb = 0.512 ´ 0.322
DTb = 0.165
NHCOCH3
\Boiling point of solution = 373 + 0.16 (Paracetamol)

= 373.16 K
58. (a) In the given structure, ring one (I) has one o-atom and is
54. (a) The strength of acidic character of boron trihalides a 6 membered ring, thus is called pyranose ring, whereas
depends upon pp - pp back bonding. ring (II) has one o-atom and is a five membered ring, thus is
In boron trihalides, pp-pp-back bonding occurs due to called furanose.
empty orbital of boron and filled orbitals of halogens. The disaccharide is linked by a glycosidic bond between
C—1 of glucose (in the a-position, a linkage) and C—2 of
F F fructose (in the b-position, b-linkage).
F ——B F ——B
F F 59. (c) Due to size and geometries of bases, the only possible
pairing in DNA are between G (guanine) and C (cytosine)
Phenomenon of back through three H-bonds and between A (adenine) and T (
bonding in BF3 molecule thymine) through two H-bonds.
SOLVED PAPER 2019 21
Thus,the complementary strand of DNA will be For reaction (iii)
A T C G T A T G 0.0591 1
E3 = E(°cell) - log
¯ ¯ ¯ ¯ ¯ ¯ ¯ ¯ (complementary strand) 2 0.1
T A G C A T A C 0.0591
E3 = E(°cell) -
2
60. (a) When Na 2SO3 reacts with hot and dil H2SO4 , it gives
SO2(g ) which decolourises bromine-water. Hence, E1 > E2 > E3.
The reaction proceed as follows: 66. (c) This test is used for the presence of sulphur in any
(i) Na 2SO3 + H2SO4 ¾® Na 2SO4 + SO2 + H2O compound. When sodium nitroprusside, react with
sulphide ion, (e.g. Na 2S) the following reaction
(ii) Br2 + H2O ¾® 2HBr + [ O ] takes place
(iii) SO2 + [ O ] ¾® SO3 Na 2S + Na 2[Fe(CN)5 NO] ¾®Na 4[Fe(CN)5 NO5]
(decolourisation of bromine-water) (Sod. nitroprusside) purple coloration
67. (c) Threshold
(sod. thionitroprusside)
61. (b) The given reaction is for preparation of DDT
energy (E)
(dichlorodiphenyl trichloroethane). Complete reaction is Et

(Energy of reactant)
given as follows.
Cl
Cl2 Ea' (energy of
(i) FeCl3 Ea EP product)
E
Benzene (X) Chlorobenzene (Y) B (energy of
DE
the reaction)
Cl ER
A
H2SO4
(ii) 2 . + CCl3CHO Cl— —CH— —Cl Reaction coordinate
CCl3
where, Ea = activation energy of forward reaction
62. (d) The mixture of CO(g ) and H2(g ) is known water gas as Ea ¢ = activation energy of backward reaction
it is used in the synthesis of other compounds, The above energy profile diagram shows that Ea > Ea ¢
CO(g ) + 2H2 (g ) ¾ Cobalt
¾¾ ¾® CH3OH(l )
Catalyst The potential energy of the product is greater than that of
Water gas the reactant, so the reaction is endothermic.

63. (c) Chromium hydroxide, i.e. Cr(OH)3 when reacts with Ea = Ea ¢ + DE


NaOH and Na 2O2, gives yellow colour of sodium Et = Ea or Et > Ea ¢
chromate. The whole reaction is as follows
68. (c) The H2C == CH2 + C6H6 on reaction produce ethyl
3Na 2O2 + 2Cr(OH)3 ¾® 2Na 2CrO4 + 2H2O + 2NaOH
benzene as follows
(yellow)
CH2–CH3
64. (a) Cl is the good leaving group, i.e is the weakest
nucleophile among the given options anhy. AlCl3
+ CH2==CH2
95ºC
[Cl, NH2, OC2H5 and OCOCH3], the rate of reaction is
faster, when ‘Z’ is ‘Cl’. (ethyl benzene)

65. (b) Key-idea 69. (b) Given, pH of solution A = 3


Use Nernst equation, i.e.
0.0591 [ Zn2+ ] [ H+ ] of A = 10-3M and, pH of solution B = 2
°
E(emf) = Ecell - log
2 [ Cu2+ ] [ H+ ] of B = 10-2 M
For reaction (i) Thus, total [ H+ ] are = 10-3 + 10-2 = 11
. ´ 10-3 M
0.0591 0.1
E1 = E(°cell) - log \ pH(final) = -log [H+ ]
2 1
0.0591 . ´ 10-3]
= - log [11
E1 = E(°cell) +
2
For reaction (ii) = 3 - log 11
.
0.0591 1 = 3 - 1. 04 = 1. 9
E2 = E(°cell) - log = 0
2 1
E2 = E(°cell)
22 SELF STUDY GUIDE BITSAT

70. (a) Freundlich isotherm is related with the pressure of gas Thus, only p 2p x and p 2p y are degenerate orbitals.
x
adsorbed, as follows = k . p1/n 76. (c) Total energy (E ) of any photon is given by the relation :
m
hc
x E=
where, = amount of gas (x ) adsorbed over the surface l
m where, h = Planck’s constant = 6.6 ´ 10-34 J-s
of mass (m ).
c = velocity of light = 3 ´ 10-10 m/s
p = pressure of gas
l = wave-length = 1. 5 ´ 10-10 m, i.e, (150 pm)
k = freundlich constant
hc 6 . 6 ´ 10-34 ´ 3 ´ 108
Thus, value of ‘n’ is always greater than 1 and therefore Thus, E = = ; E = 1. 32 ´ 10-15 J
value of 1/ n lies between 0 and 1. l 1. 5 ´ 10-10
71. (c) Electronic configuration of Mn is ( Z = 25) = [ Ar ] 3d 5 4s 2 and, energy of ejected electron (E¢ )
\Electronic configuration of Mn 4+ 4+
= [ Ar ] 3d 3 1
is Mn E ¢ = mv 2,
4+ 2
Thus, Mn has 3-unpaired electron and its spin only
magnetic moment (m) is, m = n(n + 2) BM where,
m = mass of electron = 9.1 ´ 10-31kg
where, n = number of unpaired electrons.
m = 3( 3 + 2) . BM Þ m = 15 = 3 . 87 BM » 4 BM v = velocity of electron = 1. 5 ´ 107m/s
1 1
72. (a) Q Gram equivalents of an acid E¢ = mv 2 = ´ 9.1 ´ 10-31 ´ (1. 5 ´ 107 )2
2 2
0.45 20 ´ 0 . 5
= Gram equivalents of KOH i.e. = E¢ = 1. 024 ´ 10-16
E 1000
0.45 ´ 1000 Thus, total energy of photon = binding energy of electron
where, E = Equivalent mass E = = 45 (B ) + energy of ejected electron (E¢ )
20 ´ 0 . 5
Thus, 1. 32 ´ 10-15 = (B ) + (E )
90
Thus, basicity = =2
45 \ . ´ 10-15] - [1024
(B ) = (E ) - (E ¢ ) = [132 . ´ 10-16]
é Molecular mass ù = 1. 2176 ´ 10-15 J
êQ Basicity = Equivalent mass ú
ë û
1. 2176 ´ 10-15
73. (a) Given, A+B c C+D = eV = 7.6 ´ 103 eV
1. 6 ´ 10-19
initial conc. 4 4 0 0
77. (b) The degree of unsaturation for compound C7H8 is three
at equilibrium ( 4 - 2) ( 4 - 2) 2 2
(3), as it reacts with Cl2/D. Hence C7H8 is toluene, i.e
i.e. 2 2 2 2 C6H5. CH3. The whole reaction occurs as follows:
2 ´2
\ K( eq) = =1 CH3 CCl3 C.Cl3 CH3
2 ´2
3Cl2/D Br2/Fe Zn/HCl
74. (b) Given, 125 ml of 1 M AgNO3
Br Br
which is equal to 13.5 g of Ag (Toluene) (A) (B) (m-bromo toluene)
Q 1000 ml of AgNO3 = 108 g of Ag
78. (c)
108 ´ 125
\125 ml of AgNO3 = = 13. 5 g
1000 Ratio Atomic Mol Simplest
Element (by weight) weight Ratio Mol Ratio
Also, Q 108 g of Ag is deposited by = 96500 C
9 0.75
96500 ´ 13.5 C 9 12 = 0.75 =3
\13.5 g of Ag is deposited by = = 12062. 5 C 12 0.25
108
1 100
.
Q Q = it H 1 1 = 100
. =4
1 0.25
12062. 50
\ t (time) = Q / i = , t = 50 sec 3 .5
241.25 = 0.25 0.25
N 3.5 14 14 =1
0.25
75. (b) Orbitals of nearly equal energy are called degenerated
orbitals. As, energy of,
a. s2s > s1s b. p 2p x » 2p y \ Empirical formula = C3H4N

c. p * 2p x < s* 2p z d. s2p x < s* 2p z The empirical formula mass = 3 ´ 12 + 4 ´ 1 + 14 = 54 g


Given, molecular mass = 108 g
SOLVED PAPER 2019 23
108 sp 3 sp sp sp 3
Thus, n= =2 b. CH3 ¾ C ºº C ¾ CH3
54
Hence, molecular formula = (Empirical formula) ´ 2 sp 2
c. CH2 == C == CH2 d. CH2 == CH¾ C ºº CH
(C3H4N) ´ 2 = C6H8N2 sp 2
sp
sp 2 sp 2 sp sp

79. (a) Key idea pi-bond do not participate in hybridisation 80. (a) In bassemer converter, copper-sulphide is partially
sp sp sp sp oxidised to cuprous oxide, which further reacts with the
a. H¾ C ºº C ¾ C ºº C ¾ H remaining copper sulphide to give copper and sulphur
di oxide, i.e. Cu2S + 2Cu2O ¾® 6Cu + SO2

English Proficiency
81. (b) ‘Many a’ takes singular noun, as well as singular verb. 88. (b) Taint (Noun) means a mark of disgrace or blemish.
So replace ‘have’ by ‘has’. The correct antonym will be ‘clear’, which means free of
e.g. Many a player is injured lately. ambiguity or doubt.
Antonyms Blank, esteem, respect.
82. (a) The correct syntax for interrogative sentences is ‘wh’
word + do/does+sub+verb+? Synonyms Contamination, corruption, stigma e.g. His
reputation was tainted by allegations of illegal activity.
So, we should replace ‘it takes’ by ‘does it take’ e.g. How
long does it take to build a new house? 89. (b) Vivacious (Adjective) means full of life and energy or
animated.
83. (c) Replace ‘his’ by ‘ones’ to make the possessive correct.
So, its correct synonym will be ‘Lively’.
e.g. One should respect one’s parents.
Synonyms Spirited, active, jolly
84. (a) Assured is followed by preposition ‘of’. Antonyms Boring, unhappy, dull
e.g. I am assured of my promotion by next month.
e.g. She is a dark-haired and vivacious lady.
85. (d) Preposition ‘to’ should be used with ‘submit’ here. 90. (b) Sporadic (Adjective) means rare and scattered in
e.g., When exams are over then candidates are required occurrence. Here ‘scattered’ has the same sense. It is the
to submit their papers. correct synonym.
86. (d) Audacious (Adjective) means bold or fearless. So, its Synonyms Desultory, isolated, random.
correct antonym will be ‘timid’, which means ‘lacking in Antonyms Common, systematic, usual
courage or confidence.’ e.g. Suddenly, sporadic fighting broke out in the town.
Antonyms Careful, meck, mild 91. (d) Both official and corporate India is allergic to ‘mention
Synonyms Daring, bold, unafraid. of clean technology.’
e.g. He is an audacious soldier. 92. (d) ‘Failure In crops’ is a life and death question to many
87. (c) Cogent (Adjective), which means reasonable Indians.
and convincing. The correct antonym will be ‘dissuasive’, 93. (a) The most similar in meaning to the word ‘profligacy’ is
which means divert from the measure or purpose. ‘wastefulness’.
Antonyms Unconvincing, muddle, vague 94. (a) India cannot tolerate any further ‘crop failure’.
Synonyms Convincing, logical, compelling.
95. (d) The reason could be ‘US wants to use it as a handle
e.g. He put forward some cogent reasons for initiating the against the developing countries in the forthcoming meet’.
project.

Logical Reasoning
96. (d) In the ‘College’, education is given to ‘Students’, in the This series contains two separate series. In first series, 1
same way, treatment is given to the ‘Patient’ in ‘Hospital’. is added in first two steps and 4 is added in third and
fourth steps.
97. (b) Except 35, all others are multiples of 9, but 35 is the In second series, 2 and 4 is added alternately.
multiple of 5 and 7.
98. (c) Pattern of the series is as shown below 99. (b) From given information,
Ramesh > Mohan > Shyam …(i)
+2 +4 +2
Gautam > Ramesh > Rajat …(ii)
1 2 2 4 3 8 7 10 11
Shyam > Rajat …(iii)
+1 +1 +4 +4
Combining Eqs. (i), (ii) and (iii), we get,
24 SELF STUDY GUIDE BITSAT

Gautam > Ramesh > Mohan > Shyam > Rajat i.e.

Hence, Rajat is the shortest among all.


100. (b) If figure shown in option (b), is placed in the place of
missing portion of the original figure, then it is completed 103. (a) In each row, the number of objects increases by 1 at
as shown below each step from left to right. So, in third row, the missing
segment contain 2 + 1 = 3 dots.
104. (b) One of the three dots occupies the region which is
common to the circle, rectangle and triangle; another dot
occupies the region which is common to the triangle, circle,
rectangle and square and the third dot occupies the region
which is common to the circle, rectangle and square.
These three characteristics as shown by these three dots
101. (b) Figure given in option b. can be formed by joining the
are found in figure b.. It possesses region which is common
pieces given in question figure, as shown below. to the circle, rectangle and triangle, a region which is
common to the triangle circle, rectangle, and square and a
region which is common to the circle, rectangle and square
Common to the triangle, rectangle and
circle
102. (d) In figure (X), the square sheet of paper is being
folded along the vertical line of symmetry, so that right Common to the triangle, square, circle
half of the sheet overlaps the left half. and rectangle
In figure (Y), the sheet is folded further to a quarter. Common to the circle, rectangle and square
In figure (Z), two squares are punched in the folded
sheet. Clearly, the punched squares will be created in 105. (a) Naming the figure,
each quarter of the paper and after unfolding the first fold E F G H
the figure will look like as,
O P Q

A B C D

Clearly, there are 28 triangles in the given figure namely,


then after unfolding the last fold the transparent sheet will EOF, AOE, AOB, BOF, ABF, BEF, ABE, AEF, BPF, FPG,
look like as given in option d. CPG, BPC, BFG, BCG, CFG, BCF, GQC, CDQ, DQH,
GQH, GDC, GDH, GHC, CDH, AFC, BGD, EBG and FCH.

Mathematics
106. (d) Given, a = 3b = tan-1 (n 2 + n + 1) - tan-1 1
b2 b2 æ n2 + n + 1-1 ö æ n2 + n ö
e = 1- 2
Þ e = 1- = tan-1 çç ÷ = tan-1ç
÷
÷
çn2 + n + 2÷
a 9b 2 2
è 1 + (n + n + 1) ø è ø
1 8 2 2 20
Þ e = 1- = = 108. (b) Given, ( 2x 2 + 3x + 4)10 = S ar x r
9 3 3 r =0
n æ 2m ö 2
107. (a) We have, S tan-1çç 4 ÷ Replacing x by , we get
m =1 2
èm + m + 2 ÷ø x
10 20 r
æ 8 ö
S ar æç ö÷
6 2
n æ (m 2 + m + 1) - (m 2 - m + 1) ö ç 2 + + 4÷ =
= S tan -1 ç ÷
ç 1 + (m 2 + m + 1) (m 2 - m + 1) ÷ èx x ø r =0 èx ø
m =1
è ø 20
n Þ 210 ( 2x 2 + 3x + 4)10 = S ar 2r x 20 - r
= S [tan-1 (m 2 + m + 1) - tan-1 (m 2 – m + 1)] r =0
m =1
20 20
= (tan-1 3 - tan-1 1) + (tan-1 7 - tan-1 3) Þ 210 S ar x r = S ar 2r x 20 - r
r =0 r =0

+ (tan-1(13) - tan-1 7) + ... + (tan-1(n 2 + n + 1) Comparing coefficient of x 8 both sides, we get


- tan-1 (n 2 - n + 1)) a8 212
210a 8 = a12 212 Þ = 10 = 4
a12 2
SOLVED PAPER 2019 25
109. (d) In DABC, D be the mid-point of BC and AB = ADDABD Þ 2 < x - 1 < 3 Þ 5 < x < 10
is an isosceles.
\Equation have infinite many solutions.
\AM is perpendicular bisector of BD
BM = MD 112. (d) We have,
AM ax + y + z = a … (i)
In DABM, tan B =
BM x + by + z = b … (ii)

A x + y + cz = c … (iii)
are inconsistent.
a 1 1 a 1 1
\ 1 b 1 = 0 and b b 1 ¹ 0
1 1 c c 1 c
a 1 1
Þ 1 b 1=0
B C
M D 1 1 c
AM Þ a (bc - 1) - 1(c - 1) + 1(1 - b ) = 0
In DAMC, tan C =
CM abc - a - c + 1 + 1 - b = 0
tan B CM 3BM
\ = = =3 [QCM = DM + CD] abc - a - b - c + 2 = 0
tan C BM BM
a 1 1
110. (a) We have, and b b 1 ¹0
1 dx
L = lim n ò c 1 c
n ®¥ 0 (1 + x 2 )n
It is possible only when a ¹ 1, b ¹ 1 and c ¹ 1
We know that, (1 + x ) > 1 + nx 2
2 n
113. (a) For atleast 4 times an even number, probability is
1 1
Þ < 4
æ 1ö æ 1ö
3
æ 1ö æ 1ö
5
æ 1ö æ 1ö
2 6 1
(1 + x 2 )n 1 + nx 2 = 7C4 ç ÷ ç ÷ + 4C5 ç ÷ ç ÷ + 7C6 ç ÷ ç ÷
è ø è ø
2 2 è ø è ø
2 2 è 2ø è 2ø
1 dx 1 1
Þ ò0 (1 + x 2 )n < ò0 1 + nx 2 dx æ 1ö
7
+ 7C7 ç ÷
1 1 1 è 2ø
Þ ò0 (1 + x 2 )n < [tan-1 n x ]10
n 1
=
1 2
= tan-1 n
n 114. (b) Equation of plane containing line x - y - 1 = 0 and
1 1 1 z - 1 = 0 is (x - y - 1) + l ( z - 1) = 0
L = lim n ò0 (1 + x 2 )n dx < lim n tan-1 n
n ®¥ n ®¥ n Þ x - y + lz - 1 - l = 0 …(i)
p Since, this plane is parallel to line
\ L=
2 x z
- = 1 and y = 3
2 3
111. (d) We have, x + 3 - 4 x - 1 + x + 8 - 6 x - 1 = 1
x -1 y - 3 z
= = …(ii)
( x - 1)2 - 2 ´ 2 x - 1 + 4 + ( x - 1)2 - 2 ´ 3 x - 1 + 9 = 1 2 0 3

Þ ( x - 1 - 2 )2 + ( x - 1 - 3 ) 2 = 1 \ 2(1) + ( 0) ( - 1) + 3( l ) = 0
Þ l = - 2/3
Þ | x - 1 - 2| + | x - 1 - 3| = 1
Putting the value of l in Eq. (i), we get
Þ | x - 1 - 2| + | x - 1 - 3| 2 2
x – y – z – 1 + = 0; 3x - 3y - 2z = 1
= ( x - 1 - 2) - ( x - 1 - 3) 3 3
40
We know that, 115. (d) We have, (1 + x + x 2 )20 = S ar x r
r =0
If | x - a| + | x - b| = (x - a ) - (x - b )
1
then, (x - a ) (x - b ) < 0 x is replaced by - , we get
x
\ ( x - 1 - 2) ( x - 1 - 3) < 0 20 r
40
æ 1ö
S ( - 1)r ar æç ö÷
1 1
ç1 - + 2 ÷ =
è x x ø r =0 èx ø
26 SELF STUDY GUIDE BITSAT

40 y p
(1 - x + x 2 )20 = S ( - 1)r ar x 40 - r \ tan–1 =
r =0 x 6
39 y p 1
S ( - 1)r ar . ar + 1 is coefficient of x in product of = tan =
r =0 x 6 3
æ 1 1ö
20
Þ x = 3y
(1 + x + x 2 )20 ç1 - + 2 ÷ =0
è x x ø Also given,
116. (a) Let | z - 2 3i| = l
2
4 sin x cos x - cos x + sin x | x + iy - 2 3 i| = l
L = lim
3p sin x + cos x
x ®
4 x 2 + (y - 2 3 )2 = l2
2 sin x (sin2 x + cos2 x + 2 sin x cos x - 1) This is the equation of circle whose centre is (0, 2 3)
- cos x + sin x and r = l. Now, x – 3y = 0 touches the circle
L = lim
3p sin x + cos x
x ® 0 - 3 (2 3)
4 \ l=
2
2 sin x [(sin x + cos x ) - 1] - cos x + sin x 1+ 3
L = lim
x ®
3p sin x + cos x 6
4 l= =3
2
2 sin x (sin x + cos x )2 - 2 sin x - cos x + sin x
L = lim \ l=3
x ®
3p sin x + cos x
4 119. (a) We have,
(sin x + cos x ) ( 2 sin x (sin x + cos (x - 1) l+m+n=0 …(i)
L = lim
x ®
3p sin x + cos x 2 2 2
4 and l =m + n …(ii)
L = lim 2 sin x (sin x + cos x ) - 1 = - 1 Putting the value of l in Eq. (ii), we get
3p
x ®
4 (m + n )2 = m 2 + n 2
117. (b) Given, Þ m 2 + n 2 + 2mn = m 2 + n 2
a ´ b= c Þ mn = 0
b´c= a Þ m = 0, n = 0
c´a = b when m = 0, l = - n
( b ´ c) × a = a × a æ 1 1 ö
\Direction cosines are ç - , 0, ÷.
(c ´ a) × b = b × b è 2 2ø
a 2 - b2 = ( b ´ c) × a - ( c ´ a ) × b When n = 0, l = - m
= [ b c a] - [ c a b] = 0 æ 1 1 ö
\Direction cosines are ç - , , 0÷ .
\ 2
| a | - | b| = 02
è 2 2 ø
| a | - | b| = 0 Angle between lines are
We know that, æ –1 1 ö æ 1 ö æ 1 ö
cos q = ç ´- ÷ + ç0 ´ ÷+ç ´ 0÷
| a ´ b b ´ c c ´ a] = [ a b c] 2 è 2 2ø è 2ø è 2 ø
1
[ a b c] = [ a b c] 2 cos q = , q = p / 3
2
| a b c] [ - [ a b c] - 1] = 0
120. (c) Let the foot of perpendicular be (h, k ) .
[ a b c] = 1
Equation of tangent with slope m passing (h, k ) is
| a| | b| | c | = 1
y = mx ± 6m 2 + 2, where m = - h / k
\ | a| = | b| = | c | ¹ 2
6h 2 h2 + k 2
118. (a) Let z = x + iy Þ + 2 =
k2 k
æy ö
arg z = tan-1ç ÷
èx ø 6h 2 + 2k 2 = (h 2 + k 2 )2
p So, required locus is 6x 2 + 2y 2 = (x 2 + y 2 )2.
arg ( z ) =
6
SOLVED PAPER 2019 27
121. (d) Let | l| = 3 Þ l = ± 3
h(x ) = f (x ) - 2g(x ) \ Equation of plane is
h( 0) = f ( 0) - 2g( 0) x - 2y + 2z = 3
h( 0) = 2 - 2( 0) [Q f ( 0) = 2, g( 0) = 0] 125. (b) Let A be the event that maximum is 6.
h( 0) = 2 B be the event that minimum is 3.
h(1) = f (1) - 2g(1) 5
C2
P(A) = (number dess than 6 is 5)
h(1) = 6 - 2( 2) [Q f (1) = 6, g(1) = 2] 8
C3
h(1) = 2 5
C2
h( 0) = h(1) P (B ) = 8
(number greater than 3 is 5)
C3
h ¢ (x ) = f ¢ (x ) - 2g ¢ (x ) 2
C1
By Rolle's theorem h ¢ (c ) = 0 P(A ÇB) = 8
C3
\ f ¢ (c ) - 2g ¢ (c ) = 0, f ¢ (c ) = 2g ¢ (c )
\Required probability
122. (b) We have,
P ( A Ç B ) 2C1 2 1
+
1 = P (B / A ) = = 5 = =
æ 1ö x P(A) C2 10 5
I = ò ç1 + x – ÷e x dx
è xø
sin( la ) cos( la )
x +
1
+
1 126. (b) We have, - = l -1
æ 1ö x sin a cos a
I = òe x dx + ò çx - ÷e x dx
è x ø sin( la ) cos a - cos( la ) sin a = ( l - 1) sin a cos a
1 1
x + æ 1ö x + sin( l - 1) a = ( l - 1) sin a cos a
I = òe x dx + ò x ç1 - 2 ÷e x dx
è x ø l -1
sin( l - 1) a = sin 2a
x +
1
x +
1
x +
1 2
I = òe x dx + x e x - òe x dx It is possible, when l - 1 = 0 Þ l = 1
1 l -1
x + and = 1Þ l = 3
I = xe x +C 2
123. (d) We have, \l has only two solution.
2
(1 + ax + bx ) (1 - 2x ) 18
127. (b) Given, y = ax 2 + bx + c has two x-intercepts one is
positive and one is negative and vertex is ( 2, - 2).The
(1+ ax + bx 2 )(1 - 18C12x + 18
C2( 2x 2 ) -
graph of y = ax 2 + bx + c is
18
C3( 2x )3 + 18
C4( 2x )4 …)
Coefficient of x 3 is - 18C3( 2)3 + a × 18C2( 2)2 - b18C1( 2)
and coefficient of x 4 is 18C4( 2)4 - 18C3( 2)3a + 18
C 2 ( 2 )2 b
Coefficient of x 3and x 4 are zero.
(0, 0)
\ - 18C3( 2)3 + 18
C2( 2)2(a ) - 18C1( 2) b = 0
(2, –2)
4 ´ 17 ´ 16
Þ - 17a + b = 0 … (i)
3 ´2
Clearly from graph
32
and 80 - a+b=0 …(ii) c >0
3
b
Solving Eqs. (i) and (ii), we get - >0 Þ -b > 0
a
272
a = 16, b = Þ b<0
3
f (1) = a + b + c < 0
124. (a) Equation of plane parallel to the plane
ab < 0, ac < 0, bc > 0
x - 2y + 2z - 5 = 0 is x - 2y + 2z + l = 0.
128. (b) Let A = (1, 2) equation of line is
Since, this plane is unit distance from origin
x -1 y - 2 6
l = =r =
\ 1= cos q sin q 3
1+ 4 + 4
6 6
\ x = cos q + 1, y = sin q + 2
3 3
28 SELF STUDY GUIDE BITSAT

This point lies on line x + y = 4 Y


(1, 2)
6 6
\ cos q + sin q = 1
3 3
3 x
Þ sin q + cos q = x
2 x+
y=
æp ö 3
Þ sin ç + q÷ = y=x–x x
è4 ø 2
X
p 0 (1, 0)
Þ + q = 60° or 120°
4
Þ q = 15° or 75°
\ Larger angle = 75°. 1
\Area bounded = ò0 {(x + x x ) – (x – x x )}dx
129. (d) [P + 1] = [P ] + 1, Let [P ] = n, then n is integer
\ ([P + 1], [P ]) = (n + 1, n ) lie inside the region of circles
1 4
= 2ò x 3/ 2dx = sq unit
0 5
S1 = x 2 + y 2 - 2x - 15 = 0, C1 = (1, 0), r1 = 4
x
and S 2 = x 2 + y 2 - 2x - 7 = 0, C2 = (1, 0), r2 = 2 2 132. (d) We have, f (x ) = ò ( 3 sin x + 4 cos x )dx
0

Both circles are concentric. æ 4ö


Þ f ¢ (x ) = 3 sin x + 4 cos x = 5 sin çx + tan-1 ÷
2 2
\ (n + 1) + n - 2(n + 1) - 7 > 0 è 3ø
5p 4p
and (n + 1)2 + n 2 - 2(n + 1) - 15 < 0 Þ 4 < n 2 < 8 <x <
4 3
Which is not possible for any integer. Þ p < x + tan-1 4 / 3 < 2p
130. (a) We have,
Qf ¢ (x ) < 0, i.e. f (x ) is decreasing.
dy -y
= ex (e x - e y ) æ 5p 4p ö æ 4p ö
dx Q least value of f (x )is ç , ÷ = f ç ÷
è 4 3ø è 3ø
dy
Þ ey + e y e x = e 2x 4 p/ 3
dx least f (x ) = ò ( 3 sin x + 4 cos x ) dx
Put ey =v 0

dy dv = [ - 3 cos x + 4 sin x ] 40p / 3


\ ey =
dx dx æ 1 ö æ 3ö
dv = - 3 ç - - 1÷ + 4 çç - ÷
÷
\ + ve x = e 2x è 2 ø è 2 ø
dx
9-4 3
This is linear equation =
2
e x dx
I.F = e ò
x
= ee
133. (a) Let z1 = rei q
\Required solution is
x x
then, z1 = re - i q
v × ee = ò ee × e 2x dx + C
z1
x x
= e 2i q = e 2pi /n

v ×e e
=e e x
(e - 1) + C z1

ex ex
p
Þ e ×ey
=e (e x - 1) + C Þ q=
n
x
Þ e y = e x - 1 + ce -e Also,
Im ( z1) p
= tan q = tan = 2 - 1
Re ( z1) n
131. (d) Given, curve is (y – x )2 = x 3
p
Þ tan = 2 -1
Þ y–x =±x x 8
Þ y =x ± x x \ n=8
SOLVED PAPER 2019 29
134. (b) (1 + x + x 3 + x 4 )10 = [(1 + x )(1 + x 3 )]10 1- a 1 1
10 3 10 \ 1 1- b 1 =0
= (1 + x ) (1 + x )
1 1 1- c
10 10
= (1 + C1 + C2 x 2 + 10
C3 x 3 + 10
C4 x 4¼ )
Þ ab + ac + bc = abc
10
´ (1 + C1 x 3 + 10
C2 x 6 + ¼ ) ab + bc + ac
\ ³ (a 2 b 2c 2 )1/ 3 = (abc )1/ 3 ³ 3
\Coefficient of x 4 = (10C1)(10C1) + 10
C4 3
\ Minimum of abc = ( 3)3 = 27
= 100 + 210 = 310
139. (c) Given,
135. (c) Since only one letters can not be in wrong envelope so
that at least two letters are in wrong envelope means all AA ¢ = A ¢ A
the letters are not in right envelopes. and B = A -1A ¢
\x = 6 ! - 1 = 720 - 1 = 719
BB ¢ = ( A -1A ¢ )( A -1A ¢ )¢
Y = number of ways, so that all the letters are in wrong
envelops = ( A -1A ¢ )[( A ¢ )¢ ( A -1)¢ ]

æ 1 1 1 1 1 1ö = ( A -1A ¢ ) [ A( A ¢ )-1]
= 6 ! ç1 - + - + - + ÷
è 1! 2 ! 3 ! 4 ! 5 ! 6 ! ø [Q ( A ¢ )¢ = A, ( A –1)¢ = ( A ¢ )–1]
= 360 - 120 + 30 - 6 + 1 = 265 = A -1( A ¢ A )( A ¢ )-1
\ x - y = 719 - 265 = 454
= A -1( AA ¢ ( A ¢ )-1
136. (c) We have, x = log5 3 + log7 5 + log9 7
= ( A -1A ) ( A –1 A )¢
AM ³ GM
=I×I = I
log5 3 + log7 5 + log9 7
\
3 140. (a) We have,
³ (log5 3 × log7 5 × log9 7) 1/ 3 p
sin-1 x + tan-1 x = Þ 0 <x <1
1/ 3
2
æ log 3 log 5 log 7 ö p
Þ x ³3ç ´ ´ ÷ \ tan-1 x = - sin-1 x = cos-1 x
è log 5 log 7 log 9 ø 2
1/ 3
æ 1ö 3 æ 1- x 2 ö
Þ x >3ç ÷ Þx > 3 Þ tan-1 x = tan-1ç ÷
è 2ø 2 ç x ÷
è ø
137. (c) x 2 - 2x + A = 0
Þ x 2 = 1- x 2
Þ x = 1± 1- A
- 1± 5
\ p = 1 - 1 - A, q = 1 + 1 - A Þ x 4 + x 2 - 1= 0 Þ x 2 =
2
Similarly, r = 9 - 81 - B , s = 9 + 81 - B Þ x2 > 0
p, q, r , s are in AP. 5 -1
\ x2 =
Q q - p =s - r 2
Þ 2 1 - A = 2 81 - B Þ = 2x 2 + 1 = 5

Þ B = 80 + A 141. (a) Given, | cos x | = 2[x ]


and q - p =r -q 0 £ | cos x | £ 1
Þ 3 1 - A = 8 - 81 - B \ 0 £ 2 [x ] £ 1
1
Þ A = - 3, B = 77 0 £ [x ] £
2
138. (c) We have,
x =0
(1 - a )x + y + z = 0
\ | cos x | = 0
x + (1 - b ) y + z = 0
p
x + y + (1 - c ) z = 0 x :n p +
2
has many solutions For any value of n Î N, [n] ¹ 0
Hence, the equation has no solution.
30 SELF STUDY GUIDE BITSAT

p p f ¢ ¢ (x )
142. (c) 0 £ a, b £ ; a + b £ Þ =1
4 2 f ¢ (x )
cos( a + b ) = 4 / 5 Þ tan ( a + b ) = 3 / 4 f ¢ ¢ (x )
sin ( a - b ) = 5 / 13 Þ a - b Î q1
Þ ò f ¢ (x )
dx = ò dx

and tan ( a - b ) = 5 / 12 Þ log f ¢ (x ) = x + C


tan 2a = tan ( a + b + a - b ) Þ f ¢ (x ) = Ae x
tan( a + b ) + tan ( a - b )
= Þ f (x ) = Ae x + K
1 - tan ( a + b ) tan ( a - b )
f ( 0) = A + K = 1 …(ii)
3 / 4 + 5 / 12 56
= = 1
ò0 ( Ae
x x x
5 33 \ Ae = Ae + K + + k ) dx
1- 3 / 4 ´
12
Þ k + [ Ae x + Kx ]10 = 0
143. (c) 18 draws are required for 2 aces means in the first 17
draws, there is one ace and 16 non-ace and 18th draw Þ k + Ae - A + K = 0
2nd ace. Þ A(e - 1) + 2k = 0 …(iii)
48
C16 ´ 4C1 3 561 From Eqs. (ii) and (iii), we get
\ Required probability = 52
´ =
C17 35 15925 2 1- e
A= ,k =
144. (c) ( a + pb) is normal to c 3 -e 3 -e

\ ( a + pb) × c = 0 2e x 1- e
\ f (x ) = +
3 -e 3 -e
((1 - p ) i + ( 2 + 2p ) j + ( 3 + p )k$ ) × ( 3$i + $j ) = 0
$ $

3 - 3p + 2 + 2p = 0 Þ p = 5 148. (b) We have,


y = e 2x + x 2
145. (b) We have,
f (x ) = x [x ] at x = 0, y = 1
f (x ) = px [Q [x ] = p] dy
= 2e 2x + 2x
dx
f ¢ (x ) = p = [x ]
æ dy ö
\ f ¢ (x ) = [x ] ç ÷ =2
è dx ø x =0
146. (b) We have,
2 \Equation of normal at (0, 1) is
f (x ) = x 2e - x 1
y - 1= - (x )
-x 2 3 -x 2 2
f ¢ (x ) = 2xe - 2x e
2 Þ x + 2y - 2 = 0
f ¢ (x ) = 2xe - x (1 - x 2 )
2
\Distance from (0, 0) from normal =
f ¢ (x ) = 0, x = 0, - 1, 1 5
f ¢ ( - h ) < 0, f ¢ (h ) > 0, So minimum at x = 0 x -1 y - 2 z - 4
149. (c) Given line = = is passes through
f ¢ (1 - h ) > 0, f ¢ (1 + h ) < 0, So maximum at x = 1 -1 3 1
f ¢ ( - 1 - h ) > 0, f ¢ ( - 1 + h ) < 0, So maximum at x = - 1 P(1, 2, 4).
Minimum f (x ) = f ( 0) = 0 To find the reflection of line we need one more point on
1 the line clearly M (0, 5, 5) also lie on line.
Maximum f (x ) = e -1 =
e Let N ( a , b, g ) be the reflection M in the
Since, f (x ) ³ 0 " x x +y +z =7
So, difference between maximum and minimum values a b+5 g+5
\ + + +7
1 1 2 2 2
= -0=
e e Þ a+b+g=4
147. (a) Given, Also, MN is perpendicular, i.e. parallel to the normal of the
1 plane
f ¢ (x ) = f (x ) + ò0 f (x ) dx , f ( 0) = 1 …(i)

Þ f ¢ ¢ (x ) = f ¢ (x ) + 0
SOLVED PAPER 2019 31
a b -5 g -5 Þ y 2 + x 2 - 5x - 5y + xy + 8 = 0
\ = = =l
1 1 1
Þ y 2 + y (x - 5) + (x 2 - 5x + 8) = 0
Þ a = l, b = l + 5, g = l - 5
Þ Since y Î R
\l + l + 5 + l + 5 = 4
(x - 5)2 - 4 (x 2 - 5x + 8) ³ 0
Þ l=-2
\ N = ( - 2, 3, 5) Þ x 2 - 10x + 25 - 4x 2 + 20x - 32 ³ 0
x -1 y - 2 z - 4 Þ 3x 2 - 10x + 7 £ 0
Equation PN is = =
-3 1 -1
Þ 3x 2 - 7x - 3x + 7 £ 0
150. (b) We have,
Þ ( 3x - 7) (x - 1) £ 0
x + y + z =5
é 7ù
x2 + y2 + z2 =9 Þ x Î ê1, ú
ë 3û
Þ x 2 + y 2 + (5 - x - y )2 = 9 æ7 ö 4
Length of interval = ç - 1÷ =
è3 ø 3

You might also like